КулЛиб - Классная библиотека! Скачать книги бесплатно 

Задачи по геометрии [Автор Неизвестен] (pdf) читать онлайн

Книга в формате pdf! Изображения и текст могут не отображаться!


 [Настройки текста]  [Cбросить фильтры]
  [Оглавление]
úÁÄÁÞÉ ÐÏ ÐÌÁÎÉÍÅÔÒÉÉ
ëÕÒÓ ÐÌÁÎÉÍÅÔÒÉÉ (ò. ë. çÏÒÄÉÎ) ÓÏÓÔÏÑÌ ÐÏÞÔÉ ÐÏÌÎÏÓÔØÀ ÉÚ ÒÅÛÅÎÉÑ ÚÁÄÁÞ. ðÒÉÈÏÄÑ
× ËÌÁÓÓ, ÛËÏÌØÎÉËÉ ÕÖÅ ÇÏÄ ÚÁÎÉÍÁÌÉÓØ ÇÅÏÍÅÔÒÉÅÊ, ÎÏ ÐÏ ÒÁÚÎÙÍ ÕÞÅÂÎÉËÁÍ É × ÒÁÚÎÏÊ
ÓÔÅÐÅÎÉ, ÐÏÜÔÏÍÕ ÐÏ ÓÕÝÅÓÔ×Õ ×Ó£ ÎÁÞÉÎÁÌÏÓØ ÓÎÁÞÁÌÁ.

184

119. ðÏ×ÔÏÒÅÎÉÅ. éÚÍÅÒÅÎÉÅ ÏÔÒÅÚËÏ× É ÕÇÌÏ×

119.1.

îÁ ÐÒÑÍÏÊ ×ÙÂÒÁÎÙ ÞÅÔÙÒÅ ÔÏÞËÉ

þÅÍÕ ÍÏÖÅÔ ÂÙÔØ ÒÁ×ÎÏ
119.2.

A, B, C É D, ÐÒÉÞÅÍ AB 1, BC 2, CD 4.

AD? õËÁÖÉÔÅ ×ÓÅ ×ÏÚÍÏÖÎÏÓÔÉ.

îÁ ÄÅÒÅ×ÑÎÎÏÊ ÌÉÎÅÊËÅ ÏÔÍÅÞÅÎÙ ÔÒÉ ÄÅÌÅÎÉÑ:

=

=

=

0, 7 É 11. ëÁË ÏÔÌÏÖÉÔØ Ó ÅÅ

8; (Â) 5?
ôÏÞËÁ B ÌÅÖÉÔ ÎÁ ÏÔÒÅÚËÅ AC ÄÌÉÎÏÊ 5. îÁÊÄÉÔÅ ÒÁÓÓÔÏÑÎÉÅ ÍÅÖÄÕ ÓÅÒÅÄÉÎÁÍÉ
ÏÔÒÅÚËÏ× AB É BC.
÷ ÄÅÒÅ×ÎÅ Õ ÐÒÑÍÏÊ ÄÏÒÏÇÉ ÓÔÏÑÔ ÞÅÔÙÒÅ ÉÚÂÙ A, B, C É D ÎÁ ÒÁÓÓÔÏÑÎÉÉ 50 ÍÅÐÏÍÏÝØÀ ÏÔÒÅÚÏË ÄÌÉÎÏÊ (Á)
119.3.

119.4.

ÔÒÏ× ÄÒÕÇ ÏÔ ÄÒÕÇÁ. ÷ ËÁËÏÊ ÔÏÞËÅ ÄÏÒÏÇÉ ÎÁÄÏ ÐÏÓÔÒÏÉÔØ ËÏÌÏÄÅÃ, ÞÔÏÂÙ ÓÕÍÍÁ ÒÁÓÓÔÏÑÎÉÊ
ÏÔ ËÏÌÏÄÃÁ ÄÏ ×ÓÅÈ ÞÅÔÙÒ£È ÉÚ ÂÙÌÁ ÂÙ ÎÁÉÍÅÎØÛÅÊ?
119.5.

A ÖÉ×ÅÔ 50 ÛËÏÌØÎÉËÏ×, × ÄÅÒÅ×ÎÅ B ÖÉ×ÅÔ 100 ÛËÏÌØÎÉËÏ×. òÁÓÓÔÏÑÎÉÅ
3 ËÉÌÏÍÅÔÒÁ. ÷ ËÁËÏÊ ÔÏÞËÅ ÄÏÒÏÇÉ ÉÚ A × B ÎÁÄÏ ÐÏÓÔÒÏÉÔØ ÛËÏÌÕ, ÞÔÏÂÙ

÷ ÄÅÒÅ×ÎÅ

ÍÅÖÄÕ ÄÅÒÅ×ÎÑÍÉ

ÓÕÍÍÁÒÎÏÅ ÒÁÓÓÔÏÑÎÉÅ, ÐÒÏÈÏÄÉÍÏÅ ×ÓÅÍÉ ÛËÏÌØÎÉËÁÍÉ, ÂÙÌÏ ËÁË ÍÏÖÎÏ ÍÅÎØÛÅ?

ôÏÞËÉ A, B, C ÒÁÓÐÏÌÏÖÅÎÙ ÎÁ ÏÄÎÏÊ ÐÒÑÍÏÊ; ÏÔÎÏÛÅÎÉÅ ÒÁÓÓÔÏÑÎÉÊ AC BC
m n. îÁÊÄÉÔÅ ÏÔÎÏÛÅÎÉÑ AC AB É BC AB. (õËÁÖÉÔÅ ×ÓÅ ×ÏÚÍÏÖÎÏÓÔÉ.)
ôÏÞËÁ B ÄÅÌÉÔ ÏÔÒÅÚÏË AC × ÏÔÎÏÛÅÎÉÉ 2 1 (ÓÞÉÔÁÑ ÏÔ ÔÏÞËÉ A). ôÏÞËÁ D ÄÅÌÉÔ
ÏÔÒÅÚÏË AB × ÏÔÎÏÛÅÎÉÉ 3 2 (ÓÞÉÔÁÑ ÏÔ A). ÷ ËÁËÏÍ ÏÔÎÏÛÅÎÉÉ ÄÅÌÉÔ ÔÏÞËÁ D ÏÔÒÅÚÏË
AB?
éÍÅÅÔÓÑ ÄÅÒÅ×ÑÎÎÙÊ ÕÇÏÌØÎÉË Ó ÕÇÌÏÍ × 19. ëÁË ÐÏÓÔÒÏÉÔØ Ó ÅÇÏ ÐÏÍÏÝØÀ ÕÇÏÌ

× 1 ?
þÅÒÅÚ ÔÏÞËÕ ÎÁ ÐÌÏÓËÏÓÔÉ ÐÒÏ×ÅÌÉ 10 ÐÒÑÍÙÈ, ÐÏÓÌÅ ÞÅÇÏ ÐÌÏÓËÏÓÔØ ÒÁÚÒÅÚÁÌÉ ÐÏ

ÜÔÉÍ ÐÒÑÍÙÍ ÎÁ ÕÇÌÙ. äÏËÁÖÉÔÅ, ÞÔÏ ÈÏÔÑ ÂÙ ÏÄÉÎ ÉÚ ÜÔÉÈ ÕÇÌÏ× ÍÅÎØÛÅ 20 .
119.6.

ÒÁ×ÎÏ

:

:

:

:

119.7.

:

:

119.8.

119.9.

119.10.

(Á) îÁ ÓËÏÌØËÏ ÇÒÁÄÕÓÏ× ÐÏ×ÏÒÁÞÉ×ÁÅÔÓÑ ÚÁ ÍÉÎÕÔÕ ÍÉÎÕÔÎÁÑ ÓÔÒÅÌËÁ? þÁÓÏ×ÁÑ

ÓÔÒÅÌËÁ? (Â) ëÁËÏÊ ÕÇÏÌ ÏÂÒÁÚÕÀÔ ÍÉÎÕÔÎÁÑ É ÞÁÓÏ×ÁÑ ÓÔÒÅÌËÁ × ÐÑÔØ ÍÉÎÕÔ ÞÅÔ×£ÒÔÏÇÏ? (×)
÷ ÐÏÌÄÅÎØ ÍÉÎÕÔÎÁÑ É ÞÁÓÏ×ÁÑ ÓÔÒÅÌËÁ ÓÏ×ÐÁÌÉ. ëÏÇÄÁ ÏÎÉ ÓÏ×ÐÁÄÕÔ × ÓÌÅÄÕÀÝÉÊ ÒÁÚ? (Ç)
óËÏÌØËÏ ÒÁÚ × ÔÅÞÅÎÉÅ ÓÕÔÏË ÞÁÓÏ×ÁÑ É ÍÉÎÕÔÎÁÑ ÓÔÒÅÌËÉ ÓÏ×ÐÁÄÁÀÔ? ïÂÒÁÚÕÀÔ ÒÁÚ×£ÒÎÕÔÙÊ
ÕÇÏÌ? ïÂÒÁÚÕÀÔ ÐÒÑÍÏÊ ÕÇÏÌ?
119.11.

äÏËÁÖÉÔÅ, ÞÔÏ (Á) ÂÉÓÓÅËÔÒÉÓÙ Ä×ÕÈ ÓÍÅÖÎÙÈ ÕÇÌÏ× ÐÅÒÐÅÎÄÉËÕÌÑÒÎÙ; (Â) ÂÉÓ-

ÓÅËÔÒÉÓÙ Ä×ÕÈ ×ÅÒÔÉËÁÌØÎÙÈ ÕÇÌÏ× ÌÅÖÁÔ ÎÁ ÏÄÎÏÊ ÐÒÑÍÏÊ.
119.12.

éÚ ÔÏÞËÉ ÎÁ ÌÉÓÔÅ ÂÕÍÁÇÉ ÐÒÏ×ÅÌÉ ÞÅÔÙÒÅ ÌÕÞÁ, ÄÅÌÑÝÉÈ ÐÌÏÓËÏÓÔØ ÎÁ ÞÅÔÙÒÅ

ÕÇÌÁ. úÁÔÅÍ ÌÉÓÔ ÒÁÚÒÅÚÁÌÉ ÐÏ ÂÉÓÓÅËÔÒÉÓÁÍ ÜÔÉÈ ÕÇÌÏ× ÎÁ ÞÅÔÙÒÅ ÞÁÓÔÉ (ËÏÔÏÒÙÅ ÔÁËÖÅ
Ñ×ÌÑÀÔÓÑ ÕÇÌÁÍÉ). äÏËÁÖÉÔÅ, ÞÔÏ Ä×Á ÉÚ ÜÔÉÈ ÕÇÌÏ× ÏÂÒÁÚÕÀÔ × ÓÕÍÍÅ
ÔÏÖÅ.

180, É Ä×Á ÄÒÕÇÉÈ |

éÚ ÔÏÞËÉ O ÎÁ ÐÌÏÓËÏÓÔÉ ×ÙÈÏÄÑÔ 4 ÌÕÞÁ, ÓÌÅÄÕÀÝÉÅ ÄÒÕÇ ÚÁ ÄÒÕÇÏÍ ÐÏ ÞÁÓÏ×ÏÊ
OA, OB, OC É OD. éÚ×ÅÓÔÎÏ, ÞÔÏ ÓÕÍÍÁ ÕÇÌÏ× AOB É COD ÒÁ×ÎÁ 180. äÏËÁÖÉÔÅ,
ÞÔÏ ÂÉÓÓÅËÔÒÉÓÙ ÕÇÌÏ× AOC É BOD ÐÅÒÐÅÎÄÉËÕÌÑÒÎÙ.
119.13.

ÓÔÒÅÌËÅ:

185

120. ðÏ×ÔÏÒÅÎÉÅ. ðÒÉÚÎÁËÉ ÒÁ×ÅÎÓÔ×Á ÔÒÅÕÇÏÌØÎÉËÏ×

120.1.

òÁ×ÎÙ ÌÉ ÔÒÅÕÇÏÌØÎÉËÉ (Á) ÐÏ Ä×ÕÍ ÓÔÏÒÏÎÁÍ É ÕÇÌÕ; (Â) ÐÏ ÓÔÏÒÏÎÅ É Ä×ÕÍ ÕÇÌÁÍ?

120.2.

ó ÐÏÍÏÝØÀ ÃÉÒËÕÌÑ É ÌÉÎÅÊËÉ ÐÏÓÔÒÏÊÔÅ ÔÒÅÕÇÏÌØÎÉË (Á) ÐÏ ÔÒ£Í ÓÔÏÒÏÎÁÍ; (Â)

ÐÏ Ä×ÕÍ ÓÔÏÒÏÎÁÍ É ÕÇÌÕ ÍÅÖÄÕ ÎÉÍÉ; (×) ÐÏ ÓÔÏÒÏÎÅ É Ä×ÕÍ ÐÒÉÌÅÖÁÝÉÍ Ë ÎÅÊ ÕÇÌÁÍ.
120.3.

äÏËÁÖÉÔÅ, ÞÔÏ (Á) × ÒÁ×ÎÏÂÅÄÒÅÎÎÏÍ ÔÒÅÕÇÏÌØÎÉËÅ ÕÇÌÙ ÐÒÉ ÏÓÎÏ×ÁÎÉÉ ÒÁ×ÎÙ;

(Â) ÅÓÌÉ Ä×Á ÕÇÌÁ ÔÒÅÕÇÏÌØÎÉËÁ ÒÁ×ÎÙ ÍÅÖÄÕ ÓÏÂÏÊ, ÔÏ ÔÒÅÕÇÏÌØÎÉË ÒÁ×ÎÏÂÅÄÒÅÎÎÙÊ.
120.4.

íÅÄÉÁÎÁ ÔÒÅÕÇÏÌØÎÉËÁ Ñ×ÌÑÅÔÓÑ ÅÇÏ ×ÙÓÏÔÏÊ. äÏËÁÖÉÔÅ, ÞÔÏ ÔÒÅÕÇÏÌØÎÉË ÒÁ×ÎÏ-

ÂÅÄÒÅÎÎÙÊ.
120.5.

äÏËÁÖÉÔÅ, ÞÔÏ ÂÉÓÓÅËÔÒÉÓÁ ÒÁ×ÎÏÂÅÄÒÅÎÎÏÇÏ ÔÒÅÕÇÏÌØÎÉËÁ, ÐÒÏ×ÅÄ£ÎÎÁÑ ÉÚ ×ÅÒ-

ÛÉÎÙ, Ñ×ÌÑÅÔÓÑ ÍÅÄÉÁÎÏÊ É ×ÙÓÏÔÏÊ.
120.6.

äÏËÁÖÉÔÅ, ÞÔÏ ×ÙÓÏÔÁ ÒÁ×ÎÏÂÅÄÒÅÎÎÏÇÏ ÔÒÅÕÇÏÌØÎÉËÁ, ÐÒÏ×ÅÄ£ÎÎÁÑ ÉÚ ×ÅÒÛÉÎÙ,

Ñ×ÌÑÅÔÓÑ ÍÅÄÉÁÎÏÊ É ÂÉÓÓÅËÔÒÉÓÏÊ.

ABC ÍÅÄÉÁÎÁ AM ÐÒÏÄÏÌÖÅÎÁ ÚÁ ÔÏÞËÕ M ÎÁ ÒÁÓÓÔÏÑÎÉÅ, ÒÁ×ÎÏÅ
AM. îÁÊÄÉÔÅ ÒÁÓÓÔÏÑÎÉÅ ÏÔ ÐÏÌÕÞÅÎÎÏÊ ÔÏÞËÉ ÄÏ ×ÅÒÛÉÎ B É C, ÅÓÌÉ AB = 4, AC = 5.
120.7.

120.8.

÷ ÔÒÅÕÇÏÌØÎÉËÅ

âÉÓÓÅËÔÒÉÓÁ ÔÒÅÕÇÏÌØÎÉËÁ Ñ×ÌÑÅÔÓÑ ÅÇÏ ÍÅÄÉÁÎÏÊ. äÏËÁÖÉÔÅ, ÞÔÏ ÔÒÅÕÇÏÌØÎÉË

ÒÁ×ÎÏÂÅÄÒÅÎÎÙÊ.

ABC É A 0B 0C 0 ÒÁ×ÎÙ Ä×Å ÓÔÏÒÏÎÙ É ÕÇÏÌ: \A = \A 0, AB =
A 0B 0, BC = B 0C 0. ðÒÉ ÜÔÏÍ \C = 80, Á ÕÇÏÌ C 0 ÔÕÐÏÊ. îÁÊÄÉÔÅ ÅÇÏ ×ÅÌÉÞÉÎÕ.
120.9.

÷ ÔÒÅÕÇÏÌØÎÉËÁÈ

120.10.

óÆÏÒÍÕÌÉÒÕÊÔÅ É ÄÏËÁÖÉÔÅ ÐÒÉÚÎÁËÉ ÒÁ×ÅÎÓÔ×Á ÐÒÑÍÏÕÇÏÌØÎÙÈ ÔÒÅÕÇÏÌØÎÉËÏ×

(Á) ÐÏ Ä×ÕÍ ËÁÔÅÔÁÍ; (Â) ÐÏ ËÁÔÅÔÕ É ÇÉÐÏÔÅÎÕÚÅ; (×) ÐÏ ËÁÔÅÔÕ É ÏÓÔÒÏÍÕ ÕÇÌÕ.
120.11.

ó ÐÏÍÏÝØÀ ÃÉÒËÕÌÑ É ÌÉÎÅÊËÉ ÐÏÓÔÒÏÊÔÅ ÔÒÅÕÇÏÌØÎÉË (Á) ÐÏ ÕÇÌÕ, ×ÙÓÏÔÅ É

ÂÉÓÓÅËÔÒÉÓÅ, ÐÒÏ×ÅÄ£ÎÎÙÍ ÉÚ ×ÅÒÛÉÎÙ ÜÔÏÇÏ ÕÇÌÁ; (Â) ÐÏ ÓÔÏÒÏÎÅ, ÍÅÄÉÁÎÅ, ÐÒÏ×ÅÄ£ÎÎÏÊ Ë
ÜÔÏÊ ÓÔÏÒÏÎÅ, É ×ÙÓÏÔÅ, ÏÐÕÝÅÎÎÏÊ ÎÁ ÄÒÕÇÕÀ ÓÔÏÒÏÎÕ.
120.12.

äÏËÁÖÉÔÅ ÒÁ×ÅÎÓÔ×Ï ÔÒÅÕÇÏÌØÎÉËÏ× (Á) ÐÏ Ä×ÕÍ ÓÔÏÒÏÎÁÍ É ÍÅÄÉÁÎÅ, ×ÙÈÏÄÑÝÉÍ

ÉÚ ÏÄÎÏÊ ×ÅÒÛÉÎÙ; (Â) ÐÏ ÍÅÄÉÁÎÅ É Ä×ÕÍ ÕÇÌÁÍ, ÎÁ ËÏÔÏÒÙÅ ÒÁÚÂÉ×ÁÅÔ ÜÔÁ ÍÅÄÉÁÎÁ ÕÇÏÌ
ÔÒÅÕÇÏÌØÎÉËÁ.
120.13.

äÏËÁÖÉÔÅ, ÞÔÏ ÓÅÒÅÄÉÎÎÙÊ ÐÅÒÐÅÎÄÉËÕÌÑÒ Ë ÏÔÒÅÚËÕ ÅÓÔØ ÇÅÏÍÅÔÒÉÞÅÓËÏÅ ÍÅÓÔÏ

ÔÏÞÅË, ÒÁ×ÎÏÕÄÁÌ£ÎÎÙÈ ÏÔ ËÏÎÃÏ× ÜÔÏÇÏ ÏÔÒÅÚËÁ.
120.14.

äÏËÁÖÉÔÅ, ÞÔÏ × ÒÁ×ÎÙÈ ÔÒÅÕÇÏÌØÎÉËÁÈ ÓÏÏÔ×ÅÔÓÔ×ÕÀÝÉÅ (Á) ÍÅÄÉÁÎÙ; (Â) ÂÉÓ-

ÓÅËÔÒÉÓÙ; (×) ×ÙÓÏÔÙ ÒÁ×ÎÙ ÍÅÖÄÕ ÓÏÂÏÊ.
120.15.

ä×Å ×ÙÓÏÔÙ ÔÒÅÕÇÏÌØÎÉËÁ ÒÁ×ÎÙ ÍÅÖÄÕ ÓÏÂÏÊ. äÏËÁÖÉÔÅ, ÞÔÏ ÔÒÅÕÇÏÌØÎÉË ÒÁ×-

ÎÏÂÅÄÒÅÎÎÙÊ.
120.16.

îÁÊÄÉÔÅ ÇÅÏÍÅÔÒÉÞÅÓËÏÅ ÍÅÓÔÏ ×ÎÕÔÒÅÎÎÉÈ ÔÏÞÅË ÕÇÌÁ, ÒÁ×ÎÏÕÄÁÌ£ÎÎÙÈ ÏÔ ÅÇÏ

ÓÔÏÒÏÎ.

AB Ñ×ÌÑÀÔÓÑ ÃÅÎÔÒÁÍÉ Ä×ÕÈ ÏËÒÕÖÎÏÓÔÅÊ, ÐÅÒÅÓÅËÁÀÝÉÈÓÑ ×
ÔÏÞËÁÈ C É D. äÏËÁÖÉÔÅ, ÞÔÏ CD ÐÅÒÐÅÎÄÉËÕÌÑÒÎÏ AB.
120.17.

ëÏÎÃÙ ÏÔÒÅÚËÁ

120.18.

òÁÚÄÅÌÉÔÅ ÄÁÎÎÙÊ ÏÔÒÅÚÏË ÐÏÐÏÌÁÍ Ó ÐÏÍÏÝØÀ ÃÉÒËÕÌÑ É ÌÉÎÅÊËÉ.

120.19.

éÚ ÔÏÞËÉ ×ÎÅ ÐÒÑÍÏÊ ÏÐÕÓÔÉÔÅ ÐÅÒÐÅÎÄÉËÕÌÑÒ ÎÁ ÜÔÕ ÐÒÑÍÕÀ Ó ÐÏÍÏÝØÀ ÃÉÒËÕÌÑ

É ÌÉÎÅÊËÉ, ÐÒÏ×ÏÄÑ ÎÅ ÂÏÌÅÅ ÔÒ£È ÌÉÎÉÊ.
120.20.

ó ÐÏÍÏÝØÀ ÃÉÒËÕÌÑ É ÌÉÎÅÊËÉ ÐÏÓÔÒÏÊÔÅ ÔÒÅÕÇÏÌØÎÉË ÐÏ Ä×ÕÍ ÓÔÏÒÏÎÁÍ É

ÍÅÄÉÁÎÅ.

186

120.21.

äÏËÁÖÉÔÅ ÐÒÉÚÎÁË ÒÁ×ÅÎÓÔ×Á ÐÒÑÍÏÕÇÏÌØÎÙÈ ÔÒÅÕÇÏÌØÎÉËÏ× ÐÏ ÇÉÐÏÔÅÎÕÚÅ É

ÏÓÔÒÏÍÕ ÕÇÌÕ.
120.22.

îÁÊÄÉÔÅ ÇÅÏÍÅÔÒÉÞÅÓËÏÅ ÍÅÓÔÏ ×ÎÕÔÒÅÎÎÉÈ ÔÏÞÅË ÄÁÎÎÏÇÏ ÕÇÌÁ, ÓÕÍÍÁ ÒÁÓÓÔÏÑ-

ÎÉÊ ÏÔ ËÏÔÏÒÙÈ ÄÏ ÓÔÏÒÏÎ ÜÔÏÇÏ ÕÇÌÁ ÒÁ×ÎÁ ÚÁÄÁÎÎÏÊ ×ÅÌÉÞÉÎÅ.

187

121. ðÏ×ÔÏÒÅÎÉÅ. ðÁÒÁÌÌÅÌØÎÏÓÔØ. óÕÍÍÁ ÕÇÌÏ× ÔÒÅÕÇÏÌØÎÉËÁ

121.1.

þÅÒÅÚ ÔÏÞËÕ, ÎÅ ÌÅÖÁÝÕÀ ÎÁ ÄÁÎÎÏÊ ÐÒÑÍÏÊ, ÐÒÏ×ÅÄÉÔÅ (ÃÉÒËÕÌÅÍ É ÌÉÎÅÊËÏÊ)

ÐÒÑÍÕÀ, ÐÁÒÁÌÌÅÌØÎÕÀ ÄÁÎÎÏÊ.
121.2.

ä×Å ÐÁÒÁÌÌÅÌØÎÙÅ ÐÒÑÍÙÅ ÐÅÒÅÓÅÞÅÎÙ ÔÒÅÔØÅÊ. îÁÊÄÉÔÅ ÕÇÏÌ ÍÅÖÄÕ ÂÉÓÓÅËÔÒÉ-

ÓÁÍÉ ×ÎÕÔÒÅÎÎÉÈ ÏÄÎÏÓÔÏÒÏÎÎÉÈ ÕÇÌÏ×.
121.3.

äÏËÁÖÉÔÅ, ÞÔÏ Ä×Å ÐÒÑÍÙÅ, ÐÁÒÁÌÌÅÌØÎÙÅ ÔÒÅÔØÅÊ, ÐÁÒÁÌÌÅÌØÎÙ ÍÅÖÄÕ ÓÏÂÏÊ.

121.4.

îÁÊÄÉÔÅ ÇÅÏÍÅÔÒÉÞÅÓËÏÅ ÍÅÓÔÏ ÔÏÞÅË, ÒÁ×ÎÏÕÄÁÌ£ÎÎÙÈ ÏÔ ÄÁÎÎÏÊ ÐÒÑÍÏÊ.

121.5.

ðÏÓÔÒÏÊÔÅ ÔÒÅÕÇÏÌØÎÉË ÐÏ Ä×ÕÍ ÓÔÏÒÏÎÁÍ É ×ÙÓÏÔÅ, ÏÐÕÝÅÎÎÏÊ ÎÁ ÏÄÎÕ ÉÚ ÎÉÈ.

121.6.

äÏËÁÖÉÔÅ, ÞÔÏ ÂÉÓÓÅËÔÒÉÓÁ ×ÎÅÛÎÅÇÏ ÕÇÌÁ ÐÒÉ ×ÅÒÛÉÎÅ ÒÁ×ÎÏÂÅÄÒÅÎÎÏÇÏ ÔÒÅ-

ÕÇÏÌØÎÉËÁ ÐÁÒÁÌÌÅÌØÎÁ ÏÓÎÏ×ÁÎÉÀ. ÷ÅÒÎÏ ÌÉ ÏÂÒÁÔÎÏÅ?

180.

121.7.

äÏËÁÖÉÔÅ, ÞÔÏ ÓÕÍÍÁ ÕÇÌÏ× ÔÒÅÕÇÏÌØÎÉËÁ ÒÁ×ÎÁ

121.8.

äÏËÁÖÉÔÅ, ÞÔÏ ×ÎÅÛÎÉÊ ÕÇÏÌ ÔÒÅÕÇÏÌØÎÉËÁ ÒÁ×ÅÎ ÓÕÍÍÅ Ä×ÕÈ ÎÅ ÓÍÅÖÎÙÈ Ó ÎÉÍ

×ÎÕÔÒÅÎÎÉÈ.
121.9.

ôÒÅÕÇÏÌØÎÉË

ABC | ÒÁ×ÎÏÂÅÄÒÅÎÎÙÊ (AB = BC). ïÔÒÅÚÏË AM ÄÅÌÉÔ ÅÇÏ ÎÁ
AB É MC. îÁÊÄÉÔÅ ÕÇÏÌ B.

Ä×Á ÒÁ×ÎÏÂÅÄÒÅÎÎÙÈ ÔÒÅÕÇÏÌØÎÉËÁ Ó ÏÓÎÏ×ÁÎÉÑÍÉ
121.10.

íÅÄÉÁÎÁ ÔÒÅÕÇÏÌØÎÉËÁ ÒÁ×ÎÁ ÐÏÌÏ×ÉÎÅ ÓÔÏÒÏÎÙ, Ë ËÏÔÏÒÏÊ ÏÎÁ ÐÒÏ×ÅÄÅÎÁ. äÏ-

ËÁÖÉÔÅ, ÞÔÏ ÔÒÅÕÇÏÌØÎÉË ÐÒÑÍÏÕÇÏÌØÎÙÊ. ÷ÅÒÎÏ ÌÉ ÏÂÒÁÔÎÏÅ?
121.11.

äÏËÁÖÉÔÅ, ÞÔÏ × ÐÒÑÍÏÕÇÏÌØÎÏÍ ÔÒÅÕÇÏÌØÎÉËÅ ËÁÔÅÔ, ÌÅÖÁÝÉÊ ÐÒÏÔÉ× ÕÇÌÁ ×

ÇÒÁÄÕÓÏ×, ÒÁ×ÅÎ ÐÏÌÏ×ÉÎÅ ÇÉÐÏÔÅÎÕÚÙ.

30

A É B. ðÏÌØÚÕÑÓØ ÔÏÌØËÏ ÃÉÒËÕÌÅÍ, ÕÄ×ÏÊÔÅ ÏÔÒÅÚÏË AB, Ô.Å. ÐÏÓÔÒÏÊÔÅ ÔÁËÕÀ ÔÏÞËÕ C, ÞÔÏÂÙ ÔÏÞËÉ A, B É C ÌÅÖÁÌÉ ÎÁ ÏÄÎÏÊ ÐÒÑÍÏÊ É AC = 2 BC.
121.12.

äÁÎÙ ÔÏÞËÉ

121.13.

ëÁËÉÅ ÚÎÁÞÅÎÉÑ ÍÏÖÅÔ ÐÒÉÎÉÍÁÔØ (Á) ÎÁÉÂÏÌØÛÉÊ ÕÇÏÌ ÔÒÅÕÇÏÌØÎÉËÁ; (Â) ÎÁÉ-

ÍÅÎØÛÉÊ ÕÇÏÌ ÔÒÅÕÇÏÌØÎÉËÁ; (×) ÓÒÅÄÎÉÊ ÐÏ ×ÅÌÉÞÉÎÅ ÕÇÏÌ ÔÒÅÕÇÏÌØÎÉËÁ?
121.14.

îÁÊÄÉÔÅ ÓÕÍÍÕ ×ÎÕÔÒÅÎÎÉÈ ÕÇÌÏ× (Á) ÞÅÔÙÒÅÈÕÇÏÌØÎÉËÁ; (Â) ×ÙÐÕËÌÏÇÏ ÐÑÔÉ-

ÕÇÏÌØÎÉËÁ; (×) ×ÙÐÕËÌÏÇÏ

n-ÕÇÏÌØÎÉËÁ.

121.15.

îÁÊÄÉÔÅ ÓÕÍÍÕ ÐÑÔÉ ÕÇÌÏ× ÐÒÉ ×ÅÒÛÉÎÁÈ ÐÑÔÉËÏÎÅÞÎÏÊ Ú×ÅÚÄÙ.

121.16.

îÁÊÄÉÔÅ ÓÕÍÍÕ ×ÎÅÛÎÉÈ ÕÇÌÏ× ÐÒÉ ×ÅÒÛÉÎÁÈ ×ÙÐÕËÌÏÇÏ

121.17.

ðÒÏÄÏÌÖÅÎÉÑ Ä×ÕÈ ÐÒÏÔÉ×ÏÐÏÌÏÖÎÙÈ ÓÔÏÒÏÎ

ABCD ÐÅÒÅÓÅËÁÀÔÓÑ ÐÏÄ ÕÇÌÏÍ 20
AD É BC) | ÔÏÖÅ. äÏËÁÖÉÔÅ, ÞÔÏ
ÏÔÌÉÞÁÀÔÓÑ ÎÁ 40.

É

, ÐÒÏÄÏÌÖÅÎÉÑ Ä×ÕÈ ÄÒÕÇÉÈ ÐÒÏÔÉ×ÏÐÏÌÏÖÎÙÈ ÓÔÏÒÏÎ

(

121.18.

AB

n-ÕÇÏÌØÎÉËÁ.
CD ÞÅÔÙÒÅÈÕÇÏÌØÎÉËÁ

Ä×Á ÕÇÌÁ × ÞÅÔÙÒÅÈÕÇÏÌØÎÉËÅ ÒÁ×ÎÙ, Á Ä×Á ÄÒÕÇÉÈ

ðÏÓÔÒÏÊÔÅ ÔÒÅÕÇÏÌØÎÉË ÐÏ ÓÔÏÒÏÎÅ, ÐÒÏÔÉ×ÏÌÅÖÁÝÅÍÕ ÕÇÌÕ É ÓÕÍÍÅ Ä×ÕÈ ÄÒÕÇÉÈ

ÓÔÏÒÏÎ.
121.19.

ðÏÓÔÒÏÊÔÅ ÔÒÅÕÇÏÌØÎÉË ÐÏ ÐÅÒÉÍÅÔÒÕ É Ä×ÕÍ ÕÇÌÁÍ.

121.20.

îÁ ËÁÔÅÔÁÈ

AC É BC ÐÒÑÍÏÕÇÏÌØÎÏÇÏ ÔÒÅÕÇÏÌØÎÉËÁ ABC ×ÎÅ ÅÇÏ ÐÏÓÔÒÏÅÎÙ
Ë×ÁÄÒÁÔÙ ACDE É BCKF. éÚ ÔÏÞÅË E É F ÎÁ ÐÒÑÍÕÀ AB ÏÐÕÝÅÎÙ ÐÅÒÐÅÎÄÉËÕÌÑÒÙ EM É
FN. äÏËÁÖÉÔÅ, ÞÔÏ EF + FN = AB.
îÁ ÓÔÏÒÏÎÁÈ BC É CD Ë×ÁÄÒÁÔÁ ABCD ÐÏÓÔÒÏÅÎÙ ×ÎÅÛÎÉÍ ÏÂÒÁÚÏÍ ÐÒÁ×ÉÌØÎÙÅ
ÔÒÅÕÇÏÌØÎÉËÉ BCK É DCL. äÏËÁÖÉÔÅ, ÞÔÏ ÔÒÅÕÇÏÌØÎÉË AKL ÐÒÁ×ÉÌØÎÙÊ.
îÁ ÓÔÏÒÏÎÁÈ AB É BC ÔÒÅÕÇÏÌØÎÉËÁ ABC ×ÎÅ ÅÇÏ ÐÏÓÔÒÏÅÎÙ Ë×ÁÄÒÁÔÙ AMNB
É BKLC. äÏËÁÖÉÔÅ, ÞÔÏ ÏÔÒÅÚÏË KN ×Ä×ÏÅ ÂÏÌØÛÅ ÍÅÄÉÁÎÙ BP ÔÒÅÕÇÏÌØÎÉËÁ ABC.
121.21.

121.22.

188

122. îÅÒÁ×ÅÎÓÔ×Ï ÔÒÅÕÇÏÌØÎÉËÁ

122.1.

óÔÏÒÏÎÙ ÒÁ×ÎÏÂÅÄÒÅÎÎÏÇÏ ÔÒÅÕÇÏÌØÎÉËÁ ÒÁ×ÎÙ 3 É 7. ëÁËÁÑ ÉÚ ÓÔÏÒÏÎ Ñ×ÌÑÅÔÓÑ

ÏÓÎÏ×ÁÎÉÅÍ?
122.2.

íÏÖÅÔ ÌÉ × ÔÒÅÕÇÏÌØÎÉËÅ ÓÔÏÒÏÎÁ ÂÙÔØ ×Ä×ÏÅ ÂÏÌØÛÅ ÄÒÕÇÏÊ ÓÔÏÒÏÎÙ É ×Ä×ÏÅ

ÍÅÎØÛÅ ÔÒÅÔØÅÊ?
122.3.

÷ ÔÒÅÕÇÏÌØÎÉËÅ Ä×Å ÓÔÏÒÏÎÙ ÒÁ×ÎÙ

ÉÚ×ÅÓÔÎÏ, ÞÔÏ ÅÅ ÄÌÉÎÁ ÒÁ×ÎÁ ÃÅÌÏÍÕ ÞÉÓÌÕ.

3;14 É 0;67. îÁÊÄÉÔÅ ÔÒÅÔØÀ ÓÔÏÒÏÎÕ, ÅÓÌÉ

A, B, C É D. äÏËÁÖÉÔÅ, ÞÔÏ AD 6 AB + BC +

122.4.

îÁ ÐÌÏÓËÏÓÔÉ ÄÁÎÙ ÞÅÔÙÒÅ ÔÏÞËÉ

122.5.

äÏËÁÖÉÔÅ, ÞÔÏ × ÔÒÅÕÇÏÌØÎÉËÅ ÌÀÂÁÑ ÓÔÏÒÏÎÁ ÍÅÎØÛÅ ÐÏÌÏ×ÉÎÙ ÐÅÒÉÍÅÔÒÁ.

122.6.

äÏËÁÖÉÔÅ, ÞÔÏ × ÞÅÔÙÒ£ÈÕÇÏÌØÎÉËÅ ÌÀÂÁÑ ÄÉÁÇÏÎÁÌØ ÍÅÎØÛÅ ÐÏÌÏ×ÉÎÙ ÐÅÒÉÍÅÔÒÁ.

122.7.

ä×Å ÏËÒÕÖÎÏÓÔÉ ÒÁÄÉÕÓÏ×

CD.

3 É 5 ÐÅÒÅÓÅËÁÀÔÓÑ. äÏËÁÖÉÔÅ, ÞÔÏ ÒÁÓÓÔÏÑÎÉÅ ÍÅÖÄÕ
ÉÈ ÃÅÎÔÒÁÍÉ (Á) ÎÅ ÂÏÌØÛÅ 8; (Â) ÎÅ ÍÅÎØÛÅ 2.
òÁÓÓÔÏÑÎÉÅ ÍÅÖÄÕ ÃÅÎÔÒÁÍÉ ÏËÒÕÖÎÏÓÔÅÊ ÒÁÄÉÕÓÏ× 2 É 3 ÒÁ×ÎÏ 8. îÁÊÄÉÔÅ ÎÁÉ122.8.

ÍÅÎØÛÅÅ É ÎÁÉÂÏÌØÛÅÅ ÉÚ ÒÁÓÓÔÏÑÎÉÊ ÍÅÖÄÕ ÔÏÞËÁÍÉ, ÏÄÎÁ ÉÚ ËÏÔÏÒÙÈ ÌÅÖÉÔ ÎÁ ÐÅÒ×ÏÊ

ÏËÒÕÖÎÏÓÔÉ, Á ÄÒÕÇÁÑ | ÎÁ ×ÔÏÒÏÊ.
122.9.

þÅÔÙÒÅ ÄÏÍÁ ÒÁÓÐÏÌÏÖÅÎÙ × ×ÅÒÛÉÎÁÈ ×ÙÐÕËÌÏÇÏ ÞÅÔÙÒ£ÈÕÇÏÌØÎÉËÁ. çÄÅ ÎÕÖÎÏ

×ÙÒÙÔØ ËÏÌÏÄÅÃ, ÞÔÏÂÙ ÓÕÍÍÁ ÒÁÓÓÔÏÑÎÉÊ ÏÔ ÎÅÇÏ ÄÏ ÞÅÔÙÒÅÈ ÄÏÍÏ× ÂÙÌÁ ÎÁÉÍÅÎØÛÅÊ?

ABC, ÐÒÏ×ÅÄÅÎÎÁÑ ÉÚ ×ÅÒÛÉÎÙ A, ÍÅÎØÛÅ
AB É AC, ÎÏ ÂÏÌØÛÅ ÉÈ ÐÏÌÕÒÁÚÎÏÓÔÉ.
ôÏÞËÁ M ÒÁÓÐÏÌÏÖÅÎÁ ×ÎÕÔÒÉ ÔÒÅÕÇÏÌØÎÉËÁ ABC. äÏËÁÖÉÔÅ, ÞÔÏ ÕÇÏÌ AMC
ÂÏÌØÛÅ ÕÇÌÁ ABC.
óÔÏÒÏÎÁ AB ÔÒÅÕÇÏÌØÎÉËÁ ABC ÄÌÉÎÎÅÅ ÓÔÏÒÏÎÙ AC, É ÎÁ ÎÅÊ ÏÔÌÏÖÅÎ ÏÔÒÅÚÏË
AD, ÒÁ×ÎÙÊ ÏÔÒÅÚËÕ AC. äÏËÁÖÉÔÅ, ÞÔÏ ÕÇÏÌ ÐÒÉ ÏÓÎÏ×ÁÎÉÉ ÒÁ×ÎÏÂÅÄÒÅÎÎÏÇÏ ÔÒÅÕÇÏÌØÎÉËÁ
ACD ÎÁÓÔÏÌØËÏ ÖÅ ÂÏÌØÛÅ ÕÇÌÁ B, ÎÁÓËÏÌØËÏ ÍÅÎØÛÅ ÕÇÌÁ C. (úÁÏÄÎÏ ÂÕÄÅÔ ÄÏËÁÚÁÎÏ, ÞÔÏ
ÕÇÏÌ C ÂÏÌØÛÅ ÕÇÌÁ B, Ô.Å. ÞÔÏ ÐÒÏÔÉ× ÂÏÌØÛÅÊ ÓÔÏÒÏÎÙ ÔÒÅÕÇÏÌØÎÉËÁ ÌÅÖÉÔ ÂÏÌØÛÉÊ ÕÇÏÌ.)
122.10.

äÏËÁÖÉÔÅ, ÞÔÏ ÍÅÄÉÁÎÁ ÔÒÅÕÇÏÌØÎÉËÁ

ÐÏÌÕÓÕÍÍÙ ÓÔÏÒÏÎ
122.11.

122.12.

122.13.

äÏËÁÖÉÔÅ, ÞÔÏ ÐÒÏÔÉ× ÂÏÌØÛÅÇÏ ÕÇÌÁ ÔÒÅÕÇÏÌØÎÉËÁ ÌÅÖÉÔ ÂÏÌØÛÁÑ ÓÔÏÒÏÎÁ.

122.14.

÷ ÔÒÅÕÇÏÌØÎÉËÅ

ABC ÓÔÏÒÏÎÁ AB ÄÌÉÎÎÅÅ ÓÔÏÒÏÎÙ BC. íÅÄÉÁÎÁ BM ÄÅÌÉÔ ÕÇÏÌ

B ÎÁ Ä×Á ÕÇÌÁ. ëÁËÏÊ ÉÚ ÎÉÈ ÂÏÌØÛÅ?
÷ ÔÒÅÕÇÏÌØÎÉËÅ ABC Ó ÔÕÐÙÍ ÕÇÌÏÍ C ÔÏÞËÉ M É N ÒÁÓÐÏÌÏÖÅÎÙ ÓÏÏÔ×ÅÔÓÔ×ÅÎÎÏ
ÎÁ ÓÔÏÒÏÎÁÈ AC É BC. äÏËÁÖÉÔÅ, ÞÔÏ ÏÔÒÅÚÏË MN ÄÉÎÎÅÅ ÏÔÒÅÚËÁ AB.
ôÏÞËÁ M ÒÁÓÐÏÌÏÖÅÎÁ ×ÎÕÔÒÉ ÔÒÅÕÇÏÌØÎÉËÁ ABC. äÏËÁÖÉÔÅ, ÞÔÏ BM + CM <
AB + AC.
122.15.

122.16.

122.17.

äÏËÁÖÉÔÅ, ÞÔÏ ÓÕÍÍÁ ÒÁÓÓÔÏÑÎÉÊ ÏÔ ÌÀÂÏÊ ÔÏÞËÉ ×ÎÕÔÒÉ ÔÒÅÕÇÏÌØÎÉËÁ ÄÏ ÔÒÅÈ

ÅÇÏ ×ÅÒÛÉÎ ÂÏÌØÛÅ ÐÏÌÕÐÅÒÉÍÅÔÒÁ, ÎÏ ÍÅÎØÛÅ ÐÅÒÉÍÅÔÒÁ ÔÒÅÕÇÏÌØÎÉËÁ.

ABC É A1 B1C1, ÐÒÉÞÅÍ AB = A1B1, AC = A1 C1, Á ÕÇÏÌ
BAC ÂÏÌØÛÅ ÕÇÌÁ B1A1C1. äÏËÁÖÉÔÅ, ÞÔÏ BC ÂÏÌØÛÅ B1C1. (õ×ÅÌÉÞÉ×ÁÑ ÕÇÏÌ ÍÅÖÄÕ Ä×ÕÍÑ
122.18.

äÁÎÙ ÔÒÅÕÇÏÌØÎÉËÉ

ÛÁÒÎÉÒÎÏ ÓÏÅÄÉΣÎÎÙÍÉ ÏÔÒÅÚËÁÍÉ, ÍÙ Õ×ÅÌÉÞÉ×ÁÅÍ ÒÁÓÓÔÏÑÎÉÅ ÍÅÖÄÕ ÉÈ ËÏÎÃÁÍÉ.)
122.19.

÷ ÓÔÒÁÎÅ ÅÓÔØ ÎÅÓËÏÌØËÏ ÇÏÒÏÄÏ×, ÐÒÉÞÅÍ ×ÓÅ ÒÁÓÓÔÏÑÎÉÑ ÍÅÖÄÕ ÎÉÍÉ ÐÏÐÁÒÎÏ

ÒÁÚÌÉÞÎÙ. ÷ ÏÄÎÏ ÐÒÅËÒÁÓÎÏÅ ÕÔÒÏ ÉÚ ËÁÖÄÏÇÏ ÇÏÒÏÄÁ ×ÙÌÅÔÁÅÔ ÐÏ ÏÄÎÏÍÕ ÓÁÍÏÌÅÔÕ, ËÏÔÏÒÙÊ
ÐÒÉÚÅÍÌÑÅÔÓÑ × ÂÌÉÖÁÊÛÅÍ ÇÏÒÏÄÅ. íÏÖÅÔ ÌÉ × ÏÄÎÏÍ ÇÏÒÏÄÅ ÐÒÉÚÅÍÌÉÔØÓÑ ÂÏÌÅÅ ÐÑÔÉ
ÓÁÍÏÌÅÔÏ×?

189

123. ðÁÒÁÌÌÅÌÏÇÒÁÍÍ

ðÁÒÁÌÌÅÌÏÇÒÁÍÍÏÍ ÎÁÚÙ×ÁÅÔÓÑ ÞÅÔÙÒ£ÈÕÇÏÌØÎÉË, Õ ËÏÔÏÒÏÇÏ ÐÒÏÔÉ×ÏÐÏÌÏÖÎÙÅ ÓÔÏÒÏÎÙ
ÐÏÐÁÒÎÏ ÐÁÒÁÌÌÅÌØÎÙ.
123.1.

äÏËÁÖÉÔÅ ÓÌÅÄÕÀÝÉÅ Ó×ÏÊÓÔ×Á ÐÁÒÁÌÌÅÌÏÇÒÁÍÍÁ: (Á) ÕÇÌÙ ÐÒÉ ÓÏÓÅÄÎÉÈ ×ÅÒÛÉ-

ÎÁÈ ÐÁÒÁÌÌÅÌÏÇÒÁÍÍÁ ÄÁÀÔ × ÓÕÍÍÅ

180, Á ÕÇÌÙ ÐÒÉ ÐÒÏÔÉ×ÏÐÏÌÏÖÎÙÈ ×ÅÒÛÉÎÁÈ ÒÁ×ÎÙ;

(Â) ÄÉÁÇÏÎÁÌØ ÄÅÌÉÔ ÐÁÒÁÌÌÅÌÏÇÒÁÍÍ ÎÁ Ä×Á ÒÁ×ÎÙÈ ÔÒÅÕÇÏÌØÎÉËÁ; (×) ÐÒÏÔÉ×ÏÐÏÌÏÖÎÙÅ
ÓÔÏÒÏÎÙ ÐÁÒÁÌÌÅÌÏÇÒÁÍÍÁ ÒÁ×ÎÙ; (Ç) ÔÏÞËÁ ÐÅÒÅÓÅÞÅÎÉÑ ÄÉÁÇÏÎÁÌÅÊ ÐÁÒÁÌÌÅÌÏÇÒÁÍÍÁ ÄÅÌÉÔ
ËÁÖÄÕÀ ÉÚ ÄÉÁÇÏÎÁÌÅÊ ÐÏÐÏÌÁÍ.
123.2.

äÏËÁÖÉÔÅ ÓÌÅÄÕÀÝÉÅ ÐÒÉÚÎÁËÉ ÐÁÒÁÌÌÅÌÏÇÒÁÍÍÁ: (Á) ÅÓÌÉ × ÞÅÔÙÒ£ÈÕÇÏÌØÎÉËÅ

ÐÒÏÔÉ×ÏÐÏÌÏÖÎÙÅ ÓÔÏÒÏÎÙ ÐÏÐÁÒÎÏ ÒÁ×ÎÙ, ÔÏ ÜÔÏ ÐÁÒÁÌÌÅÌÏÇÒÁÍÍ; (Â) ÅÓÌÉ × ÞÅÔÙÒ£ÈÕÇÏÌØÎÉËÅ ÐÒÏÔÉ×ÏÐÏÌÏÖÎÙÅ ÕÇÌÙ ÐÏÐÁÒÎÏ ÒÁ×ÎÙ, ÔÏ ÜÔÏ ÐÁÒÁÌÌÅÌÏÇÒÁÍÍ; (×) ÅÓÌÉ × ÞÅÔÙÒ£ÈÕÇÏÌØÎÉËÅ Ä×Å ÐÒÏÔÉ×ÏÐÏÌÏÖÎÙÅ ÓÔÏÒÏÎÙ ÒÁ×ÎÙ É ÐÁÒÁÌÌÅÌØÎÙ, ÔÏ ÜÔÏ ÐÁÒÁÌÌÅÌÏÇÒÁÍÍ; (Ç)
ÅÓÌÉ ÄÉÁÇÏÎÁÌÉ ÞÅÔÙÒ£ÈÕÇÏÌØÎÉËÁ ÄÅÌÑÔÓÑ ÔÏÞËÏÊ ÉÈ ÐÅÒÅÓÅÞÅÎÉÑ ÐÏÐÏÌÁÍ, ÔÏ ÜÔÏ ÐÁÒÁÌÌÅÌÏÇÒÁÍÍ.
123.3.

âÉÓÓÅËÔÒÉÓÁ ÕÇÌÁ ÐÁÒÁÌÌÅÌÏÇÒÁÍÍÁ ÄÅÌÉÔ ÓÔÏÒÏÎÕ ÐÁÒÁÌÌÅÌÏÇÒÁÍÍÁ ÎÁ ÏÔÒÅÚËÉ

3 É 5. îÁÊÄÉÔÅ ÓÔÏÒÏÎÙ ÐÁÒÁÌÌÅÌÏÇÒÁÍÍÁ.
÷ ÔÒÅÕÇÏÌØÎÉËÅ ABC ÍÅÄÉÁÎÁ AM ÐÒÏÄÏÌÖÅÎÁ ÚÁ ÔÏÞËÕ M ÄÏ ÔÏÞËÉ D ÎÁ ÒÁÓÓÔÏÑÎÉÅ, ÒÁ×ÎÏÅ AM (ÔÁË ÞÔÏ AM = MD). äÏËÁÖÉÔÅ, ÞÔÏ ABCD | ÐÁÒÁÌÌÅÌÏÇÒÁÍÍ.
ÄÌÉÎÏÊ

123.4.

ôÏÞËÕ ÐÅÒÅÓÅÞÅÎÉÑ ÄÉÁÇÏÎÁÌÅÊ ÐÁÒÁÌÌÅÌÏÇÒÁÍÍÁ ÎÁÚÙ×ÁÀÔ ÅÇÏ
123.5.

ÃÅÎÔÒÏÍ.

äÏËÁÖÉÔÅ, ÞÔÏ ÏÔÒÅÚÏË, ÓÏÅÄÉÎÑÀÝÉÊ ÓÅÒÅÄÉÎÙ ÐÒÏÔÉ×ÏÐÏÌÏÖÎÙÈ ÓÔÏÒÏÎ ÐÁÒÁÌ-

ÌÅÌÏÇÒÁÍÍÁ, ÐÒÏÈÏÄÉÔ ÞÅÒÅÚ ÅÇÏ ÃÅÎÔÒ.
123.6.

ðÏÓÔÒÏÊÔÅ ÐÁÒÁÌÌÅÌÏÇÒÁÍÍ ÐÏ Ä×ÕÍ ÓÔÏÒÏÎÁÍ É ÄÉÁÇÏÎÁÌÉ, ÉÓÈÏÄÑÝÉÍ ÉÚ ÏÄÎÏÊ

×ÅÒÛÉÎÙ.
123.7.

îÁ ÓÔÏÒÏÎÁÈ ÐÁÒÁÌÌÅÌÏÇÒÁÍÍÁ ×ÚÑÔÙ ÞÅÔÙÒÅ ÔÏÞËÉ, ÄÅÌÑÝÉÅ ÜÔÉ ÓÔÏÒÏÎÙ × ÏÄÎÏÍ

É ÔÏÍ ÖÅ ÏÔÎÏÛÅÎÉÉ (ÓÞÉÔÁÑ ÐÏ ÞÁÓÏ×ÏÊ ÓÔÒÅÌËÅ). äÏËÁÖÉÔÅ, ÞÔÏ ÏÎÉ Ñ×ÌÑÀÔÓÑ ×ÅÒÛÉÎÁÍÉ
ÐÁÒÁÌÌÅÌÏÇÒÁÍÍÁ, ÃÅÎÔÒ ËÏÔÏÒÏÇÏ ÓÏ×ÐÁÄÁÅÔ Ó ÃÅÎÔÒÏÍ ÉÓÈÏÄÎÏÇÏ.
123.8.

þÅÒÅÚ ÄÁÎÎÕÀ ÔÏÞËÕ ×ÎÕÔÒÉ ÕÇÌÁ ÐÒÏ×ÅÄÉÔÅ ÐÒÑÍÕÀ, ÏÔÒÅÚÏË ËÏÔÏÒÏÊ, ÚÁËÌÀÞÅÎ-

ÎÙÊ ×ÎÕÔÒÉ ÜÔÏÇÏ ÕÇÌÁ, ÄÅÌÉÔÓÑ ÄÁÎÎÏÊ ÔÏÞËÏÊ ÐÏÐÏÌÁÍ.

îÁ ÓÔÏÒÏÎÁÈ AB É AC ÔÒÅÕÇÏÌØÎÉËÁ ABC ÐÏÓÔÒÏÊÔÅ ÓÏÏÔ×ÅÔÓÔ×ÅÎÎÏ ÔÏÞËÉ M É
N ÔÁË, ÞÔÏ BM = AN É MN k BC.
123.9.

123.10.

òÁÎÅÅ ÂÙÌÏ ÄÏËÁÚÁÎÏ, ÞÔÏ ÂÉÓÓÅËÔÒÉÓÙ ÔÒÅÕÇÏÌØÎÉËÁ ÐÅÒÅÓÅËÁÀÔÓÑ × ÏÄÎÏÊ ÔÏÞ-

ËÅ É ÓÅÒÅÄÉÎÎÙÅ ÐÅÒÐÅÎÄÉËÕÌÑÒÙ Ë ÓÔÏÒÏÎÁÍ ÔÒÅÕÇÏÌØÎÉËÁ ÐÅÒÅÓÅËÁÀÔÓÑ × ÏÄÎÏÊ ÔÏÞËÅ.
äÏËÁÖÉÔÅ, ÞÔÏ É ×ÙÓÏÔÙ ÔÒÅÕÇÏÌØÎÉËÁ ÐÅÒÅÓÅËÁÀÔÓÑ × ÏÄÎÏÊ ÔÏÞËÅ.

190

124. ðÒÑÍÏÕÇÏÌØÎÉË, ÒÏÍÂ, Ë×ÁÄÒÁÔ

ðÁÒÁÌÌÅÌÏÇÒÁÍÍ, × ËÏÔÏÒÏÍ ×ÓÅ ÕÇÌÙ ÐÒÑÍÙÅ, ÎÁÚÙ×ÁÅÔÓÑ
124.1.

äÏËÁÖÉÔÅ, ÞÔÏ ÐÁÒÁÌÌÅÌÏÇÒÁÍÍ Ñ×ÌÑÅÔÓÑ ÐÒÑÍÏÕÇÏÌØÎÉËÏÍ ÔÏÇÄÁ É ÔÏÌØËÏ ÔÏÇÄÁ,

ËÏÇÄÁ ÅÇÏ ÄÉÁÇÏÎÁÌÉ ÒÁ×ÎÙ.
124.2.

äÉÁÇÏÎÁÌÉ ÐÒÑÍÏÕÇÏÌØÎÉËÁ ÒÁ×ÎÙ

ÛÕÀ ÓÔÏÒÏÎÕ ÐÒÑÍÏÕÇÏÌØÎÉËÁ.
124.3.

ÐÒÑÍÏÕÇÏÌØÎÉËÏÍ.

8 É ÐÅÒÅÓÅËÁÀÔÓÑ ÐÏÄ ÕÇÌÏÍ 60. îÁÊÄÉÔÅ ÍÅÎØ-

äÏËÁÖÉÔÅ, ÞÔÏ ÔÏÞËÉ ÐÏÐÁÒÎÏÇÏ ÐÅÒÅÓÅÞÅÎÉÑ ÂÉÓÓÅËÔÒÉÓ ×ÓÅÈ ÞÅÔÙÒ£È ÕÇÌÏ× ÐÁ-

ÒÁÌÌÅÌÏÇÒÁÍÍÁ Ñ×ÌÑÀÔÓÑ ×ÅÒÛÉÎÁÍÉ ÐÒÑÍÏÕÇÏÌØÎÉËÁ.
124.4.

÷ÙÓÏÔÁ ÐÒÑÍÏÕÇÏÌØÎÏÇÏ ÔÒÅÕÇÏÌØÎÉËÁ, ÏÐÕÝÅÎÎÁÑ ÎÁ ÇÉÐÏÔÅÎÕÚÕ, ÒÁ×ÎÁ

ÉÚ ÏÓÔÒÙÈ ÕÇÌÏ× ÒÁ×ÅÎ

15. îÁÊÄÉÔÅ ÇÉÐÏÔÅÎÕÚÕ.

1, ÏÄÉÎ

òÏÍÂÏÍ ÎÁÚÙ×ÁÅÔÓÑ ÐÁÒÁÌÌÅÌÏÇÒÁÍÍ, × ËÏÔÏÒÏÍ ×ÓÅ ÓÔÏÒÏÎÙ ÒÁ×ÎÙ.
124.5.

äÏËÁÖÉÔÅ, ÞÔÏ ÄÉÁÇÏÎÁÌÉ ÒÏÍÂÁ (Á) ×ÚÁÉÍÎÏ ÐÅÒÐÅÎÄÉËÕÌÑÒÎÙ; (Â) Ñ×ÌÑÀÔÓÑ ÂÉÓ-

ÓÅËÔÒÉÓÁÍÉ ÕÇÌÏ× ÒÏÍÂÁ. ÷ÅÒÎÙ ÌÉ ÏÂÒÁÔÎÙÅ ÕÔ×ÅÒÖÄÅÎÉÑ?
124.6.

ÒÁ×ÅÎ

îÁÊÄÉÔÅ ÒÁÓÓÔÏÑÎÉÅ ÏÔ ÃÅÎÔÒÁ ÒÏÍÂÁ ÄÏ ÅÇÏ ÓÔÏÒÏÎÙ, ÅÓÌÉ ÏÓÔÒÙÊ ÕÇÏÌ ÒÏÍÂÁ

30, Á ÓÔÏÒÏÎÁ ÒÁ×ÎÁ 4.

124.7.

ôÒÉ ÒÁ×ÎÙÈ ÏËÒÕÖÎÏÓÔÉ ÐÒÏÈÏÄÑÔ ÞÅÒÅÚ ÏÄÎÕ ÔÏÞËÕ É ÐÏÐÁÒÎÏ ÐÅÒÅÓÅËÁÀÔÓÑ × ÔÒ£È

ÄÒÕÇÉÈ ÔÏÞËÁÈ

A, B É C. äÏËÁÖÉÔÅ, ÞÔÏ ÔÒÅÕÇÏÌØÎÉË ABC ÒÁ×ÅÎ ÔÒÅÕÇÏÌØÎÉËÕ Ó ×ÅÒÛÉÎÁÍÉ

× ÃÅÎÔÒÁÈ ÏËÒÕÖÎÏÓÔÅÊ.

ðÒÑÍÏÕÇÏÌØÎÉË, Ñ×ÌÑÀÝÉÊÓÑ ÏÄÎÏ×ÒÅÍÅÎÎÏ ÒÏÍÂÏÍ, ÎÁÚÙ×ÁÅÔÓÑ

Ë×ÁÄÒÁÔÏÍ ; Õ ÎÅÇÏ ×ÓÅ

ÓÔÏÒÏÎÙ ÒÁ×ÎÙ, Á ×ÓÅ ÕÇÌÙ ÐÒÑÍÙÅ.
124.8.

îÁ ËÁÖÄÏÊ ÓÔÏÒÏÎÅ Ë×ÁÄÒÁÔÁ ×ÚÑÌÉ ÐÏ ÏÄÎÏÊ ÔÏÞËÅ. ðÒÉ ÜÔÏÍ ÏËÁÚÁÌÏÓØ, ÞÔÏ ÜÔÉ

ÔÏÞËÉ Ñ×ÌÑÀÔÓÑ ×ÅÒÛÉÎÁÍÉ ÐÒÑÍÏÕÇÏÌØÎÉËÁ, ÓÔÏÒÏÎÙ ËÏÔÏÒÏÇÏ ÐÁÒÁÌÌÅÌØÎÙ ÄÉÁÇÏÎÁÌÑÍ
Ë×ÁÄÒÁÔÁ. îÁÊÄÉÔÅ ÐÅÒÉÍÅÔÒ ÐÒÑÍÏÕÇÏÌØÎÉËÁ, ÅÓÌÉ ÄÉÁÇÏÎÁÌØ Ë×ÁÄÒÁÔÁ ÒÁ×ÎÁ
124.9.

6.

þÅÒÅÚ ÃÅÎÔÒ Ë×ÁÄÒÁÔÁ ÐÒÏ×ÅÄÅÎÙ Ä×Å ×ÚÁÉÍÎÏ ÐÅÒÐÅÎÄÉËÕÌÑÒÎÙÅ ÐÒÑÍÙÅ. äÏ-

ËÁÖÉÔÅ, ÞÔÏ ÔÏÞËÉ ÐÅÒÅÓÅÞÅÎÉÑ ÜÔÉÈ ÐÒÑÍÙÈ ÓÏ ÓÔÏÒÏÎÁÍÉ Ë×ÁÄÒÁÔÁ Ñ×ÌÑÀÔÓÑ ×ÅÒÛÉÎÁÍÉ
ÄÒÕÇÏÇÏ Ë×ÁÄÒÁÔÁ. á ÅÓÌÉ ×ÍÅÓÔÏ ÃÅÎÔÒÁ ×ÚÑÔØ ÐÒÏÉÚ×ÏÌØÎÕÀ ÔÏÞËÕ ×ÎÕÔÒÉ Ë×ÁÄÒÁÔÁ?
124.10.

äÏËÁÖÉÔÅ, ÞÔÏ ÔÏÞËÉ ÐÏÐÁÒÎÏÇÏ ÐÅÒÅÓÅÞÅÎÉÑ ÂÉÓÓÅËÔÒÉÓ ×ÓÅÈ ÞÅÔÙÒ£È ÕÇÌÏ×

ÐÒÑÍÏÕÇÏÌØÎÉËÁ Ñ×ÌÑÀÔÓÑ ×ÅÒÛÉÎÁÍÉ Ë×ÁÄÒÁÔÁ.

ôÏÞËÁ K | ÓÅÒÅÄÉÎÁ ÓÔÏÒÏÎÙ AB Ë×ÁÄÒÁÔÁ ABCD, ÔÏÞËÁ L ÒÁÓÐÏÌÏÖÅÎÁ ÎÁ
AC, ÐÒÉÞÅÍ AL LC . îÁÊÄÉÔÅ \KLD.

124.11.

ÄÉÁÇÏÎÁÌÉ

124.12.

:

???

îÁ ÓÔÏÒÏÎÁÈ ÐÁÒÁÌÌÅÌÏÇÒÁÍÍÁ ×ÎÅ ÅÇÏ ÐÏÓÔÒÏÅÎÙ Ë×ÁÄÒÁÔÙ. äÏËÁÖÉÔÅ, ÞÔÏ ÉÈ

ÃÅÎÔÒÙ ÓÁÍÉ ÏÂÒÁÚÕÀÔ Ë×ÁÄÒÁÔ.

M ÒÁÓÐÏÌÏÖÅÎÁ ×ÎÕÔÒÉ Ë×ÁÄÒÁÔÁ ABCD, ÐÒÉÞÅÍ \MAB \MBA 15.
äÏËÁÖÉÔÅ, ÞÔÏ ÔÒÅÕÇÏÌØÎÉË DMC ÐÒÁ×ÉÌØÎÙÊ.
124.13.

ôÏÞËÁ

124.14.

îÁ ËÁÖÄÏÊ ÓÔÏÒÏÎÅ Ë×ÁÄÒÁÔÁ ÏÔÍÅÔÉÌÉ ÐÏ ÔÏÞËÅ. úÁÔÅÍ ×Ó£, ËÒÏÍÅ ÜÔÉÈ ÔÏÞÅË,

=

ÓÔ£ÒÌÉ. ÷ÏÓÓÔÁÎÏ×ÉÔÅ Ë×ÁÄÒÁÔ Ó ÐÏÍÏÝØÀ ÃÉÒËÕÌÑ É ÌÉÎÅÊËÉ.

191

=

125. óÒÅÄÎÑÑ ÌÉÎÉÑ ÔÒÅÕÇÏÌØÎÉËÁ

óÒÅÄÎÅÊ ÌÉÎÉÅÊ ÔÒÅÕÇÏÌØÎÉËÁ ÎÁÚÙ×ÁÅÔÓÑ ÏÔÒÅÚÏË, ÓÏÅÄÉÎÑÀÝÉÊ ÓÅÒÅÄÉÎÙ Ä×ÕÈ ÓÔÏÒÏÎ
ÔÒÅÕÇÏÌØÎÉËÁ.
125.1.

äÏËÁÖÉÔÅ, ÞÔÏ ÏÎÁ ÐÁÒÁÌÌÅÌØÎÁ ÔÒÅÔØÅÊ ÓÔÏÒÏÎÅ É ÒÁ×ÎÁ Å£ ÐÏÌÏ×ÉÎÅ.

125.2.

äÏËÁÖÉÔÅ, ÞÔÏ ÔÒÉ ÓÒÅÄÎÉÅ ÌÉÎÉÉ ÒÁÚÂÉ×ÁÀÔ ÔÒÅÕÇÏÌØÎÉË ÎÁ ÞÅÔÙÒÅ ÒÁ×ÎÙÈ ÔÒÅ-

ÕÇÏÌØÎÉËÁ.
125.3.

ó ÐÏÍÏÝØÀ ÃÉÒËÕÌÑ É ÌÉÎÅÊËÉ ÐÏÓÔÒÏÊÔÅ ÔÒÅÕÇÏÌØÎÉË ÐÏ ÓÅÒÅÄÉÎÁÍ ÔÒÅÈ ÅÇÏ

ÓÔÏÒÏÎ.
125.4.

(ôÅÏÒÅÍÁ ÷ÁÒÉÎØÏÎÁ) äÏËÁÖÉÔÅ, ÞÔÏ ÓÅÒÅÄÉÎÙ ÓÔÏÒÏÎ ÌÀÂÏÇÏ ÞÅÔÙÒÅÈÕÇÏÌØÎÉËÁ

Ñ×ÌÑÀÔÓÑ ×ÅÒÛÉÎÁÍÉ ÐÁÒÁÌÌÅÌÏÇÒÁÍÍÁ.
125.5.

äÏËÁÖÉÔÅ, ÞÔÏ ÓÅÒÅÄÉÎÙ Ä×ÕÈ ÐÒÏÔÉ×ÏÐÏÌÏÖÎÙÈ ÓÔÏÒÏÎ ÌÀÂÏÇÏ ÞÅÔÙÒÅÈÕÇÏÌØÎÉËÁ

É ÓÅÒÅÄÉÎÙ ÅÇÏ ÄÉÁÇÏÎÁÌÅÊ Ñ×ÌÑÀÔÓÑ ×ÅÒÛÉÎÁÍÉ ÐÁÒÁÌÌÅÌÏÇÒÁÍÍÁ.
125.6.

äÏËÁÖÉÔÅ, ÞÔÏ (Á) ÅÓÌÉ ÏÔÒÅÚËÉ, ÓÏÅÄÉÎÑÀÝÉÅ ÓÅÒÅÄÉÎÙ ÐÒÏÔÉ×ÏÐÏÌÏÖÎÙÈ ÓÔÏÒÏÎ

ÞÅÔÙÒÅÈÕÇÏÌØÎÉËÁ ÒÁ×ÎÙ, ÔÏ ÄÉÁÇÏÎÁÌÉ ÞÅÔÙÒÅÈÕÇÏÌØÎÉËÁ ÐÅÒÐÅÎÄÉËÕÌÑÒÎÙ; (Â) ÅÓÌÉ ÜÔÉ
ÏÔÒÅÚËÉ ÐÅÒÐÅÎÄÉËÕÌÑÒÎÙ, ÔÏ ÄÉÁÇÏÎÁÌÉ ÞÅÔÙÒÅÈÕÇÏÌØÎÉËÁ ÒÁ×ÎÙ.

÷ ×ÙÐÕËÌÏÍ ÞÅÔÙÒÅÈÕÇÏÌØÎÉËÅ ABCD ÏÔÒÅÚÏË, ÓÏÅÄÉÎÑÀÝÉÊ ÓÅÒÅÄÉÎÙ ÓÔÏÒÏÎ
AB É CD ÒÁ×ÅÎ 1. ðÒÑÍÙÅ BC É AD ÐÅÒÐÅÎÄÉËÕÌÑÒÎÙ. îÁÊÄÉÔÅ ÏÔÒÅÚÏË, ÓÏÅÄÉÎÑÀÝÉÊ
ÓÅÒÅÄÉÎÙ ÄÉÁÇÏÎÁÌÅÊ AC É BD.
125.7.

125.8.

îÁÊÄÉÔÅ ÇÅÏÍÅÔÒÉÞÅÓËÏÅ ÍÅÓÔÏ ÓÅÒÅÄÉÎ ×ÓÅÈ ÏÔÒÅÚËÏ×, ÓÏÅÄÉÎÑÀÝÉÈ ÄÁÎÎÕÀ ÔÏÞËÕ

ÓÏ ×ÓÅÍÉ ÔÏÞËÁÍÉ ÄÁÎÎÏÊ ÐÒÑÍÏÊ.

A ÔÒÅÕÇÏÌØÎÉËÁ ABC ÏÐÕÝÅÎÙ ÐÅÒÐÅÎÄÉËÕÌÑÒÙ AM É AP ÎÁ ÂÉÓÓÅËÔÒÉÓÙ ×ÎÅÛÎÉÈ ÕÇÌÏ× B É C. îÁÊÄÉÔÅ ÏÔÒÅÚÏË PM, ÅÓÌÉ ÐÅÒÉÍÅÔÒ ÔÒÅÕÇÏÌØÎÉËÁ ABC
125.9.

éÚ ×ÅÒÛÉÎÙ

ÒÁ×ÅÎ 10.

125.10.

äÏËÁÖÉÔÅ, ÞÔÏ ÍÅÄÉÁÎÙ ÔÒÅÕÇÏÌØÎÉËÁ ÐÅÒÅÓÅËÁÀÔÓÑ × ÏÄÎÏÊ ÔÏÞËÅ É ÄÅÌÑÔÓÑ ÅÀ

× ÏÔÎÏÛÅÎÉÉ 2:1, ÓÞÉÔÁÑ ÏÔ ×ÅÒÛÉÎ ÔÒÅÕÇÏÌØÎÉËÁ.
125.11.

ä×Å ÍÅÄÉÁÎÙ ÔÒÅÕÇÏÌØÎÉËÁ ÒÁ×ÎÙ. äÏËÁÖÉÔÅ, ÞÔÏ ÔÒÅÕÇÏÌØÎÉË ÒÁ×ÎÏÂÅÄÒÅÎÎÙÊ.

125.12.

ðÏÓÔÒÏÊÔÅ ÐÁÒÁÌÌÅÌÏÇÒÁÍÍ ÐÏ ×ÅÒÛÉÎÅ É ÓÅÒÅÄÉÎÁÍ ÓÔÏÒÏÎ, ÎÅ ÓÏÄÅÒÖÁÝÉÍ ÜÔÕ

×ÅÒÛÉÎÕ.
125.13.

äÏËÁÖÉÔÅ, ÞÔÏ ÓÕÍÍÁ ÔÒÅÈ ÍÅÄÉÁÎ ÔÒÅÕÇÏÌØÎÉËÁ ÍÅÎØÛÅ ÐÅÒÉÍÅÔÒÁ, ÎÏ ÂÏÌØÛÅ

ÔÒÅÈ ÞÅÔ×ÅÒÔÅÊ ÐÅÒÉÍÅÔÒÁ ÔÒÅÕÇÏÌØÎÉËÁ.

M É N | ÓÅÒÅÄÉÎÙ ÓÏÓÅÄÎÉÈ ÓÔÏÒÏÎ BC É CD ÐÁÒÁÌÌÅÌÏÇÒÁÍÍÁ ABCD.
äÏËÁÖÉÔÅ, ÞÔÏ ÐÒÑÍÙÅ AM É AN ÄÅÌÑÔ ÄÉÁÇÏÎÁÌØ BD ÎÁ ÔÒÉ ÒÁ×ÎÙÅ ÞÁÓÔÉ.
125.14.

ôÏÞËÉ

125.15.

ðÏÓÔÒÏÊÔÅ ÔÒÅÕÇÏÌØÎÉË ÐÏ ×ÙÓÏÔÁÍ, ÐÒÏ×ÅÄÅÎÎÙÍ ÉÚ Ä×ÕÈ ×ÅÒÛÉÎ, É ÍÅÄÉÁÎÅ,

ÐÒÏ×ÅÄÅÎÎÏÊ ÉÚ ÔÒÅÔØÅÊ.

AB É BC ÒÁ×ÎÏÂÅÄÒÅÎÎÏÇÏ ÔÒÅÕÇÏÌØÎÉËÁ ABC ×ÚÑÔÙ ÓÏÏÔ×ÅÔÓÔ×ÅÎÎÏ ÔÏÞËÉ M É N, ÐÒÉÞ£Í BM = CN. äÏËÁÖÉÔÅ, ÞÔÏ ÓÅÒÅÄÉÎÁ ÏÔÒÅÚËÁ MN ÌÅÖÉÔ
ÎÁ ÓÒÅÄÎÅÊ ÌÉÎÉÉ ÔÒÅÕÇÏÌØÎÉËÁ ABC, ÐÁÒÁÌÌÅÌØÎÏÊ ÅÇÏ ÏÓÎÏ×ÁÎÉÀ.
125.16.

îÁ ÂÏËÏ×ÙÈ ÓÔÏÒÏÎÁÈ

125.17.

ðÏÓÔÒÏÊÔÅ ÔÒÅÕÇÏÌØÎÉË ÐÏ ÓÔÏÒÏÎÅ É ÍÅÄÉÁÎÁÍ, ÐÒÏ×ÅÄÅÎÎÙÍ Ë Ä×ÕÍ ÄÒÕÇÉÍ

ÓÔÏÒÏÎÁÍ.
125.18.

ðÏÓÔÒÏÊÔÅ ÔÒÅÕÇÏÌØÎÉË ÐÏ ÔÒ£Í ÍÅÄÉÁÎÁÍ.

192

125.19.

äÏËÁÖÉÔÅ ÐÒÉÚÎÁË ÒÁ×ÅÎÓÔ×Á ÔÒÅÕÇÏÌØÎÉËÏ× ÐÏ ÔÒ£Í ÍÅÄÉÁÎÁÍ.
* * *

125.20. ðÏÓÔÒÏÊÔÅ ÐÑÔÉÕÇÏÌØÎÉË ÐÏ ÓÅÒÅÄÉÎÁÍ ÅÇÏ ÓÔÏÒÏÎ.
125.21. ôÏÞËÉ

A1 B1 C1
BC AC AB
,

É

ÓÔÏÒÏÎ ÓÏÏÔ×ÅÔÓÔ×ÅÎÎÏ

ÏÂÒÁÚÙ ÐÒÏÉÚ×ÏÌØÎÏÊ ÔÏÞËÉ

,

É

ÔÒÅÕÇÏÌØÎÉËÁ

ÐÅÒÅÓÅËÁÀÔÓÑ × ÏÄÎÏÊ ÔÏÞËÅ.

ABCD
AF CE BF DE

125.22. ÷ ÞÅÔÙÒÅÈÕÇÏÌØÎÉËÅ
ÞÔÏ ÓÅÒÅÄÉÎÙ ÏÔÒÅÚËÏ×

,

,

ÔÏÞËÁ

É

E

ABC

M

ÐÒÉ ÓÉÍÍÅÔÒÉÉ ÏÔÎÏÓÉÔÅÌØÎÏ ÓÅÒÅÄÉÎ

. äÏËÁÖÉÔÅ, ÞÔÏ ÐÒÑÍÙÅ

AB

| ÓÅÒÅÄÉÎÁ

, ÔÏÞËÁ

F

AA1 BB1 CC1

| ÓÅÒÅÄÉÎÁ

,

É

CD

. äÏËÁÖÉÔÅ,

Ñ×ÌÑÀÔÓÑ ×ÅÒÛÉÎÁÍÉ ÐÁÒÁÌÌÅÌÏÇÒÁÍÍÁ.

125.23. ÷ ×ÙÐÕËÌÏÍ ÞÅÔÙÒÅÈÕÇÏÌØÎÉËÅ ÐÒÑÍÁÑ, ÐÒÏÈÏÄÑÝÁÑ ÞÅÒÅÚ ÓÅÒÅÄÉÎÙ Ä×ÕÈ ÐÒÏÔÉ×ÏÐÏÌÏÖÎÙÈ ÓÔÏÒÏÎ, ÏÂÒÁÚÕÅÔ ÒÁ×ÎÙÅ ÕÇÌÙ Ó ÄÉÁÇÏÎÁÌÑÍÉ ÞÅÔÙÒÅÈÕÇÏÌØÎÉËÁ. äÏËÁÖÉÔÅ, ÞÔÏ ÜÔÉ ÄÉÁÇÏÎÁÌÉ
ÒÁ×ÎÙ.
125.24. äÏËÁÖÉÔÅ, ÞÔÏ ÒÁÓÓÔÏÑÎÉÅ ÏÔ ×ÅÒÛÉÎÙ ÔÒÅÕÇÏÌØÎÉËÁ ÄÏ ÔÏÞËÉ ÐÅÒÅÓÅÞÅÎÉÑ ×ÙÓÏÔ ×Ä×ÏÅ
ÂÏÌØÛÅ, ÞÅÍ ÒÁÓÓÔÏÑÎÉÅ ÏÔ ÃÅÎÔÒÁ ÏÐÉÓÁÎÎÏÇÏ ËÒÕÇÁ ÄÏ ÐÒÏÔÉ×ÏÐÏÌÏÖÎÏÊ ÓÔÏÒÏÎÙ.
125.25. ðÏÓÔÒÏÊÔÅ ÔÒÅÕÇÏÌØÎÉË, ÚÎÁÑ ÔÒÉ ÔÏÞËÉ, ÓÉÍÍÅÔÒÉÞÎÙÅ ÃÅÎÔÒÕ ÅÇÏ ÏÐÉÓÁÎÎÏÊ ÏËÒÕÖÎÏÓÔÉ
ÏÔÎÏÓÉÔÅÌØÎÏ ÓÔÏÒÏÎ.
125.26. äÉÁÇÏÎÁÌÉ ×ÙÐÕËÌÏÇÏ ÞÅÔÙÒÅÈÕÇÏÌØÎÉËÁ
ÄÉÎÙ ÓÔÏÒÏÎ

CB

AB AD
É

BC CA
É

ABC
P
PM PK
×ÚÑÔÁ ÔÏÞËÁ

ÏÐÕÝÅÎÙ ÐÅÒÐÅÎÄÉËÕÌÑÒÙ

äÏËÁÖÉÔÅ, ÞÔÏ

×ÚÁÉÍÎÏ ÐÅÒÐÅÎÄÉËÕÌÑÒÎÙ. þÅÒÅÚ ÓÅÒÅ-

ÐÒÏ×ÅÄÅÎÙ ÐÒÑÍÙÅ, ÐÅÒÐÅÎÄÉËÕÌÑÒÎÙÅ ÐÒÏÔÉ×ÏÐÏÌÏÖÎÙÍ ÓÔÏÒÏÎÁÍ

ÓÏÏÔ×ÅÔÓÔ×ÅÎÎÏ. äÏËÁÖÉÔÅ, ÞÔÏ ÜÔÉ ÐÒÑÍÙÅ É ÐÒÑÍÁÑ
125.27. ÷ÎÕÔÒÉ ÔÒÅÕÇÏÌØÎÉËÁ

DK = DM
AB BC
APB BQC
PQK K

É

125.28. îÁ ÓÔÏÒÏÎÁÈ

îÁÊÄÉÔÅ ÕÇÌÙ ÔÒÅÕÇÏÌØÎÉËÁ

É

É

AC

, ÐÒÉÞ£Í

ÉÍÅÀÔ ÏÂÝÕÀ ÔÏÞËÕ.

\PAC = \PBC

ÓÏÏÔ×ÅÔÓÔ×ÅÎÎÏ. ðÕÓÔØ

.

ÕÇÏÌØÎÙÅ ÔÒÅÕÇÏÌØÎÉËÉ

AE

ABCD

ÔÒÅÕÇÏÌØÎÉËÁ

ABC

| ÓÅÒÅÄÉÎÁ ÓÔÏÒÏÎÙ

ABC CDE
M N K
MNK

125.29. ä×Á ÒÁ×ÎÏÓÔÏÒÏÎÎÉÈ ÔÒÅÕÇÏÌØÎÉËÁ
É ÉÍÅÀÔ ÅÄÉÎÓÔ×ÅÎÎÕÀ ÏÂÝÕÀ ÔÏÞËÕ

C

É

. ðÕÓÔØ

ÓÏÏÔ×ÅÔÓÔ×ÅÎÎÏ. äÏËÁÖÉÔÅ, ÞÔÏ ÔÒÅÕÇÏÌØÎÉË

ÎÁ ÓÔÏÒÏÎÙ

| ÓÅÒÅÄÉÎÁ ÓÔÏÒÏÎÙ

AB

.

AC

ÐÒÉ ÉÈ ÏÂÝÅÊ ×ÅÒÛÉÎÅ

B

.

.

ÒÁÓÐÏÌÏÖÅÎÙ ÐÏ ÏÄÎÕ ÓÔÏÒÏÎÕ ÏÔ ÐÒÑÍÏÊ

,

É

| ÓÅÒÅÄÉÎÙ ÏÔÒÅÚËÏ×

| ÒÁ×ÎÏÓÔÏÒÏÎÎÉÊ.

193

P

É

ËÁË ÎÁ ÇÉÐÏÔÅÎÕÚÁÈ ÐÏÓÔÒÏÅÎÙ ×ÎÅ ÅÇÏ ÐÒÑÍÏ-

Ó ÏÄÉÎÁËÏ×ÙÍÉ ÕÇÌÁÍÉ ×ÅÌÉÞÉÎÙ

, ÇÄÅ

D

. éÚ ÔÏÞËÉ

CD

BD AC CE
,

É

126. ôÒÁÐÅÃÉÑ

ôÒÁÐÅÃÉÅÊ ÎÁÚÙ×ÁÅÔÓÑ ÞÅÔÙÒ£ÈÕÇÏÌØÎÉË, Õ ËÏÔÏÒÏÇÏ Ä×Å ÐÒÏÔÉ×ÏÐÏÌÏÖÎÙÅ ÓÔÏÒÏÎÙ
(ÏÓÎÏ×ÁÎÉÑ ) ÐÁÒÁÌÌÅÌØÎÙ, Á Ä×Å ÄÒÕÇÉÅ (ÂÏËÏ×ÙÅ ÓÔÏÒÏÎÙ ) ÎÅÔ.
óÒÅÄÎÅÊ ÌÉÎÉÅÊ ÔÒÁÐÅÃÉÉ ÎÁÚÙ×ÁÅÔÓÑ ÏÔÒÅÚÏË, ÓÏÅÄÉÎÑÀÝÉÊ ÓÅÒÅÄÉÎÙ ÂÏËÏ×ÙÈ ÓÔÏÒÏÎ.
126.1.

óÒÅÄÎÑÑ ÌÉÎÉÑ ÔÒÁÐÅÃÉÉ ÐÁÒÁÌÌÅÌØÎÁ ÏÓÎÏ×ÁÎÉÑÍ É ÒÁ×ÎÁ ÉÈ ÐÏÌÕÓÕÍÍÅ.

126.2.

ôÒÁÐÅÃÉÑ ÎÁÚÙ×ÁÅÔÓÑ ÒÁ×ÎÏÂÅÄÒÅÎÎÏÊ, ÅÓÌÉ ÅÅ ÂÏËÏ×ÙÅ ÓÔÏÒÏÎÙ ÒÁ×ÎÙ ÍÅÖÄÕ

ÓÏÂÏÊ. äÏËÁÖÉÔÅ ÓÌÅÄÕÀÝÉÅ ÕÔ×ÅÒÖÄÅÎÉÑ:
(Á) ÕÇÌÙ ÐÒÉ ÏÓÎÏ×ÁÎÉÉ ÒÁ×ÎÏÂÅÄÒÅÎÎÏÊ ÔÒÁÐÅÃÉÉ ÒÁ×ÎÙ;
(Â) ÅÓÌÉ ÕÇÌÙ ÐÒÉ ÏÄÎÏÍ ÉÚ ÏÓÎÏ×ÁÎÉÊ ÔÒÁÐÅÃÉÉ ÒÁ×ÎÙ, ÔÏ ÏÎÁ ÒÁ×ÎÏÂÅÄÒÅÎÎÁÑ;
(×) ÄÉÁÇÏÎÁÌÉ ÒÁ×ÎÏÂÅÄÒÅÎÎÏÊ ÔÒÁÐÅÃÉÉ ÒÁ×ÎÙ;
(Ç) ÅÓÌÉ ÄÉÁÇÏÎÁÌÉ ÔÒÁÐÅÃÉÉ ÒÁ×ÎÙ, ÔÏ ÏÎÁ ÒÁ×ÎÏÂÅÄÒÅÎÎÁÑ.

P | ÏÓÎÏ×ÁÎÉÅ ÐÅÒÐÅÎÄÉËÕÌÑÒÁ, ÏÐÕÝÅÎÎÏÇÏ ÉÚ ×ÅÒÛÉÎÙ C ÍÅÎØÛÅÇÏ ÏÓÎÏ×ÁÎÉÑ BC ÒÁ×ÎÏÂÅÄÒÅÎÎÏÊ ÔÒÁÐÅÃÉÉ ABCD ÎÁ ÂÏÌØÛÅÅ ÏÓÎÏ×ÁÎÉÅ AD. îÁÊÄÉÔÅ DP É AP,
ÅÓÌÉ ÏÓÎÏ×ÁÎÉÑ ÔÒÁÐÅÃÉÉ ÒÁ×ÎÙ a É b (a > b).
÷ ÒÁ×ÎÏÂÅÄÒÅÎÎÏÊ ÔÒÁÐÅÃÉÉ ÏÓÔÒÙÊ ÕÇÏÌ ÒÁ×ÅÎ 60. äÏËÁÖÉÔÅ, ÞÔÏ ÍÅÎØÛÅÅ ÏÓÎÏ126.3.

ðÕÓÔØ

126.4.

×ÁÎÉÅ ÒÁ×ÎÏ ÒÁÚÎÏÓÔÉ ÂÏÌØÛÅÇÏ ÏÓÎÏ×ÁÎÉÑ É ÂÏËÏ×ÏÊ ÓÔÏÒÏÎÙ.
126.5.

äÉÁÇÏÎÁÌØ ÒÁ×ÎÏÂÅÄÒÅÎÎÏÊ ÔÒÁÐÅÃÉÉ ÒÁ×ÎÁ

ÔÒÁÐÅÃÉÉ. îÁÊÄÉÔÅ ÓÒÅÄÎÀÀ ÌÉÎÉÀ ÔÒÁÐÅÃÉÉ.

10 É ÏÂÒÁÚÕÅÔ ÕÇÏÌ 60 Ó ÏÓÎÏ×ÁÎÉÅÍ

ôÏÞËÉ M É N | ÓÅÒÅÄÉÎÙ ÂÏËÏ×ÙÈ ÓÔÏÒÏÎ AB É CD ÔÒÁÐÅÃÉÉ ABCD. íÏÇÕÔ ÌÉ
BN É DM ÂÙÔØ ÐÁÒÁÌÌÅÌØÎÙÍÉ?
ïÔÒÅÚËÉ AB É BC | ÓÏÏÔ×ÅÔÓÔ×ÅÎÎÏ ÂÏËÏ×ÁÑ ÓÔÏÒÏÎÁ É ÍÅÎØÛÅÅ ÏÓÎÏ×ÁÎÉÅ ÔÒÁÐÅÃÉÉ ABCD. éÚ×ÅÓÔÎÏ, ÞÔÏ AB = 2;6 É BC = 2;5. ëÁËÏÊ ÉÚ ÏÔÒÅÚËÏ× ÐÅÒÅÓÅËÁÅÔ ÂÉÓÓÅËÔÒÉÓÁ
ÕÇÌÁ A: ÏÓÎÏ×ÁÎÉÅ BC ÉÌÉ ÂÏËÏ×ÕÀ ÓÔÏÒÏÎÕ CD ?
126.6.

ÐÒÑÍÙÅ

126.7.

126.8.

äÏËÁÖÉÔÅ, ÞÔÏ ÂÉÓÓÅËÔÒÉÓÙ ÕÇÌÏ× ÐÒÉ ÂÏËÏ×ÏÊ ÓÔÏÒÏÎÅ ÔÒÁÐÅÃÉÉ ÐÅÒÅÓÅËÁÀÔÓÑ ÎÁ

Å£ ÓÒÅÄÎÅÊ ÌÉÎÉÉ.

äÁÎÁ ÔÒÁÐÅÃÉÑ ABCD Ó ÏÓÎÏ×ÁÎÉÅÍ AD. âÉÓÓÅËÔÒÉÓÙ ×ÎÅÛÎÉÈ ÕÇÌÏ× ÐÒÉ ×ÅÒA É B ÐÅÒÅÓÅËÁÀÔÓÑ × ÔÏÞËÅ P, Á ÐÒÉ ×ÅÒÛÉÎÁÈ C É D | × ÔÏÞËÅ Q. äÏËÁÖÉÔÅ, ÞÔÏ
ÏÔÒÅÚÏË PQ ÒÁ×ÅÎ ÐÏÌÕÐÅÒÉÍÅÔÒÕ ÔÒÁÐÅÃÉÉ.
ïÓÎÏ×ÁÎÉÑ ÔÒÁÐÅÃÉÉ ÒÁ×ÎÙ a É b, ÐÒÉÞ£Í a > b. îÁÊÄÉÔÅ ÄÌÉÎÕ ÏÔÒÅÚËÁ,
126.9.

ÛÉÎÁÈ

126.10.

ÓÏÅÄÉÎÑÀÝÅÇÏ ÓÅÒÅÄÉÎÙ ÄÉÁÇÏÎÁÌÅÊ ÔÒÁÐÅÃÉÉ.
126.11.

îÁÊÄÉÔÅ ÏÔÎÏÛÅÎÉÅ ÏÓÎÏ×ÁÎÉÊ ÔÒÁÐÅÃÉÉ, ÅÓÌÉ ÅÅ ÓÒÅÄÎÑÑ ÌÉÎÉÑ ÄÅÌÉÔÓÑ ÄÉÁÇÏ-

ÎÁÌÑÍÉ ÎÁ ÔÒÉ ÒÁ×ÎÙÅ ÞÁÓÔÉ.
126.12.

óÕÍÍÁ ÕÇÌÏ× ÐÒÉ ÏÄÎÏÍ ÉÚ ÏÓÎÏ×ÁÎÉÊ ÔÒÁÐÅÃÉÉ ÒÁ×ÎÁ

90. äÏËÁÖÉÔÅ, ÞÔÏ ÏÔÒÅÚÏË,

ÓÏÅÄÉÎÑÀÝÉÊ ÓÅÒÅÄÉÎÙ ÏÓÎÏ×ÁÎÉÊ ÔÒÁÐÅÃÉÉ, ÒÁ×ÅÎ ÉÈ ÐÏÌÕÒÁÚÎÏÓÔÉ.
126.13.

âÏËÏ×ÁÑ ÓÔÏÒÏÎÁ ÔÒÁÐÅÃÉÉ ÒÁ×ÎÁ ÏÄÎÏÍÕ ÏÓÎÏ×ÁÎÉÀ É ×Ä×ÏÅ ÍÅÎØÛÅ ÄÒÕÇÏÇÏ.

äÏËÁÖÉÔÅ, ÞÔÏ ×ÔÏÒÁÑ ÂÏËÏ×ÁÑ ÓÔÏÒÏÎÁ ÐÅÒÐÅÎÄÉËÕÌÑÒÎÁ ÏÄÎÏÊ ÉÚ ÄÉÁÇÏÎÁÌÅÊ ÔÒÁÐÅÃÉÉ.
126.14.

ðÏÓÔÒÏÊÔÅ ÔÒÁÐÅÃÉÀ ÐÏ ÏÓÎÏ×ÁÎÉÑÍ É ÂÏËÏ×ÙÍ ÓÔÏÒÏÎÁÍ.

126.15.

ðÏÓÔÒÏÊÔÅ ÔÒÁÐÅÃÉÀ ÐÏ ÏÓÎÏ×ÁÎÉÑÍ É ÄÉÁÇÏÎÁÌÑÍ.

126.16.

äÉÁÇÏÎÁÌÉ ÔÒÁÐÅÃÉÉ ×ÚÁÉÍÎÏ ÐÅÒÐÅÎÄÉËÕÌÑÒÎÙ. ïÄÎÁ ÉÚ ÎÉÈ ÒÁ×ÎÁ

ÏÂÒÁÚÕÅÔ Ó ÏÓÎÏ×ÁÎÉÅÍ ÕÇÏÌ, ÒÁ×ÎÙÊ
126.17.

30. îÁÊÄÉÔÅ ÓÒÅÄÎÀÀ ÌÉÎÉÀ ÔÒÁÐÅÃÉÉ.

6, Á ×ÔÏÒÁÑ

óÒÅÄÎÑÑ ÌÉÎÉÑ ÔÒÁÐÅÃÉÉ ÒÁ×ÎÁ 5, Á ÏÔÒÅÚÏË, ÓÏÅÄÉÎÑÀÝÉÊ ÓÅÒÅÄÉÎÙ ÏÓÎÏ×ÁÎÉÊ,

ÒÁ×ÅÎ 3. õÇÌÙ ÐÒÉ ÂÏÌØÛÅÍ ÏÓÎÏ×ÁÎÉÉ ÔÒÁÐÅÃÉÉ ÒÁ×ÎÙ

194

30 É 60. îÁÊÄÉÔÅ ÏÓÎÏ×ÁÎÉÑ É

ÍÅÎØÛÕÀ ÂÏËÏ×ÕÀ ÓÔÏÒÏÎÕ ÔÒÁÐÅÃÉÉ.
* * *
126.18. ïÄÎÁ ÉÚ ÂÏËÏ×ÙÈ ÓÔÏÒÏÎ ÔÒÁÐÅÃÉÉ ÒÁ×ÎÁ ÓÕÍÍÅ ÏÓÎÏ×ÁÎÉÊ. äÏËÁÖÉÔÅ, ÞÔÏ ÂÉÓÓÅËÔÒÉÓÙ
ÕÇÌÏ× ÐÒÉ ÜÔÏÊ ÓÔÏÒÏÎÅ ÐÅÒÅÓÅËÁÀÔÓÑ ÎÁ ÄÒÕÇÏÊ ÂÏËÏ×ÏÊ ÓÔÏÒÏÎÅ.
126.19. ïÔÒÅÚÏË, ÓÏÅÄÉÎÑÀÝÉÊ ÓÅÒÅÄÉÎÙ Ä×ÕÈ ÐÒÏÔÉ×ÏÐÏÌÏÖÎÙÈ ÓÔÏÒÏÎ ÞÅÔÙÒÅÈÕÇÏÌØÎÉËÁ, ÒÁ×ÅÎ ÐÏÌÕÓÕÍÍÅ Ä×ÕÈ ÄÒÕÇÉÈ ÓÔÏÒÏÎ. äÏËÁÖÉÔÅ, ÞÔÏ ÜÔÏÔ ÞÅÔÙÒÅÈÕÇÏÌØÎÉË | ÔÒÁÐÅÃÉÑ ÉÌÉ ÐÁÒÁÌÌÅÌÏÇÒÁÍÍ.
126.20. ïÄÎÉÍ ÐÒÑÍÏÌÉÎÅÊÎÙÍ ÒÁÚÒÅÚÏÍ ÏÔÒÅÖØÔÅ ÏÔ ÔÒÅÕÇÏÌØÎÉËÁ ÔÒÁÐÅÃÉÀ, Õ ËÏÔÏÒÏÊ ÍÅÎØÛÅÅ
ÏÓÎÏ×ÁÎÉÅ ÒÁ×ÎÏ ÓÕÍÍÅ ÂÏËÏ×ÙÈ ÓÔÏÒÏÎ.
126.21. óÕÝÅÓÔ×ÕÀÔ ÌÉ Ä×Å ÔÒÁÐÅÃÉÉ, ÏÓÎÏ×ÁÎÉÑ ÐÅÒ×ÏÊ ÉÚ ËÏÔÏÒÙÈ ÓÏÏÔ×ÅÔÓÔ×ÅÎÎÏ ÒÁ×ÎÙ ÂÏËÏ×ÙÍ ÓÔÏÒÏÎÁÍ ×ÔÏÒÏÊ, Á ÏÓÎÏ×ÁÎÉÑ ×ÔÏÒÏÊ | ÂÏËÏ×ÙÍ ÓÔÏÒÏÎÁÍ ÐÅÒ×ÏÊ?

195

127. ðÌÏÝÁÄÉ

ðÌÏÝÁÄØ ÅÓÔØ ÎÅÏÔÒÉÃÁÔÅÌØÎÁÑ ×ÅÌÉÞÉÎÁ. òÁ×ÎÙÅ ÆÉÇÕÒÙ ÉÍÅÀÔ ÒÁ×ÎÙÅ ÐÌÏÝÁÄÉ. åÓÌÉ
ÆÉÇÕÒÁ ÒÁÚÒÅÚÁÎÁ ÎÁ Ä×Å ÆÉÇÕÒÙ, ÔÏ ÅÅ ÐÌÏÝÁÄØ ÒÁ×ÎÁ ÓÕÍÍÅ ÐÌÏÝÁÄÅÊ ÜÔÉÈ ÆÉÇÕÒ. ðÌÏÝÁÄØ
ÐÒÑÍÏÕÇÏÌØÎÉËÁ ÒÁ×ÎÁ ÐÒÏÉÚ×ÅÄÅÎÉÀ ÅÇÏ ÓÏÓÅÄÎÉÈ ÓÔÏÒÏÎ.
æÉÇÕÒÙ, ÉÍÅÀÝÉÅ ÒÁ×ÎÙÅ ÐÌÏÝÁÄÉ, ÎÁÚÙ×ÁÀÔÓÑ
127.1.

ÒÁ×ÎÏ×ÅÌÉËÉÍÉ.

äÏËÁÖÉÔÅ, ÞÔÏ (Á) ÐÌÏÝÁÄØ ÔÒÅÕÇÏÌØÎÉËÁ ÒÁ×ÎÁ ÐÏÌÏ×ÉÎÅ ÐÒÏÉÚ×ÅÄÅÎÉÑ ÅÇÏ ÏÓÎÏ-

×ÁÎÉÑ ÎÁ ×ÙÓÏÔÕ; (Â) ÐÌÏÝÁÄØ ÐÁÒÁÌÌÅÌÏÇÒÁÍÍÁ ÒÁ×ÎÁ ÐÒÏÉÚ×ÅÄÅÎÉÀ ÅÇÏ ÏÓÎÏ×ÁÎÉÑ ÎÁ ×ÙÓÏÔÕ; (×) ÐÌÏÝÁÄØ ÒÏÍÂÁ ÒÁ×ÎÁ ÐÏÌÏ×ÉÎÅ ÐÒÏÉÚ×ÅÄÅÎÉÑ ÅÇÏ ÄÉÁÇÏÎÁÌÅÊ; (Ç) ÐÌÏÝÁÄØ ÔÒÁÐÅÃÉÉ
ÒÁ×ÎÁ ÐÒÏÉÚ×ÅÄÅÎÉÀ Å£ ÓÒÅÄÎÅÊ ÌÉÎÉÉ ÎÁ ×ÙÓÏÔÕ.

100. íÏÖÅÔ ÌÉ ÅÇÏ ÐÌÏÝÁÄØ ÂÙÔØ ÍÅÎØÛÅ

127.2.

ëÁÖÄÁÑ ÓÔÏÒÏÎÁ ÔÒÅÕÇÏÌØÎÉËÁ ÂÏÌØÛÅ

127.3.

ïÓÎÏ×ÁÎÉÑ ÒÁ×ÎÏÂÅÄÒÅÎÎÏÊ ÔÒÁÐÅÃÉÉ ÒÁ×ÎÙ

0;01?

ÐÌÏÝÁÄØ ÔÒÁÐÅÃÉÉ.

a É b, ÏÓÔÒÙÊ ÕÇÏÌ ÒÁ×ÅÎ 45. îÁÊÄÉÔÅ

127.4.

ëÁËÕÀ ÞÁÓÔØ ÐÌÏÝÁÄÉ ÔÒÅÕÇÏÌØÎÉËÁ ÏÔÓÅËÁÅÔ ÏÔ ÎÅÇÏ ÓÒÅÄÎÑÑ ÌÉÎÉÑ?

127.5.

äÉÁÇÏÎÁÌÉ ÒÁÚÂÉ×ÁÀÔ ÔÒÁÐÅÃÉÀ ÎÁ ÞÅÔÙÒÅ ÔÒÅÕÇÏÌØÎÉËÁ. äÏËÁÖÉÔÅ, ÞÔÏ ÔÒÅÕÇÏÌØ-

ÎÉËÉ, ÐÒÉÌÅÖÁÝÉÅ Ë ÂÏËÏ×ÙÍ ÓÔÏÒÏÎÁÍ, ÒÁ×ÎÏ×ÅÌÉËÉ. ÷ÅÒÎÏ ÌÉ ÏÂÒÁÔÎÏÅ (ÅÓÌÉ ÏÎÉ ÒÁ×ÎÏ×ÅÌÉËÉ, ÔÏ ÞÅÔÙÒÅÈÕÇÏÌØÎÉË | ÔÒÁÐÅÃÉÑ)?
127.6.

äÏËÁÖÉÔÅ, ÞÔÏ ÍÅÄÉÁÎÁ ÒÁÚÂÉ×ÁÅÔ ÔÒÅÕÇÏÌØÎÉË ÎÁ Ä×Á ÒÁ×ÎÏ×ÅÌÉËÉÈ ÔÒÅÕÇÏÌØÎÉËÁ.

127.7.

äÏËÁÖÉÔÅ, ÞÔÏ ÅÓÌÉ ÄÉÁÇÏÎÁÌØ ÞÅÔÙÒ£ÈÕÇÏÌØÎÉËÁ ÄÅÌÉÔ ÄÒÕÇÕÀ ÄÉÁÇÏÎÁÌØ ÐÏÐÏ-

ÌÁÍ, ÔÏ ÏÎÁ ÄÅÌÉÔ ÐÏÐÏÌÁÍ É ÐÌÏÝÁÄØ ÞÅÔÙÒ£ÈÕÇÏÌØÎÉËÁ.

ABC. îÁÊÄÉÔÅ ÇÅÏÍÅÔÒÉÞÅÓËÏÅ ÍÅÓÔÏ ÔÏÞÅË M, ÄÌÑ ËÏÔÏÒÙÈ ( )
ÔÒÅÕÇÏÌØÎÉËÉ AMB É ABC ÒÁ×ÎÏ×ÅÌÉËÉ; ( ) ÔÒÅÕÇÏÌØÎÉËÉ AMB É AMC ÒÁ×ÎÏ×ÅÌÉËÉ; ( )
ÔÒÅÕÇÏÌØÎÉËÉ AMB, AMC É BMC ÒÁ×ÎÏ×ÅÌÉËÉ.
ôÏÞËÁ M ÄÅÌÉÔ ÓÔÏÒÏÎÕ AB ÔÒÅÕÇÏÌØÎÉËÁ ABC × ÏÔÎÏÛÅÎÉÉ AM : MB = m : n.
äÏËÁÖÉÔÅ, ÞÔÏ ÐÌÏÝÁÄÉ ÔÒÅÕÇÏÌØÎÉËÏ× CAM É CBM ÏÔÎÏÓÑÔÓÑ ËÁË m : n.
127.8.

äÁÎ ÔÒÅÕÇÏÌØÎÉË

Á

Â

×

127.9.

127.10.

äÏËÁÖÉÔÅ, ÞÔÏ ÍÅÄÉÁÎÙ ÔÒÅÕÇÏÌØÎÉËÁ ÄÅÌÑÔ ÅÇÏ ÎÁ ÛÅÓÔØ ÒÁ×ÎÏ×ÅÌÉËÉÈ ÞÁÓÔÅÊ.

127.11.

ôÏÞËÉ

M É N ÌÅÖÁÔ ÎÁ ÓÔÏÒÏÎÁÈ AB É AC ÔÒÅÕÇÏÌØÎÉËÁ ABC ÉÌÉ ÉÈ ÐÒÏÄÏÌÖÅÎÉÑÈ, ÐÒÉÞ£Í AM : AB = m : n, AN : AC = p : q. äÏËÁÖÉÔÅ, ÞÔÏ ÐÌÏÝÁÄÉ ÔÒÅÕÇÏÌØÎÉËÏ×
AMN É ABC ÏÔÎÏÓÑÔÓÑ ËÁË (m=n) (p=q).
óÔÏÒÏÎÙ ÔÒÅÕÇÏÌØÎÉËÁ ÐÌÏÝÁÄÉ 1 ÒÁÚÄÅÌÅÎÙ × ÏÔÎÏÛÅÎÉÉ 2 : 1, ÓÞÉÔÁÑ ÐÏ ÞÁÓÏ

127.12.

×ÏÊ ÓÔÒÅÌËÅ. îÁÊÄÉÔÅ ÐÌÏÝÁÄØ ÔÒÅÕÇÏÌØÎÉËÁ Ó ×ÅÒÛÉÎÁÍÉ × ÔÏÞËÁÈ ÄÅÌÅÎÉÑ.
127.13.

äÁÎÎÙÊ ÐÁÒÁÌÌÅÌÏÇÒÁÍÍ ÒÁÚÄÅÌÉÔÅ ÎÁ ÔÒÉ ÒÁ×ÎÏ×ÅÌÉËÉÅ ÞÁÓÔÉ ÐÒÑÍÙÍÉ, ×ÙÈÏ-

ÄÑÝÉÍÉ ÉÚ ÏÄÎÏÊ ×ÅÒÛÉÎÙ.
127.14.

îÁÊÄÉÔÅ ÐÌÏÝÁÄØ ÞÅÔÙÒ£ÈÕÇÏÌØÎÉËÁ Ó ×ÅÒÛÉÎÁÍÉ × ÓÅÒÅÄÉÎÁÈ ÓÔÏÒÏÎ ×ÙÐÕËÌÏÇÏ

ÞÅÔÙÒ£ÈÕÇÏÌØÎÉËÁ, ÐÌÏÝÁÄØ ËÏÔÏÒÏÇÏ ÒÁ×ÎÁ 1.
127.15.

äÏËÁÖÉÔÅ, ÞÔÏ ÓÕÍÍÁ ÒÁÓÓÔÏÑÎÉÊ ÏÔ ÐÒÏÉÚ×ÏÌØÎÏÊ ÔÏÞËÉ ×ÎÕÔÒÉ ÒÁ×ÎÏÓÔÏÒÏÎÎÅÇÏ

ÔÒÅÕÇÏÌØÎÉËÁ ÄÏ ÅÇÏ ÓÔÏÒÏÎ ×ÓÅÇÄÁ ÏÄÎÁ É ÔÁ ÖÅ.
127.16.

îÁÊÄÉÔÅ ÐÌÏÝÁÄØ ÐÒÑÍÏÕÇÏÌØÎÏÇÏ ÔÒÅÕÇÏÌØÎÉËÁ Ó ÇÉÐÏÔÅÎÕÚÏÊ, ÒÁ×ÎÏÊ

ÏÓÔÒÙÍ ÕÇÌÏÍ
127.17.

15

.

c,

É

÷ ÐÁÒÁÌÌÅÌÏÇÒÁÍÍÅ ÓÅÒÅÄÉÎÁ ËÁÖÄÏÊ ÓÔÏÒÏÎÙ ÓÏÅÄÉÎÅÎÁ Ó ËÏÎÃÏÍ ÓÌÅÄÕÀÝÅÊ

ÓÔÏÒÏÎÙ; ÐÏÌÕÞÉÌÓÑ ×ÎÕÔÒÅÎÎÉÊ ÐÁÒÁÌÌÅÌÏÇÒÁÍÍ. äÏËÁÖÉÔÅ, ÞÔÏ ÅÇÏ ÐÌÏÝÁÄØ ÓÏÓÔÁ×ÌÑÅÔ

1=5 ÐÌÏÝÁÄÉ ÄÁÎÎÏÇÏ ÐÁÒÁÌÌÅÌÏÇÒÁÍÍÁ.

196

127.18.

ðÒÏÉÚ×ÏÌØÎÙÊ ÞÅÔÙÒ£ÈÕÇÏÌØÎÉË ÒÁÚÄÅÌÅÎ ÄÉÁÇÏÎÁÌÑÍÉ ÎÁ ÞÅÔÙÒÅ ÔÒÅÕÇÏÌØÎÉËÁ;

ÐÌÏÝÁÄÉ ÔÒ£È ÉÚ ÎÉÈ ÒÁ×ÎÙ

10, 20 É 30, É ËÁÖÄÁÑ ÍÅÎØÛÅ ÐÌÏÝÁÄÉ ÞÅÔ×ÅÒÔÏÇÏ ÔÒÅÕÇÏÌØÎÉËÁ.

îÁÊÄÉÔÅ ÐÌÏÝÁÄØ ÞÅÔÙÒ£ÈÕÇÏÌØÎÉËÁ.

âÏËÏ×ÁÑ ÓÔÏÒÏÎÁ AB É ÏÓÎÏ×ÁÎÉÅ BC ÔÒÁÐÅÃÉÉ ABCD ×Ä×ÏÅ ÍÅÎØÛŠţ ÏÓÎÏ×ÁAD. îÁÊÄÉÔÅ ÐÌÏÝÁÄØ ÔÒÁÐÅÃÉÉ, ÅÓÌÉ AC = a, CD = b.

127.19.

ÎÉÑ

127.20.

ïÔÒÅÚÏË, ÓÏÅÄÉÎÑÀÝÉÊ ÓÅÒÅÄÉÎÙ Ä×ÕÈ ÐÒÏÔÉ×ÏÐÏÌÏÖÎÙÈ ÓÔÏÒÏÎ ×ÙÐÕËÌÏÇÏ ÞÅ-

ÔÙÒÅÈÕÇÏÌØÎÉËÁ, ÄÅÌÉÔ ÅÇÏ ÎÁ Ä×Á ÒÁ×ÎÏ×ÅÌÉËÉÈ ÞÅÔÙÒ£ÈÕÇÏÌØÎÉËÁ. äÏËÁÖÉÔÅ, ÞÔÏ ÜÔÉ ÓÔÏÒÏÎÙ ÐÁÒÁÌÌÅÌØÎÙ.

P | ÓÅÒÅÄÉÎÁ ÓÔÏÒÏÎÙ AB ×ÙÐÕËÌÏÇÏ ÞÅÔÙÒ£ÈÕÇÏÌØÎÉËÁ ABCD. äÏËÁÖÉÔÅ, ÞÔÏ ÅÓÌÉ ÐÌÏÝÁÄØ ÔÒÅÕÇÏÌØÎÉËÁ PDC ÒÁ×ÎÁ ÐÏÌÏ×ÉÎÅ ÐÌÏÝÁÄÉ ÞÅÔÙÒ£ÈÕÇÏÌØÎÉËÁ ABCD,
ÔÏ ÓÔÏÒÏÎÙ BC É AD ÐÁÒÁÌÌÅÌØÎÙ.
127.21.

127.22.

ðÕÓÔØ

(ôÅÏÒÅÍÁ ðÉÆÁÇÏÒÁ) äÏËÁÚÁÔØ, ÞÔÏ ÐÌÏÝÁÄØ Ë×ÁÄÒÁÔÁ, ÐÏÓÔÒÏÅÎÎÏÇÏ ÎÁ ÇÉÐÏ-

ÔÅÎÕÚÅ ÐÒÑÍÏÕÇÏÌØÎÏÇÏ ÔÒÅÕÇÏÌØÎÉËÁ, ÒÁ×ÎÁ ÓÕÍÍÅ ÐÌÏÝÁÄÅÊ Ë×ÁÄÒÁÔÏ×, ÐÏÓÔÒÏÅÎÎÙÈ ÎÁ
ËÁÔÅÔÁÈ.
* * *

M É N ÌÅÖÁÔ ÎÁ ÓÔÏÒÏÎÁÈ AB É AC ÔÒÅÕÇÏÌØÎÉËÁ ABC, ÐÒÉÞÅÍ AM = CN É
AN = BM. äÏËÁÖÉÔÅ, ÞÔÏ ÐÌÏÝÁÄØ ÞÅÔÙÒÅÈÕÇÏÌØÎÉËÁ BMNC ÐÏ ËÒÁÊÎÅÊ ÍÅÒÅ × ÔÒÉ ÒÁÚÁ ÂÏÌØÛÅ
ÐÌÏÝÁÄÉ ÔÒÅÕÇÏÌØÎÉËÁ AMN.
÷ÎÕÔÒÉ Ë×ÁÄÒÁÔÁ ÓÏ ÓÔÏÒÏÎÏÊ, ÒÁ×ÎÏÊ 1, ÐÒÏÉÚ×ÏÌØÎÏ ÂÅÒÕÔ 101 ÔÏÞËÕ, ÐÒÉÞÅÍ ÎÉËÁËÉÅ
3 ÉÚ ÎÉÈ ÎÅ ÌÅÖÁÔ ÎÁ ÏÄÎÏÊ ÐÒÑÍÏÊ. äÏËÁÖÉÔÅ, ÞÔÏ ÓÕÝÅÓÔ×ÕÅÔ ÔÒÅÕÇÏÌØÎÉË Ó ×ÅÒÛÉÎÁÍÉ × ÜÔÉÈ
ÔÏÞËÁÈ, ÐÌÏÝÁÄØ ËÏÔÏÒÏÇÏ ÎÅ ÂÏÌØÛÅ 1=100.
îÁÊÄÉÔÅ ÇÅÏÍÅÔÒÉÞÅÓËÏÅ ÍÅÓÔÏ ÔÏÞÅË X, ÌÅÖÁÝÉÈ ×ÎÕÔÒÉ ÉÌÉ ÎÁ ÓÔÏÒÏÎÁÈ ÔÒÁÐÅÃÉÉ
ABCD Ó ÏÓÎÏ×ÁÎÉÑÍÉ BC É AD, ÄÌÑ ËÏÔÏÒÙÈ S(XAB) = S(XCD).
ðÕÓÔØ M É N | ÓÅÒÅÄÉÎÙ ÐÒÏÔÉ×ÏÐÏÌÏÖÎÙÈ ÓÔÏÒÏÎ BC É AD ×ÙÐÕËÌÏÇÏ ÞÅÔÙÒ£ÈÕÇÏÌØÎÉËÁ ABCD, ÏÔÒÅÚËÉ AM É BN ÐÅÒÅÓÅËÁÀÔÓÑ × ÔÏÞËÅ P, Á ÏÔÒÅÚËÉ DM É CN | × ÔÏÞËÅ
Q. äÏËÁÖÉÔÅ, ÞÔÏ ÓÕÍÍÁ ÐÌÏÝÁÄÅÊ ÔÒÅÕÇÏÌØÎÉËÏ× APB É CQD ÒÁ×ÎÁ ÐÌÏÝÁÄÉ ÞÅÔÙÒÅÈÕÇÏÌØÎÉËÁ
MPNQ.
ôÏÞËÁ ×ÎÕÔÒÉ ÐÒÁ×ÉÌØÎÏÇÏ 2n-ÕÇÏÌØÎÉËÁ ÓÏÅÄÉÎÅÎÁ Ó ×ÅÒÛÉÎÁÍÉ. ÷ÏÚÎÉËÛÉÅ 2n ÔÒÅ127.23.

ôÏÞËÉ

127.24.

127.25.

127.26.

127.27.

ÕÇÏÌØÎÉËÏ× ÒÁÓËÒÁÛÅÎÙ ÐÏÐÅÒÅÍÅÎÎÏ × ÇÏÌÕÂÏÊ É ËÒÁÓÎÙÊ Ã×ÅÔ. äÏËÁÖÉÔÅ, ÞÔÏ ÓÕÍÍÁ ÐÌÏÝÁÄÅÊ

ÇÏÌÕÂÙÈ ÔÒÅÕÇÏÌØÎÉËÏ× ÒÁ×ÎÁ ÓÕÍÍÅ ÐÌÏÝÁÄÅÊ ËÒÁÓÎÙÈ (Á) ÄÌÑ
ÐÒÏÉÚ×ÏÌØÎÏÇÏ
127.28.

n.

n

=

4;

(Â) ÄÌÑ

n

=

3;

(×) ÄÌÑ

éÚ ÓÅÒÅÄÉÎÙ ËÁÖÄÏÊ ÓÔÏÒÏÎÙ ÏÓÔÒÏÕÇÏÌØÎÏÇÏ ÔÒÅÕÇÏÌØÎÉËÁ ÏÐÕÝÅÎÙ ÐÅÒÐÅÎÄÉËÕÌÑÒÙ

ÎÁ Ä×Å ÄÒÕÇÉÅ ÓÔÏÒÏÎÙ. äÏËÁÖÉÔÅ, ÞÔÏ ÐÌÏÝÁÄØ ÏÇÒÁÎÉÞÅÎÎÏÇÏ ÉÍÉ ÛÅÓÔÉÕÇÏÌØÎÉËÁ ÒÁ×ÎÁ ÐÏÌÏ×ÉÎÅ
ÐÌÏÝÁÄÉ ÔÒÅÕÇÏÌØÎÉËÁ.
127.29.

îÁ ËÁÖÄÏÊ ÓÔÏÒÏÎÅ ÐÁÒÁÌÌÅÌÏÇÒÁÍÍÁ ×ÚÑÔÏ ÐÏ ÔÏÞËÅ. ðÌÏÝÁÄØ ÞÅÔÙÒ£ÈÕÇÏÌØÎÉËÁ Ó ×ÅÒ-

ÛÉÎÁÍÉ × ÜÔÉÈ ÔÏÞËÁÈ ÒÁ×ÎÁ ÐÏÌÏ×ÉÎÅ ÐÌÏÝÁÄÉ ÐÁÒÁÌÌÅÌÏÇÒÁÍÍÁ. äÏËÁÖÉÔÅ, ÞÔÏ ÈÏÔÑ ÂÙ ÏÄÎÁ ÉÚ
ÄÉÁÇÏÎÁÌÅÊ ÞÅÔÙÒÅÈÕÇÏÌØÎÉËÁ ÐÁÒÁÌÌÅÌØÎÁ ÏÄÎÏÊ ÉÚ ÓÔÏÒÏÎ ÐÁÒÁÌÌÅÌÏÇÒÁÍÍÁ.

197

128. ïËÒÕÖÎÏÓÔØ
ïËÒÕÖÎÏÓÔØÀ ÎÁÚÙ×ÁÅÔÓÑ ÇÅÏÍÅÔÒÉÞÅÓËÏÅ ÍÅÓÔÏ ÔÏÞÅË, ÕÄÁÌÅÎÎÙÈ ÏÔ ÄÁÎÎÏÊ ÔÏÞËÉ (ÃÅÎÔÒÁ ), ÎÁ ÏÄÎÏ É ÔÏ ÖÅ ÐÏÌÏÖÉÔÅÌØÎÏÅ ÒÁÓÓÔÏÑÎÉÅ (ÒÁÄÉÕÓ ). ïÔÒÅÚÏË, ÓÏÅÄÉÎÑÀÝÉÊ ÔÏÞËÕ

ÏËÒÕÖÎÏÓÔÉ Ó ÃÅÎÔÒÏÍ, ÔÁËÖÅ ÎÁÚÙ×ÁÀÔ ÒÁÄÉÕÓÏÍ. ïÔÒÅÚÏË, ÓÏÅÄÉÎÑÀÝÉÊ Ä×Å ÔÏÞËÉ ÏËÒÕÖÎÏÓÔÉ, ÎÁÚÙ×ÁÀÔ ÈÏÒÄÏÊ. äÉÁÍÅÔÒ ÏËÒÕÖÎÏÓÔÉ | ÈÏÒÄÁ, ÐÒÏÈÏÄÑÝÁÑ ÞÅÒÅÚ ÃÅÎÔÒ.
128.1.

äÏËÁÖÉÔÅ ÓÌÅÄÕÀÝÉÅ Ó×ÏÊÓÔ×Á ÏËÒÕÖÎÏÓÔÉ: (Á) ÄÉÁÍÅÔÒ ÅÓÔØ ÎÁÉÂÏÌØÛÁÑ ÈÏÒÄÁ

ÏËÒÕÖÎÏÓÔÉ; (Â) ÄÉÁÍÅÔÒ, ÐÅÒÐÅÎÄÉËÕÌÑÒÎÙÊ ÈÏÒÄÅ, ÄÅÌÉÔ Å£ ÐÏÐÏÌÁÍ; (×) ÏËÒÕÖÎÏÓÔØ ÓÉÍÍÅÔÒÉÞÎÁ ÏÔÎÏÓÉÔÅÌØÎÏ ËÁÖÄÏÇÏ Ó×ÏÅÇÏ ÄÉÁÍÅÔÒÁ; (Ç) ÄÕÇÉ ÏËÒÕÖÎÏÓÔÉ, ÚÁËÌÀÞÅÎÎÙÅ ÍÅÖÄÕ
ÐÁÒÁÌÌÅÌØÎÙÍÉ ÈÏÒÄÁÍÉ, ÒÁ×ÎÙ; (Ä) ÒÁ×ÎÙÅ ÈÏÒÄÙ ÕÄÁÌÅÎÙ ÎÁ ÒÁ×ÎÙÅ ÒÁÓÓÔÏÑÎÉÑ ÏÔ ÃÅÎÔÒÁ;
(Ö) ÉÚ Ä×ÕÈ ÎÅÒÁ×ÎÙÈ ÈÏÒÄ ÂÏÌØÛÁÑ ÕÄÁÌÅÎÁ ÏÔ ÃÅÎÔÒÁ ÎÁ ÍÅÎØÛÅÅ ÒÁÓÓÔÏÑÎÉÅ.
128.2.

þÅÒÅÚ ÔÏÞËÕ ÏËÒÕÖÎÏÓÔÉ ÐÒÏ×ÅÄÅÎÙ ÄÉÁÍÅÔÒ É ÈÏÒÄÁ, ÒÁ×ÎÁÑ ÒÁÄÉÕÓÕ. îÁÊÄÉÔÅ

ÕÇÏÌ ÍÅÖÄÕ ÎÉÍÉ.

O. îÁ ÐÒÏÄÏÌÖÅÎÉÉ ÈÏÒÄÙ AB ÚÁ ÔÏÞËÕ B ÏÔÌÏÖÅÎ
ÏÔÒÅÚÏË BC, ÒÁ×ÎÙÊ ÒÁÄÉÕÓÕ. þÅÒÅÚ ÔÏÞËÉ C É O ÐÒÏ×ÅÄÅÎÁ ÓÅËÕÝÁÑ CD (ÐÒÉ ÜÔÏÍ D |
ÔÏÞËÁ ÐÅÒÅÓÅÞÅÎÉÑ Ó ÏËÒÕÖÎÏÓÔØÀ, ÌÅÖÁÝÁÑ ×ÎÅ ÏÔÒÅÚËÁ CO). äÏËÁÖÉÔÅ, ÞÔÏ \AOD =
3\ACD.
128.3.

äÁÎÁ ÏËÒÕÖÎÏÓÔØ Ó ÃÅÎÔÒÏÍ

128.4.

äÉÁÇÏÎÁÌÉ ×ÐÉÓÁÎÎÏÇÏ ÞÅÔÙÒÅÈÕÇÏÌØÎÉËÁ ×ÚÁÉÍÎÏ ÐÅÒÐÅÎÄÉËÕÌÑÒÎÙ. äÏËÁÖÉÔÅ,

ÞÔÏ ÒÁÓÓÔÏÑÎÉÅ ÏÔ ÔÏÞËÉ ÐÅÒÅÓÅÞÅÎÉÑ ÄÉÁÇÏÎÁÌÅÊ ÄÏ ÃÅÎÔÒÁ ÏÐÉÓÁÎÎÏÊ ÏËÒÕÖÎÏÓÔÉ ÒÁ×ÎÏ
ÒÁÓÓÔÏÑÎÉÀ ÍÅÖÄÕ ÓÅÒÅÄÉÎÁÍÉ ÄÉÁÇÏÎÁÌÅÊ.
128.5.

þÅÒÅÚ ÄÁÎÎÕÀ ÔÏÞËÕ ×ÎÕÔÒÉ ËÒÕÇÁ ÐÒÏ×ÅÄÉÔÅ ÈÏÒÄÕ ÎÁÉÍÅÎØÛÅÊ ÄÌÉÎÙ.

128.6.

îÁÊÄÉÔÅ ÇÅÏÍÅÔÒÉÞÅÓËÏÅ ÍÅÓÔÏ ÔÏÞÅË

ÐÏÄ ÐÒÑÍÙÍ ÕÇÌÏÍ (

\AMD = 90 ).


M, ÉÚ ËÏÔÏÒÙÈ ÄÁÎÎÙÊ ÏÔÒÅÚÏË AB ×ÉÄÅÎ

128.7.

îÁÊÄÉÔÅ ÃÅÎÔÒ ÄÁÎÎÏÊ ÏËÒÕÖÎÏÓÔÉ Ó ÐÏÍÏÝØÀ ÞÅÒÔ£ÖÎÏÇÏ ÕÇÏÌØÎÉËÁ.

128.8.

ðÏÓÔÒÏÊÔÅ ÐÒÑÍÏÕÇÏÌØÎÙÊ ÔÒÅÕÇÏÌØÎÉË ÐÏ ÇÉÐÏÔÅÎÕÚÅ É ×ÙÓÏÔÅ, ÐÒÏ×ÅÄÅÎÎÏÊ Ë

ÇÉÐÏÔÅÎÕÚÅ.
128.9.

þÅÒÅÚ ÄÁÎÎÕÀ ÔÏÞËÕ ÏËÒÕÖÎÏÓÔÉ ÐÒÏ×ÅÄÉÔÅ ÈÏÒÄÕ, ËÏÔÏÒÁÑ ÄÅÌÉÔÓÑ ÄÁÎÎÏÊ ÈÏÒÄÏÊ

ÐÏÐÏÌÁÍ.
128.10.

÷ÐÉÛÉÔÅ × ÏËÒÕÖÎÏÓÔØ ÐÒÑÍÏÕÇÏÌØÎÙÊ ÔÒÅÕÇÏÌØÎÉË, ËÁÔÅÔÙ ËÏÔÏÒÏÇÏ ÐÒÏÈÏÄÑÔ

ÞÅÒÅÚ Ä×Å ÄÁÎÎÙÅ ÔÏÞËÉ.
128.11.

òÁÓÓÔÏÑÎÉÅ ÍÅÖÄÕ ÒÁ×ÎÙÍÉ ÐÁÒÁÌÌÅÌØÎÙÍÉ ÈÏÒÄÁÍÉ ÒÁ×ÎÏ ÒÁÄÉÕÓÕ ÏËÒÕÖÎÏÓÔÉ.

îÁÊÄÉÔÅ ÕÇÏÌ ÍÅÖÄÕ ÐÒÑÍÙÍÉ, ÐÒÏÈÏÄÑÝÉÍÉ ÞÅÒÅÚ ËÏÎÃÙ ÜÔÉÈ ÈÏÒÄ.
128.12.

ðÒÏÄÏÌÖÅÎÉÑ ×ÙÓÏÔÙ É ÍÅÄÉÁÎÙ ÐÒÑÍÏÕÇÏÌØÎÏÇÏ ÔÒÅÕÇÏÌØÎÉËÁ

ÄÅÎÎÙÈ ÉÚ ×ÅÒÛÉÎÙ

C ÐÒÑÍÏÇÏ ÕÇÌÁ, ÐÅÒÅÓÅËÁÀÔ

ABC, ÐÒÏ×Å-

ÏÐÉÓÁÎÎÕÀ ÏËÒÕÖÎÏÓÔØ ÔÒÅÕÇÏÌØÎÉËÁ ×

M É N ÓÏÏÔ×ÅÔÓÔ×ÅÎÎÏ. îÁÊÄÉÔÅ ÐÌÏÝÁÄØ ÞÅÔÙÒÅÈÕÇÏÌØÎÉËÁ ABNM, ÅÓÌÉ ÐÌÏÝÁÄØ
ABC ÒÁ×ÎÁ S É \ABC = 30.
äÁÎÙ Ä×Å ÔÏÞËÉ A É B. îÁÊÄÉÔÅ ÇÅÏÍÅÔÒÉÞÅÓËÏÅ ÍÅÓÔÏ ÔÏÞÅË, ËÁÖÄÁÑ ÉÚ ËÏÔÏÒÙÈ
ÓÉÍÍÅÔÒÉÞÎÁ ÔÏÞËÅ A ÏÔÎÏÓÉÔÅÌØÎÏ ÎÅËÏÔÏÒÏÊ ÐÒÑÍÏÊ, ÐÒÏÈÏÄÑÝÅÊ ÞÅÒÅÚ ÔÏÞËÕ B.
þÅÔÙÒÅÈÕÇÏÌØÎÉË ABCD, ÄÉÁÇÏÎÁÌÉ ËÏÔÏÒÏÇÏ ×ÚÁÉÍÎÏ ÐÅÒÐÅÎÄÉËÕÌÑÒÎÙ, ×ÐÉÓÁÎ × ÏËÒÕÖÎÏÓÔØ Ó ÃÅÎÔÒÏÍ O. äÏËÁÖÉÔÅ, ÞÔÏ ÌÏÍÁÎÁÑ AOC ÄÅÌÉÔ ÅÇÏ ÎÁ Ä×Å ÒÁ×ÎÏ×ÅÌÉËÉÅ

ÔÏÞËÁÈ

ÔÒÅÕÇÏÌØÎÉËÁ
128.13.

128.14.

ÞÁÓÔÉ.

128.15.

þÅÒÅÚ ÔÏÞËÕ ÐÅÒÅÓÅÞÅÎÉÑ Ä×ÕÈ ÏËÒÕÖÎÏÓÔÅÊ ÐÒÏ×ÅÄÉÔÅ ÓÅËÕÝÕÀ ÔÁË, ÞÔÏÂÙ ÞÁÓÔØ

ÅÅ, ÚÁËÌÀÞ£ÎÎÁÑ ×ÎÕÔÒÉ ÏËÒÕÖÎÏÓÔÅÊ, ÉÍÅÌÁ ÚÁÄÁÎÎÕÀ ÄÌÉÎÕ.

198

128.16.

÷ ÔÒÅÕÇÏÌØÎÉËÅ

ABC ÎÁ ÎÁÉÂÏÌØÛÅÊ ÓÔÏÒÏÎÅ BC, ÒÁ×ÎÏÊ a, ×ÙÂÉÒÁÅÔÓÑ ÔÏÞËÁ

M. îÁÊÄÉÔÅ ÎÁÉÍÅÎØÛÅÅ ÒÁÓÓÔÏÑÎÉÅ ÍÅÖÄÕ ÃÅÎÔÒÁÍÉ ÏËÒÕÖÎÏÓÔÅÊ, ÏÐÉÓÁÎÎÙÈ ÏËÏÌÏ ÔÒÅAMB É AMC.

ÕÇÏÌØÎÉËÏ×
128.17.

þÅÒÅÚ ÔÏÞËÕ ÐÅÒÅÓÅÞÅÎÉÑ Ä×ÕÈ ÏËÒÕÖÎÏÓÔÅÊ ÐÒÏ×ÅÄÉÔÅ ÐÒÑÍÕÀ, ÎÁ ËÏÔÏÒÏÊ

ÏËÒÕÖÎÏÓÔÉ ×ÙÓÅËÁÀÔ ÈÏÒÄÙ Ó ÎÁÉÂÏÌØÛÅÊ ÓÕÍÍÏÊ. (ãÅÎÔÒÙ ÏËÒÕÖÎÏÓÔÅÊ ÒÁÓÐÏÌÏÖÅÎÙ ÐÏ
ÒÁÚÎÙÅ ÓÔÏÒÏÎÙ ÏÔ ÉÈ ÏÂÝÅÊ ÈÏÒÄÙ).
128.18.

îÁ ÓÔÏÒÏÎÁÈ ×ÙÐÕËÌÏÇÏ ÞÅÔÙÒÅÈÕÇÏÌØÎÉËÁ ËÁË ÎÁ ÄÉÁÍÅÔÒÁÈ ÐÏÓÔÒÏÅÎÙ ÞÅÔÙÒÅ

ËÒÕÇÁ. äÏËÁÖÉÔÅ, ÞÔÏ ÏÎÉ ÐÏËÒÙ×ÁÀÔ ×ÅÓØ ÞÅÔÙÒÅÈÕÇÏÌØÎÉË.
128.19.

ä×Á ÐÒÏÔÉ×ÏÐÏÌÏÖÎÙÈ ÕÇÌÁ ×ÙÐÕËÌÏÇÏ ÞÅÔÙÒÅÈÕÇÏÌØÎÉËÁ | ÔÕÐÙÅ. äÏËÁÖÉÔÅ,

ÞÔÏ ÓÏÅÄÉÎÑÀÝÁÑ ÉÈ ÄÉÁÇÏÎÁÌØ ËÏÒÏÞÅ ÄÒÕÇÏÊ ÄÉÁÇÏÎÁÌÉ.
* * *

äÁÎÁ ÏËÒÕÖÎÏÓÔØ, ÅÅ ÄÉÁÍÅÔÒ AB É ÔÏÞËÁ C ÎÁ ÜÔÏÍ ÄÉÁÍÅÔÒÅ. ðÏÓÔÒÏÊÔÅ ÎÁ ÏËÒÕÖÎÏÓÔÉ
X É Y, ÓÉÍÍÅÔÒÉÞÎÙÅ ÏÔÎÏÓÉÔÅÌØÎÏ AB, ÄÌÑ ËÏÔÏÒÙÈ ÐÒÑÍÁÑ YC ÐÅÒÐÅÎÄÉËÕÌÑÒÎÁ ÐÒÑÍÏÊ

128.20.

Ä×Å ÔÏÞËÉ

XA.

128.21.

ó ÐÏÍÏÝØÀ ÏÄÎÏÊ ÌÉÎÅÊËÉ ÏÐÕÓÔÉÔÅ ÐÅÒÐÅÎÄÉËÕÌÑÒ ÉÚ ÄÁÎÎÏÊ ÔÏÞËÉ ÎÁ ÄÁÎÎÙÊ ÄÉÁÍÅÔÒ

ÄÁÎÎÏÊ ÏËÒÕÖÎÏÓÔÉ (ÔÏÞËÁ ÎÅ ÌÅÖÉÔ ÎÉ ÎÁ ÏËÒÕÖÎÏÓÔÉ, ÎÉ ÎÁ ÄÉÁÍÅÔÒÅ).
128.22.

þÅÔÙÒÅÈÕÇÏÌØÎÉË

ÏËÒÕÖÎÏÓÔØ Ó ÃÅÎÔÒÏÍ

CD = 8.

128.23.

O.

ABCD,

ÄÉÁÇÏÎÁÌÉ ËÏÔÏÒÏÇÏ ×ÚÁÉÍÎÏ ÐÅÒÐÅÎÄÉËÕÌÑÒÎÙ, ×ÐÉÓÁÎ ×

îÁÊÄÉÔÅ ÒÁÓÓÔÏÑÎÉÅ ÏÔ ÔÏÞËÉ

äÏËÁÖÉÔÅ, ÞÔÏ × ÌÀÂÏÍ ÔÒÅÕÇÏÌØÎÉËÅ

O

ÄÏ ÓÔÏÒÏÎÙ

AB,

ÅÓÌÉ ÉÚ×ÅÓÔÎÏ, ÞÔÏ

ABC ÓÅÒÅÄÉÎÁ ÓÔÏÒÏÎÙ BC ÌÅÖÉÔ ÎÁ ÏÔÒÅÚËÅ,

ÓÏÅÄÉÎÑÀÝÅÍ ÔÏÞËÕ ÐÅÒÅÓÅÞÅÎÉÑ ×ÙÓÏÔ Ó ÔÏÞËÏÊ ÏËÒÕÖÎÏÓÔÉ, ÏÐÉÓÁÎÎÏÊ ÏËÏÌÏ ÜÔÏÇÏ ÔÒÅÕÇÏÌØÎÉËÁ,

A, É ÄÅÌÉÔ ÜÔÏÔ ÏÔÒÅÚÏË ÐÏÐÏÌÁÍ.
A. ôÏÞËÉ B É C | ÐÒÏÅËÃÉÉ
ÔÏÞËÉ M ÎÁ ÜÔÉ ÐÒÑÍÙÅ. îÁÊÄÉÔÅ ÕÇÏÌ ÍÅÖÄÕ ÐÒÑÍÏÊ BC É ÐÒÑÍÏÊ, ÐÒÏÈÏÄÑÝÅÊ ÞÅÒÅÚ ÓÅÒÅÄÉÎÙ
ÏÔÒÅÚËÏ× AM É BC.
÷ ÏÓÔÒÏÕÇÏÌØÎÏÍ ÔÒÅÕÇÏÌØÎÉËÅ ABC ÐÒÏ×ÅÄÅÎÙ ×ÙÓÏÔÙ BD É CE. éÚ ×ÅÒÛÉÎ B É C ÎÁ
ÐÒÑÍÕÀ ED ÏÐÕÝÅÎÙ ÐÅÒÐÅÎÄÉËÕÌÑÒÙ BF É CG. äÏËÁÖÉÔÅ, ÞÔÏ EF = DG.
÷ ×ÙÐÕËÌÏÍ ÞÅÔÙÒÅÈÕÇÏÌØÎÉËÅ ABCD ÐÒÏÔÉ×ÏÐÏÌÏÖÎÙÅ ÕÇÌÙ A É C ÐÒÑÍÙÅ. îÁ ÄÉÁÇÏÎÁÌØ AC ÏÐÕÝÅÎÙ ÐÅÒÐÅÎÄÉËÕÌÑÒÙ BE É DF. äÏËÁÖÉÔÅ, ÞÔÏ CE = FA.
îÁ ÏÔÒÅÚËÅ AB ×ÚÑÔÁ ÔÏÞËÁ C. ðÒÑÍÁÑ, ÐÒÏÈÏÄÑÝÁÑ ÞÅÒÅÚ ÔÏÞËÕ C, ÐÅÒÅÓÅËÁÅÔ ÏËÒÕÖÎÏÓÔÉ Ó ÄÉÁÍÅÔÒÁÍÉ AC É BC × ÔÏÞËÁÈ K É L, Á ÔÁËÖÅ ÏËÒÕÖÎÏÓÔØ Ó ÄÉÁÍÅÔÒÏÍ AB × ÔÏÞËÁÈ M É
N. äÏËÁÖÉÔÅ, ÞÔÏ KM = LN.
ÄÉÁÍÅÔÒÁÌØÎÏ ÐÒÏÔÉ×ÏÐÏÌÏÖÎÏÊ ×ÅÒÛÉÎÅ
128.24.

ä×Å ÎÅÐÅÒÐÅÎÄÉËÕÌÑÒÎÙÅ ÐÒÑÍÙÅ ÐÅÒÅÓÅËÁÀÔÓÑ × ÔÏÞËÅ

128.25.

128.26.

128.27.

199

129. ëÁÓÁÔÅÌØÎÁÑ Ë ÏËÒÕÖÎÏÓÔÉ

ëÁÓÁÔÅÌØÎÁÑ Ë ÏËÒÕÖÎÏÓÔÉ | ÐÒÑÍÁÑ, ÉÍÅÀÝÁÑ Ó ÏËÒÕÖÎÏÓÔØÀ ÅÄÉÎÓÔ×ÅÎÎÕÀ ÏÂÝÕÀ
ÔÏÞËÕ (ÎÁÚÙ×ÁÅÍÕÀ ÔÏÞËÏÊ ËÁÓÁÎÉÑ ).
129.1. (ôÅÏÒÅÍÁ Ï ËÁÓÁÔÅÌØÎÏÊ) òÁÄÉÕÓ ÏËÒÕÖÎÏÓÔÉ, ÐÒÏ×ÅÄÅÎÎÙÊ × ÔÏÞËÕ ËÁÓÁÎÉÑ, ÐÅÒÐÅÎÄÉËÕÌÑÒÅÎ ËÁÓÁÔÅÌØÎÏÊ.
129.2. äÏËÁÖÉÔÅ ÏÂÒÁÔÎÏÅ ÕÔ×ÅÒÖÄÅÎÉÅ: ÐÒÑÍÁÑ, ÐÒÏÈÏÄÑÝÁÑ ÞÅÒÅÚ ÎÅËÏÔÏÒÕÀ ÔÏÞËÕ
ÏËÒÕÖÎÏÓÔÉ ÐÅÒÐÅÎÄÉËÕÌÑÒÎÏ ÒÁÄÉÕÓÕ, Ñ×ÌÑÅÔÓÑ ËÁÓÁÔÅÌØÎÏÊ.
129.3. òÁÓÓÔÏÑÎÉÑ ÏÔ ËÏÎÃÏ× ÎÅËÏÔÏÒÏÇÏ ÄÉÁÍÅÔÒÁ ÄÏ ËÁÓÁÔÅÌØÎÏÊ Ë ÏËÒÕÖÎÏÓÔÉ ÒÁ×ÎÙ
É . îÁÊÄÉÔÅ ÒÁÄÉÕÓ ÏËÒÕÖÎÏÓÔÉ.
129.4. þÅÒÅÚ ÔÏÞËÕ
ÐÒÏ×ÅÄÅÎÙ Ä×Å ËÁÓÁÔÅÌØÎÙÅ
É
Ë ÎÅËÏÔÏÒÏÊ ÏËÒÕÖÎÏÓÔÉ
( É | ÔÏÞËÉ ËÁÓÁÎÉÑ). äÏËÁÖÉÔÅ, ÞÔÏ
=
.
129.5. þÅÒÅÚ ÄÁÎÎÕÀ ÔÏÞËÕ ÐÒÏ×ÅÄÉÔÅ ËÁÓÁÔÅÌØÎÕÀ Ë ÄÁÎÎÏÊ ÏËÒÕÖÎÏÓÔÉ.
129.6. ðÏÓÔÒÏÊÔÅ ÔÒÅÕÇÏÌØÎÉË, ÅÓÌÉ ÉÚ×ÅÓÔÎÙ ÏÔÒÅÚËÉ, ÎÁ ËÏÔÏÒÙÅ ×ÐÉÓÁÎÎÁÑ ÏËÒÕÖÎÏÓÔØ
ÄÅÌÉÔ ÅÇÏ ÓÔÏÒÏÎÕ, É ÒÁÄÉÕÓ ×ÐÉÓÁÎÎÏÊ ÏËÒÕÖÎÏÓÔÉ.
129.7. ðÏÓÔÒÏÊÔÅ ÐÒÑÍÕÀ, ËÁÓÁÀÝÕÀÓÑ ÄÁÎÎÏÊ ÏËÒÕÖÎÏÓÔÉ × ÄÁÎÎÏÊ ÔÏÞËÅ. (ãÅÎÔÒ
ÏËÒÕÖÎÏÓÔÉ ÎÅ ÚÁÄÁÎ.)
129.8. äÏËÁÖÉÔÅ, ÞÔÏ ÃÅÎÔÒ ÏËÒÕÖÎÏÓÔÉ, ×ÐÉÓÁÎÎÏÊ × ÕÇÏÌ, ÒÁÓÐÏÌÏÖÅÎ ÎÁ ÅÇÏ ÂÉÓÓÅËÔÒÉÓÅ.
129.9. ðÏÓÔÒÏÊÔÅ ÔÒÅÕÇÏÌØÎÉË ÐÏ ÓÔÏÒÏÎÅ, ÐÒÉÌÅÖÁÝÅÍÕ Ë ÎÅÊ ÕÇÌÕ É ÒÁÄÉÕÓÕ ×ÐÉÓÁÎÎÏÇÏ
ËÒÕÇÁ.
129.10. ôÏÞËÁ
ÌÅÖÉÔ ÎÁ ÓÔÏÒÏÎÅ
ÔÒÅÕÇÏÌØÎÉËÁ
. ÷ ÔÒÅÕÇÏÌØÎÉËÉ
É
×ÐÉÓÁÎÙ ÏËÒÕÖÎÏÓÔÉ Ó ÃÅÎÔÒÁÍÉ 1 É 2 . äÏËÁÖÉÔÅ, ÞÔÏ ÏÔÒÅÚÏË 1 2 ×ÉÄÅÎ ÉÚ ÔÏÞËÉ
ÐÏÄ ÐÒÑÍÙÍ ÕÇÌÏÍ.
129.11. ãÅÎÔÒ ÏËÒÕÖÎÏÓÔÉ, ÏÐÉÓÁÎÎÏÊ ÏËÏÌÏ ÔÒÅÕÇÏÌØÎÉËÁ, ÓÏ×ÐÁÄÁÅÔ Ó ÃÅÎÔÒÏÍ ×ÐÉÓÁÎÎÏÊ × ÎÅÇÏ ÏËÒÕÖÎÏÓÔÉ. îÁÊÄÉÔÅ ÕÇÌÙ ÔÒÅÕÇÏÌØÎÉËÁ.
129.12. ÷ ÔÒÁÐÅÃÉÀ ×ÐÉÓÁÎÁ ÏËÒÕÖÎÏÓÔØ. äÏËÁÖÉÔÅ, ÞÔÏ ÏÔÒÅÚËÉ, ÓÏÅÄÉÎÑÀÝÉÅ ÃÅÎÔÒ
ÜÔÏÊ ÏËÒÕÖÎÏÓÔÉ Ó ËÏÎÃÁÍÉ ÏÄÎÏÊ ÉÚ ÂÏËÏ×ÙÈ ÓÔÏÒÏÎ, ×ÚÁÉÍÎÏ ÐÅÒÐÅÎÄÉËÕÌÑÒÎÙ.
129.13. ïËÒÕÖÎÏÓÔØ ×ÙÓÅËÁÅÔ ÎÁ ÓÔÏÒÏÎÁÈ ÞÅÔÙÒÅÈÕÇÏÌØÎÉËÁ ÒÁ×ÎÙÅ ÈÏÒÄÙ. äÏËÁÖÉÔÅ,
ÞÔÏ × ÜÔÏÔ ÞÅÔÙÒÅÈÕÇÏÌØÎÉË ÍÏÖÎÏ ×ÐÉÓÁÔØ ÏËÒÕÖÎÏÓÔØ.
129.14. ÷ ÐÒÑÍÏÊ ÕÇÏÌ ×ÐÉÓÁÎÁ ÏËÒÕÖÎÏÓÔØ ÒÁÄÉÕÓÁ , ËÁÓÁÀÝÁÑÓÑ ÓÔÏÒÏÎ ÕÇÌÁ × ÔÏÞËÁÈ
É . þÅÒÅÚ ÎÅËÏÔÏÒÕÀ ÔÏÞËÕ ÎÁ ÍÅÎØÛÅÊ ÄÕÇÅ
ÏËÒÕÖÎÏÓÔÉ ÐÒÏ×ÅÄÅÎÁ ËÁÓÁÔÅÌØÎÁÑ,
ÏÔÓÅËÁÀÝÁÑ ÏÔ ÄÁÎÎÏÇÏ ÕÇÌÁ ÔÒÅÕÇÏÌØÎÉË. îÁÊÄÉÔÅ ÅÇÏ ÐÅÒÉÍÅÔÒ.
129.15. ë ÏËÒÕÖÎÏÓÔÉ, ×ÐÉÓÁÎÎÏÊ × ÒÁ×ÎÏÓÔÏÒÏÎÎÉÊ ÔÒÅÕÇÏÌØÎÉË ÓÏ ÓÔÏÒÏÎÏÊ
, ÐÒÏ×ÅÄÅÎÁ ËÁÓÁÔÅÌØÎÁÑ, ÐÅÒÅÓÅËÁÀÝÁÑ Ä×Å ÅÇÏ ÓÔÏÒÏÎÙ. îÁÊÄÉÔÅ ÐÅÒÉÍÅÔÒ ÏÔÓÅÞ£ÎÎÏÇÏ ÔÒÅÕÇÏÌØÎÉËÁ.
129.16. ÷ ÒÁ×ÎÏÂÅÄÒÅÎÎÙÊ ÔÒÅÕÇÏÌØÎÉË Ó ÏÓÎÏ×ÁÎÉÅÍ, ÒÁ×ÎÙÍ
, ×ÐÉÓÁÎÁ ÏËÒÕÖÎÏÓÔØ.
ë ÎÅÊ ÐÒÏ×ÅÄÅÎÙ ÔÒÉ ËÁÓÁÔÅÌØÎÙÅ, ÏÔÓÅËÁÀÝÉÅ ÏÔ ÄÁÎÎÏÇÏ ÔÒÅÕÇÏÌØÎÉËÁ ÔÒÉ ÍÁÌÅÎØËÉÈ
ÔÒÅÕÇÏÌØÎÉËÁ, ÓÕÍÍÁ ÐÅÒÉÍÅÔÒÏ× ËÏÔÏÒÙÈ ÒÁ×ÎÁ . îÁÊÄÉÔÅ ÂÏËÏ×ÕÀ ÓÔÏÒÏÎÕ ÄÁÎÎÏÇÏ ÔÒÅÕÇÏÌØÎÉËÁ.
129.17. þÅÒÅÚ ÄÁÎÎÕÀ ÔÏÞËÕ ÐÒÏ×ÅÄÉÔÅ ÐÒÑÍÕÀ, ÎÁ ËÏÔÏÒÏÊ ÄÁÎÎÁÑ ÏËÒÕÖÎÏÓÔØ ×ÙÓÅËÁÌÁ
ÂÙ ÈÏÒÄÕ ÄÁÎÎÏÊ ÄÌÉÎÙ.

7 3

M

A B

D

MA MB

BC
O O

MA MB

ABC

A B

AB

OO

ABD ACD
D

R

a

12

48

200

ïËÒÕÖÎÏÓÔØ, ×ÐÉÓÁÎÎÁÑ × ÔÒÅÕÇÏÌØÎÉË ABC, ËÁÓÁÅÔÓÑ ÅÇÏ ÓÔÏÒÏÎ AB, BC É AC
× ÔÏÞËÁÈ K, M É N ÓÏÏÔ×ÅÔÓÔ×ÅÎÎÏ. îÁÊÄÉÔÅ ÕÇÏÌ KMN, ÅÓÌÉ \A = 60 .
129.19. ðÕÓÔØ r | ÒÁÄÉÕÓ ÏËÒÕÖÎÏÓÔÉ, ×ÐÉÓÁÎÎÏÊ × ÐÒÑÍÏÕÇÏÌØÎÙÊ ÔÒÅÕÇÏÌØÎÉË Ó ËÁÔÅÔÁÍÉ a É b É ÇÉÐÏÔÅÎÕÚÏÊ c. äÏËÁÖÉÔÅ, ÞÔÏ r = (a + b - c)=2.
129.20. äÏËÁÖÉÔÅ, ÞÔÏ ÐÌÏÝÁÄØ ÔÒÅÕÇÏÌØÎÉËÁ ÒÁ×ÎÁ ÐÒÏÉÚ×ÅÄÅÎÉÀ ÅÇÏ ÐÏÌÕÐÅÒÉÍÅÔÒÁ
ÎÁ ÒÁÄÉÕÓ ×ÐÉÓÁÎÎÏÊ ÏËÒÕÖÎÏÓÔÉ.
129.21. ÷ ÔÒÅÕÇÏÌØÎÉË ABC ×ÐÉÓÁÎÁ ÏËÒÕÖÎÏÓÔØ, ËÁÓÁÀÝÁÑÓÑ ÓÔÏÒÏÎÙ AB × ÔÏÞËÅ M.
ðÕÓÔØ AM = x, BC = a, ÐÏÌÕÐÅÒÉÍÅÔÒ ÔÒÅÕÇÏÌØÎÉËÁ ÒÁ×ÅÎ p. äÏËÁÖÉÔÅ, ÞÔÏ x = p - a.
129.22. îÁÊÄÉÔÅ ÒÁÓÓÔÏÑÎÉÅ ÍÅÖÄÕ ÔÏÞËÁÍÉ ËÁÓÁÎÉÑ ÓÏ ÓÔÏÒÏÎÏÊ AC ÏËÒÕÖÎÏÓÔÅÊ, ×ÐÉÓÁÎÎÙÈ × ÔÒÅÕÇÏÌØÎÉËÉ ABC É ADC, ÅÓÌÉ (Á) AB = 5, BC = 7, CD = AD; (Â) AB = 5,
BC = CD, AD = 7.
129.23. äÏËÁÖÉÔÅ, ÞÔÏ Õ ÞÅÔÙÒÅÈÕÇÏÌØÎÉËÁ, ÏÐÉÓÁÎÎÏÇÏ ÏËÏÌÏ ÏËÒÕÖÎÏÓÔÉ, ÓÕÍÍÙ ÐÒÏÔÉ×ÏÐÏÌÏÖÎÙÈ ÓÔÏÒÏÎ ÒÁ×ÎÙ ÍÅÖÄÕ ÓÏÂÏÊ.
129.24. ÷ÎÅ×ÐÉÓÁÎÎÁÑ ÏËÒÕÖÎÏÓÔØ ËÁÓÁÅÔÓÑ ÓÔÏÒÏÎÙ BC ÔÒÅÕÇÏÌØÎÉËÁ ABC × ÔÏÞËÅ M
É ÐÒÏÄÏÌÖÅÎÉÊ ÓÔÏÒÏÎ AB É AC × ÔÏÞËÁÈ N É P (ÓÏÏÔ×ÅÔÓÔ×ÅÎÎÏ). ÷ÐÉÓÁÎÎÁÑ ÏËÒÕÖÎÏÓÔØ
ÜÔÏÇÏ ÔÒÅÕÇÏÌØÎÉËÁ ËÁÓÁÅÔÓÑ ÓÔÏÒÏÎÙ BC × ÔÏÞËÅ K, Á ÓÔÏÒÏÎÙ AB - × ÔÏÞËÅ L. äÏËÁÖÉÔÅ,
ÞÔÏ (Á) AN ÒÁ×ÎÏ ÐÏÌÕÐÅÒÉÍÅÔÒÕ ÔÒÅÕÇÏÌØÎÉËÁ ABC; (Â) BM = CK; (×) NL = BC.
129.25. ÷ ÔÒÅÕÇÏÌØÎÉË ÓÏ ÓÔÏÒÏÎÁÍÉ 6, 10 É 12 ×ÐÉÓÁÎÁ ÏËÒÕÖÎÏÓÔØ. ë ÏËÒÕÖÎÏÓÔÉ
ÐÒÏ×ÅÄÅÎÁ ËÁÓÁÔÅÌØÎÁÑ, ÐÅÒÅÓÅËÁÀÝÁÑ Ä×Å ÂÏÌØÛÉÅ ÓÔÏÒÏÎÙ. îÁÊÄÉÔÅ ÐÅÒÉÍÅÔÒ ÏÔÓÅÞÅÎÎÏÇÏ
ÔÒÅÕÇÏÌØÎÉËÁ.
129.26. þÅÒÅÚ ÄÁÎÎÕÀ ÔÏÞËÕ ÐÒÏ×ÅÄÉÔÅ ÐÒÑÍÕÀ, ÏÔÓÅËÁÀÝÕÀ ÏÔ ÄÁÎÎÏÇÏ ÕÇÌÁ ÔÒÅÕÇÏÌØÎÉË ÚÁÄÁÎÎÏÇÏ ÐÅÒÉÍÅÔÒÁ.
129.27. ðÏÓÔÒÏÊÔÅ ÏÂÝÉÅ ËÁÓÁÔÅÌØÎÙÅ Ë Ä×ÕÍ ÄÁÎÎÙÍ ÏËÒÕÖÎÏÓÔÑÍ.
129.28. çÏ×ÏÒÑÔ, ÞÔÏ Ä×Å ÏËÒÕÖÎÏÓÔÉ ËÁÓÁÀÔÓÑ, ÅÓÌÉ ÏÎÉ ÉÍÅÀÔ ÅÄÉÎÓÔ×ÅÎÎÕÀ ÏÂÝÕÀ
ÔÏÞËÕ. äÏËÁÖÉÔÅ, ÞÔÏ Ä×Å ÏËÒÕÖÎÏÓÔÉ ËÁÓÁÀÔÓÑ ÔÏÇÄÁ É ÔÏÌØËÏ ÔÏÇÄÁ, ËÏÇÄÁ ÏÎÉ ËÁÓÁÀÔÓÑ
ÎÅËÏÔÏÒÏÊ ÐÒÑÍÏÊ × ÏÄÎÏÊ É ÔÏÊ ÖÅ ÔÏÞËÅ.
129.29. ðÒÑÍÁÑ, ÐÒÏÈÏÄÑÝÁÑ ÞÅÒÅÚ ÃÅÎÔÒÙ Ä×ÕÈ ÏËÒÕÖÎÏÓÔÅÊ, ÎÁÚÙ×ÁÅÔÓÑ ÉÈ ÌÉÎÉÅÊ
ÃÅÎÔÒÏ×. äÏËÁÖÉÔÅ, ÞÔÏ ÌÉÎÉÑ ÃÅÎÔÒÏ× Ä×ÕÈ ËÁÓÁÀÝÉÈÓÑ ÏËÒÕÖÎÏÓÔÅÊ ÐÒÏÈÏÄÉÔ ÞÅÒÅÚ ÔÏÞËÕ
ÉÈ ËÁÓÁÎÉÑ.

129.30. ÷ ÏÓÔÒÙÊ ÕÇÏÌ, ÒÁ×ÎÙÊ 60 , ×ÐÉÓÁÎÙ Ä×Å ÏËÒÕÖÎÏÓÔÉ, ÉÚ×ÎÅ ËÁÓÁÀÝÉÅÓÑ ÄÒÕÇ
ÄÒÕÇÁ. òÁÄÉÕÓ ÍÅÎØÛÅÊ ÏËÒÕÖÎÏÓÔÉ ÒÁ×ÅÎ r. îÁÊÄÉÔÅ ÒÁÄÉÕÓ ÂÏÌØÛÅÊ ÏËÒÕÖÎÏÓÔÉ.
129.31. ä×Å ÏËÒÕÖÎÏÓÔÉ ËÁÓÁÀÔÓÑ × ÔÏÞËÅ A. ðÒÑÍÁÑ, ÐÒÏÈÏÄÑÝÁÑ ÞÅÒÅÚ ÔÏÞËÕ A, ÐÅÒÅÓÅËÁÅÔ ÜÔÉ ÏËÒÕÖÎÏÓÔÉ ×ÔÏÒÉÞÎÏ × ÔÏÞËÁÈ B É C ÓÏÏÔ×ÅÔÓÔ×ÅÎÎÏ. äÏËÁÖÉÔÅ, ÞÔÏ ËÁÓÁÔÅÌØÎÙÅ,
ÐÒÏ×ÅÄÅÎÎÙÅ Ë ÜÔÉÍ ÏËÒÕÖÎÏÓÔÑÍ × ÔÏÞËÁÈ B É C, ÐÁÒÁÌÌÅÌØÎÙ.
129.32. (Á) ðÏÓÔÒÏÊÔÅ ÏËÒÕÖÎÏÓÔØ, ËÁÓÁÀÝÕÀÓÑ ÄÁÎÎÏÊ ÐÒÑÍÏÊ É ÄÁÎÎÏÊ ÏËÒÕÖÎÏÓÔÉ ×
ÄÁÎÎÏÊ ÎÁ ÎÅÊ ÔÏÞËÅ. (Â) ðÏÓÔÒÏÊÔÅ ÏËÒÕÖÎÏÓÔØ, ËÁÓÁÀÝÕÀÓÑ ÄÁÎÎÏÊ ÏËÒÕÖÎÏÓÔÉ É ÄÁÎÎÏÊ
ÐÒÑÍÏÊ × ÄÁÎÎÏÊ ÎÁ ÎÅÊ ÔÏÞËÅ.
129.33. òÁÓÓÔÏÑÎÉÅ ÍÅÖÄÕ ÃÅÎÔÒÁÍÉ ÎÅÐÅÒÅÓÅËÁÀÝÉÈÓÑ ÏËÒÕÖÎÏÓÔÅÊ ÒÁ×ÎÏ a. äÏËÁÖÉÔÅ, ÞÔÏ ÞÅÔÙÒÅ ÔÏÞËÉ ÐÅÒÅÓÅÞÅÎÉÑ ÏÂÝÉÈ ×ÎÅÛÎÉÈ ËÁÓÁÔÅÌØÎÙÈ Ó ÏÂÝÉÍÉ ×ÎÕÔÒÅÎÎÉÍÉ
ËÁÓÁÔÅÌØÎÙÍÉ ÌÅÖÁÔ ÎÁ ÏÄÎÏÊ ÏËÒÕÖÎÏÓÔÉ É ÎÁÊÄÉÔÅ ÒÁÄÉÕÓ ÜÔÏÊ ÏËÒÕÖÎÏÓÔÉ.
129.18.

* * *
129.34.

þÅÒÅÚ ×ÅÒÛÉÎÕ ÔÒÅÕÇÏÌØÎÉËÁ ÐÒÏ×ÅÄÉÔÅ ÐÒÑÍÕÀ, ÄÅÌÑÝÕÀ ÐÅÒÉÍÅÔÒ ÔÒÅÕÇÏÌØÎÉËÁ ÐÏÐÏ-

ÌÁÍ.

201

ABC, ËÁÓÁÅÔÓÑ ÓÔÏÒÏÎÙ BC × ÔÏÞËÅ M. äÏËÁÖÉÔÅ,
ABM É ACM, ËÁÓÁÀÔÓÑ ÏÔÒÅÚËÁ AM × ÏÄÎÏÊ ÔÏÞËÅ.
îÁ ÓÔÏÒÏÎÁÈ BC, CA É AB ÔÒÅÕÇÏÌØÎÉËÁ ABC ×ÚÑÔÙ (ÓÏÏÔ×ÅÔÓÔ×ÅÎÎÏ) ÔÏÞËÉ A1 , B1 É
C1, ÐÒÉÞÅÍ AC1 = AB1, BA1 = BC1 É CA1 = CB1. äÏËÁÖÉÔÅ, ÞÔÏ A1, B1 É C1 | ÔÏÞËÉ ËÁÓÁÎÉÑ
129.35.

ïËÒÕÖÎÏÓÔØ, ×ÐÉÓÁÎÎÁÑ × ÔÒÅÕÇÏÌØÎÉË

ÞÔÏ ÏËÒÕÖÎÏÓÔÉ, ×ÐÉÓÁÎÎÙÅ × ÔÒÅÕÇÏÌØÎÉËÉ
129.36.

×ÐÉÓÁÎÎÏÊ ÏËÒÕÖÎÏÓÔÉ ÓÏ ÓÔÏÒÏÎÁÍÉ ÔÒÅÕÇÏÌØÎÉËÁ.
129.37.

ÔÏÞËÁÈ.
129.38.

äÁÎÙ ÔÒÉ ÔÏÞËÉ

A, B É C.

ðÏÓÔÒÏÊÔÅ ÔÒÉ ÏËÒÕÖÎÏÓÔÉ, ÐÏÐÁÒÎÏ ËÁÓÁÀÝÉÅÓÑ × ÜÔÉÈ

129.39.

A, B É C.
A, B É C ÐÅÒÅÓÅËÁÀÔÓÑ × ÏÄÎÏÊ ÔÏÞËÅ.

ôÒÉ ÏËÒÕÖÎÏÓÔÉ ÐÏÐÁÒÎÏ ËÁÓÁÀÔÓÑ ÄÒÕÇ ÄÒÕÇÁ ×ÎÅÛÎÉÍ ÏÂÒÁÚÏÍ × ÔÏÞËÁÈ

äÏËÁÖÉÔÅ, ÞÔÏ ËÁÓÁÔÅÌØÎÙÅ Ë ÜÔÉÍ ÏËÒÕÖÎÏÓÔÑÍ × ÔÏÞËÁÈ

äÏËÁÖÉÔÅ, ÞÔÏ × ÞÅÔÙÒÅÈÕÇÏÌØÎÉË, ÓÕÍÍÙ ÐÒÏÔÉ×ÏÌÅÖÁÝÉÈ ÓÔÏÒÏÎ ËÏÔÏÒÏÇÏ ÒÁ×ÎÙ ÍÅÖÄÕ

ÓÏÂÏÊ, ÍÏÖÎÏ ×ÐÉÓÁÔØ ÏËÒÕÖÎÏÓÔØ.

202

130. õÇÌÙ, Ó×ÑÚÁÎÎÙÅ Ó ÏËÒÕÖÎÏÓÔØÀ

ãÅÎÔÒÁÌØÎÙÊ ÕÇÏÌ | ÕÇÏÌ ÍÅÖÄÕ Ä×ÕÍÑ ÒÁÄÉÕÓÁÍÉ. õÇÌÏ×ÁÑ ×ÅÌÉÞÉÎÁ ÄÕÇÉ

^ AB) ÏËÒÕÖÎÏÓÔÉ Ó ÃÅÎÔÒÏÍ O | ÕÇÌÏ×ÁÑ ×ÅÌÉÞÉÎÁ ÓÏÏÔ×ÅÔÓÔ×ÕÀÝÅÇÏ ÃÅÎÔÒÁÌØÎÏÇÏ
AOB. ÷ÐÉÓÁÎÎÙÊ ÕÇÏÌ | ÕÇÏÌ, ×ÅÒÛÉÎÁ ËÏÔÏÒÏÇÏ ÌÅÖÉÔ ÎÁ ÏËÒÕÖÎÏÓÔÉ, Á ÓÔÏÒÏÎÙ

ÞÁÅÔÓÑ
ÕÇÌÁ

AB (ÏÂÏÚÎÁ-

Ñ×ÌÑÀÔÓÑ ÈÏÒÄÁÍÉ.
130.1.

(ôÅÏÒÅÍÁ Ï ×ÐÉÓÁÎÎÏÍ ÕÇÌÅ) ÷ÐÉÓÁÎÎÙÊ ÕÇÏÌ ÒÁ×ÅÎ ÐÏÌÏ×ÉÎÅ ÄÕÇÉ, ÚÁËÌÀÞ£ÎÎÏÊ

ÍÅÖÄÕ ÅÇÏ ÓÔÏÒÏÎÁÍÉ.

A, B, C É D ÐÏÓÌÅÄÏ×ÁÔÅÌØÎÏ ÒÁÓÐÏÌÏÖÅÎÙ ÎÁ ÏËÒÕÖÎÏÓÔÉ. éÚ×ÅÓÔÎÏ, ÞÔÏ
ÕÇÌÏ×ÙÅ ×ÅÌÉÞÉÎÙ ÄÕÇ AB, BC, CD É DA ÏÔÎÏÓÑÔÓÑ ËÁË 1 : 3 : 5 : 6. îÁÊÄÉÔÅ ÕÇÌÙ
ÞÅÔÙÒ£ÈÕÇÏÌØÎÉËÁ ABCD.
130.2.

ôÏÞËÉ

130.3.

äÏËÁÖÉÔÅ, ÞÔÏ Õ ÞÅÔÙÒ£ÈÕÇÏÌØÎÉËÁ, ×ÐÉÓÁÎÎÏÇÏ × ÏËÒÕÖÎÏÓÔØ, ÓÕÍÍÁÐÒÏÔÉ×ÏÐÏ-

ÌÏÖÎÙÈ ÕÇÌÏ× ÒÁ×ÎÁ
130.4.

180.

òÁÓÓÍÏÔÒÉÍ ÞÅÔÙÒÅ ÓÅÇÍÅÎÔÁ, ÏÔÓÅËÁÅÍÙÈ ÏÔ ÏËÒÕÖÎÏÓÔÉ ×ÐÉÓÁÎÎÙÍ × ÎÅÅ ÞÅÔÙ-

ÒÅÈÕÇÏÌØÎÉËÏÍ (É ÎÁÈÏÄÑÝÉÈÓÑ ×ÎÅ ÜÔÏÇÏ ÞÅÔÙÒÅÈÕÇÏÌØÎÉËÁ). îÁÊÄÉÔÅ ÓÕÍÍÕ ÕÇÌÏ×, ×ÐÉÓÁÎÎÙÈ × ÜÔÉ ÓÅÇÍÅÎÔÙ.
130.5.

÷ ËÒÕÇÅ ÐÒÏ×ÅÌÉ ÔÒÉ ÈÏÒÄÙ

äÏËÁÖÉÔÅ, ÞÔÏ

\BMN = \NKC.

AB, BC, CD É ÏÔÍÅÔÉÌÉ ÉÈ ÓÅÒÅÄÉÎÙ: M, N, K.

AA1 É BB1 | ×ÙÓÏÔÙ ÏÓÔÒÏÕÇÏÌØÎÏÇÏ ÔÒÅÕÇÏÌØÎÉËÁ ABC. äÏËÁÖÉÔÅ, ÞÔÏ
\CA1B1 = \CAB.
éÚ ÔÏÞËÉ P, ÒÁÓÐÏÌÏÖÅÎÎÏÊ ×ÎÕÔÒÉ ÏÓÔÒÏÇÏ ÕÇÌÁ BAC, ÏÐÕÝÅÎÙ ÐÅÒÐÅÎÄÉËÕÌÑÒÙ
PC1 É PB1 ÎÁ ÐÒÑÍÙÅ AB É AC. äÏËÁÖÉÔÅ, ÞÔÏ \C1AP = \C1B1P.

ôÒÉ ÐÒÑÍÙÅ, ÐÒÏÈÏÄÑÝÉÅ ÞÅÒÅÚ ÔÏÞËÕ O, ÏÂÒÁÚÕÀÔ ÄÒÕÇ Ó ÄÒÕÇÏÍ ÕÇÌÙ × 60 .
äÏËÁÖÉÔÅ, ÞÔÏ ÐÒÏÅËÃÉÉ ÌÀÂÏÊ ÔÏÞËÉ (ËÒÏÍÅ ÓÁÍÏÊ ÔÏÞËÉ O) ÎÁ ÜÔÉ ÐÒÑÍÙÅ Ñ×ÌÑÀÔÓÑ
130.6.

ðÕÓÔØ

130.7.

130.8.

×ÅÒÛÉÎÁÍÉ ÐÒÁ×ÉÌØÎÏÇÏ ÔÒÅÕÇÏÌØÎÉËÁ.
130.9.

÷ÅÒÛÉÎÙ ÕÇÏÌØÎÉËÁ ÓËÏÌØÚÑÔ ÐÏ ÓÔÏÒÏÎÁÍ ÐÒÑÍÏÇÏ ÕÇÌÁ. îÁÊÄÉÔÅ ÔÒÁÅËÔÏÒÉÀ

×ÅÒÛÉÎÙ ÐÒÑÍÏÇÏ ÕÇÌÁ ÕÇÏÌØÎÉËÁ.

A É B. ðÒÏÄÏÌÖÅÎÉÑ ÈÏÒÄ AC É BD
ÐÅÒ×ÏÊ ÏËÒÕÖÎÏÓÔÉ ÐÅÒÅÓÅËÁÀÔ ×ÔÏÒÕÀ ÏËÒÕÖÎÏÓÔØ × ÔÏÞËÁÈ E É F. äÏËÁÖÉÔÅ, ÞÔÏ ÐÒÑÍÙÅ
CD É EF ÐÁÒÁÌÌÅÌØÎÙ.
130.10.

ä×Å ÏËÒÕÖÎÏÓÔÉ ÐÅÒÅÓÅËÁÀÔÓÑ × ÔÏÞËÁÈ

130.11.

õÇÌÏ×ÙÅ ×ÅÌÉÞÉÎÙ ÐÒÏÔÉ×ÏÐÏÌÏÖÎÙÈ ÄÕÇ, ×ÙÓÅËÁÅÍÙÈ ÎÁ ÏËÒÕÖÎÏÓÔÉ ÐÅÒÅÓÅËÁ-

ÀÝÉÍÉÓÑ ÈÏÒÄÁÍÉ, ÒÁ×ÎÙ
130.12.

É

. îÁÊÄÉÔÅ ÕÇÏÌ ÍÅÖÄÕ ÈÏÒÄÁÍÉ.

õÇÌÏ×ÙÅ ×ÅÌÉÞÉÎÙ ÄÕÇ, ÚÁËÌÀÞ£ÎÎÙÈ ÍÅÖÄÕ Ä×ÕÍÑ ÈÏÒÄÁÍÉ, ÐÒÏÄÏÌÖÅÎÉÑ ËÏÔÏ-

ÒÙÈ ÐÅÒÅÓÅËÁÀÔÓÑ ×ÎÅ ËÒÕÇÁ, ÒÁ×ÎÙ

É

, ÐÒÉ ÜÔÏÍ

ÐÒÏÄÏÌÖÅÎÉÑ ÈÏÒÄ?

>

. ðÏÄ ËÁËÉÍ ÕÇÌÏÍ ÐÅÒÅÓÅËÁÀÔÓÑ

A, B, C, D ÌÅÖÁÔ ÎÁ ÏËÒÕÖÎÏÓÔÉ. ôÏÞËÉ M, N, K, L | ÓÅÒÅÄÉÎÙ ÄÕÇ
AB, BC, CD É DA ÓÏÏÔ×ÅÔÓÔ×ÅÎÎÏ. äÏËÁÖÉÔÅ, ÞÔÏ MK ? NL.
ðÒÏÄÏÌÖÅÎÉÑ ÐÒÏÔÉ×ÏÐÏÌÏÖÎÙÈ ÓÔÏÒÏÎ AB É CD ×ÐÉÓÁÎÎÏÇÏ ÞÅÔÙÒ£ÈÕÇÏÌØÎÉËÁ
ABCD ÐÅÒÅÓÅËÁÀÔÓÑ × ÔÏÞËÅ M, Á ÓÔÏÒÏÎ AD É BC - × ÔÏÞËÅ N. äÏËÁÖÉÔÅ, ÞÔÏ ÂÉÓÓÅËÔÒÉÓÙ
ÕÇÌÏ× AMD É DNC ×ÚÁÉÍÎÏ ÐÅÒÐÅÎÄÉËÕÌÑÒÎÙ.
130.13.

ôÏÞËÉ

130.14.

130.15.

îÁÊÄÉÔÅ ÇÅÏÍÅÔÒÉÞÅÓËÏÅ ÍÅÓÔÏ ÔÏÞÅË, ÉÚ ËÏÔÏÒÙÈ ÄÁÎÎÙÊ ÏÔÒÅÚÏË ×ÉÄÅÎ ÐÏÄ

ÄÁÎÎÙÍ ÕÇÌÏÍ.

203

130.16.

ó ÐÏÍÏÝØÀ ÃÉÒËÕÌÑ É ÌÉÎÅÊËÉ ÐÏÓÔÒÏÊÔÅ ÔÒÅÕÇÏÌØÎÉË ÐÏ ÓÔÏÒÏÎÅ, ÐÒÏÔÉ×ÏÌÅ-

ÖÁÝÅÍÕ ÕÇÌÕ É ×ÙÓÏÔÅ, ÐÒÏ×ÅÄ£ÎÎÏÊ ÉÚ ×ÅÒÛÉÎÙ ÜÔÏÇÏ ÕÇÌÁ.

A É ÃÅÎÔÒ ×ÐÉÓÁÎÎÏÊ ÏËÒÕÖÎÏÓÔÉ ÔÒÅÕÇÏÌØÎÉËÁ
ABC, ×ÔÏÒÉÞÎÏ ÐÅÒÅÓÅËÁÅÔ ÏÐÉÓÁÎÎÕÀ ÏËÒÕÖÎÏÓÔØ ÜÔÏÇÏ ÔÒÅÕÇÏÌØÎÉËÁ × ÔÏÞËÅ M. äÏËÁÖÉÔÅ,
ÞÔÏ ÔÒÅÕÇÏÌØÎÉËÉ BOM É COM | ÒÁ×ÎÏÂÅÄÒÅÎÎÙÅ.
ôÏÞËÁ O | ÃÅÎÔÒ ÏËÒÕÖÎÏÓÔÉ, ÏÐÉÓÁÎÎÏÊ ÏËÏÌÏ ÔÒÅÕÇÏÌØÎÉËÁ ABC, ÐÒÉÞ£Í
\AOC = 60. îÁÊÄÉÔÅ ÕÇÏÌ AMC, ÇÄÅ M | ÃÅÎÔÒ ÏËÒÕÖÎÏÓÔÉ, ×ÐÉÓÁÎÎÏÊ × ÔÒÅÕÇÏÌØÎÉË
ABC.
ôÏÞËÉ A É B | ÆÉËÓÉÒÏ×ÁÎÎÙÅ ÔÏÞËÉ ÏËÒÕÖÎÏÓÔÉ, ÔÏÞËÁ C | ÐÒÏÉÚ×ÏÌØÎÁÑ
130.17.

ðÒÑÍÁÑ, ÐÒÏÈÏÄÑÝÁÑ ÞÅÒÅÚ ÔÏÞËÕ

130.18.

130.19.

ÔÏÞËÁ ÏËÒÕÖÎÏÓÔÉ. îÁÊÄÉÔÅ ÇÅÏÍÅÔÒÉÞÅÓËÏÅ ÍÅÓÔÏ ÔÏÞÅË ÐÅÒÅÓÅÞÅÎÉÑ (Á) ÂÉÓÓÅËÔÒÉÓ; (Â)
×ÙÓÏÔ ÔÒÅÕÇÏÌØÎÉËÁ
130.20.

ABC.

ABC ÐÅÒÅÓÅËÁÀÔ ÏÐÉA1, B1, C1. äÏËÁÖÉÔÅ, ÞÔÏ ×ÙÓÏÔÙ ÔÒÅÕÇÏÌØÎÉËÁ

ðÒÏÄÏÌÖÅÎÉÑ ÂÉÓÓÅËÔÒÉÓ ÏÓÔÒÏÕÇÏÌØÎÏÇÏ ÔÒÅÕÇÏÌØÎÉËÁ

ÓÁÎÎÕÀ ×ÏËÒÕÇ ÎÅÇÏ ÏËÒÕÖÎÏÓÔØ × ÔÏÞËÁÈ

A1B1C1 ÌÅÖÁÔ ÎÁ ÐÒÑÍÙÈ AA1 , BB1, CC1.

ABC ÐÅÒÅÓÅËÁÀÔ ÏÐÉÓÁÎÎÕÀ
A1, B1, C1. äÏËÁÖÉÔÅ, ÞÔÏ ÂÉÓÓÅËÔÒÉÓÙ ÔÒÅÕÇÏÌØÎÉËÁ A1 B1 C1 ÌÅÖÁÔ ÎÁ ÐÒÑÍÙÈ AA1 , BB1, CC1.
÷Ï ×ÐÉÓÁÎÎÏÍ ÞÅÔÙÒ£ÈÕÇÏÌØÎÉËÅ ABCD ÉÚ×ÅÓÔÎÙ ÕÇÌÙ \DAB = , \ABC =
É ÕÇÏÌ \BKC =
(ÚÄÅÓØ K | ÔÏÞËÁ ÐÅÒÅÓÅÞÅÎÉÑ ÄÉÁÇÏÎÁÌÅÊ). îÁÊÄÉÔÅ ÕÇÏÌ ACD.
130.21.

ðÒÏÄÏÌÖÅÎÉÑ ×ÙÓÏÔ ÏÓÔÒÏÕÇÏÌØÎÏÇÏ ÔÒÅÕÇÏÌØÎÉËÁ

ÏËÒÕÖÎÏÓÔØ ÜÔÏÇÏ ÔÒÅÕÇÏÌØÎÉËÁ × ÔÏÞËÁÈ
130.22.

130.23.

ÒÁ×ÎÁ

äÏËÁÖÉÔÅ, ÞÔÏ ÏËÏÌÏ ÞÅÔÙÒ£ÈÕÇÏÌØÎÉËÁ, ÓÕÍÍÁ ÐÒÏÔÉ×ÏÐÏÌÏÖÎÙÈ ÕÇÌÏ× ËÏÔÏÒÏÇÏ

180, ÍÏÖÎÏ ÏÐÉÓÁÔØ ÏËÒÕÖÎÏÓÔØ.

130.24.

äÏËÁÖÉÔÅ, ÞÔÏ ÔÏÞËÁ, ÓÉÍÍÅÔÒÉÞÎÁÑ ÔÏÞËÅ ÐÅÒÅÓÅÞÅÎÉÑ ×ÙÓÏÔ (ÏÒÔÏÃÅÎÔÒÕ) ÔÒÅ-

ÕÇÏÌØÎÉËÁ ÏÔÎÏÓÉÔÅÌØÎÏ ÏÄÎÏÊ ÉÚ ÓÔÏÒÏÎ, ÌÅÖÉÔ ÎÁ ÏÐÉÓÁÎÎÏÊ ÏËÒÕÖÎÏÓÔÉ.

O | ÃÅÎÔÒ ÏÐÉÓÁÎÎÏÊ ×ÏËÒÕÇ ÔÒÅÕÇÏÌØÎÉËÁ ABC ÏËÒÕÖÎÏÓÔÉ, AH |
×ÙÓÏÔÁ. äÏËÁÖÉÔÅ, ÞÔÏ \BAH = \OAC.
ðÕÓÔØ AA1 É BB1 | ×ÙÓÏÔÙ ÏÓÔÒÏÕÇÏÌØÎÏÇÏ ÔÒÅÕÇÏÌØÎÉËÁ ABC, ÔÏÞËÁ O |
ÃÅÎÔÒ ÏÐÉÓÁÎÎÏÊ ÏËÒÕÖÎÏÓÔÉ. äÏËÁÖÉÔÅ, ÞÔÏ CO ? A1 B1 .
þÅÔÙÒ£ÈÕÇÏÌØÎÉË ABCD, ÄÉÁÇÏÎÁÌÉ ËÏÔÏÒÏÇÏ ×ÚÁÉÍÎÏ ÐÅÒÐÅÎÄÉËÕÌÑÒÎÙ, ×ÐÉÓÁÎ × ÏËÒÕÖÎÏÓÔØ. ðÅÒÐÅÎÄÉËÕÌÑÒÙ, ÏÐÕÝÅÎÎÙÅ ÎÁ ÓÔÏÒÏÎÕ AD ÉÚ ×ÅÒÛÉÎ B É C, ÐÅÒÅÓÅËÁÀÔ
ÄÉÁÇÏÎÁÌÉ AC É BD × ÔÏÞËÁÈ E É F ÓÏÏÔ×ÅÔÓÔ×ÅÎÎÏ. éÚ×ÅÓÔÎÏ, ÞÔÏ BC = 1. îÁÊÄÉÔÅ EF.
130.25.

ðÕÓÔØ

130.26.

130.27.

130.28.

ðÏÓÔÒÏÊÔÅ Ó ÐÏÍÏÝØÀ ÃÉÒËÕÌÑ É ÌÉÎÅÊËÉ ÔÒÅÕÇÏÌØÎÉË ÐÏ ÔÏÞËÁÍ ÐÅÒÅÓÅÞÅÎÉÑ

Ó ÏÐÉÓÁÎÎÏÊ ÏËÒÕÖÎÏÓÔØÀ ÐÒÏÄÏÌÖÅÎÉÊ ÅÇÏ ×ÙÓÏÔÙ, ÍÅÄÉÁÎÙ É ÂÉÓÓÅËÔÒÉÓÙ, ÐÒÏ×ÅÄ£ÎÎÙÈ
ÉÚ ÏÄÎÏÊ ×ÅÒÛÉÎÙ.
130.29.

äÏËÁÖÉÔÅ, ÞÔÏ ÕÇÏÌ ÍÅÖÄÕ ËÁÓÁÔÅÌØÎÏÊ É ÈÏÒÄÏÊ, ÐÒÏ×ÅÄÅÎÎÏÊ ÞÅÒÅÚ ÔÏÞËÕ ËÁ-

ÓÁÎÉÑ, ÒÁ×ÅÎ ÐÏÌÏ×ÉÎÅ ÕÇÌÏ×ÏÊ ×ÅÌÉÞÉÎÙ ÄÕÇÉ, ÚÁËÌÀÞÅÎÎÏÊ ÍÅÖÄÕ ÎÉÍÉ.

K É M, ÐÒÏ×ÅÄÅÎÁ ÏÂÝÁÑ ËÁÓÁÔÅÌØÎÁÑ. äÏËÁÖÉÔÅ, ÞÔÏ ÅÓÌÉ A É B - ÔÏÞËÉ ËÁÓÁÎÉÑ, ÔÏ \AMB + \AKB = 180.
ëÁÓÁÔÅÌØÎÁÑ × ÔÏÞËÅ A Ë ÏÐÉÓÁÎÎÏÊ ÏËÒÕÖÎÏÓÔÉ ÔÒÅÕÇÏÌØÎÉËÁ ABC ÐÅÒÅÓÅËÁÅÔ
ÐÒÑÍÕÀ BC × ÔÏÞËÅ E; ÏÔÒÅÚÏË AD | ÂÉÓÓÅËÔÒÉÓÁ ÔÒÅÕÇÏÌØÎÉËÁ ABC. äÏËÁÖÉÔÅ, ÞÔÏ AE =
ED.
ä×Å ÏËÒÕÖÎÏÓÔÉ ÐÅÒÅÓÅËÁÀÔÓÑ × ÔÏÞËÁÈ A É B. þÅÒÅÚ ÔÏÞËÕ K ÐÅÒ×ÏÊ ÏËÒÕÖÎÏÓÔÉ
ÐÒÏ×ÏÄÑÔÓÑ ÐÒÑÍÙÅ KA É KB, ÐÅÒÅÓÅËÁÀÝÉÅ ×ÔÏÒÕÀ ÏËÒÕÖÎÏÓÔØ × ÔÏÞËÁÈ P É Q. äÏËÁÖÉÔÅ,
ÞÔÏ ÈÏÒÄÁ PQ ×ÔÏÒÏÊ ÏËÒÕÖÎÏÓÔÉ ÐÅÒÐÅÎÄÉËÕÌÑÒÎÁ ÄÉÁÍÅÔÒÕ KM ÐÅÒ×ÏÊ ÏËÒÕÖÎÏÓÔÉ.
130.30.

ë Ä×ÕÍ ÏËÒÕÖÎÏÓÔÑÍ, ÐÅÒÅÓÅËÁÀÝÉÍÓÑ × ÔÏÞËÁÈ

130.31.

130.32.

204

ABC ÏÐÉÓÁÎÁ ÏËÒÕÖÎÏÓÔØ, É ÎÁ ÄÕÇÅ BC
×ÚÑÔÁ ÐÒÏÉÚ×ÏÌØÎÁÑ ÔÏÞËÁ M. äÏËÁÖÉÔÅ, ÞÔÏ AM = BM + CM.
ôÒÅÕÇÏÌØÎÉË Ó ×ÅÒÛÉÎÁÍÉ × ÏÓÎÏ×ÁÎÉÑÈ ×ÙÓÏÔ ÔÒÅÕÇÏÌØÎÉËÁ ABC ÎÁÚÙ×ÁÅÔÓÑ
ÏÒÔÏÔÒÅÕÇÏÌØÎÉËÏÍ ÔÒÅÕÇÏÌØÎÉËÁ ABC. äÏËÁÖÉÔÅ, ÞÔÏ ×ÙÓÏÔÙ ÔÒÅÕÇÏÌØÎÉËÁ ABC Ñ×ÌÑÀÔÓÑ
ïËÏÌÏ ÒÁ×ÎÏÓÔÏÒÏÎÎÅÇÏ ÔÒÅÕÇÏÌØÎÉËÁ

130.33.

130.34.

ÂÉÓÓÅËÔÒÉÓÁÍÉ ÅÇÏ ÏÒÔÏÔÒÅÕÇÏÌØÎÉËÁ.

* * *

A ÐÒÏ×ÅÄÅÎÙ Ë ÏËÒÕÖÎÏÓÔÉ Ä×Å ËÁÓÁÔÅÌØÎÙÅ AP É AQ (ÔÏÞËÉ P É Q | ÔÏÞËÉ
AKL (ÔÏÞËÁ K ÌÅÖÉÔ ÍÅÖÄÕ A É L). ðÕÓÔØ M | ÓÅÒÅÄÉÎÁ ÏÔÒÅÚËÁ KL. äÏËÁÖÉÔÅ,
ÞÔÏ \AMP = \AMQ.
éÚ ÔÏÞËÉ

130.35.

ËÁÓÁÎÉÑ) É ÓÅËÕÝÁÑ

äÏËÁÖÉÔÅ, ÞÔÏ ÏÓÎÏ×ÁÎÉÑ ÐÅÒÐÅÎÄÉËÕÌÑÒÏ×, ÏÐÕÝÅÎÎÙÈ ÉÚ ÐÒÏÉÚ×ÏÌØÎÏÊ ÔÏÞËÉ ÏÐÉÓÁÎ-

130.36.

ÎÏÊ ÏËÒÕÖÎÏÓÔÉ ÎÁ ÓÔÏÒÏÎÙ ÔÒÅÕÇÏÌØÎÉËÁ (ÉÌÉ ÉÈ ÐÒÏÄÏÌÖÅÎÉÑ), ÌÅÖÁÔ ÎÁ ÏÄÎÏÊ ÐÒÑÍÏÊ (ÐÒÑÍÁÑ

óÉÍÓÏÎÁ ).

AMB.

M. ðÕÓÔØ AB | ÈÏÒÄÁ ÂÏÌØÛÅÊ
T . äÏËÁÖÉÔÅ, ÞÔÏ MT | ÂÉÓÓÅËÔÒÉÓÁ ÕÇÌÁ

ä×Å ÏËÒÕÖÎÏÓÔÉ ËÁÓÁÀÔÓÑ ×ÎÕÔÒÅÎÎÉÍ ÏÂÒÁÚÏÍ × ÔÏÞËÅ

130.37.

ÏËÒÕÖÎÏÓÔÉ, ËÁÓÁÀÝÁÑÓÑ ÍÅÎØÛÅÊ ÏËÒÕÖÎÏÓÔÉ × ÔÏÞËÅ

A É B. þÅÒÅÚ ÔÏÞËÕ B ÐÒÏ×ÏÄÉÔÓÑ ÐÒÑÍÁÑ, ÐÅÒÅÓÅËÁÀÝÁÑ ÏËÒÕÖÎÏÓÔÉ × ÔÏÞËÁÈ C É D, Á ÚÁÔÅÍ ÞÅÒÅÚ ÔÏÞËÉ C É D ÐÒÏ×ÏÄÑÔÓÑ ËÁÓÁÔÅÌØÎÙÅ Ë ÜÔÉÍ
ÏËÒÕÖÎÏÓÔÑÍ. äÏËÁÖÉÔÅ, ÞÔÏ ÔÏÞËÉ A, C, D É ÔÏÞËÁ P ÐÅÒÅÓÅÞÅÎÉÑ ËÁÓÁÔÅÌØÎÙÈ ÌÅÖÁÔ ÎÁ ÏÄÎÏÊ
130.38.

ä×Å ÏËÒÕÖÎÏÓÔÉ ÐÅÒÅÓÅËÁÀÔÓÑ × ÔÏÞËÁÈ

ÏËÒÕÖÎÏÓÔÉ.

S2 ÐÒÏÈÏÄÉÔ ÞÅÒÅÚ ÃÅÎÔÒ O ÏËÒÕÖÎÏÓÔÉ S1 É ÐÅÒÅÓÅËÁÅÔ ÅÅ × ÔÏÞËÁÈ A É B.
A ÐÒÏ×ÅÄÅÎÁ ËÁÓÁÔÅÌØÎÁÑ Ë ÏËÒÕÖÎÏÓÔÉ S2. ôÏÞËÁ D | ×ÔÏÒÁÑ ÔÏÞËÁ ÐÅÒÅÓÅÞÅÎÉÑ ÜÔÏÊ
ËÁÓÁÔÅÌØÎÏÊ Ó ÏËÒÕÖÎÏÓÔØÀ S1 . äÏËÁÖÉÔÅ, ÞÔÏ AD = AB.
130.39.

ïËÒÕÖÎÏÓÔØ

þÅÒÅÚ ÔÏÞËÕ
130.40.

ôÏÞËÉ ËÁÓÁÎÉÑ ×ÐÉÓÁÎÎÏÇÏ × ÄÁÎÎÙÊ ÔÒÅÕÇÏÌØÎÉË ËÒÕÇÁ ÓÏÅÄÉÎÅÎÙ ÏÔÒÅÚËÁÍÉ É × ÐÏÌÕ-

ÞÅÎÎÏÍ ÔÒÅÕÇÏÌØÎÉËÅ ÐÒÏ×ÅÄÅÎÙ ×ÙÓÏÔÙ. äÏËÁÖÉÔÅ, ÞÔÏ ÐÒÑÍÙÅ, ÓÏÅÄÉÎÑÀÝÉÅ ÏÓÎÏ×ÁÎÉÑ ÜÔÉÈ ×ÙÓÏÔ,
ÐÁÒÁÌÌÅÌØÎÙ ÓÔÏÒÏÎÁÍ ÉÓÈÏÄÎÏÇÏ ÔÒÅÕÇÏÌØÎÉËÁ.

AD É BC ÐÅÒÅÓÅËÁÀÔÓÑ × ÔÏÞËÅ O. äÏËÁÖÉÔÅ, ÞÔÏ
AOD É BOC, ËÁÓÁÀÔÓÑ ÄÒÕÇ ÄÒÕÇÁ.
ïËÒÕÖÎÏÓÔÉ S1 É S2 ÐÅÒÅÓÅËÁÀÔÓÑ × ÔÏÞËÁÈ A É P. þÅÒÅÚ ÔÏÞËÕ A ÐÒÏ×ÅÄÅÎÁ ËÁÓÁÔÅÌØÎÁÑ
AB Ë ÏËÒÕÖÎÏÓÔÉ S1 , Á ÞÅÒÅÚ ÔÏÞËÕ P | ÐÒÑÍÁÑ CD, ÐÁÒÁÌÌÅÌØÎÁÑ ÐÒÑÍÏÊ AB (ÔÏÞËÉ B É C ÌÅÖÁÔ
ÎÁ S2 , ÔÏÞËÁ D | ÎÁ S1 ). äÏËÁÖÉÔÅ, ÞÔÏ ABCD | ÐÁÒÁÌÌÅÌÏÇÒÁÍÍ.
ïËÒÕÖÎÏÓÔØ S1 ËÁÓÁÅÔÓÑ ÓÔÏÒÏÎ ÕÇÌÁ ABC × ÔÏÞËÁÈ A É C. ïËÒÕÖÎÏÓÔØ S2 ËÁÓÁÅÔÓÑ
ÐÒÑÍÏÊ AC × ÔÏÞËÅ C É ÐÒÏÈÏÄÉÔ ÞÅÒÅÚ ÔÏÞËÕ B. ïËÒÕÖÎÏÓÔØ S1 ÏÎÁ ÐÅÒÅÓÅËÁÅÔ × ÔÏÞËÅ M. äÏËÁÖÉÔÅ,
ÞÔÏ ÐÒÑÍÁÑ AM ÄÅÌÉÔ ÏÔÒÅÚÏË BC ÐÏÐÏÌÁÍ.
ë Ä×ÕÍ ÏËÒÕÖÎÏÓÔÑÍ ÒÁÚÌÉÞÎÏÇÏ ÒÁÄÉÕÓÁ ÐÒÏ×ÅÄÅÎÙ ÏÂÝÉÅ ×ÎÅÛÎÉÅ ËÁÓÁÔÅÌØÎÙÅ AB
É CD. äÏËÁÖÉÔÅ, ÞÔÏ ÞÅÔÙÒÅÈÕÇÏÌØÎÉË ABCD ÏÐÉÓÁÎÎÙÊ ÔÏÇÄÁ É ÔÏÌØËÏ ÔÏÇÄÁ, ËÏÇÄÁ ÏËÒÕÖÎÏÓÔÉ
130.41.

äÉÁÇÏÎÁÌÉ ÔÒÁÐÅÃÉÉ Ó ÏÓÎÏ×ÁÎÉÑÍÉ

ÏËÒÕÖÎÏÓÔÉ, ÏÐÉÓÁÎÎÙÅ ÏËÏÌÏ ÔÒÅÕÇÏÌØÎÉËÏ×
130.42.

130.43.

130.44.

ËÁÓÁÀÔÓÑ.

205

131. ôÅÏÒÅÍÁ æÁÌÅÓÁ

131.1.

(ôÅÏÒÅÍÁ æÁÌÅÓÁ) åÓÌÉ ÎÁ ÏÄÎÏÊ ÉÚ ÓÔÏÒÏÎ ÕÇÌÁ ÏÔÌÏÖÉÔØ ÒÁ×ÎÙÅ ÏÔÒÅÚËÉ É ÞÅÒÅÚ

ÉÈ ËÏÎÃÙ ÐÒÏ×ÅÓÔÉ ÐÁÒÁÌÌÅÌØÎÙÅ ÐÒÑÍÙÅ, ÐÅÒÅÓÅËÁÀÝÉÅ ×ÔÏÒÕÀ ÓÔÏÒÏÎÕ ÕÇÌÁ, ÔÏ ÎÁ ×ÔÏÒÏÊ
ÓÔÏÒÏÎÅ ÏÂÒÁÚÕÀÔÓÑ ÔÁËÖÅ ÒÁ×ÎÙÅ ÏÔÒÅÚËÉ.
131.2.

ó ÐÏÍÏÝØÀ ÃÉÒËÕÌÑ É ÌÉÎÅÊËÉ ÒÁÚÄÅÌÉÔÅ ÄÁÎÎÙÊ ÏÔÒÅÚÏË ÎÁ

131.3.

÷ÅÒÎÁ ÌÉ ÔÅÏÒÅÍÁ, ÏÂÒÁÔÎÁÑ ÔÅÏÒÅÍÅ æÁÌÅÓÁ?

n ÒÁ×ÎÙÈ ÞÁÓÔÅÊ.

(ïÂÏÂÝÅÎÉÅ ÔÅÏÒÅÍÙ æÁÌÅÓÁ) ðÁÒÁÌÌÅÌØÎÙÅ ÐÒÑÍÙÅ, ÐÅÒÅÓÅËÁÀÝÉÅ ÓÔÏÒÏÎÙ

131.4.

ÕÇÌÁ, ÏÔÓÅËÁÀÔ ÎÁ ÎÉÈ ÐÒÏÐÏÒÃÉÏÎÁÌØÎÙÅ ÏÔÒÅÚËÉ. äÏËÁÖÉÔÅ ÜÔÕ ÔÅÏÒÅÍÕ ÄÌÑ ÓÌÕÞÁÑ, ËÏÇÄÁ

e ÎÁÚÙ×ÁÅÔÓÑ
a É b, ÅÓÌÉ ÏÎ ÃÅÌÏÅ ÞÉÓÌÏ ÒÁÚ ÕËÌÁÄÙ×ÁÅÔÓÑ × ÜÔÉÈ ÏÔÒÅÚËÁÈ.)

ÏÔÒÅÚËÉ ÎÁ ÏÄÎÏÊ ÉÚ ÓÔÏÒÏÎ ÕÇÌÁ ÓÏÉÚÍÅÒÉÍÙ, Ô.Å. ÉÍÅÀÔ ÏÂÝÕÀ ÍÅÒÕ. (ïÔÒÅÚÏË
ÏÂÝÅÊ ÍÅÒÏÊ ÏÔÒÅÚËÏ×
131.5.

äÁÎ ÕÇÏÌ É ÔÏÞËÁ ×ÎÕÔÒÉ ÎÅÇÏ. ðÒÏ×ÅÄÉÔÅ ÞÅÒÅÚ ÜÔÕ ÔÏÞËÕ ÐÒÑÍÕÀ, ÏÔÒÅÚÏË ËÏÔÏ-

ÒÏÊ, ÚÁËÌÀÞÅÎÎÙÊ ×ÎÕÔÒÉ ÄÁÎÎÏÇÏ ÕÇÌÁ, ÄÅÌÉÌÓÑ ÂÙ ÄÁÎÎÏÊ ÔÏÞËÏÊ × ÚÁÄÁÎÎÏÍ ÏÔÎÏÛÅÎÉÉ.

a, b É c. ðÏÓÔÒÏÊÔÅ ÔÁËÏÊ ÏÔÒÅÚÏË x, ÞÔÏ x a b c
óÔÏÒÏÎÁ AB ÔÒÅÕÇÏÌØÎÉËÁ ABC ÒÁÚÄÅÌÅÎÁ ÎÁ ÔÒÉ ÒÁ×ÎÙÅ ÞÁÓÔÉ É ÞÅÒÅÚ ÔÏÞËÉ
ÄÅÌÅÎÉÑ ÐÒÏ×ÅÄÅÎÙ ÐÒÑÍÙÅ, ÐÁÒÁÌÌÅÌØÎÙÅ ÓÔÏÒÏÎÅ BC. îÁÊÄÉÔÅ ÏÔÒÅÚËÉ ÜÔÉÈ ÐÒÑÍÙÈ, ÚÁËÌÀÞÅÎÎÙÅ ×ÎÕÔÒÉ ÔÒÅÕÇÏÌØÎÉËÁ, ÅÓÌÉ BC
12.
ëÁÖÄÁÑ ÉÚ ÓÔÏÒÏÎ AB É AC ÔÒÅÕÇÏÌØÎÉËÁ ABC ÒÁÚÄÅÌÅÎÁ ÓÏÏÔ×ÅÔÓÔ×ÅÎÎÏ ÔÏÞËÁÍÉ
M É N × ÏÔÎÏÛÅÎÉÉ 2:3, ÓÞÉÔÁÑ ÏÔ ÔÏÞËÉ A. ( ) äÏËÁÖÉÔÅ, ÞÔÏ MNkBC. ( ) îÁÊÄÉÔÅ MN,
ÅÓÌÉ BC
20.
îÁ ÓÔÏÒÏÎÁÈ AB É BC ÔÒÅÕÇÏÌØÎÉËÁ ABC ÒÁÓÐÏÌÏÖÅÎÙ ÔÏÞËÉ M É N ÓÏÏÔ×ÅÔÓÔ×ÅÎÎÏ, ÐÒÉÞÅÍ AM
MB AN NC m n (m É n | ÎÁÔÕÒÁÌØÎÙÅ). îÁÊÄÉÔÅ
ÏÔÎÏÛÅÎÉÅ MN BC.
ïÓÎÏ×ÁÎÉÑ ÔÒÁÐÅÃÉÉ ÒÁ×ÎÙ a É b. ëÁÖÄÁÑ ÂÏËÏ×ÁÑ ÓÔÏÒÏÎÁ ÔÒÁÐÅÃÉÉ ÒÁÚÄÅÌÅÎÁ
131.6.

äÁÎÙ ÏÔÒÅÚËÉ

:

=

:

131.7.

=

131.8.

Á

Â

=

131.9.

:

=

:

=

:

:

131.10.

ÎÁ ÔÒÉ ÒÁ×ÎÙÈ ÞÁÓÔÉ. îÁÊÄÉÔÅ ÏÔÒÅÚËÉ, ÓÏÅÄÉÎÑÀÝÉÅ ÓÏÏÔ×ÅÔÓÔ×ÕÀÝÉÅ ÔÏÞËÉ ÄÅÌÅÎÉÑ.
131.11.

âÏËÏ×ÁÑ ÓÔÏÒÏÎÁ ÔÒÁÐÅÃÉÉ ÒÁÚÄÅÌÅÎÁ ÎÁ ÐÑÔØ ÒÁ×ÎÙÈ ÞÁÓÔÅÊ, É ÞÅÒÅÚ ÔÒÅÔØÀ

ÔÏÞËÕ ÄÅÌÅÎÉÑ (ÓÞÉÔÁÑ ÏÔ ×ÅÒÛÉÎÙ ÍÅÎØÛÅÇÏ ÏÓÎÏ×ÁÎÉÑ) ÐÒÏ×ÅÄÅÎÁ ÐÒÑÍÁÑ, ÐÁÒÁÌÌÅÌØÎÁÑ
ÏÓÎÏ×ÁÎÉÑÍ ÔÒÁÐÅÃÉÉ. îÁÊÄÉÔÅ ÏÔÒÅÚÏË ÐÒÑÍÏÊ, ÚÁËÌÀÞÅÎÎÙÊ ÍÅÖÄÕ ÓÔÏÒÏÎÁÍÉ ÔÒÁÐÅÃÉÉ,
ÅÓÌÉ ÏÓÎÏ×ÁÎÉÑ ÔÒÁÐÅÃÉÉ ÒÁ×ÎÙ

a É b.

ëÁÖÄÁÑ ÉÚ ÂÏËÏ×ÙÈ ÓÔÏÒÏÎ ÔÒÁÐÅÃÉÉ ÒÁÚÄÅÌÅÎÁ ÎÁ 5 ÒÁ×ÎÙÈ ÞÁÓÔÅÊ. ðÕÓÔØ M É
N | ×ÔÏÒÙÅ ÔÏÞËÉ ÄÅÌÅÎÉÑ ÎÁ ÂÏËÏ×ÙÈ ÓÔÏÒÏÎÁÈ, ÓÞÉÔÁÑ ÏÔ ×ÅÒÛÉÎ ÍÅÎØÛÅÇÏ ÏÓÎÏ×ÁÎÉÑ.
îÁÊÄÉÔÅ MN, ÅÓÌÉ ÏÓÎÏ×ÁÎÉÑ ÔÒÁÐÅÃÉÉ ÒÁ×ÎÙ a É b.
ïÓÎÏ×ÁÎÉÑ AD É BC ÔÒÁÐÅÃÉÉ ABCD ÒÁ×ÎÙ ÓÏÏÔ×ÅÔÓÔ×ÅÅÎÏ a É b. äÉÁÇÏÎÁÌØ
AC ÒÁÚÄÅÌÅÎÁ ÎÁ ÔÒÉ ÒÁ×ÎÙÅ ÞÁÓÔÉ É ÞÅÒÅÚ ÂÌÉÖÁÊÛÕÀ Ë A ÔÏÞËÕ ÄÅÌÅÎÉÑ M ÐÒÏ×ÅÄÅ131.12.

131.13.

ÎÁ ÐÒÑÍÁÑ, ÐÁÒÁÌÌÅÌØÎÁÑ ÏÓÎÏ×ÁÎÉÑÍ. îÁÊÄÉÔÅ ÏÔÒÅÚÏË ÜÔÏÊ ÐÒÑÍÏÊ, ÚÁËÌÀÞÅÎÎÙÊ ÍÅÖÄÕ

ÄÉÁÇÏÎÁÌÑÍÉ.
131.14.

ÐÒÉÞÅÍ

AM MC
a É b.
:

ÓÏÏÔ×ÅÔÓÔ×ÅÎÎÏ
131.15.

AC É BD ÔÒÁÐÅÃÉÉ ABCD ×ÚÑÔÙ ÓÏÏÔ×ÅÔÓÔ×ÅÎÎÏ ÔÏÞËÉ M É N,
DN NB 1 4. îÁÊÄÉÔÅ MN, ÅÓÌÉ ÏÓÎÏ×ÁÎÉÑ AD É BC ÒÁ×ÎÙ

îÁ ÄÉÁÇÏÎÁÌÑÈ
=

:

=

:

þÅÒÅÚ ÔÏÞËÕ ÎÁ ÓÔÏÒÏÎÅ ÞÅÔÙÒÅÈÕÇÏÌØÎÉËÁ ÐÒÏ×ÅÄÅÎÁ ÐÒÑÍÁÑ, ÐÁÒÁÌÌÅÌØÎÁÑ ÄÉÁ-

ÇÏÎÁÌÉ, ÄÏ ÐÅÒÅÓÅÞÅÎÉÑ Ó ÓÏÓÅÄÎÅÊ ÓÔÏÒÏÎÏÊ ÞÅÔÙÒÅÈÕÇÏÌØÎÉËÁ. þÅÒÅÚ ÐÏÌÕÞÅÎÎÕÀ ÔÏÞËÕ
ÐÒÏ×ÅÄÅÎÁ ÐÒÑÍÁÑ, ÐÁÒÁÌÌÅÌØÎÁÑ ÄÒÕÇÏÊ ÄÉÁÇÏÎÁÌÉ, É Ô.Ä. äÏËÁÖÉÔÅ, ÞÔÏ ÐÑÔÁÑ ÔÏÞËÁ, ÐÏÌÕÞÅÎÎÁÑ ÔÁËÉÍ ÓÐÏÓÏÂÏÍ, ÓÏ×ÐÁÄÅÔ Ó ÉÓÈÏÄÎÏÊ.

206

131.16.

þÅÒÅÚ ÔÏÞËÕ ÎÁ ÓÔÏÒÏÎÅ ÔÒÅÕÇÏÌØÎÉËÁ ÐÒÏ×ÅÄÅÎÁ ÐÒÑÍÁÑ, ÐÁÒÁÌÌÅÌØÎÁÑ ÄÒÕÇÏÊ

ÓÔÏÒÏÎÅ, ÄÏ ÐÅÒÅÓÅÞÅÎÉÑ Ó ÔÒÅÔØÅÊ ÓÔÏÒÏÎÏÊ ÔÒÅÕÇÏÌØÎÉËÁ. þÅÒÅÚ ÐÏÌÕÞÅÎÎÕÀ ÔÏÞËÕ ÐÒÏ×ÅÄÅÎÁ ÐÒÑÍÁÑ, ÐÁÒÁÌÌÅÌØÎÁÑ ÐÅÒ×ÏÊ ÓÔÏÒÏÎÅ ÔÒÅÕÇÏÌØÎÉËÁ É Ô.Ä. äÏËÁÖÉÔÅ, ÞÔÏ (Á) ÅÓÌÉ
ÉÓÈÏÄÎÁÑ ÔÏÞËÁ ÓÏÐÁÄÁÅÔ Ó ÓÅÒÅÄÉÎÏÊ ÓÔÏÒÏÎÙ ÔÒÅÕÇÏÌØÎÉËÁ, ÔÏ ÞÅÔ×ÅÒÔÁÑ ÔÏÞËÁ, ÐÏÌÕÞÅÎÎÁÑ
ÔÁËÉÍ ÓÐÏÓÏÂÏÍ, ÓÏ×ÐÁÄÅÔ Ó ÉÓÈÏÄÎÏÊ; (Â) ÅÓÌÉ ÉÓÈÏÄÎÁÑ ÔÏÞËÁ ÏÔÌÉÞÎÁ ÏÔ ÓÅÒÅÄÉÎÙ ÓÔÏÒÏÎÙ
ÔÒÅÕÇÏÌØÎÉËÁ, ÔÏ ÓÅÄØÍÁÑ ÔÏÞËÁ, ÐÏÌÕÞÅÎÎÁÑ ÔÁËÉÍ ÓÐÏÓÏÂÏÍ, ÓÏ×ÐÁÄÅÔ Ó ÉÓÈÏÄÎÏÊ.
131.17.

äÏËÁÖÉÔÅ, ÞÔÏ ÄÉÁÇÏÎÁÌÉ ÔÒÁÐÅÃÉÉ ÄÅÌÑÔÓÑ ÔÏÞËÏÊ ÐÅÒÅÓÅÞÅÎÉÑ ÎÁ ÏÔÒÅÚËÉ,

ÐÒÏÐÏÒÃÉÏÎÁÌØÎÙÅ ÏÓÎÏ×ÁÎÉÑÍ.
131.18.

ëÁÖÄÁÑ ÓÔÏÒÏÎÁ ×ÙÐÕËÌÏÇÏ ÞÅÔÙÒÅÈÕÇÏÌØÎÉËÁ ÐÏÄÅÌÅÎÁ ÎÁ ÔÒÉ ÒÁ×ÎÙÅ ÞÁÓÔÉ.

óÏÏÔ×ÅÔÓÔ×ÕÀÝÉÅ ÔÏÞËÉ ÄÅÌÅÎÉÑ ÎÁ ÐÒÏÔÉ×ÏÐÏÌÏÖÎÙÈ ÓÔÏÒÏÎÁÈ ÓÏÅÄÉÎÅÎÙ ÏÔÒÅÚËÁÍÉ. äÏËÁÖÉÔÅ, ÞÔÏ ÜÔÉ ÏÔÒÅÚËÉ ÄÅÌÑÔ ÄÒÕÇ ÄÒÕÇÁ ÎÁ ÔÒÉ ÒÁ×ÎÙÅ ÞÁÓÔÉ.

ABC ÔÏÞËÁ K ÎÁ ÓÔÏÒÏÎÅ AB É ÔÏÞËÁ M ÎÁ ÓÔÏÒÏÎÅ AC ÒÁÓAK KB 3 2, Á AM MC 4 5. îÁÊÄÉÔÅ ÏÔÎÏÛÅÎÉÅ, × ËÏÔÏÒÏÍ
ÐÒÑÍÁÑ, ÐÒÏÈÏÄÑÝÁÑ ÞÅÒÅÚ ÔÏÞËÕ K ÐÁÒÁÌÌÅÌØÎÏ ÓÔÏÒÏÎÅ BC, ÄÅÌÉÔ ÏÔÒÅÚÏË BM.
÷ ÔÒÅÕÇÏÌØÎÉËÅ ABC ÔÏÞËÁ M ÌÅÖÉÔ ÎÁ ÓÔÏÒÏÎÅ AC, Á ÔÏÞËÁ L ÎÁ ÓÔÏÒÏÎÅ BC
ÒÁÓÐÏÌÏÖÅÎÁ ÔÁË, ÞÔÏ BL LC
2 5. ðÒÑÍÁÑ, ÐÒÏÈÏÄÑÝÁÑ ÞÅÒÅÚ ÔÏÞËÕ L ÐÁÒÁÌÌÅÌØÎÏ ÓÔÏÒÏÎÅ AB, ÐÅÒÅÓÅËÁÅÔ ÏÔÒÅÚÏË BM × ÔÏÞËÅ O, ÐÒÉÞÅÍ BO OM
7 4. îÁÊÄÉÔÅ ÏÔÎÏÛÅÎÉÅ,
× ËÏÔÏÒÏÍ ÔÏÞËÁ M ÄÅÌÉÔ ÓÔÏÒÏÎÕ AC.
131.19.

÷ ÔÒÅÕÇÏÌØÎÉËÅ

ÐÏÌÏÖÅÎÙ ÔÁË, ÞÔÏ

:

=

:

:

=

:

131.20.

:

=

:

:

131.21.

=

:

(úÁÍÅÞÁÔÅÌØÎÏÅ Ó×ÏÊÓÔ×Ï ÔÒÁÐÅÃÉÉ) äÏËÁÖÉÔÅ, ÞÔÏ ÔÏÞËÁ ÐÅÒÅÓÅÞÅÎÉÑ ÄÉÁÇÏÎÁ-

ÌÅÊ, ÔÏÞËÁ ÐÅÒÅÓÅÞÅÎÉÑ ÐÒÏÄÏÌÖÅÎÉÊ ÂÏËÏ×ÙÈ ÓÔÏÒÏÎ É ÓÅÒÅÄÉÎÙ ÏÓÎÏ×ÁÎÉÊ ÌÀÂÏÊ ÔÒÁÐÅÃÉÉ
ÌÅÖÁÔ ÎÁ ÏÄÎÏÊ ÐÒÑÍÏÊ.
131.22.

äÁÎÙ Ä×Å ÐÁÒÁÌÌÅÌØÎÙÅ ÐÒÑÍÙÅ

l É l1. ó ÐÏÍÏÝØÀ ÏÄÎÏÊ ÌÉÎÅÊËÉ ÒÁÚÄÅÌÉÔÅ

ÐÏÐÏÌÁÍ ÏÔÒÅÚÏË, ÒÁÓÐÏÌÏÖÅÎÎÙÊ ÎÁ ÏÄÎÏÊ ÉÚ ÎÉÈ.

l É l1. ó ÐÏÍÏÝØÀ ÏÄÎÏÊ ÌÉÎÅÊËÉ ÐÒÏ×ÅÄÉÔÅ
ÞÅÒÅÚ ÄÁÎÎÕÀ ÔÏÞËÕ M ÐÒÑÍÕÀ, ÐÁÒÁÌÌÅÌØÎÕÀ ÐÒÑÍÙÍ l É l1 .
131.23.

äÁÎÙ Ä×Å ÐÁÒÁÌÌÅÌØÎÙÅ ÐÒÑÍÙÅ

207

132. ðÏÄÏÂÎÙÅ ÔÒÅÕÇÏÌØÎÉËÉ

ä×Á ÔÒÅÕÇÏÌØÎÉËÁ ÎÁÚÙ×ÁÀÔÓÑ

ÐÏÄÏÂÎÙÍÉ, ÅÓÌÉ ÉÈ ÕÇÌÙ ÓÏÏÔ×ÅÔÓÔ×ÅÎÎÏ ÒÁ×ÎÙ, Á ÓÏÏÔ-

×ÅÔÓÔ×ÕÀÝÉÅ ÓÔÏÒÏÎÙ ÐÒÏÐÏÒÃÉÏÎÁÌØÎÙ:

ABC A1B1C1 ,
'

, \A

=

\A1; \B = \B1; \C = \C1; BBC
C
1

=

1

AC = AB
A1 C1 A1 B1

ïÔÎÏÛÅÎÉÅ ÓÏÏÔ×ÅÔÓÔ×ÕÀÝÉÈ ÓÔÏÒÏÎ ÐÏÄÏÂÎÙÈ ÔÒÅÕÇÏÌØÎÉËÏ× ÎÁÚÙ×ÁÅÔÓÑ ËÏÜÆÆÉÃÉÅÎÔÏÍ
ÐÏÄÏÂÉÑ.
132.1.

äÏËÁÖÉÔÅ, ÞÔÏ ÐÒÑÍÁÑ, ÐÁÒÁÌÌÅÌØÎÁÑ ÓÔÏÒÏÎÅ ÄÁÎÎÏÇÏ ÔÒÅÕÇÏÌØÎÉËÁ É ÐÅÒÅÓÅËÁÀ-

ÝÁÑ Ä×Å ÄÒÕÇÉÅ ÅÇÏ ÓÔÏÒÏÎÙ (ÉÌÉ ÉÈ ÐÒÏÄÏÌÖÅÎÉÑ), ÏÂÒÁÚÕÅÔ Ó ÜÔÉÍÉ ÓÔÏÒÏÎÁÍÉ ÔÒÅÕÇÏÌØÎÉË,
ÐÏÄÏÂÎÙÊ ÄÁÎÎÏÍÕ.
132.2.

(ðÒÉÚÎÁËÉ ÐÏÄÏÂÉÑ ÔÒÅÕÇÏÌØÎÉËÏ×) äÏËÁÖÉÔÅ, ÞÔÏ Ä×Á ÔÒÅÕÇÏÌØÎÉËÁ ÐÏÄÏÂÎÙ,

ÅÓÌÉ (Á) Ä×Á ÕÇÌÁ ÏÄÎÏÇÏ ÉÚ ÎÉÈ ÓÏÏÔ×ÅÔÓÔ×ÅÎÎÏ ÒÁ×ÎÙ Ä×ÕÍ ÕÇÌÁÍ ÄÒÕÇÏÇÏ; (Â) Ä×Å ÓÔÏÒÏÎÙ
ÏÄÎÏÇÏ ÉÚ ÎÉÈ ÓÏÏÔ×ÅÔÓÔ×ÅÎÎÏ ÐÒÏÐÏÒÃÉÏÎÁÌØÎÙ Ä×ÕÍ ÓÔÏÒÏÎÁÍ ÄÒÕÇÏÇÏ, Á ÕÇÌÙ ÍÅÖÄÕ ÜÔÉÍÉ
ÓÔÏÒÏÎÁÍÉ ÒÁ×ÎÙ; (×) ÔÒÉ ÓÔÏÒÏÎÙ ÏÄÎÏÇÏ ÉÚ ÎÉÈ ÓÏÏÔ×ÅÔÓÔ×ÅÎÎÏ ÐÒÏÐÏÒÃÉÏÎÁÌØÎÙ ÔÒÅÍ
ÓÔÏÒÏÎÁÍ ÄÒÕÇÏÇÏ.

ABCD ÒÁÚÄÅÌÅÎÁ ÎÁ n ÒÁ×ÎÙÈ ÞÁÓÔÅÊ. ðÅÒ×ÁÑ
ÔÏÞËÁ ÄÅÌÅÎÉÑ P ÓÏÅÄÉÎÅÎÁ Ó ×ÅÒÛÉÎÏÊ B. äÏËÁÖÉÔÅ, ÞÔÏ ÐÒÑÍÁÑ BP ÏÔÓÅËÁÅÔ ÎÁ ÄÉÁÇÏÎÁÌÉ
AC ÞÁÓÔØ AQ, ËÏÔÏÒÁÑ ÒÁ×ÎÁ 1=(n + 1) ×ÓÅÊ ÄÉÁÇÏÎÁÌÉ.
÷ ÐÒÑÍÏÕÇÏÌØÎÙÊ ÔÒÅÕÇÏÌØÎÉË Ó ËÁÔÅÔÁÍÉ, ÒÁ×ÎÙÍÉ 6 É 8, ×ÐÉÓÁÎ Ë×ÁÄÒÁÔ, ÉÍÅ132.3.

óÔÏÒÏÎÁ

AD

ÐÁÒÁÌÌÅÌÏÇÒÁÍÍÁ

132.4.

ÀÝÉÊ Ó ÔÒÅÕÇÏÌØÎÉËÏÍ ÏÂÝÉÊ ÐÒÑÍÏÊ ÕÇÏÌ. îÁÊÄÉÔÅ ÓÔÏÒÏÎÕ Ë×ÁÄÒÁÔÁ.
132.5.

äÁÎ ÕÇÏÌ É ÔÏÞËÁ ×ÎÕÔÒÉ ÎÅÇÏ. ðÒÏ×ÅÄÉÔÅ ÞÅÒÅÚ ÜÔÕ ÔÏÞËÕ ÐÒÑÍÕÀ, ÏÔÒÅÚÏË ËÏÔÏ-

ÒÏÊ, ÚÁËÌÀÞÅÎÎÙÊ ×ÎÕÔÒÉ ÄÁÎÎÏÇÏ ÕÇÌÁ, ÄÅÌÉÌÓÑ ÂÙ ÄÁÎÎÏÊ ÔÏÞËÏÊ × ÚÁÄÁÎÎÏÍ ÏÔÎÏÛÅÎÉÉ.
132.6.

ðÏÓÔÒÏÊÔÅ ÐÒÑÍÏÕÇÏÌØÎÙÊ ÔÒÅÕÇÏÌØÎÉË ÐÏ ÏÔÎÏÛÅÎÉÀ ÅÇÏ ËÁÔÅÔÏ× É ×ÙÓÏÔÅ, ÏÐÕ-

ÝÅÎÎÏÊ ÎÁ ÇÉÐÏÔÅÎÕÚÕ.
132.7.

ðÏÓÔÒÏÊÔÅ ÐÒÑÍÏÕÇÏÌØÎÙÊ ÔÒÅÕÇÏÌØÎÉË ÐÏ ÇÉÐÏÔÅÎÕÚÅ É ÏÔÎÏÛÅÎÉÀ ËÁÔÅÔÏ×.

132.8.

ïÔÒÅÚÏË ÐÒÑÍÏÊ, ÐÁÒÁÌÌÅÌØÎÏÊ ÏÓÎÏ×ÁÎÉÑÍ ÔÒÁÐÅÃÉÉ, ÚÁËÌÀÞÅÎÎÙÊ ×ÎÕÔÒÉ ÔÒÁ-

ÐÅÃÉÉ, ÒÁÚÂÉ×ÁÅÔÓÑ ÅÅ ÄÉÁÇÏÎÁÌÑÍÉ ÎÁ ÔÒÉ ÞÁÓÔÉ. äÏËÁÖÉÔÅ, ÞÔÏ ÏÔÒÅÚËÉ, ÐÒÉÌÅÇÁÀÝÉÅ Ë
ÂÏËÏ×ÙÍ ÓÔÏÒÏÎÁÍ, ÒÁ×ÎÙ ÍÅÖÄÕ ÓÏÂÏÊ.
132.9.

þÅÒÅÚ ÔÏÞËÕ ÐÅÒÓÅÞÅÎÉÑ ÄÉÁÇÏÎÁÌÅÊ ÔÒÁÐÅÃÉÉ Ó ÏÓÎÏ×ÁÎÉÑÍÉ

a É b ÐÒÏ×ÅÄÅÎÁ

ÐÒÑÍÁÑ, ÐÁÒÁÌÌÅÌØÎÁÑ ÏÓÎÏ×ÁÎÉÑÍ. îÁÊÄÉÔÅ ÏÔÒÅÚÏË ÜÔÏÊ ÐÒÑÍÏÊ, ÚÁËÌÀÞÅÎÎÙÊ ÍÅÖÄÕ ÂÏËÏ×ÙÍÉ ÓÔÏÒÏÎÁÍÉ ÔÒÁÐÅÃÉÉ.
132.10.

ðÁÒÁÌÌÅÌØÎÏ ÏÓÎÏ×ÁÎÉÑÍ ÔÒÁÐÅÃÉÉ ÐÒÏ×ÅÄÉÔÅ ÐÒÑÍÕÀ, ÏÔÒÅÚÏË ËÏÔÏÒÏÊ, ÚÁ-

ËÌÀÞÅÎÎÙÊ ×ÎÕÔÒÉ ÔÒÁÐÅÃÉÉ, ÄÅÌÉÌÓÑ ÂÙ ÅÅ ÄÉÁÇÏÎÁÌÑÍÉ × ÏÔÎÏÛÅÎÉÉ
132.11.

1 : 3 : 1.

îÅÐÁÒÁÌÌÅÌØÎÙÅ ÓÔÏÒÏÎÙ ÔÒÁÐÅÃÉÉ ÐÒÏÄÏÌÖÅÎÙ ÄÏ ×ÚÁÉÍÎÏÇÏ ÐÅÒÅÓÅÞÅÎÉÑ É

ÞÅÒÅÚ ÐÏÌÕÞÅÎÎÕÀ ÔÏÞËÕ ÐÒÏ×ÅÄÅÎÁ ÐÒÑÍÁÑ, ÐÁÒÁÌÌÅÌØÎÁÑ ÏÓÎÏ×ÁÎÉÑÍ ÔÒÁÐÅÃÉÉ. îÁÊÄÉÔÅ
ÄÌÉÎÕ ÏÔÒÅÚËÁ ÜÔÏÊ ÐÒÑÍÏÊ, ÏÇÒÁÎÉÞÅÎÎÏÇÏ ÐÒÏÄÏÌÖÅÎÉÑÍÉ ÄÉÁÇÏÎÁÌÅÊ, ÅÓÌÉ ÄÌÉÎÙ ÏÓÎÏ×ÁÎÉÊ ÔÒÁÐÅÃÉÉ ÒÁ×ÎÙ

a É b.

AA1 É BB1 { ×ÙÓÏÔÙ ÏÓÔÒÏÕÇÏÌØÎÏÇÏ ÔÒÅÕÇÏÌØÎÉËÁ ABC. äÏËÁÖÉÔÅ, ÞÔÏ: ( ) ÔÒÅÕÇÏÌØÎÉË AA1 C ÐÏÄÏÂÅÎ ÔÒÅÕÇÏÌØÎÉËÕ BB1C; ( ) ÔÒÅÕÇÏÌØÎÉË ABC ÐÏÄÏÂÅÎ ÔÒÅÕÇÏÌØÎÉËÕ
A1B1C.
132.12.

Á

Â

208

60

132.13.

÷ ÔÒÅÕÇÏÌØÎÉËÅ

132.14.

èÏÒÄÙ

o É

BC = 6.

MB = CM MD.

ABC ÐÒÏ×ÅÄÅÎÙ ×ÙÓÏÔÙ BB1 É CC1. îÁÊÄÉÔÅ B1C1, ÅÓÌÉ \A =

AB É CD ÏËÒÕÖÎÏÓÔÉ ÐÅÒÅÓÅËÁÀÔÓÑ × ÔÏÞËÅ M. äÏËÁÖÉÔÅ, ÞÔÏ AM





ABCD, ÄÉÁÇÏÎÁÌÉ ËÏÔÏÒÏÇÏ ÐÅÒÅÓÅËÁÀÔÓÑ ×
ÔÏÞËÅ K, ÉÚ×ÅÓÔÎÏ, ÞÔÏ AB = a, BK = b, AK = c, CD = d. îÁÊÄÉÔÅ AC.
ä×Å ÏËÒÕÖÎÏÓÔÉ ÐÅÒÅÓÅËÁÀÔÓÑ × ÔÏÞËÁÈ A É B. ÷ ËÁÖÄÏÊ ÉÚ ÜÔÉÈ ÏËÒÕÖÎÏÓÔÅÊ
ÐÒÏ×ÅÄÅÎÙ ÈÏÒÄÙ AC É AD ÔÁË, ÞÔÏ ÈÏÒÄÁ ÏÄÎÏÊ ÏËÒÕÖÎÏÓÔÉ ËÁÓÁÅÔÓÑ ÄÒÕÇÏÊ ÏËÒÕÖÎÏÓÔÉ.
îÁÊÄÉÔÅ AB, ÅÓÌÉ CB = a, DB = b.
132.15.

÷Ï ×ÐÉÓÁÎÎÏÍ ÞÅÔÙÒÅÈÕÇÏÌØÎÉËÅ

132.16.

132.17.

÷ ÒÁ×ÎÏÂÅÄÒÅÎÎÙÊ ÔÒÅÕÇÏÌØÎÉË ×ÐÉÓÁÎÁ ÏËÒÕÖÎÏÓÔØ. ôÏÞËÉ ËÁÓÁÎÉÑ ÄÅÌÑÔ ËÁ-

ÖÄÕÀ ÂÏËÏ×ÕÀ ÓÔÏÒÏÎÕ ÎÁ ÏÔÒÅÚËÉ ÄÌÉÎÏÊ

m É n, ÓÞÉÔÁÑ ÏÔ ×ÅÒÛÉÎÙ. ë ÏËÒÕÖÎÏÓÔÉ ÐÒÏ-

×ÅÄÅÎÙ ÔÒÉ ËÁÓÁÔÅÌØÎÙÅ, ÐÁÒÁÌÌÅÌØÎÙÅ ËÁÖÄÏÊ ÉÚ ÓÔÏÒÏÎ ÔÒÅÕÇÏÌØÎÉËÁ. îÁÊÄÉÔÅ ÄÌÉÎÙ
ÏÔÒÅÚËÏ× ËÁÓÁÔÅÌØÎÙÈ, ÚÁËÌÀÞÅÎÎÙÈ ÍÅÖÄÕ ÓÔÏÒÏÎÁÍÉ ÔÒÅÕÇÏÌØÎÉËÁ.

K É M ÌÅÖÁÔ ÎÁ ÓÔÏÒÏÎÁÈ AB É BC ÔÒÅÕÇÏÌØÎÉËÁ ABC, ÐÒÉÞÅÍ AK :
BK = 3 : 2, BM : MC = 3 : 1. þÅÒÅÚ ÔÏÞËÕ B ÐÒÏ×ÅÄÅÎÁ ÐÒÑÍÁÑ l, ÐÁÒÁÌÌÅÌØÎÁÑ AC. ðÒÑÍÁÑ
KM ÐÅÒÅÓÅËÁÅÔ ÐÒÑÍÕÀ l × ÔÏÞËÅ P, Á ÐÒÑÍÕÀ AC × ÔÏÞËÅ N. îÁÊÄÉÔÅ BP É CN, ÅÓÌÉ
AC = a.
äÁÎ ÔÒÅÕÇÏÌØÎÉË ABC. îÁ ÐÒÏÄÏÌÖÅÎÉÉ ÓÔÏÒÏÎÙ AC ÚÁ ÔÏÞËÕ C ×ÚÑÔÁ ÔÏÞËÁ N
ÔÁË, ÞÔÏ CN = AC. ôÏÞËÁ K - ÓÅÒÅÄÉÎÁ ÓÔÏÒÏÎÙ AB. ÷ ËÁËÏÍ ÏÔÎÏÛÅÎÉÉ ÐÒÑÍÁÑ KN ÄÅÌÉÔ
ÓÔÏÒÏÎÕ BC ?
äÁÎ ÔÒÅÕÇÏÌØÎÉË ABC. îÁ ÐÒÏÄÏÌÖÅÎÉÉ ÓÔÏÒÏÎÙ AC ÚÁ ÔÏÞËÕ C ×ÚÑÔÁ ÔÏÞËÁ N
ÔÁË, ÞÔÏ CN = (2=3) AC. ôÏÞËÁ K ÌÅÖÉÔ ÎÁ ÓÔÏÒÏÎÅ AB, ÐÒÉÞÅÍ AK : KB = 3 : 2. ÷ ËÁËÏÍ
ÏÔÎÏÛÅÎÉÉ ÐÒÑÍÁÑ KN ÄÅÌÉÔ ÓÔÏÒÏÎÕ BC ?
äÁÎ ÔÒÅÕÇÏÌØÎÉË ABC. îÁ ÐÒÏÄÏÌÖÅÎÉÉ ÓÔÏÒÏÎÙ AC ÚÁ ÔÏÞËÕ C ×ÚÑÔÁ ÔÏÞËÁ N
ÔÁË, ÞÔÏ AC = 2 CN. ôÏÞËÁ M ÌÅÖÉÔ ÎÁ ÓÔÏÒÏÎÅ BC, ÐÒÉÞÅÍ BM : MC = 1 : 3. ÷ ËÁËÏÍ
ÏÔÎÏÛÅÎÉÉ ÐÒÑÍÁÑ MN ÄÅÌÉÔ ÓÔÏÒÏÎÕ AB ?
ôÏÞËÉ K É M ÌÅÖÁÔ ÓÏÏÔ×ÅÔÓÔ×ÅÎÎÏ ÎÁ ÓÔÏÒÏÎÁÈ AB É BC ÔÒÅÕ ÇÏÌØÎÉËÁ ABC,
ÐÒÉÞÅÍ BK : KA = 1 : 4, BM : MC = 3 : 2. ðÒÑÍÁÑ MK ÐÅÒÅÓÅËÁÅÔ ÐÒÏÄÏÌÖÅÎÉÅ ÓÔÏÒÏÎÙ
AC × ÔÏÞËÅ N. îÁÊÄÉÔÅ AC : CN.
ôÏÞËÉ M É N ÌÅÖÁÔ ÓÏÏÔ×ÅÔÓÔ×ÅÎÎÏ ÎÁ ÓÔÏÒÏÎÁÈ AB É BC ÐÁÒÁÌÌÅÌÏÇÒÁÍÍÁ
ABCD, ÐÒÉÞÅÍ AM : MB = 1 : 2, AN : ND = 3 : 2. ïÔÒÅÚËÉ DM É CN ÐÅÒÅÓÅËÁÀÔÓÑ ×
ÔÏÞËÅ K. îÁÊÄÉÔÅ ÏÔÎÏÛÅÎÉÑ DK : KM, CK : KN.
AD | ÍÅÄÉÁÎÁ ÔÒÅÕÇÏÌØÎÉËÁ ABC. ôÏÞËÁ E - ÌÅÖÉÔ ÎÁ ÓÔÏÒÏÎÅ AB, ÐÒÉÞÅÍ
AE : EB = 1 : 2. ïÔÒÅÚËÉ AD É CE ÐÅÒÅÓÅËÁÀÔÓÑ × ÔÏÞËÅ M. îÁÊÄÉÔÅ ÏÔÎÏÛÅÎÉÑ AM : MD,
CM : ME.
ôÏÞËÉ K É E ÌÅÖÁÔ ÓÏÏÔ×ÅÔÓÔ×ÅÎÎÏ ÎÁ ÓÔÏÒÏÎÁÈ BC É AB ÔÒÅÕÇÏÌØÎÉËÁ ABC.
ïÔÒÅÚËÉ AK É CE ÐÅÒÅÓÅËÁÀÔÓÑ × ÔÏÞËÅ M. ÷ ËÁËÏÍ ÏÔÎÏÛÅÎÉÉ ÐÒÑÍÁÑ BM ÄÅÌÉÔ ÓÔÏÒÏÎÕ
AC, ÅÓÌÉ BK : KC = 1 : 2, AE : EB = 2 : 3 ?
îÁ ÍÅÄÉÁÎÅ AD ÔÒÅÕÇÏÌØÎÉËÁ ABC ×ÚÑÔÁ ÔÏÞËÁ M, ÐÒÉÞÅÍ AM : MD = 1 : 3.
÷ ËÁËÏÍ ÏÔÎÏÛÅÎÉÉ ÐÒÑÍÁÑ BM ÄÅÌÉÔ ÓÔÏÒÏÎÕ AC?
132.18.

ôÏÞËÉ

132.19.

132.20.



132.21.



132.22.

132.23.

132.24.

132.25.

132.26.

132.27.

äÏËÁÖÉÔÅ, ÞÔÏ ÂÉÓÓÅËÔÒÉÓÁ ÔÒÅÕÇÏÌØÎÉËÁ ÄÅÌÉÔ ÏÓÎÏ×ÁÎÉÅ ÎÁ ÏÔÒÅÚËÉ, ÐÒÏÐÏÒ-

ÃÉÏÎÁÌØÎÙÅ ÂÏËÏ×ÙÍ ÓÔÏÒÏÎÁÍ.
132.28.

÷ ÔÒÅÕÇÏÌØÎÉËÅ

ABC ÉÚ×ÅÓÔÎÏ, ÞÔÏ AB = c, BC = a, AC = b. ÷ ËÁËÏÍ ÏÔÎÏÛÅC?

ÎÉÉ ÃÅÎÔÒ ×ÐÉÓÁÎÎÏÊ ÏËÒÕÖÎÏÓÔÉ ÔÒÅÕÇÏÌØÎÉËÁ ÄÅÌÉÔ ÂÉÓÓÅËÔÒÉÓÕ ÕÇÌÁ

209

ABC ×ÐÉÓÁÎ ÒÏÍ DECF ÔÁË, ÞÔÏ ×ÅÒÛÉÎÁ E
ÌÅÖÉÔ ÎÁ ÏÔÒÅÚËÅ BC, ×ÅÒÛÉÎÁ F ÌÅÖÉÔ ÎÁ ÏÔÒÅÚËÅ AC É ×ÅÒÛÉÎÁ D ÌÅÖÉÔ ÎÁ ÏÔÒÅÚËÅ AB.
îÁÊÄÉÔÅ ÄÌÉÎÕ ÓÔÏÒÏÎÙ ÒÏÍÂÁ, ÅÓÌÉ AB = BC = 12, AC = 6.
÷ ÔÒÅÕÇÏÌØÎÉËÅ ABC ÓÔÏÒÏÎÁ AC ÒÁ×ÎÁ b, ÓÔÏÒÏÎÁ AB ÒÁ×ÎÁ c, Á ÂÉÓÓÅËÔÒÉÓÁ
A ÐÅÒÅÓÅËÁÅÔÓÑ ÓÏ ÓÔÏÒÏÎÏÊ BC × ÔÏÞËÅ D, ÔÁËÏÊ, ÞÔÏ DA = DB. îÁÊÄÉÔÅ ÓÔÏÒÏÎÕ BC.
âÉÓÓÅËÔÒÉÓÁ ×ÎÅÛÎÅÇÏ ÕÇÌÁ A ÔÒÅÕÇÏÌØÎÉËÁ ABC ÐÅÒÅÓÅËÁÅÔ ÐÒÏÄÏÌÖÅÎÉÅ ÓÔÏÒÏÎÙ BC É ÔÏÞËÅ M. äÏËÁÖÉÔÅ, ÞÔÏ BM=MC = AB=AC.
îÁ ÓÔÏÒÏÎÅ BC ÔÒÅÕÇÏÌØÎÉËÁ ABC ×ÚÑÔÁ ÔÏÞËÁ D ÔÁË, ÞÔÏ BD=AB = DC=AC.
äÏËÁÖÉÔÅ, ÞÔÏ AD | ÂÉÓÓÅËÔÒÉÓÁ ÔÒÅÕÇÏÌØÎÉËÁ ABC.
ïËÒÕÖÎÏÓÔØ É ÐÒÑÍÁÑ ËÁÓÁÀÔÓÑ × ÔÏÞËÅ M. éÚ ÔÏÞÅË A É B ÜÔÏÊ ÏËÒÕÖÎÏÓÔÉ
ÏÐÕÝÅÎÙ ÐÅÒÐÅÎÄÉËÕÌÑÒÙ ÎÁ ÐÒÑÍÕÀ, ÒÁ×ÎÙÅ a É b ÓÏÏÔ×ÅÔÓÔ×ÅÎÎÏ. îÁÊÄÉÔÅ ÒÁÓÓÔÏÑÎÉÅ ÏÔ
ÔÏÞËÉ M ÄÏ ÐÒÑÍÏÊ AB.
éÚ ÔÏÞËÉ M, ÌÅÖÁÝÅÊ ×ÎÅ ÏËÒÕÖÎÏÓÔÉ, ÐÒÏ×ÅÄÅÎÙ Ë ÜÔÏÊ ÏËÒÕÖÎÏÓÔÉ Ä×Å ËÁÓÁÔÅÌØÎÙÅ. òÁÓÓÔÏÑÎÉÑ ÏÔ ÔÏÞËÉ C, ÌÅÖÁÝÅÊ ÎÁ ÏËÒÕÖÎÏÓÔÉ, ÄÏ ËÁÓÁÔÅÌØÎÙÈ ÒÁ×ÎÙ a É b.
îÁÊÄÉÔÅ ÒÁÓÓÔÏÑÎÉÅ ÏÔ ÔÏÞËÉ C ÄÏ ÐÒÑÍÏÊ AB, ÇÄÅ A É B - ÔÏÞËÉ ËÁÓÁÎÉÑ.
÷ ÒÁ×ÎÏÂÅÄÒÅÎÎÙÊ ÔÒÅÕÇÏÌØÎÉË

132.29.

132.30.

132.31.

132.32.

132.33.

132.34.

* * *
132.35.

(ôÅÏÒÅÍÁ íÅÎÅÌÁÑ) äÁÎ ÔÒÅÕÇÏÌØÎÉË

ABC. ðÒÑÍÁÑ ÐÅÒÅÓÅËÁÅÔ ÅÇÏ ÓÔÏÒÏÎÙ AB, BC É

AC × ÔÏÞËÁÈ C1 , A1, B1 ÓÏÏÔ×ÅÔÓÔ×ÅÎÎÏ. äÏËÁÖÉÔÅ, ÞÔÏ
BA1 CB1 AC1 = 1
A1C B1A C1B
(ôÅÏÒÅÍÁ þÅ×Ù) ðÕÓÔØ ÔÏÞËÉ A1 , B1 É C1 ÐÒÉÎÁÄÌÅÖÁÔ ÓÏÏÔ×ÅÔÓÔ×ÅÎÎÏ ÓÔÏÒÏÎÁÍ BC,
AC É AB ÔÒÅÕÇÏÌØÎÉËÁ ABC. äÏËÁÖÉÔÅ, ÞÔÏ ÏÔÒÅÚËÉ AA1, BB1, CC1 ÐÅÒÅÓÅËÁÀÔÓÑ × ÏÄÎÏÊ ÔÏÞËÅ
ÔÏÇÄÁ É ÔÏÌØËÏ ÔÏÇÄÁ, ËÏÇÄÁ
AB1 CA1 BC1 = 1:
B1C A1C C1A

ÐÒÏÄÏÌÖÅÎÉÅ ÓÔÏÒÏÎÙ





132.36.



132.37.



÷ ÔÒÅÕÇÏÌØÎÉË ×ÐÉÓÁÎÁ ÏËÒÕÖÎÏÓÔØ. ôÏÞËÉ ËÁÓÁÎÉÑ ÓÏ ÓÔÏÒÏÎÁÍÉ ÔÒÅÕÇÏÌØÎÉËÁ ÓÏÅÄÉÎÅÎÙ

Ó ÐÒÏÔÉ×ÏÐÏÌÏÖÎÙÍÉ ×ÅÒÛÉÎÁÍÉ. äÏËÁÖÉÔÅ ÞÔÏ ÔÒÉ ÐÏÌÕÞÅÎÎÙÈ ÏÔÒÅÚËÁ ÐÅÒÅÓÅËÁÀÔÓÑ × ÏÄÎÏÊ ÔÏÞËÅ.
132.38.

ïËÏÌÏ ÏËÒÕÖÎÏÓÔÉ ÏÐÉÓÁÎÁ ÒÁ×ÎÏÂÅÄÒÅÎÎÁÑ ÔÒÁÐÅÃÉÑ. âÏËÏ×ÁÑ ÓÔÏÒÏÎÁ ÔÒÁÐÅÃÉÉ ÒÁ×ÎÁ

a, ÏÔÒÅÚÏË, ÓÏÅÄÉÎÑÀÝÉÊ ÔÏÞËÉ ËÁÓÁÎÉÑ ÂÏËÏ×ÙÈ ÓÔÏÒÏÎ Ó ÏËÒÕÖÎÏÓÔØÀ, ÒÁ×ÅÎ b. îÁÊÄÉÔÅ ÄÉÁÍÅÔÒ
ÏËÒÕÖÎÏÓÔÉ.

þÅÔÙÒÅÈÕÇÏÌØÎÉË ABCD ×ÐÉÓÁÎ × ÏËÒÕÖÎÏÓÔØ. äÉÁÇÏÎÁÌØ AC Ñ×ÌÑÅÔÓÑ ÂÉÓÓÅËÔÒÉÓÏÊ
BAD É ÐÅÒÅÓÅËÁÅÔÓÑ Ó ÄÉÁÇÏÎÁÌØÀ BD × ÔÏÞËÅ K. îÁÊÄÉÔÅ KC, ÅÓÌÉ BC = 4 É AK = 6.
ðÒÏÄÏÌÖÅÎÉÅ ÍÅÄÉÁÎÙ ÔÒÅÕÇÏÌØÎÉËÁ ABC, ÐÒÏ×ÅÄÅÎÎÏÊ ÉÚ ×ÅÒÛÉÎÙ A, ÐÅÒÅÓÅËÁÅÔ ÏÐÉÓÁÎÎÕÀ ÏËÒÕÖÎÏÓÔØ × ÔÏÞËÅ D. îÁÊÄÉÔÅ BC, ÅÓÌÉ AC = DC = 1.
þÅÒÅÚ ÔÏÞËÕ P ÍÅÄÉÁÎÙ CC1 ÔÒÅÕÇÏÌØÎÉËÁ ABC ÐÒÏ×ÅÄÅÎÙ ÐÒÑÍÙÅ AA1 É BB1 (ÔÏÞËÉ
A1 É B1 ÌÅÖÁÔ ÎÁ ÓÔÏÒÏÎÁÈ BC É CA). äÏËÁÖÉÔÅ, ÞÔÏ A1B1 AB.
ðÒÑÍÁÑ, ÓÏÅÄÉÎÑÀÝÁÑ ÔÏÞËÕ P ÐÅÒÅÓÅÞÅÎÉÑ ÄÉÁÇÏÎÁÌÅÊ ÞÅÔÙÒÅÈÕÇÏÌØÎÉËÁ ABCD Ó ÔÏÞËÏÊ Q ÐÅÒÅÓÅÞÅÎÉÑ ÐÒÑÍÙÈ AB É CD, ÄÅÌÉÔ ÓÔÏÒÏÎÕ AD ÐÏÐÏÌÁÍ. äÏËÁÖÉÔÅ, ÞÔÏ ÏÎÁ ÄÅÌÉÔ ÐÏÐÏÌÁÍ
É ÓÔÏÒÏÎÕ BC.
132.39.

ÕÇÌÁ

132.40.

132.41.

k

132.42.

132.43.

óÔÏÒÏÎÁ ÔÒÅÕÇÏÌØÎÉËÁ ÒÁ×ÎÁ ÓÒÅÄÎÅÍÕ ÁÒÉÆÍÅÔÉÞÅÓËÏÍÕ Ä×ÕÈ ÄÒÕÇÉÈ ÓÔÏÒÏÎ. äÏËÁÖÉÔÅ,

ÞÔÏ ÐÒÑÍÁÑ, ÐÒÏÈÏÄÑÝÁÑ ÞÅÒÅÚ ÔÏÞËÕ ÐÅÒÅÓÅÞÅÎÉÑ ÍÅÄÉÁÎ É ÔÏÞËÕ ÐÅÒÅÓÅÞÅÎÉÑ ÂÉÓÓÅËÔÒÉÓ ÔÒÅÕÇÏÌØÎÉËÁ, ÐÁÒÁÌÌÅÌØÎÁ ÜÔÏÊ ÓÔÏÒÏÎÅ.
132.44.

äÏËÁÖÉÔÅ, ÞÔÏ × ÌÀÂÏÍ ÔÒÅÕÇÏÌØÎÉËÅ ÔÏÞËÁ

H ÐÅÒÅÓÅÞÅÎÉÑ ×ÙÓÏÔ (ÏÒÔÏÃÅÎÔÒ), ÃÅÎÔÒ

O ÏÐÉÓÁÎÎÏÊ ÏËÒÕÖÎÏÓÔÉ É ÔÏÞËÁ M ÐÅÒÅÓÅÞÅÎÉÑ ÍÅÄÉÁÎ (ÃÅÎÔÒ ÔÑÖÅÓÔÉ) ÌÅÖÁÔ ÎÁ ÏÄÎÏÊ ÐÒÑÍÏÊ
M ÒÁÓÐÏÌÏÖÅÎÁ ÍÅÖÄÕ ÔÏÞËÁÍÉ O É H, É MH = 2 MO.

(€ÐÒÑÍÁÑ üÊÌÅÒÁ), ÐÒÉÞÅÍ ÔÏÞËÁ



210

ABC. îÅËÏÔÏÒÁÑ ÐÒÑÍÁÑ, ÐÁÒÁÌÌÅÌØÎÁÑ ÐÒÑÍÏÊ AC, ÐÅÒÅAB É BC × ÔÏÞËÁÈ M É P ÓÏÏÔ×ÅÔÓÔ×ÅÎÎÏ. ôÏÞËÁ D - ÃÅÎÔÒ ÐÒÁ×ÉÌØÎÏÇÏ ÔÒÅÕÇÏÌØÎÉËÁ
PMB, ÔÏÞËÁ E - ÓÅÒÅÄÉÎÁ ÏÔÒÅÚËÁ AP. ïÐÒÅÄÅÌÉÔÅ ÕÇÌÙ ÔÒÅÕÇÏÌØÎÉËÁ DEC.
þÅÒÅÚ ÃÅÎÔÒ O ÏËÒÕÖÎÏÓÔÉ, ÏÐÉÓÁÎÎÏÊ ÏËÏÌÏ ÏÓÔÒÏÕÇÏÌØÎÏÇÏ ÔÒÅÕÇÏÌØÎÉËÁ ABC, ÐÒÏ×ÅÄÅÎÁ ÐÒÑÍÁÑ, ÐÅÒÐÅÎÄÉËÕÌÑÒÎÁÑ BO É ÐÅÒÅÓÅËÁÀÝÁÑ ÏÔÒÅÚÏË AB × ÔÏÞËÅ P, É ÐÒÏÄÏÌÖÅÎÉÅ ÏÔÒÅÚËÁ
BC ÚÁ ÔÏÞËÕ C × ÔÏÞËÅ Q. ÷ÙÞÉÓÌÉÔÅ ÄÌÉÎÕ ÏÔÒÅÚËÁ BP, ÅÓÌÉ ÉÚ×ÅÓÔÎÙ ÄÌÉÎÙ ÓÔÏÒÏÎ ÔÒÅÕÇÏÌØÎÉËÁ
AB = c, BC = a, É ÄÌÉÎÁ ÏÔÒÅÚËÁ BQ ÒÁ×ÎÁ p.
þÅÒÅÚ ÔÏÞËÕ D, ×ÚÑÔÕÀ ÎÁ ÓÔÏÒÏÎÅ AB ÔÒÅÕÇÏÌØÎÉËÁ ABC, ÐÒÏ×ÅÄÅÎÁ ÐÒÑÍÁÑ, ÐÁÒÁÌÌÅÌØÎÁÑ AC, É ÐÅÒÅÓÅËÁÀÝÁÑ ÓÔÏÒÏÎÕ BC × ÔÏÞËÅ E. äÏËÁÖÉÔÅ, ÞÔÏ AE, CD É ÍÅÄÉÁÎÁ, ÐÒÏ×ÅÄÅÎÎÁÑ
ÞÅÒÅÚ ×ÅÒÛÉÎÕ B, ÐÅÒÅÓÅËÁÀÔÓÑ × ÏÄÎÏÊ ÔÏÞËÅ.
132.45.

äÁÎ ÐÒÁ×ÉÌØÎÙÊ ÔÒÅÕÇÏÌØÎÉË

ÓÅËÁÅÔ ÐÒÑÍÙÅ
132.46.

132.47.

132.48.

ó ÐÏÍÏÝØÀ ÃÉÒËÕÌÑ É ÌÉÎÅÊËÉ ÐÏÓÔÒÏÊÔÅ ÔÒÅÕÇÏÌØÎÉË ÐÏ ÔÒÅÍ ×ÙÓÏÔÁÍ.

132.49.

ïËÒÕÖÎÏÓÔØ

S1

ABC × ÔÏÞËÁÈ A É C. ïËÒÕÖÎÏÓÔØ S2 ËÁÓÁÅÔÓÑ
ÐÒÑÍÏÊ AC × ÔÏÞËÅ C É ÐÒÏÈÏÄÉÔ ÞÅÒÅÚ ÔÏÞËÕ B. ïËÒÕÖÎÏÓÔØ S1 ÏÎÁ ÐÅÒÅÓÅËÁÅÔ × ÔÏÞËÅ M. äÏËÁÖÉÔÅ,
ÞÔÏ ÐÒÑÍÁÑ AM ÄÅÌÉÔ ÏÔÒÅÚÏË BC ÐÏÐÏÌÁÍ.
ðÑÔÉÕÇÏÌØÎÉË ABCDE ×ÐÉÓÁÎ × ÏËÒÕÖÎÏÓÔØ. òÁÓÓÔÏÑÎÉÑ ÏÔ ÔÏÞËÉ A ÄÏ ÐÒÑÍÙÈ BC, DC
É DE ÒÁ×ÎÙ ÓÏÏÔ×ÅÔÓÔ×ÅÎÎÏ a, b, c. îÁÊÄÉÔÅ ÒÁÓÓÔÏÑÎÉÅ ÏÔ ×ÅÒÛÉÎÙ A ÄÏ ÐÒÑÍÏÊ BE.
ËÁÓÁÅÔÓÑ ÓÔÏÒÏÎ ÕÇÌÁ

132.50.

211

133. ôÅÏÒÅÍÁ ðÉÆÁÇÏÒÁ

133.1.

(ôÅÏÒÅÍÁ ðÉÆÁÇÏÒÁ) ë×ÁÄÒÁÔ ÇÉÐÏÔÅÎÕÚÙ ÐÒÑÍÏÕÇÏÌØÎÏÇÏ ÔÒÅÕÇÏÌØÎÉËÁ ÒÁ×ÅÎ

ÓÕÍÍÅ Ë×ÁÄÒÁÔÏ× ËÁÔÅÔÏ×. (õËÁÚÁÎÉÅ: ×ÙÓÏÔÁ ÐÒÑÍÏÕÇÏÌØÎÏÇÏ ÔÒÅÕÇÏÌØÎÉËÁ, ÏÐÕÝÅÎÎÁÑ ÉÚ
×ÅÒÛÉÎÙ ÐÒÑÍÏÇÏ ÕÇÌÁ, ÄÅÌÉÔ ÅÇÏ ÎÁ Ä×Á ÔÒÅÕÇÏÌØÎÉËÁ, ÐÏÄÏÂÎÙÈ ÉÓÈÏÄÎÏÍÕ.)
133.2.

(ôÅÏÒÅÍÁ Ï ÓÒÅÄÎÉÈ ÐÒÏÐÏÒÃÉÏÎÁÌØÎÙÈ × ÐÒÑÍÏÕÇÏÌØÎÏÍ ÔÒÅÕÇÏÌØÎÉËÅ.) ëÁÖÄÙÊ

ËÁÔÅÔ ÐÒÑÍÏÕÇÏÌØÎÏÇÏ ÔÒÅÕÇÏÌØÎÉËÁ ÅÓÔØ ÓÒÅÄÎÅÅ ÐÒÏÐÏÒÃÉÏÎÁÌØÎÏÅ ÍÅÖÄÕ ÇÉÐÏÔÅÎÕÚÏÊ É
Ó×ÏÅÊ ÐÒÏÅËÃÉÅÊ ÎÁ ÇÉÐÏÔÅÎÕÚÕ, Á ×ÙÓÏÔÁ, ÐÒÏ×ÅÄÅÎÎÁÑ ÉÚ ×ÅÒÛÉÎÙ ÐÒÑÍÏÇÏ ÕÇÌÁ, | ÓÒÅÄÎÅÅ
ÐÒÏÐÏÒÃÉÏÎÁÌØÎÏÅ ÍÅÖÄÕ ÐÒÏÅËÃÉÑÍÉ ËÁÔÅÔÏ× ÎÁ ÇÉÐÏÔÅÎÕÚÕ.
133.3.
133.4.
133.5.

2 3, 5.
äÁÎÙ ÏÔÒÅÚËÉ a É b. ðÏÓÔÒÏÊÔÅ ÏÔÒÅÚËÉ a2 + b2 ,
a2 - b2, ab.
ëÁÔÅÔÙ ÐÒÑÍÏÕÇÏÌØÎÏÇÏ ÔÒÅÕÇÏÌØÎÉËÁ ÒÁ×ÎÙ 12 É 16. îÁÊÄÉÔÅ ÍÅÄÉÁÎÕ, ÐÒÏ×Åp

äÁÎ ÏÔÒÅÚÏË, ÒÁ×ÎÙÊ 1. ðÏÓÔÒÏÊÔÅ ÏÔÒÅÚËÉ,
ÒÁ×ÎÙÅ p ,
p

p

p

p

ÄÅÎÎÕÀ Ë ÇÉÐÏÔÅÎÕÚÅ.
133.6.

îÁÊÄÉÔÅ ×ÙÓÏÔÕ ÒÁ×ÎÏÂÅÄÒÅÎÎÏÇÏ ÔÒÅÕÇÏÌØÎÉËÁ, ÐÒÏ×ÅÄÅÎÎÕÀ Ë ÂÏËÏ×ÏÊ ÓÔÏÒÏÎÅ,

ÅÓÌÉ ÏÓÎÏ×ÁÎÉÅ ÒÁ×ÎÏ
133.7.

a, Á ÂÏËÏ×ÁÑ ÓÔÏÒÏÎÁ ÒÁ×ÎÁ b.
13, 14, 15. îÁÊÄÉÔÅ ×ÙÓÏÔÕ, ÐÒÏ×ÅÄÅÎÎÕÀ Ë ÂÏÌØ-

äÁÎ ÔÒÅÕÇÏÌØÎÉË ÓÏ ÓÔÏÒÏÎÁÍÉ

ÛÅÊ ÓÔÏÒÏÎÅ.
133.8.

óÆÏÒÍÕÌÉÒÕÊÔÅ ÔÅÏÒÅÍÕ, ÏÂÒÁÔÎÕÀ ÔÅÏÒÅÍÅ ðÉÆÁÇÏÒÁ. ÷ÅÒÎÁ ÌÉ ÏÎÁ?

133.9.

÷ÙÓÏÔÁ

CD ÔÒÅÕÇÏÌØÎÉËÁ ABC ÄÅÌÉÔ ÓÔÏÒÏÎÕ AB ÎÁ ÏÔÒÅÚËÉ AD É BD, ÐÒÉÞÅÍ
AD BD = CD . ÷ÅÒÎÏ ÌÉ, ÞÔÏ ÔÒÅÕÇÏÌØÎÉË ABC ÐÒÑÍÏÕÇÏÌØÎÙÊ?
îÁÊÄÉÔÅ ×ÙÓÏÔÕ ÔÒÁÐÅÃÉÉ, ÂÏËÏ×ÙÅ ÓÔÏÒÏÎÙ ËÏÔÏÒÏÊ ÒÁ×ÎÙ 6 É 8, Á ÏÓÎÏ×ÁÎÉÑ
ÒÁ×ÎÙ 4 É 14.
2



133.10.

133.11.

÷ÙÓÏÔÁ ÒÏÍÂÁ, ÐÒÏ×ÅÄÅÎÎÁÑ ÉÚ ×ÅÒÛÉÎÙ ÔÕÐÏÇÏ ÕÇÌÁ, ÄÅÌÉÔ ÅÇÏ ÓÔÏÒÏÎÕ ÎÁ ÏÔ-

ÒÅÚËÉ ÄÌÉÎÏÊ
133.12.

a É b. îÁÊÄÉÔÅ ÄÉÁÇÏÎÁÌÉ ÒÏÍÂÁ.

÷ ÐÒÑÍÏÕÇÏÌØÎÙÊ ÔÒÅÕÇÏÌØÎÉË ×ÐÉÓÁÎ Ë×ÁÄÒÁÔ ÔÁË, ÞÔÏ ÏÄÎÁ ÉÚ ÅÇÏ ÓÔÏÒÏÎ ÎÁÈÏ-

ÄÉÔÓÑ ÎÁ ÇÉÐÏÔÅÎÕÚÅ. âÏËÏ×ÙÅ ÏÔÒÅÚËÉ ÇÉÐÏÔÅÎÕÚÙ ÒÁ×ÎÙ
133.13.

m É n. îÁÊÄÉÔÅ ÐÌÏÝÁÄØ Ë×ÁÄÒÁÔÁ.

îÁÊÄÉÔÅ ÄÉÁÇÏÎÁÌØ É ÂÏËÏ×ÕÀ ÓÔÏÒÏÎÕ ÒÁ×ÎÏÂÅÄÒÅÎÎÏÊ ÔÒÁÐÅÃÉÉ Ó ÏÓÎÏ×ÁÎÉÑÍÉ

20 É 12, ÅÓÌÉ ÉÚ×ÅÓÔÎÏ, ÞÔÏ ÃÅÎÔÒ Å£ ÏÐÉÓÁÎÎÏÊ ÏËÒÕÖÎÏÓÔÉ ÌÅÖÉÔ ÎÁ ÂÏÌØÛÅÍ ÏÓÎÏ×ÁÎÉÉ.
äÉÁÇÏÎÁÌØ ÒÁ×ÎÏÂÅÄÒÅÎÎÏÊ ÔÒÁÐÅÃÉÉ ÒÁ×ÎÁ a, Á ÓÒÅÄÎÑÑ ÌÉÎÉÑ ÒÁ×ÎÁ b. îÁÊÄÉÔÅ
133.14.

×ÙÓÏÔÕ ÜÔÏÊ ÔÒÁÐÅÃÉÉ.
133.15.

ðÒÑÍÙÅ, ÓÏÄÅÒÖÁÝÉÅ ÂÏËÏ×ÙÅ ÓÔÏÒÏÎÙ ÔÒÁÐÅÃÉÉ ÐÅÒÅÓÅËÁÀÔÓÑ ÐÏÄ ÐÒÑÍÙÍ

ÕÇÌÏÍ. âÏÌØÛÁÑ ÂÏËÏ×ÁÑ ÓÔÏÒÏÎÁ ÔÒÁÐÅÃÉÉ ÒÁ×ÎÁ
ÄÉÔÅ ÍÅÎØÛÕÀ ÂÏËÏ×ÕÀ ÓÔÏÒÏÎÕ.
133.16.

íÅÄÉÁÎÙ, ÐÒÏ×ÅÄÅÎÎÙÅ Ë ËÁÔÅÔÁÍ ÐÒÑÍÏÕÇÏÌØÎÏÇÏ ÔÒÅÕÇÏÌØÎÉËÁ, ÒÁ×ÎÙ

îÁÊÄÉÔÅ ÇÉÐÏÔÅÎÕÚÕ ÔÒÅÕÇÏÌØÎÉËÁ.
133.17.

8, Á ÒÁÚÎÏÓÔØ ÏÓÎÏ×ÁÎÉÊ ÒÁ×ÎÁ 10. îÁÊ-

ä×Å ÓÔÏÒÏÎÙ ÔÒÅÕÇÏÌØÎÉËÁ ÒÁ×ÎÙ

a É b.

a É b. íÅÄÉÁÎÙ, ÐÒÏ×ÅÄÅÎÎÙÅ Ë ÜÔÉÍ ÓÔÏÒÏÎÁÍ,

×ÚÁÉÍÎÏ ÐÅÒÐÅÎÄÉËÕÌÑÒÎÙ. îÁÊÄÉÔÅ ÔÒÅÔØÀ ÓÔÏÒÏÎÕ ÔÒÅÕÇÏÌØÎÉËÁ.

15, Á ÐÒÏÅËÃÉÑ ×ÔÏÒÏÇÏ
16. îÁÊÄÉÔÅ ÇÉÐÏÔÅÎÕÚÕ É ×ÔÏÒÏÊ ËÁÔÅÔ.
îÁ ÓÔÏÒÏÎÕ BC ÒÏÍÂÁ ABCD ÏÐÕÝÅÎÁ ×ÙÓÏÔÁ DE. äÉÁÇÏÎÁÌØ AC ÒÏÍÂÁ ÐÅÒÅÓÅËÁÅÔ ×ÙÓÏÔÕ DE × ÔÏÞËÅ F ÔÁË, ÞÔÏ DF=FE = 5. îÁÊÄÉÔÅ ÓÔÏÒÏÎÕ ÒÏÍÂÁ, ÅÓÌÉ ÉÚ×ÅÓÔÎÏ,
ÞÔÏ AE = 5.
ïËÏÌÏ ÐÒÑÍÏÕÇÏÌØÎÏÇÏ ÔÒÅÕÇÏÌØÎÉËÁ ABC ÏÐÉÓÁÎÁ ÏËÒÕÖÎÏÓÔØ. òÁÓÓÔÏÑÎÉÅ ÏÔ
133.18.

ïÄÉÎ ÉÚ ËÁÔÅÔÏ× ÐÒÑÍÏÕÇÏÌØÎÏÇÏ ÔÒÅÕÇÏÌØÎÉËÁ ÒÁ×ÅÎ

ËÁÔÅÔÁ ÎÁ ÇÉÐÏÔÅÎÕÚÕ ÒÁ×ÎÁ
133.19.

133.20.

212

AB ÄÏ ÐÒÑÍÏÊ, ËÁÓÁÀÝÅÊÓÑ ÏËÒÕÖÎÏÓÔÉ × ÔÏÞËÅ C, ÒÁ×ÎÙ m É n ÓÏÏÔ×ÅÔÓÔ×ÅÎÎÏ. îÁÊÄÉÔÅ ËÁÔÅÔÙ AC É BC.

ËÏÎÃÏ× ÇÉÐÏÔÅÎÕÚÙ
133.21.

÷ ÐÒÑÍÏÕÇÏÌØÎÏÍ ÔÒÅÕÇÏÌØÎÉËÅ ÔÏÞËÁ ËÁÓÁÎÉÑ ×ÐÉÓÁÎÎÏÊ ÏËÒÕÖÎÏÓÔÉ ÄÅÌÉÔ ÇÉ-

ÐÏÔÅÎÕÚÕ ÎÁ ÏÔÒÅÚËÉ ÄÌÉÎÏÊ
133.22.

5 É 12. îÁÊÄÉÔÅ ËÁÔÅÔÙ ÔÒÅÕÇÏÌØÎÉËÁ.

âÉÓÓÅËÔÒÉÓÙ ÔÕÐÙÈ ÕÇÌÏ× ÐÒÉ ÏÓÎÏ×ÁÎÉÉ ÔÒÁÐÅÃÉÉ ÐÅÒÅÓÅËÁÀÔÓÑ ÎÁ ÄÒÕÇÏÍ ÅÅ

ÏÓÎÏ×ÁÎÉÉ. îÁÊÄÉÔÅ ×ÓÅ ÓÔÏÒÏÎÙ ÔÒÁÐÅÃÉÉ, ÅÓÌÉ ÅÅ ×ÙÓÏÔÁ ÒÁ×ÎÁ

15 É 13.

133.23.

12, Á ÂÉÓÓÅËÔÒÉÓÙ ÒÁ×ÎÙ

ïËÒÕÖÎÏÓÔØ, ×ÐÉÓÁÎÎÁÑ × ÔÒÁÐÅÃÉÀ, ÄÅÌÉÔ ÅÅ ÂÏËÏ×ÕÀ ÓÔÏÒÏÎÕ ÎÁ ÏÔÒÅÚËÉ

b. îÁÊÄÉÔÅ ÒÁÄÉÕÓ ÏËÒÕÖÎÏÓÔÉ.



äÁÎÙ ÏËÒÕÖÎÏÓÔÉ ÒÁÄÉÕÓÏ× r É R, ÐÒÉÞ£Í R > r. òÁÓÓÔÏÑÎÉÅ ÍÅÖÄÕ ÉÈ ÃÅÎÔÒÁÍÉ
a É ÂÏÌØÛÅ R + r. îÁÊÄÉÔÅ ÏÔÒÅÚËÉ ÏÂÝÉÈ ×ÎÅÛÎÉÈ É ÏÂÝÉÈ ×ÎÕÔÒÅÎÎÉÈ ËÁÓÁÔÅÌØÎÙÈ,

133.24.

ÒÁ×ÎÏ

ÚÁËÌÀÞÅÎÎÙÅ ÍÅÖÄÕ ÔÏÞËÁÍÉ ËÁÓÁÎÉÑ.

r É R (R > r) ËÁÓÁÀÔÓÑ ×ÎÅÛÎÉÍ ÏÂÒÁÚÏÍ × ÔÏÞËÅ K. ë ÎÉÍ
ÐÒÏ×ÅÄÅÎÙ Ä×Å ÏÂÝÉÅ ×ÎÅÛÎÉÅ ËÁÓÁÔÅÌØÎÙÅ. éÈ ÔÏÞËÉ ËÁÓÁÎÉÑ Ó ÍÅÎØÛÅÊ ÏËÒÕÖÎÏÓÔØÀ | A
É D, Ó ÂÏÌØÛÅÊ | B É C ÓÏÏÔ×ÅÔÓÔ×ÅÎÎÏ. ( ) îÁÊÄÉÔÅ AB É ÏÔÒÅÚÏË MN ÏÂÝÅÊ ×ÎÕÔÒÅÎÎÅÊ
ËÁÓÁÔÅÌØÎÏÊ, ÚÁËÌÀÞÅÎÎÙÊ ÍÅÖÄÕ ×ÎÅÛÎÉÍÉ ËÁÓÁÔÅÌØÎÙÍÉ. ( ) äÏËÁÖÉÔÅ, ÞÔÏ ÕÇÌÙ AKB É
O1MO2 | ÐÒÑÍÙÅ (O1 É O2 | ÃÅÎÔÒÙ ÏËÒÕÖÎÏÓÔÅÊ). ( ) îÁÊÄÉÔÅ ÒÁÄÉÕÓÙ ÏËÒÕÖÎÏÓÔÅÊ,
133.25.

ïËÒÕÖÎÏÓÔÉ ÒÁÄÉÕÓÏ×

Á

Â

×

ËÁÓÁÀÝÉÈÓÑ ÏÂÅÉÈ ÄÁÎÎÙÈ ÏËÒÕÖÎÏÓÔÅÊ É ÉÈ ÏÂÝÅÊ ×ÎÅÛÎÅÊ ËÁÓÁÔÅÌØÎÏÊ.
133.26.

ÒÁ×ÎÁ

îÁÊÄÉÔÅ ÏÓÎÏ×ÁÎÉÅ ÒÁ×ÎÏÂÅÄÒÅÎÎÏÇÏ ÔÒÅÕÇÏÌØÎÉËÁ, ÅÓÌÉ ÅÇÏ ÂÏËÏ×ÁÑ ÓÔÏÒÏÎÁ

a, Á ×ÙÓÏÔÁ, ÏÐÕÝÅÎÎÁÑ ÎÁ ÏÓÎÏ×ÁÎÉÅ, ÒÁ×ÎÁ ÏÔÒÅÚËÕ, ÓÏÅÄÉÎÑÀÝÅÍÕ ÓÅÒÅÄÉÎÕ ÏÓÎÏ-

×ÁÎÉÑ Ó ÓÅÒÅÄÉÎÏÊ ÂÏËÏ×ÏÊ ÓÔÏÒÏÎÙ.

BC ÐÒÑÍÏÕÇÏÌØÎÏÇÏ ÔÒÅÕÇÏÌØÎÉËÁ ABC ËÁË ÎÁ ÄÉÁÍÅÔÒÅ ÐÏÓÔÒÏÅD, ÐÒÉÞÅÍ AD : DB = 1 : 3. ÷ÙÓÏÔÁ,
ÏÐÕÝÅÎÎÁÑ ÎÁ ÇÉÐÏÔÅÎÕÚÕ, ÒÁ×ÎÁ 3. îÁÊÄÉÔÅ ËÁÔÅÔ BC.
÷ ÐÒÑÍÏÕÇÏÌØÎÏÍ ÔÒÅÕÇÏÌØÎÉËÅ ABC ÐÒÏ×ÅÄÅÎÁ ×ÙÓÏÔÁ ÉÚ ×ÅÒÛÉÎÙ C ÐÒÑÍÏÇÏ
133.27.

îÁ ËÁÔÅÔÅ

ÎÁ ÏËÒÕÖÎÏÓÔØ, ÐÅÒÅÓÅËÁÀÝÁÑ ÇÉÐÏÔÅÎÕÚÕ × ÔÏÞËÅ
133.28.

ÕÇÌÁ. îÁ ÜÔÏÊ ×ÙÓÏÔÅ ËÁË ÎÁ ÄÉÁÍÅÔÒÅ ÐÏÓÔÒÏÅÎÁ ÏËÒÕÖÎÏÓÔØ. éÚ×ÅÓÔÎÏ, ÞÔÏ ÜÔÁ ÏËÒÕÖÎÏÓÔØ

12 É 18. îÁÊÄÉÔÅ ËÁÔÅÔÙ ÔÒÅÕÇÏÌØÎÉËÁ ABC.
ABCD ÍÅÎØÛÁÑ ÄÉÁÇÏÎÁÌØ BD ÐÅÒÐÅÎÄÉËÕÌÑÒÎÁ ÏÓÎÏ×ÁÎÉÑÍ AD É
BC; ÓÕÍÍÁ ÏÓÔÒÙÈ ÕÇÌÏ× A É C ÒÁ×ÎÁ 90. ïÓÎÏ×ÁÎÉÑ AD = a, BC = b. îÁÊÄÉÔÅ ÂÏËÏ×ÙÅ
ÓÔÏÒÏÎÙ AB É CD.
ïÔÒÅÚÏË, ÓÏÅÄÉÎÑÀÝÉÊ ÓÅÒÅÄÉÎÙ ÏÓÎÏ×ÁÎÉÊ ÔÒÁÐÅÃÉÉ, ÒÁ×ÅÎ 3. õÇÌÙ ÐÒÉ ÂÏÌØÛÅÍ ÏÓÎÏ×ÁÎÉÉ ÔÒÁÐÅÃÉÉ ÒÁ×ÎÙ 30 É 60. îÁÊÄÉÔÅ ×ÙÓÏÔÕ ÔÒÁÐÅÃÉÉ.
èÏÒÄÙ AB É CD ÏËÒÕÖÎÏÓÔÉ ÒÁÄÉÕÓÁ R ÐÅÒÅÓÅËÁÀÔÓÑ ÐÏÄ ÐÒÑÍÙÍ ÕÇÌÏÍ. îÁÊÄÉÔÅ BD, ÅÓÌÉ AC = a.
îÁ ÇÉÐÏÔÅÎÕÚÅ AB ÐÒÑÍÏÕÇÏÌØÎÏÇÏ ÔÒÅÕÇÏÌØÎÉËÁ ABC Ó ËÁÔÅÔÁÍÉ BC = a É
AC = b ×Ï ×ÎÅÛÎÀÀ ÓÔÏÒÏÎÕ ÐÏÓÔÒÏÅÎ Ë×ÁÄÒÁÔ ABKM. îÁÊÄÉÔÅ ÒÁÓÓÔÏÑÎÉÅ ÏÔ ÔÏÞËÉ C ÄÏ
×ÙÓÅËÁÅÔ ÎÁ ËÁÔÅÔÁÈ ÏÔÒÅÚËÉ, ÒÁ×ÎÙÅ
133.29.

÷ ÔÒÁÐÅÃÉÉ

133.30.

133.31.

133.32.

ÃÅÎÔÒÁ Ë×ÁÄÒÁÔÁ.

M, ÒÁÚÎÏÓÔØ Ë×ÁÄÒÁÔÏ× ÒÁÓÓÔÏÑÎÉÊ ÏÔ ËÏA É B ÐÏÓÔÏÑÎÎÁ.
äÏËÁÖÉÔÅ, ÞÔÏ ÐÒÑÍÙÅ AB É CD ÐÅÒÐÅÎÄÉËÕÌÑÒÎÙ ÔÏÇÄÁ É ÔÏÌØËÏ ÔÏÇÄÁ, ËÏÇÄÁ
AC2 + BD2 = AD2 + BC2.
133.33.

îÁÊÄÉÔÅ ÇÅÏÍÅÔÒÉÞÅÓËÏÅ ÍÅÓÔÏ ÔÏÞÅË

ÔÏÒÙÈ ÄÏ Ä×ÕÈ ÄÁÎÎÙÈ ÔÏÞÅË
133.34.

133.35.

éÓÐÏÌØÚÕÑ ÒÅÚÕÌØÔÁÔ ÐÒÅÄÙÄÕÝÅÊ ÚÁÄÁÞÉ, ÄÏËÁÖÉÔÅ, ÞÔÏ ×ÙÓÏÔÙ ÔÒÅÕÇÏÌØÎÉËÁ

ÐÅÒÅÓÅËÁÀÔÓÑ × ÏÄÎÏÊ ÔÏÞËÅ.

213

÷ÙÓÏÔÙ ÔÒÅÕÇÏÌØÎÉËÁ ÒÁ×ÎÙ

133.36.

ÍÏÕÇÏÌØÎÙÊ.

12, 15 É 20. äÏËÁÖÉÔÅ ÞÔÏ ÜÔÏÔ ÔÒÅÕÇÏÌØÎÉË ÐÒÑ-

AB É CD, M | ÎÅËÏÔÏÒÁÑ ÔÏÞËÁ. éÚ×ÅÓÔÎÏ,
ÞÔÏ AM = 15, BM = 20, CM = 24. îÁÊÄÉÔÅ DM.
133.37.

÷ ËÒÕÇÅ ÐÒÏ×ÅÄÅÎÙ Ä×Á ÄÉÁÍÅÔÒÁ

133.38.

îÁÊÄÉÔÅ ÇÅÏÍÅÔÒÉÞÅÓËÏÅ ÍÅÓÔÏ ÔÏÞÅË, ËÁÓÁÔÅÌØÎÙÅ ÉÚ ËÏÔÏÒÙÈ, ÐÒÏ×ÅÄÅÎÎÙÅ Ë

Ä×ÕÍ ÄÁÎÎÙÍ ÏËÒÕÖÎÏÓÔÑÍ, ÒÁ×ÎÙ ÍÅÖÄÕ ÓÏÂÏÊ.

C ÐÒÑÍÏÇÏ ÕÇÌÁ ÐÒÑÍÏÕÇÏÌØÎÏÇÏ ÔÒÅÕÇÏÌØÎÉËÁ ABC ÏÐÕÝÅÎ ÐÅÒCD ÎÁ ÇÉÐÏÔÅÎÕÚÕ AD. òÁÄÉÕÓÙ ÏËÒÕÖÎÏÓÔÅÊ, ×ÐÉÓÁÎÎÙÈ × ÔÒÅÕÇÏÌØÎÉËÉ CAD
É CBD, ÒÁ×ÎÙ ÓÏÏÔ×ÅÔÓÔ×ÅÎÎÏ 3 É 4. îÁÊÄÉÔÅ ÒÁÄÉÕÓ ÏËÒÕÖÎÏÓÔÉ, ×ÐÉÓÁÎÎÏÊ × ÔÒÅÕÇÏÌØÎÉË
ABC.
éÚ ×ÅÒÛÉÎÙ C ÐÒÑÍÏÇÏ ÕÇÌÁ ÐÒÑÍÏÕÇÏÌØÎÏÇÏ ÔÒÅÕÇÏÌØÎÉËÁ ABC ÏÐÕÝÅÎ ÐÅÒÐÅÎÄÉËÕÌÑÒ CD ÎÁ ÇÉÐÏÔÅÎÕÚÕ AD. òÁÓÓÔÏÑÎÉÅ ÍÅÖÄÕ ÃÅÎÔÒÁÍÉ ÏËÒÕÖÎÏÓÔÅÊ, ×ÐÉÓÁÎÎÙÈ ×
ÔÒÅÕÇÏÌØÎÉËÉ CAD É CBD, ÒÁ×ÎÏ 1. îÁÊÄÉÔÅ ÒÁÄÉÕÓ ÏËÒÕÖÎÏÓÔÉ, ×ÐÉÓÁÎÎÏÊ × ÔÒÅÕÇÏÌØÎÉË
ABC.
éÚ ×ÅÒÛÉÎÙ

133.39.

ÐÅÎÄÉËÕÌÑÒ

133.40.

133.41.

ïËÒÕÖÎÏÓÔØ, ËÁÓÁÀÝÁÑÓÑ ÓÔÏÒÏÎÙ ÔÒÅÕÇÏÌØÎÉËÁ É ÐÒÏÄÏÌÖÅÎÉÊ Ä×ÕÈ ÅÇÏ ÄÒÕÇÉÈ

ÓÔÏÒÏÎ, ÎÁÚÙ×ÁÅÔÓÑ ×ÎÅ×ÐÉÓÁÎÎÏÊ ÏËÒÕÖÎÏÓÔØÀ ÔÒÅÕÇÏÌØÎÉËÁ. îÁÊÄÉÔÅ ÒÁÄÉÕÓ ×ÎÅ×ÐÉÓÁÎÎÏÊ
ÏËÒÕÖÎÏÓÔÉ, ËÁÓÁÀÝÅÊÓÑ ÍÅÎØÛÅÊ ÓÔÏÒÏÎÙ ÐÒÑÍÏÕÇÏÌØÎÏÇÏ ÔÒÅÕÇÏÌØÎÉËÁ Ó ÏÓÔÒÙÍ ÕÇÌÏÍ,
ÒÁ×ÎÙÍ

30, ÅÓÌÉ ÒÁÄÉÕÓ ×ÐÉÓÁÎÎÏÊ ÏËÒÕÖÎÏÓÔÉ ÒÁ×ÅÎ r.

133.42.

ÒÏÎÁÍÉ (Á)

îÁÊÄÉÔÅ ÒÁÄÉÕÓÙ ×ÐÉÓÁÎÎÏÊ É ×ÎÅ×ÐÉÓÁÎÎÙÈ ÏËÒÕÖÎÏÓÔÅÊ ÔÒÅÕÇÏÌØÎÉËÁ ÓÏ ÓÔÏ-

5, 12, 13; ( ) 10, 10, 12.
Â

C, ÐÒÏ×ÅÄÅÎÁ ÏÂÝÁÑ
×ÎÅÛÎÑÑ ËÁÓÁÔÅÌØÎÁÑ, A É B | ÔÏÞËÉ ËÁÓÁÎÉÑ. îÁÊÄÉÔÅ ÒÁÄÉÕÓÙ ÏËÒÕÖÎÏÓÔÅÊ, ÅÓÌÉ AC = 6,
BC = 8.
þÅÔÙÒÅÈÕÇÏÌØÎÉË ABCD ×ÐÉÓÁÎ × ÏËÒÕÖÎÏÓÔØ ÒÁÄÉÕÓÁ R. åÇÏ ÄÉÁÇÏÎÁÌÉ ×ÚÁÉÍÎÏ
ÐÅÒÐÅÎÄÉËÕÌÑÒÎÙ É ÐÅÒÅÓÅËÁÀÔÓÑ × ÔÏÞËÅ P. îÁÊÄÉÔÅ AP2 + BP2 + CP2 + DP2 É AB2 + BC2 +
CD2 + AD2.
ôÒÉ ÏËÒÕÖÎÏÓÔÉ ÒÁÄÉÕÓÏ× 1, 2 É 3 ÓÍ ËÁÓÁÀÔÓÑ ÄÒÕÇ ÄÒÕÇÁ ×ÎÅÛÎÉÍ ÏÂÒÁÚÏÍ.
133.43.

ë Ä×ÕÍ ÏËÒÕÖÎÏÓÔÑÍ, ËÁÓÁÀÝÉÍÓÑ ×ÎÅÛÎÉÍ ÏÂÒÁÚÏÍ × ÔÏÞËÅ

133.44.

133.45.

îÁÊÄÉÔÅ ÒÁÄÉÕÓ ÏËÒÕÖÎÏÓÔÉ, ÐÒÏÈÏÄÑÝÅÊ ÞÅÒÅÚ ÔÏÞËÉ ËÁÓÁÎÉÑ ÜÔÉÈ ÏËÒÕÖÎÏÓÔÅÊ.

ABC (\A = 90 É Ä×Á Ë×ÁÄÒÁÔÁ BEFC É AMNC
ÒÁÓÐÏÌÏÖÅÎÙ ÔÁË, ÞÔÏ ÔÏÞËÉ E É A ÌÅÖÁÔ ÐÏ ÒÁÚÎÙÅ ÓÔÏÒÏÎÙ ÏÔ ÐÒÑÍÏÊ BC, Á ÔÏÞËÉ M É B
| ÐÏ ÒÁÚÎÙÅ ÓÔÏÒÏÎÙ ÏÔ ÐÒÑÍÏÊ AC. îÁÊÄÉÔÅ ÒÁÓÓÔÏÑÎÉÅ ÍÅÖÄÕ ÃÅÎÔÒÁÍÉ Ë×ÁÄÒÁÔÏ×, ÅÓÌÉ
AB = a, AC = b.
éÚ ×ÅÒÛÉÎÙ C ÐÒÑÍÏÇÏ ÕÇÌÁ ÐÒÑÍÏÕÇÏÌØÎÏÇÏ ÔÒÅÕÇÏÌØÎÉËÁ ABC ÐÒÏ×ÅÄÅÎÁ ×ÙÓÏÔÁ CD, É × ÔÒÅÕÇÏÌØÎÉËÉ ACD É BCD ×ÐÉÓÁÎÙ ÏËÒÕÖÎÏÓÔÉ Ó ÃÅÎÔÒÁÍÉ P É Q. ïÂÝÁÑ
×ÎÅÛÎÑÑ ËÁÓÁÔÅÌØÎÁÑ Ë ÜÔÉÍ ÏËÒÕÖÎÏÓÔÑÍ ÐÅÒÅÓÅËÁÅÔ ËÁÔÅÔÙ AC É BC × ÔÏÞËÁÈ M É N, Á
×ÙÓÏÔÕ CD | × ÔÏÞËÅ K. äÏËÁÖÉÔÅ, ÞÔÏ ( ) ÔÒÅÕÇÏÌØÎÉËÉ CMN É CBA ÐÏÄÏÂÎÙ; ( ) ÔÏÞËÉ
C, M, N, P É Q ÌÅÖÁÔ ÎÁ ÏÄÎÏÊ ÏËÒÕÖÎÏÓÔÉ Ó ÃÅÎÔÒÏÍ K, ÒÁÄÉÕÓ ËÏÔÏÒÏÊ ÒÁ×ÅÎ ÒÁÄÉÕÓÕ
×ÐÉÓÁÎÎÏÊ ÏËÒÕÖÎÏÓÔÉ ÔÒÅÕÇÏÌØÎÉËÁ ABC.
133.46.

ðÒÑÍÏÕÇÏÌØÎÙÊ ÔÒÅÕÇÏÌØÎÉË

133.47.

Á

Â

214

134. ôÒÉÇÏÎÏÍÅÔÒÉÞÅÓËÉÅ ÆÕÎËÃÉÉ ÏÓÔÒÏÇÏ ÕÇÌÁ

óÉÎÕÓÏÍ ÏÓÔÒÏÇÏ ÕÇÌÁ ÐÒÑÍÏÕÇÏÌØÎÏÇÏ ÔÒÅÕÇÏÌØÎÉËÁ ÎÁÚÙ×ÁÅÔÓÑ ÏÔÎÏÛÅÎÉÅ ËÁÔÅÔÁ, ÐÒÏÔÉ×ÏÌÅÖÁÝÅÇÏ ÜÔÏÍÕ ÕÇÌÕ, Ë ÇÉÐÏÔÅÎÕÚÅ, ËÏÓÉÎÕÓÏÍ | ÏÔÎÏÛÅÎÉÅ ÐÒÉÌÅÖÁÝÅÇÏ ËÁÔÅÔÁ Ë
ÇÉÐÏÔÅÎÕÚÅ, ÔÁÎÇÅÎÓÏÍ | ÏÔÎÏÛÅÎÉÅ ÐÒÏÔÉ×ÏÌÅÖÁÝÅÇÏ ËÁÔÅÔÁ Ë ÐÒÉÌÅÖÁÝÅÍÕ, ËÏÔÁÎÇÅÎÓÏÍ
| ÐÒÉÌÅÖÁÝÅÇÏ ËÁÔÅÔÁ Ë ÐÒÏÔÉ×ÏÌÅÖÁÝÅÍÕ.

AC É BC | ËÁÔÅÔÙ ÐÒÑÍÏÕÇÏÌØÎÏÇÏ ÔÒÅÕÇÏÌØÎÉËÁ ABC, Á AB
BC = a, AC = b, AB = , ÔÏ
b
a
b
a
sin \A = ; cos \A = ; tg \A = ; ctg \A = ;
c
c
b
a
b
a
b
a
sin \B = ; cos \B = ; tg \A = ; ctg \A = :
c
c
a
b

äÒÕÇÉÍÉ ÓÌÏ×ÁÍÉ, ÅÓÌÉ
| ÅÇÏ ÇÉÐÏÔÅÎÕÚÁ, É

134.1.

äÏËÁÖÉÔÅ, ÞÔÏ ÓÉÎÕÓ ÏÓÔÒÏÇÏ ÕÇÌÁ ÚÁ×ÉÓÉÔ ÔÏÌØËÏ ÏÔ ÇÒÁÄÕÓÎÏÊ ÍÅÒÙ ÕÇÌÁ, ÔÏ ÅÓÔØ

ÞÔÏ ÒÁ×ÎÙÅ ÕÇÌÙ ÉÍÅÀÔ ÒÁ×ÎÙÅ ÓÉÎÕÓÙ. (áÎÁÌÏÇÉÞÎÏ ÄÌÑ ËÏÓÉÎÕÓÏ× É Ô.Ä.)
134.2.

îÁÊÄÉÔÅ ÓÉÎÕÓ, ËÏÓÉÎÕÓ, ÔÁÎÇÅÎÓ É ËÏÔÁÎÇÅÎÓ ÕÇÌÏ× ×

134.3.

äÏËÁÖÉÔÅ, ÞÔÏ ÄÌÑ ÌÀÂÏÇÏ ÏÓÔÒÏÇÏ ÕÇÌÁ
sin
cos

30, 60 É 45.

1 ;
= tg ; cos
=
ctg ; ctg =
sin
tg

+ cos2 = 1; 1 + tg2 = cos12 ; 1 + ctg2 = 12 :
sin
ôÁÎÇÅÎÓ ÏÓÔÒÏÇÏ ÕÇÌÁ
ÒÁ×ÅÎ 2. îÁÊÄÉÔÅ sin .
÷ ÐÒÑÍÏÕÇÏÌØÎÏÍ ÔÒÅÕÇÏÌØÎÉËÅ ABC (ÕÇÏÌ C ÐÒÑÍÏÊ) AB = 4, \A = 60. îÁÊÄÉÔÅ BC É AC.
÷ ÐÒÑÍÏÕÇÏÌØÎÏÍ ÔÒÅÕÇÏÌØÎÉËÅ ABC (ÕÇÏÌ C ÐÒÑÍÏÊ) \A = , BC = a. îÁÊÄÉÔÅ
sin

2

134.4.

134.5.

134.6.

ÇÉÐÏÔÅÎÕÚÕ É ×ÔÏÒÏÊ ËÁÔÅÔ.

6

30o. îÁÊÄÉÔÅ ×ÙÓÏÔÕ ÒÏÍÂÁ É ÅÇÏ ÄÉÁÇÏÎÁÌÉ.

134.7.

óÔÏÒÏÎÁ ÒÏÍÂÁ ÒÁ×ÎÁ , ÏÓÔÒÙÊ ÕÇÏÌ

134.8.

îÁÊÄÉÔÅ ×ÙÓÏÔÕ É ÒÁÄÉÕÓÙ ×ÐÉÓÁÎÎÏÊ É ÏÐÉÓÁÎÎÏÊ ÏËÒÕÖÎÏÓÔÉ ÒÁ×ÎÏÓÔÏÒÏÎÎÅÇÏ

ÔÒÅÕÇÏÌØÎÉËÁ ÓÏ ÓÔÏÒÏÎÏÊ
134.9.

a.

îÁÊÄÉÔÅ ×ÙÓÏÔÕ ÏÓÔÒÏÕÇÏÌØÎÏÇÏ ÔÒÅÕÇÏÌØÎÉËÁ, ÐÒÏ×ÅÄÅÎÎÕÀ ÉÚ ×ÅÒÛÉÎÙ ÐÒÑÍÏÇÏ

8, Á ÏÄÉÎ ÉÚ ÏÓÔÒÙÈ ÕÇÌÏ× ÒÁ×ÅÎ 60.
÷ÅÒÛÉÎÁ M ÐÒÁ×ÉÌØÎÏÇÏ ÔÒÅÕÇÏÌØÎÉËÁ ABM ÓÏ ÓÔÏÒÏÎÏÊ a ÒÁÓÐÏÌÏÖÅÎÁ ÎÁ
ÓÔÏÒÏÎÅ CD ÐÒÑÍÏÕÇÏÌØÎÉËÁ ABCD. îÁÊÄÉÔÅ ÄÉÁÇÏÎÁÌØ
ÐÒÑÍÏÕÇÏÌØÎÉËÁ ABCD.
p
äÁÎ ÐÒÑÍÏÕÇÏÌØÎÉË ÓÏ ÓÔÏÒÏÎÁÍÉ 4 3 É 4. îÁÊÄÉÔÅ ÅÇÏ ÄÉÁÇÏÎÁÌÉ É ÏÓÔÒÙÊ

ÕÇÌÁ, ÅÓÌÉ ÇÉÐÏÔÅÎÕÚÁ ÒÁ×ÎÁ
134.10.

134.11.

ÕÇÏÌ ÍÅÖÄÕ ÎÉÍÉ.
134.12.

÷ ÒÁ×ÎÏÂÅÄÒÅÎÎÏÍ ÔÒÅÕÇÏÌØÎÉËÅ

×ÁÎÉÅ ÒÁ×ÎÏ
134.13.

ÒÁ×ÅÎ

45

8. îÁÊÄÉÔÅ ÂÏËÏ×ÙÅ ÓÔÏÒÏÎÙ.

ABC ÕÇÏÌ ÐÒÉ ×ÅÒÛÉÎÅ B ÒÁ×ÅÎ 120, Á ÏÓÎÏ-

ïÓÎÏ×ÁÎÉÑ ÒÁ×ÎÏÂÅÄÒÅÎÎÏÊ ÔÒÁÐÅÃÉÉ ÒÁ×ÎÙ

8 É 4, ÕÇÏÌ ÐÒÉ ÂÏÌØÛÅÍ ÏÓÎÏ×ÁÎÉÉ

ïÓÎÏ×ÁÎÉÑ ÐÒÑÍÏÕÇÏÌØÎÏÊ ÔÒÁÐÅÃÉÉ ÒÁ×ÎÙ

6 É 8. ïÄÉÎ ÉÚ ÕÇÌÏ× ÐÒÉ ÍÅÎØÛÅÍ

o . îÁÊÄÉÔÅ ÄÉÁÇÏÎÁÌÉ ÔÒÁÐÅÃÉÉ.

134.14.

ÏÓÎÏ×ÁÎÉÉ ÒÁ×ÅÎ

120

. îÁÊÄÉÔÅ ÄÉÁÇÏÎÁÌÉ ÔÒÁÐÅÃÉÉ.

215

íÅÄÉÁÎÁ ÐÒÑÍÏÕÇÏÌØÎÏÇÏ ÔÒÅÕÇÏÌØÎÉËÁ, ÐÒÏ×ÅÄÅÎÎÁÑ Ë ÇÉÐÏÔÅÎÕÚÅ, ÒÁ×ÎÁ

134.15.

ÄÅÌÉÔ ÐÒÑÍÏÊ ÕÇÏÌ × ÏÔÎÏÛÅÎÉÉ

1 : 2. îÁÊÄÉÔÅ ÓÔÏÒÏÎÙ ÔÒÅÕÇÏÌØÎÉËÁ.

îÁÊÄÉÔÅ ÐÌÏÝÁÄØ ÐÒÑÍÏÕÇÏÌØÎÏÊ ÔÒÁÐÅÃÉÉ, ÅÓÌÉ ÅÅ ÏÓÔÒÙÊ ÕÇÏÌ ÒÁ×ÅÎ

134.16.

ÛÅÅ ÏÓÎÏ×ÁÎÉÅ ÒÁ×ÎÏ
134.17.

a, ÂÏÌØÛÁÑ ÂÏËÏ×ÁÑ ÓÔÏÒÏÎÁ ÒÁ×ÎÁ b.

ëÏÓÉÎÕÓ ÕÇÌÁ ÐÒÉ ÏÓÎÏ×ÁÎÉÉ ÒÁ×ÎÏÂÅÄÒÅÎÎÏÇÏ ÔÒÅÕÇÏÌØÎÉËÁ ÒÁ×ÅÎ

12 É

60, ÍÅÎØ-

3=5, ×ÙÓÏÔÁ,

h. îÁÊÄÉÔÅ ×ÙÓÏÔÕ, ÏÐÕÝÅÎÎÕÀ ÎÁ ÂÏËÏ×ÕÀ ÓÔÏÒÏÎÕ.
èÏÒÄÁ AC ÏËÒÕÖÎÏÓÔÉ ÒÁÄÉÕÓÁ R ÏÂÒÁÚÕÅÔ Ó ÄÉÁÍÅÔÒÏÍ AB ÕÇÏÌ, ÒÁ×ÎÙÊ .
îÁÊÄÉÔÅ ÒÁÓÓÔÏÑÎÉÅ ÏÔ ÔÏÞËÉ C ÄÏ ÄÉÁÍÅÔÒÁ AB.
çÉÐÏÔÅÎÕÚÁ ÐÒÑÍÏÕÇÏÌØÎÏÇÏ ÔÒÅÕÇÏÌØÎÉËÁ ÒÁ×ÎÁ a, ÏÄÉÎ ÉÚ ÏÓÔÒÙÈ ÕÇÌÏ× ÒÁ×ÅÎ
ÏÐÕÝÅÎÎÁÑ ÎÁ ÏÓÎÏ×ÁÎÉÅ, ÒÁ×ÎÁ
134.18.

134.19.

. îÁÊÄÉÔÅ ÒÁÓÓÔÏÑÎÉÑ ÏÔ ÏÓÎÏ×ÁÎÉÑ ×ÙÓÏÔÙ, ÏÐÕÝÅÎÎÏÊ ÎÁ ÇÉÐÏÔÅÎÕÚÕ, ÄÏ ËÁÔÅÔÏ× ÔÒÅÕÇÏÌØ-

ÎÉËÁ.

éÚ ÔÏÞËÉ M ÐÒÏ×ÅÄÅÎÙ ËÁÓÁÔÅÌØÎÙÅ MA É MB Ë ÏËÒÕÖÎÏÓÔÉ Ó ÃÅÎÔÒÏÍ O (A
B | ÔÏÞËÉ ËÁÓÁÎÉÑ). îÁÊÄÉÔÅ ÒÁÄÉÕÓ ÏËÒÕÖÎÏÓÔÉ, ÅÓÌÉ \AMB = É AB = a.
134.20.

É

134.21.

ä×Å ×ÅÒÛÉÎÙ Ë×ÁÄÒÁÔÁ ÒÁÓÐÏÌÏÖÅÎÙ ÎÁ ÏÓÎÏ×ÁÎÉÉ ÒÁ×ÎÏÂÅÄÒÅÎÎÏÇÏ ÔÒÅÕÇÏÌØ-

ÎÉËÁ, Á Ä×Å ÄÒÕÇÉÅ - ÎÁ ÅÇÏ ÂÏËÏ×ÙÈ ÓÔÏÒÏÎÁÈ. îÁÊÄÉÔÅ ÓÔÏÒÏÎÕ Ë×ÁÄÒÁÔÁ, ÅÓÌÉ ÏÓÎÏ×ÁÎÉÅ
ÔÒÅÕÇÏÌØÎÉËÁ ÒÁ×ÎÏ

a, Á ÕÇÏÌ ÐÒÉ ÏÓÎÏ×ÁÎÉÉ ÒÁ×ÅÎ 30.

134.22.

ïÂÝÁÑ ÈÏÒÄÁ Ä×ÕÈ ÐÅÒÅÓÅËÁÀÝÉÈÓÑ ÏËÒÕÖÎÏÓÔÅÊ ×ÉÄÎÁ ÉÚ ÉÈ ÃÅÎÔÒÏ× ÐÏÄ ÕÇÌÁÍÉ

134.23.

.

90o É 60o . îÁÊÄÉÔÅ ÒÁÄÉÕÓÙ ÏËÒÕÖÎÏÓÔÅÊ, ÅÓÌÉ ÒÁÓÓÔÏÑÎÉÅ ÍÅÖÄÕ ÉÈ ÃÅÎÔÒÁÍÉ ÒÁ×ÎÏ a.
òÁÄÉÕÓ ÏËÒÕÖÎÏÓÔÉ, ×ÐÉÓÁÎÎÏÊ × ÒÏÍÂ, ÒÁ×ÅÎ r, Á ÏÓÔÒÙÊ ÕÇÏÌ ÒÏÍÂÁ ÒÁ×ÅÎ
îÁÊÄÉÔÅ ÓÔÏÒÏÎÕ ÒÏÍÂÁ.
134.24.

÷ ÐÒÑÍÏÕÇÏÌØÎÙÊ ÔÒÅÕÇÏÌØÎÉË Ó ÕÇÌÏÍ

ÔÁË, ÞÔÏ ÕÇÏÌ ×

60 Õ ÎÉÈ ÏÂÝÉÊ É ×ÓÅ ×ÅÒÛÉÎÙ ÒÏÍÂÁ ÌÅÖÁÔ ÎÁ ÓÔÏÒÏÎÁÈ ÔÒÅÕÇÏÌØÎÉËÁ.

îÁÊÄÉÔÅ ÓÔÏÒÏÎÙ ÔÒÅÕÇÏÌØÎÉËÁ.

÷ÙÓÏÔÁ ÔÒÅÕÇÏÌØÎÉËÁ

134.25.

\C =

ABC, ÏÐÕÝÅÎÎÁÑ

ÎÁ ÓÔÏÒÏÎÕ

. îÁÊÄÉÔÅ ÏÓÔÁÌØÎÙÅ ×ÙÓÏÔÙ ÜÔÏÇÏ ÔÒÅÕÇÏÌØÎÉËÁ.

134.26.

ÒÁ×ÎÁ

60 ×ÐÉÓÁÎ ÒÏÍ ÓÏ ÓÔÏÒÏÎÏÊ, ÒÁ×ÎÏÊ 6,
BC, ÒÁ×ÎÁ h, Á \B =

÷ÙÓÏÔÁ ÐÒÑÍÏÕÇÏÌØÎÏÇÏ ÔÒÅÕÇÏÌØÎÉËÁ, ÐÒÏ×ÅÄÅÎÎÁÑ ÉÚ ×ÅÒÛÉÎÙ ÐÒÑÍÏÇÏ ÕÇÌÁ,

a É ÏÂÒÁÚÕÅÔ ÕÇÏÌ

Ó ÍÅÄÉÁÎÏÊ, ÐÒÏ×ÅÄÅÎÎÏÊ ÉÚ ÔÏÊ ÖÅ ×ÅÒÛÉÎÙ. îÁÊÄÉÔÅ ËÁÔÅÔÙ

ÔÒÅÕÇÏÌØÎÉËÁ.
134.27.

òÁÄÉÕÓ ÏËÒÕÖÎÏÓÔÉ, ÏÐÉÓÁÎÎÏÊ ÏËÏÌÏ ÒÁ×ÎÏÂÅÄÒÅÎÎÏÇÏ ÔÒÅÕÇÏÌØÎÉËÁ, ÒÁ×ÅÎ

õÇÏÌ ÐÒÉ ÏÓÎÏ×ÁÎÉÉ ÒÁ×ÅÎ

p

134.28.

. îÁÊÄÉÔÅ ÓÔÏÒÏÎÙ ÔÒÅÕÇÏÌØÎÉËÁ.

óÔÏÒÏÎÁ ÔÒÅÕÇÏÌØÎÉËÁ ÒÁ×ÎÁ 2, ÐÒÉÌÅÖÁÝÉÅ Ë ÎÅÊ ÕÇÌÙ ÒÁ×ÎÙ

ÏÓÔÁÌØÎÙÅ ÓÔÏÒÏÎÙ ÔÒÅÕÇÏÌØÎÉËÁ.
134.30.

äÉÁÇÏÎÁÌØ ÒÁ×ÎÏÂÅÄÒÅÎÎÏÊ ÔÒÁÐÅÃÉÉ ÒÁ×ÎÁ

ÏÓÎÏ×ÁÎÉÅÍ. îÁÊÄÉÔÅ ÏÓÎÏ×ÁÎÉÑ ÔÒÁÐÅÃÉÉ.
134.31.

R.

òÁÄÉÕÓ ÏËÒÕÖÎÏÓÔÉ, ×ÐÉÓÁÎÎÏÊ × ÐÒÑÍÏÕÇÏÌØÎÙÊ ÔÒÅÕÇÏÌØÎÉË Ó ÏÓÔÒÙÍ ÕÇÌÏÍ

60o, ÒÁ×ÅÎ 3. îÁÊÄÉÔÅ ÓÔÏÒÏÎÙ ÔÒÅÕÇÏÌØÎÉËÁ.
134.29.

,

óÔÏÒÏÎÙ ÐÁÒÁÌÌÅÌÏÇÒÁÍÍÁ ÒÁ×ÎÙ

30 É 45. îÁÊÄÉÔÅ

a É ÏÂÒÁÚÕÅÔ ÕÇÌÙ

É

a É b, Á ÕÇÏÌ ÍÅÖÄÕ ÎÉÍÉ ÒÁ×ÅÎ

Ó ÂÏÌØÛÉÍ
. îÁÊÄÉÔÅ

ÓÔÏÒÏÎÙ É ÄÉÁÇÏÎÁÌÉ ÞÅÔÙÒÅÈÕÇÏÌØÎÉËÁ, ÏÂÒÁÚÏ×ÁÎÎÏÇÏ ÐÅÒÅÓÅÞÅÎÉÅÍ ÂÉÓÓÅËÔÒÉÓ ×ÎÕÔÒÅÎÎÉÈ ÕÇÌÏ× ÐÁÒÁÌÌÅÌÌÏÇÒÁÍÍÁ.
134.32.
134.33.

15 É tg 75.
îÁÊÄÉÔÅ sin 18.
îÁÊÄÉÔÅ sin

216

135. ðÒÏÐÏÒÃÉÏÎÁÌØÎÙÅ ÏÔÒÅÚËÉ × ËÒÕÇÅ

äÏËÁÚÁÔØ, ÞÔÏ ÐÒÏÉÚ×ÅÄÅÎÉÑ ÏÔÒÅÚËÏ× ÐÅÒÅÓÅËÁÀÝÉÈÓÑ ÈÏÒÄ ÏËÒÕÖÎÏÓÔÉ ÒÁ×ÎÙ.
135.2. (ôÅÏÒÅÍÁ Ï ËÁÓÁÔÅÌØÎÏÊ É ÓÅËÕÝÅÊ) äÏËÁÚÁÔØ, ÞÔÏ ÅÓÌÉ ÉÚ ÏÄÎÏÊ ÔÏÞËÉ ÐÒÏ×ÅÄÅÎÙ
Ë ÏËÒÕÖÎÏÓÔÉ ËÁÓÁÔÅÌØÎÁÑ É ÓÅËÕÝÁÑ, ÔÏ ÐÒÏÉÚ×ÅÄÅÎÉÅ ×ÓÅÊ ÓÅËÕÝÅÊ ÎÁ ÅÅ ×ÎÅÛÎÀÀ ÞÁÓÔØ
ÒÁ×ÎÏ Ë×ÁÄÒÁÔÕ ËÁÓÁÔÅÌØÎÏÊ.
135.3. ôÏÞËÁ P ÕÄÁÌÅÎÎÁ ÎÁ ÒÁÓÓÔÏÑÎÉÅ 7 ÏÔ ÃÅÎÔÒÁ ÏËÒÕÖÎÏÓÔÉ ÒÁÄÉÕÓÁ 11. þÅÒÅÚ ÔÏÞËÕ
P ÐÒÏ×ÅÄÅÎÁ ÈÏÒÄÁ, ÒÁ×ÎÁÑ 18. îÁÊÄÉÔÅ ÏÔÒÅÚËÉ, ÎÁ ËÏÔÏÒÙÅ ÄÅÌÉÔÓÑ ÈÏÒÄÁ ÔÏÞËÏÊ P.
135.4. äÉÁÇÏÎÁÌÉ ×ÐÉÓÁÎÎÏÇÏ ÞÅÔÙÒÅÈÕÇÏÌØÎÉËÁ ABCD ÐÅÒÅÓÅËÁÀÔÓÑ × ÔÏÞËÅ K. éÚ×ÅÓÔÎÏ, ÞÔÏ AB = a, BK = b, AK = c, CD = d. îÁÊÄÉÔÅ AC.
135.5. ôÏÞËÁ M ÌÅÖÉÔ ×ÎÕÔÒÉ ÏËÒÕÖÎÏÓÔÉ ÒÁÄÉÕÓÁ R É ÕÄÁÌÅÎÁ ÏÔ ÃÅÎÔÒÁ ÎÁ ÒÁÓÓÔÏÑÎÉÅ d. äÏËÁÖÉÔÅ, ÞÔÏ ÄÌÑ ×ÓÅÈ ÈÏÒÄ AB ÜÔÏÊ ÏËÒÕÖÎÏÓÔÉ, ÐÒÏÈÏÄÑÝÉÈ ÞÅÒÅÚ ÔÏÞËÕ M,
ÐÒÏÉÚ×ÅÄÅÎÉÅ AM  BM ÏÄÎÏ É ÔÏ ÖÅ. þÅÍÕ ÏÎÏ ÒÁ×ÎÏ?
135.6. ôÏÞËÁ M ÌÅÖÉÔ ×ÎÅ ÏËÒÕÖÎÏÓÔÉ ÒÁÄÉÕÓÁ R É ÕÄÁÌÅÎÁ ÏÔ Å£ ÃÅÎÔÒÁ ÎÁ ÒÁÓÓÔÏÑÎÉÅ
d. äÏËÁÖÉÔÅ, ÞÔÏ ÄÌÑ ×ÓÅÈ ÐÒÑÍÙÈ, ÐÒÏÈÏÄÑÝÉÈ ÞÅÒÅÚ ÔÏÞËÕ M É ÐÅÒÅÓÅËÁÀÝÉÈ ÏËÒÕÖÎÏÓÔØ
× ÔÏÞËÁÈ A É B, ÐÒÏÉÚ×ÅÄÅÎÉÅ AM  BM ÏÄÎÏ É ÔÏ ÖÅ. þÅÍÕ ÏÎÏ ÒÁ×ÎÏ ?
135.7. éÚ ÔÏÞËÉ A ÐÒÏ×ÅÄÅÎÙ Ä×Á ÌÕÞÁ, ÐÅÒÅÓÅËÁÀÝÉÅ ÄÁÎÎÕÀ ÏËÒÕÖÎÏÓÔØ: ÏÄÉÎ | ×
ÔÏÞËÁÈ B É C, ÄÒÕÇÏÊ | × ÔÏÞËÁÈ D É E. éÚ×ÅÓÔÎÏ, ÞÔÏ AB = 7, BC = 7, AD = 10. îÁÊÄÉÔÅ
DE.
135.8. ÷ ÐÒÑÍÏÕÇÏÌØÎÏÍ ÔÒÅÕÇÏÌØÎÉËÅ ABC Ó ÐÒÑÍÙÍ ÕÇÌÏÍ ÐÒÉ ×ÅÒÛÉÎÅ C ËÁÔÅÔ BC
ÒÁ×ÅÎ a, ÒÁÄÉÕÓ ×ÐÉÓÁÎÎÏÊ ÏËÒÕÖÎÏÓÔÉ ÒÁ×ÅÎ r. ÷ÐÉÓÁÎÎÁÑ ÏËÒÕÖÎÏÓÔØ ËÁÓÁÅÔÓÑ ËÁÔÅÔÁ AC
× ÔÏÞËÅ D. îÁÊÄÉÔÅ ÈÏÒÄÕ, ÓÏÅÄÉÎÑÀÝÕÀ ÔÏÞËÉ ÐÅÒÅÓÅÞÅÎÉÑ ÏËÒÕÖÎÏÓÔÉ Ó ÐÒÑÍÏÊ BD.
135.9. éÚ ÔÏÞËÉ A, ÌÅÖÁÝÅÊ ×ÎÅ ÏËÒÕÖÎÏÓÔÉ, ÐÒÏ×ÅÄÅÎÙ Ë ÏËÒÕÖÎÏÓÔÉ ËÁÓÁÔÅÌØÎÁÑ É
ÓÅËÕÝÁÑ. òÁÓÓÔÏÑÎÉÅ ÏÔ ÔÏÞËÉ A ÄÏ ÔÏÞËÉ ËÁÓÁÎÉÑ ÒÁ×ÎÏ 16, Á ÒÁÓÓÔÏÑÎÉÅ ÏÔ ÔÏÞËÉ A ÄÏ
ÏÄÎÏÊ ÉÚ ÔÏÞÅË ÐÅÒÅÓÅÞÅÎÉÑ ÓÅËÕÝÅÊ Ó ÏËÒÕÖÎÏÓÔØÀ ÒÁ×ÎÏ 32. îÁÊÄÉÔÅ ÒÁÄÉÕÓ ÏËÒÕÖÎÏÓÔÉ,
ÅÓÌÉ ÒÁÓÓÔÏÑÎÉÅ ÏÔ ÃÅÎÔÒÁ ÏËÒÕÖÎÏÓÔÉ ÄÏ ÓÅËÕÝÅÊ ÒÁ×ÎÏ 5.
135.10. äÉÁÇÏÎÁÌØ AC ×ÐÉÓÁÎÎÏÇÏ × ÏËÒÕÖÎÏÓÔØ ÞÅÔÙÒÅÈÕÇÏÌØÎÉËÁ ABCD Ñ×ÌÑÅÔÓÑ ÂÉÓÓÅËÔÒÉÓÏÊ ÕÇÌÁ BAD. äÏËÁÖÉÔÅ, ÞÔÏ ÐÒÑÍÁÑ BD ÏÔÓÅËÁÅÔ ÏÔ ÔÒÅÕÇÏÌØÎÉËÁ ABC ÐÏÄÏÂÎÙÊ
ÅÍÕ ÔÒÅÕÇÏÌØÎÉË.
135.11. ðÅÒÅÓÅËÁÀÝÉÅÓÑ ÈÏÒÄÙ ÏËÒÕÖÎÏÓÔÉ ÄÅÌÑÔÓÑ ÔÏÞËÏÊ ÐÅÒÅÓÅÞÅÎÉÑ × ÏÄÎÏÍ É ÔÏÍ
ÖÅ ÏÔÎÏÛÅÎÉÉ. äÏËÁÖÉÔÅ, ÞÔÏ ÜÔÉ ÈÏÒÄÙ ÒÁ×ÎÙ ÍÅÖÄÕ ÓÏÂÏÊ.
135.12. ëÁÖÄÁÑ ÉÚ Ä×ÕÈ ÒÁ×ÎÙÈ ÐÅÒÅÓÅËÁÀÝÉÈÓÑ ÈÏÒÄ ÏËÒÕÖÎÏÓÔÉ ÄÅÌÉÔÓÑ ÔÏÞËÏÊ ÐÅÒÅÓÅÞÅÎÉÑ ÎÁ Ä×Á ÏÔÒÅÚËÁ. äÏËÁÖÉÔÅ, ÞÔÏ ÏÔÒÅÚËÉ ÐÅÒ×ÏÊ ÈÏÒÄÙ ÓÏÏÔ×ÅÔÓÔ×ÅÎÎÏ ÒÁ×ÎÙ ÏÔÒÅÚËÁÍ
×ÔÏÒÏÊ.
135.13. ÷ ËÒÕÇÅ ÐÒÏ×ÅÄÅÎÙ Ä×Å ÈÏÒÄÙ AB É CD, ÐÅÒÅÓÅËÁÀÝÉÅÓÑ × ÔÏÞËÅ M. ðÕÓÔØ K
| ÔÏÞËÁ ÐÅÒÅÓÅÞÅÎÉÑ ÂÉÓÓÅËÔÒÉÓÙ ÕÇÌÁ BMD Ó ÈÏÒÄÏÊ BD. îÁÊÄÉÔÅ ÏÔÒÅÚËÉ BK É KD,
ÅÓÌÉ BD = 3, Á ÐÌÏÝÁÄÉ ÔÒÅÕÇÏÌØÎÉËÏ× CMB É AMD ÏÔÎÏÓÑÔÓÑ ËÁË 1 : 4.
135.14. ä×Å ÏËÒÕÖÎÏÓÔÉ ÐÅÒÅÓÅËÁÀÔÓÑ × ÔÏÞËÁÈ A É B. ÷ ËÁÖÄÏÊ ÉÚ ÜÔÉÈ ÏËÒÕÖÎÏÓÔÅÊ
ÐÒÏ×ÅÄÅÎÏ ÐÏ ÈÏÒÄÅ ÉÚ ÔÏÞËÉ A (ÈÏÒÄÙ AC É AD). ðÒÉ ÜÔÏÍ ÈÏÒÄÁ ÏÄÎÏÊ ÏËÒÕÖÎÏÓÔÉ
ËÁÓÁÅÔÓÑ ÄÒÕÇÏÊ ÏËÒÕÖÎÏÓÔÉ. îÁÊÄÉÔÅ AB, ÅÓÌÉ CB = a, DB = b.
135.15. ïËÒÕÖÎÏÓÔØ ÐÒÏÈÏÄÉÔ ÞÅÒÅÚ ×ÅÒÛÉÎÙ B É C ÔÒÅÕÇÏÌØÎÉËÁ ABC É ÐÅÒÅÓÅËÁÅÔ ÅÇÏ
ÓÔÏÒÏÎÙ AB É AC × ÔÏÞËÁÈ M É N ÓÏÏÔ×ÅÔÓÔ×ÅÎÎÏ. éÚ×ÅÓÔÎÏ, ÞÔÏ BC = 3  MN É AB = 12.
îÁÊÄÉÔÅ AN.
135.1.

217

äÏËÁÖÉÔÅ, ÞÔÏ ÐÒÑÍÁÑ, ÐÒÏÈÏÄÑÝÁÑ ÞÅÒÅÚ ÔÏÞËÉ ÐÅÒÅÓÅÞÅÎÉÑ Ä×ÕÈ ÏËÒÕÖÎÏÓÔÅÊ,
ÄÅÌÉÔ ÐÏÐÏÌÁÍ ÏÂÝÕÀ ËÁÓÁÔÅÌØÎÕÀ Ë ÎÉÍ.
135.17. ÷ ÕÇÏÌ ×ÐÉÓÁÎÙ Ä×Å ÏËÒÕÖÎÏÓÔÉ; ÏÄÎÁ ÉÚ ÎÉÈ ËÁÓÁÅÔÓÑ ÓÔÏÒÏÎ ÕÇÌÁ × ÔÏÞËÁÈ
K1 É K2, Á ÄÒÕÇÁÑ - × ÔÏÞËÁÈ L1 É L2. äÏËÁÖÉÔÅ, ÞÔÏ ÐÒÑÍÁÑ K1L2 ×ÙÓÅËÁÅÔ ÎÁ ÜÔÉÈ Ä×ÕÈ
ÏËÒÕÖÎÏÓÔÑÈ ÒÁ×ÎÙÅ ÈÏÒÄÙ.
135.18. þÅÔÙÒÅÈÕÇÏÌØÎÉË ABCD ×ÐÉÓÁÎ × ÏËÒÕÖÎÏÓÔØ. äÉÁÇÏÎÁÌØ AC Ñ×ÌÑÅÔÓÑ ÂÉÓÓÅËÔÒÉÓÏÊ ÕÇÌÁ BAD É ÐÅÒÅÓÅËÁÅÔÓÑ Ó ÄÉÁÇÏÎÁÌØÀ BD × ÔÏÞËÅ K. îÁÊÄÉÔÅ KC, ÅÓÌÉ BC = 4 É
AK = 6.
135.19. ðÒÏÄÏÌÖÅÎÉÅ ÍÅÄÉÁÎÙ ÔÒÅÕÇÏÌØÎÉËÁ ABC, ÐÒÏ×ÅÄ£ÎÎÏÊ ÉÚ ×ÅÒÛÉÎÙ A, ÐÅÒÅÓÅËÁÅÔ ÏÐÉÓÁÎÎÕÀ ÏËÒÕÖÎÏÓÔØ × ÔÏÞËÅ D. îÁÊÄÉÔÅ BC, ÅÓÌÉ AC = DC = 1.
135.20. ïËÒÕÖÎÏÓÔØ ÄÅÌÉÔ ËÁÖÄÕÀ ÉÚ ÓÔÏÒÏÎ ÔÒÅÕÇÏÌØÎÉËÁ ÎÁ ÔÒÉ ÒÁ×ÎÙÅ ÞÁÓÔÉ. äÏËÁÖÉÔÅ, ÞÔÏ ÜÔÏÔ ÔÒÅÕÇÏÌØÎÉË ÐÒÁ×ÉÌØÎÙÊ.
135.21. óÔÏÒÏÎÁ Ë×ÁÄÒÁÔÁ ABCD ÒÁ×ÎÁ 1 É Ñ×ÌÑÅÔÓÑ ÈÏÒÄÏÊ ÎÅËÏÔÏÒÏÊ ÏËÒÕÖÎÏÓÔÉ, ÐÒÉÞÅÍ ÏÓÔÁÌØÎÙÅ ÓÔÏÒÏÎÙ Ë×ÁÄÒÁÔÁ ÌÅÖÁÔ ×ÎÅ ÜÔÏÊ ÏËÒÕÖÎÏÓÔÉ. ëÁÓÁÔÅÌØÎÁÑ CK, ÐÒÏ×ÅÄÅÎÎÁÑ
ÉÚ ×ÅÒÛÉÎÙ C Ë ÜÔÏÊ ÖÅ ÏËÒÕÖÎÏÓÔÉ, ÒÁ×ÎÁ 2. îÁÊÄÉÔÅ ÄÉÁÍÅÔÒ ÏËÒÕÖÎÏÓÔÉ.
135.22. ÷ ÐÒÑÍÏÕÇÏÌØÎÏÍ ÔÒÅÕÇÏÌØÎÉËÅ ABC Ó ËÁÔÅÔÁÍÉ AB = 3 É BC = 4 ÞÅÒÅÚ ÓÅÒÅÄÉÎÙ ÓÔÏÒÏÎ AB É AC ÐÒÏ×ÅÄÅÎÁ ÏËÒÕÖÎÏÓÔØ, ËÁÓÁÀÝÁÑÓÑ ËÁÔÅÔÁ BC. îÁÊÄÉÔÅ ÄÌÉÎÕ ÏÔÒÅÚËÁ
ÇÉÐÏÔÅÎÕÚÙ AC, ËÏÔÏÒÙÊ ÌÅÖÉÔ ×ÎÕÔÒÉ ÜÔÏÊ ÏËÒÕÖÎÏÓÔÉ.
135.23. ôÏÞËÁ B ÒÁÓÐÏÌÏÖÅÎÁ ÍÅÖÄÕ ÔÏÞËÁÍÉ A É C. îÁ ÏÔÒÅÚËÁÈ AB É AC ËÁË ÎÁ ÄÉÁÍÅÔÒÁÈ ÐÏÓÔÒÏÅÎÙ ÏËÒÕÖÎÏÓÔÉ. ðÒÑÍÁÑ, ÐÅÒÐÅÎÄÉËÕÌÑÒÎÁÑ AC É ÐÒÏÈÏÄÑÝÁÑ ÞÅÒÅÚ ÔÏÞËÕ
B, ÐÅÒÅÓÅËÁÅÔ ÂÏÌØÛÕÀ ÏËÒÕÖÎÏÓÔØ × ÔÏÞËÅ D. ðÒÑÍÁÑ, ÐÒÏÈÏÄÑÝÁÑ ÞÅÒÅÚ ÔÏÞËÕ C, ËÁÓÁÅÔÓÑ
ÍÅÎØÛÅÊ ÏËÒÕÖÎÏÓÔÉ × ÔÏÞËÅ K. äÏËÁÖÉÔÅ, ÞÔÏ CD = CK.
135.24. ðÏÓÔÒÏÊÔÅ ÏËÒÕÖÎÏÓÔØ, ÐÒÏÈÏÄÑÝÕÀ ÞÅÒÅÚ Ä×Å ÄÁÎÎÙÅ ÔÏÞËÉ É ËÁÓÁÀÝÕÀÓÑ
ÄÁÎÎÏÊ ÐÒÑÍÏÊ.
135.25. ïËÒÕÖÎÏÓÔØ ËÁÓÁÅÔÓÑ ÓÔÏÒÏÎ AB É BC ÔÒÅÕÇÏÌØÎÉËÁ ABC ÓÏÏÔ×ÅÔÓÔ×ÅÎÎÏ ×
ÔÏÞËÁÈ D É E. îÁÊÄÉÔÅ ×ÙÓÏÔÕ ÔÒÅÕÇÏÌØÎÉËÁ ABC, ÏÐÕÝÅÎÎÕÀ ÉÚ ÔÏÞËÉ A, ÅÓÌÉ AB = 5,
AC = 2, Á ÔÏÞËÉ A, D, E É C ÌÅÖÁÔ ÎÁ ÏÄÎÏÊ ÏËÒÕÖÎÏÓÔÉ.
135.26. ÷ ÒÁ×ÎÏÂÅÄÒÅÎÎÏÍ ÔÒÅÕÇÏÌØÎÉËÅ ABC (AB = AC) ÐÒÏ×ÅÄÅÎÙ ÂÉÓÓÅËÔÒÉÓÙ AD,
BE É CF. îÁÊÄÉÔÅ BC, ÅÓÌÉ ÉÚ×ÅÓÔÎÏ, ÞÔÏ AC = 1, Á ×ÅÒÛÉÎÁ A ÌÅÖÉÔ ÎÁ ÏËÒÕÖÎÏÓÔÉ,
ÐÒÏÈÏÄÑÝÅÊ ÞÅÒÅÚ ÔÏÞËÉ D, E É F.
135.27. éÚ ÔÏÞËÉ A Ë ÏËÒÕÖÎÏÓÔÉ ÒÁÄÉÕÓÁ R ÐÒÏ×ÅÄÅÎÁ ËÁÓÁÔÅÌØÎÁÑ AM (M | ÔÏÞËÁ
ËÁÓÁÎÉÑ). óÅËÕÝÁÑ, ÐÒÏÈÏÄÑÝÁÑ ÞÅÒÅÚ ÔÏÞËÕ A, ÐÅÒÅÓÅËÁÅÔ ÏËÒÕÖÎÏÓÔØ × ÔÏÞËÁÈ K É L,
ÐÒÉÞÅÍ L | ÓÅÒÅÄÉÎÁ ÏÔÒÅÚËÁ AK, Á ÕÇÏÌ AMK ÒÁ×ÅÎ 60. îÁÊÄÉÔÅ AM.

135.28. èÏÒÄÁ AB ÓÔÑÇÉ×ÁÅÔ ÄÕÇÕ ÏËÒÕÖÎÏÓÔÉ, ÒÁ×ÎÕÀ 120 . ôÏÞËÁ C ÌÅÖÉÔ ÎÁ ÜÔÏÊ ÄÕÇÅ,
p
Á ÔÏÞËÁ D ÌÅÖÉÔ ÎÁ ÈÏÒÄÅ AB. ðÒÉ ÜÔÏÍ AD = 2, BD = 1, DC = 2. îÁÊÄÉÔÅ ÐÌÏÝÁÄØ
ÔÒÅÕÇÏÌØÎÉËÁ ABC.
135.29. ä×Å ÏËÒÕÖÎÏÓÔÉ ×ÎÕÔÒÅÎÎÅ ËÁÓÁÀÔÓÑ. ðÒÑÍÁÑ, ÐÒÏÈÏÄÑÝÁÑ ÞÅÒÅÚ ÃÅÎÔÒ ÂÏÌØÛÅÊ
ÏËÒÕÖÎÏÓÔÉ, ÐÅÒÅÓÅËÁÅÔ Å£ × ÔÏÞËÁÈ A É D, Á ÍÅÎØÛÕÀ ÏËÒÕÖÎÏÓÔØ | × ÔÏÞËÁÈ B É C.
îÁÊÄÉÔÅ ÏÔÎÏÛÅÎÉÅ ÒÁÄÉÕÓÏ× ÏËÒÕÖÎÏÓÔÅÊ, ÅÓÌÉ AB : BC : CD = 3 : 7 : 2.
135.30. ôÏÞËÉ A, B É C ÌÅÖÁÔ ÎÁ ÏÄÎÏÊ ÐÒÑÍÏÊ (ÔÏÞËÁ B ÒÁÓÐÏÌÏÖÅÎÁ ÍÅÖÄÕ ÔÏÞËÁÍÉ
A É C). þÅÒÅÚ ÔÏÞËÉ A É B ÐÒÏ×ÏÄÑÔÓÑ ÏËÒÕÖÎÏÓÔÉ, Á ÞÅÒÅÚ ÔÏÞËÕ C | ËÁÓÁÔÅÌØÎÙÅ Ë ÎÉÍ.
îÁÊÄÉÔÅ ÇÅÏÍÅÔÒÉÞÅÓËÏÅ ÍÅÓÔÏ ÔÏÞÅË ËÁÓÁÎÉÑ.
135.31. ïËÒÕÖÎÏÓÔØ É ÐÒÑÍÁÑ ËÁÓÁÀÔÓÑ × ÔÏÞËÅ M. éÚ ÔÏÞÅË A É B ÜÔÏÊ ÏËÒÕÖÎÏÓÔÉ
ÏÐÕÝÅÎÙ ÐÅÒÐÅÎÄÉËÕÌÑÒÙ ÎÁ ÐÒÑÍÕÀ, ÒÁ×ÎÙÅ a É b ÓÏÏÔ×ÅÔÓÔ×ÅÎÎÏ. îÁÊÄÉÔÅ ÒÁÓÓÔÏÑÎÉÅ ÏÔ
135.16.

218

M ÄÏ ÐÒÑÍÏÊ AB.
éÚ ÔÏÞËÉ A, ÎÁÈÏÄÑÝÅÊÓÑ ÎÁ ÒÁÓÓÔÏÑÎÉÉ 5 ÏÔ ÃÅÎÔÒÁ ÏËÒÕÖÎÏÓÔÉ ÒÁÄÉÕÓÁ 3,
ÐÒÏ×ÅÄÅÎÙ Ä×Å ÓÅËÕÝÉÅ AKC É ALB, ÕÇÏÌ ÍÅÖÄÕ ËÏÔÏÒÙÍÉ ÒÁ×ÅÎ 30 (K, C, L, B | ÔÏÞËÉ
ÐÅÒÅÓÅÞÅÎÉÑ ÓÅËÕÝÉÈ Ó ÏËÒÕÖÎÏÓÔØÀ). îÁÊÄÉÔÅ ÐÌÏÝÁÄØ ÔÒÅÕÇÏÌØÎÉËÁ AKL, ÅÓÌÉ ÐÌÏÝÁÄØ
ÔÒÅÕÇÏÌØÎÉËÁ ABC ÒÁ×ÎÁ 10.
ÔÏÞËÉ

135.32.

÷ ÏËÒÕÖÎÏÓÔÉ ÐÒÏ×ÅÄÅÎÙ ÔÒÉ ÐÏÐÁÒÎÏ ÐÅÒÅÓÅËÁÀÝÉÅÓÑ ÈÏÒÄÙ. ëÁÖÄÁÑ ÈÏÒÄÁ
ÒÁÚÄÅÌÅÎÁ ÔÏÞËÁÍÉ ÐÅÒÅÓÅÞÅÎÉÑ ÎÁ ÔÒÉ ÒÁ×ÎÙÅ ÞÁÓÔÉ. îÁÊÄÉÔÅ ÒÁÄÉÕÓ ÏËÒÕÖÎÏÓÔÉ, ÅÓÌÉ
ÏÄÎÁ ÉÚ ÈÏÒÄ ÒÁ×ÎÁ a.
135.34. ÷ ÏËÒÕÖÎÏÓÔØ ×ÐÉÓÁÎ ÔÒÅÕÇÏÌØÎÉË. ÷ÔÏÒÁÑ ÏËÒÕÖÎÏÓÔØ, ËÏÎÃÅÎÔÒÉÞÅÓËÁÑ Ó ÐÅÒ×ÏÊ, ËÁÓÁÅÔÓÑ ÏÄÎÏÊ ÓÔÏÒÏÎÙ ÔÒÅÕÇÏÌØÎÉËÁ É ÄÅÌÉÔ ËÁÖÄÕÀ ÉÚ Ä×ÕÈ ÄÒÕÇÉÈ ÓÔÏÒÏÎ ÎÁ ÔÒÉ
ÒÁ×ÎÙÅ ÞÁÓÔÉ. îÁÊÄÉÔÅ ÏÔÎÏÛÅÎÉÅ ÒÁÄÉÕÓÏ× ÏËÒÕÖÎÏÓÔÅÊ.
135.35. ïËÒÕÖÎÏÓÔØ ËÁÓÁÅÔÓÑ ÓÔÏÒÏÎ AB É AD ÐÒÑÍÏÕÇÏÌØÎÉËÁ ABCD É ÐÒÏÈÏÄÉÔ ÞÅÒÅÚ
×ÅÒÛÉÎÕ C. óÔÏÒÏÎÕ DC ÏÎÁÐÅÒÅÓÅËÁÅÔ × ÔÏÞËÅ N. îÁÊÄÉÔÅ ÐÌÏÝÁÄØ ÔÒÁÐÅÃÉÉ ABND, ÅÓÌÉ
AB = 9 É AD = 8.
135.36. îÁÊÄÉÔÅ ÒÁÄÉÕÓ ÏËÒÕÖÎÏÓÔÉ, ËÏÔÏÒÁÑ ×ÙÓÅËÁÅÔ ÎÁ ÏÂÅÉÈ ÓÔÏÒÏÎÁÈ ÕÇÌÁ ×ÅÌÉÞÉÎÙ
ÈÏÒÄÙ ÄÌÉÎÙ a, ÅÓÌÉ ÉÚ×ÅÓÔÎÏ, ÞÔÏ ÒÁÓÓÔÏÑÎÉÅ ÍÅÖÄÕ ÂÌÉÖÁÊÛÉÍÉ ËÏÎÃÁÍÉ ÜÔÉÈ ÈÏÒÄ
ÒÁ×ÎÏ b.
135.37. äÁÎ ÕÇÏÌ Ó ×ÅÒÛÉÎÏÊ O É ÏËÒÕÖÎÏÓÔØ, ËÁÓÁÀÝÁÑÓÑ ÅÇÏ ÓÔÏÒÏÎ × ÔÏÞËÁÈ A É B. éÚ
ÔÏÞËÉ A ÐÁÒÁÌÌÅÌØÎÏ OB ÐÒÏ×ÅÄÅÎ ÌÕÞ, ÐÅÒÅÓÅËÁÀÝÉÊ ÏËÒÕÖÎÏÓÔØ × ÔÏÞËÅ C. ïÔÒÅÚÏË OC
ÐÅÒÅÓÅËÁÅÔ ÏËÒÕÖÎÏÓÔØ × ÔÏÞËÅ E. ðÒÑÍÙÅ AE É OB ÐÅÒÅÓÅËÁÀÔÓÑ × ÔÏÞËÅ K. äÏËÁÖÉÔÅ,
ÞÔÏ OK = KB.
135.38. ôÏÞËÉ A1 É B1 ÐÒÉÎÁÄÌÅÖÁÔ ÓÏÏÔ×ÅÔÓÔ×ÅÎÎÏ ÓÔÏÒÏÎÁÍ OA É OB ÕÇÌÁ AOB (ÎÅ
ÒÁ×ÎÏÇÏ 180) É OA  OA1 = OB  OB1 . äÏËÁÖÉÔÅ, ÞÔÏ ÔÏÞËÉ A, B, A1 , B1 ÐÒÉÎÁÄÌÅÖÁÔ
ÏÄÎÏÊ ÏËÒÕÖÎÏÓÔÉ.
135.39. þÅÒÅÚ ÔÏÞËÕ P , ÌÅÖÁÝÕÀ ÎÁ ÏÂÝÅÊ ÈÏÒÄÅ Ä×ÕÈ ÐÅÒÅÓÅËÁÀÝÉÈÓÑ ÏËÒÕÖÎÏÓÔÅÊ,
ÐÒÏ×ÏÄÅÎÙ ÈÏÒÄÁ KM ÐÅÒ×ÏÊ ÏËÒÕÖÎÏÓÔÉ É ÈÏÒÄÁ LN ×ÔÏÒÏÊ ÏËÒÕÖÎÏÓÔÉ. äÏËÁÖÉÔÅ, ÞÔÏ
ÞÅÔÙÒÅÈÕÇÏÌØÎÉË KLMN | ×ÐÉÓÁÎÎÙÊ.
135.40. ôÏÞËÁ M ÎÁÈÏÄÉÔÓÑ ÎÁ ÐÒÏÄÏÌÖÅÎÉÉ ÈÏÒÄÙ AB. äÏËÁÖÉÔÅ, ÞÔÏ ÅÓÌÉ ÔÏÞËÁ C
ÏËÒÕÖÎÏÓÔÉ ÔÁËÏ×Á, ÞÔÏ MC2 = MA  MB, ÔÏ MC | ËÁÓÁÔÅÌØÎÁÑ Ë ÏËÒÕÖÎÏÓÔÉ.
135.41. äÏËÁÖÉÔÅ, ÞÔÏ Ë×ÁÄÒÁÔ ÂÉÓÓÅËÔÒÉÓÙ ÔÒÅÕÇÏÌØÎÉËÁ ÒÁ×ÅÎ ÐÒÏÉÚ×ÅÄÅÎÉÀ ÓÔÏÒÏÎ,
Å£ ÚÁËÌÀÞÁÀÝÉÈ, ÂÅÚ ÐÒÏÉÚ×ÅÄÅÎÉÑ ÏÔÒÅÚËÏ× ÔÒÅÔØÅÊ ÓÔÏÒÏÎÙ, ÎÁ ËÏÔÏÒÙÅ ÏÎÁ ÒÁÚÄÅÌÅÎÁ
ÂÉÓÓÅËÔÒÉÓÏÊ.
135.42. ðÏÓÔÒÏÊÔÅ ÏËÒÕÖÎÏÓÔØ, ÐÒÏÈÏÄÑÝÕÀ ÞÅÒÅÚ Ä×Å ÄÁÎÎÙÅ ÔÏÞËÉ A É B É ËÁÓÁÀÝÕÀÓÑ ÄÁÎÎÏÊ ÏËÒÕÖÎÏÓÔÉ S.
135.33.

* * *
135.43.

þÅÒÅÚ ÄÁÎÎÕÀ ÔÏÞËÕ ÐÒÏ×ÅÄÉÔÅ ÏËÒÕÖÎÏÓÔØ, ËÁÓÁÀÝÕÀÓÑ ÄÁÎÎÏÊ ÐÒÑÍÏÊ É ÄÁÎÎÏÊ ÏËÒÕÖ-

ÎÏÓÔÉ.

KLM ÐÒÏ×ÅÄÅÎÁ ÂÉÓÓÅËÔÒÉÓÁ MN. þÅÒÅÚ ×ÅÒÛÉÎÕ M ÐÒÏÈÏÄÉÔ ÏËÒÕÖÎÏÓÔØ, ËÁÓÁÀÝÁÑÓÑ ÓÔÏÒÏÎÙ KL × ÔÏÞËÅ M É ÐÅÒÅÓÅËÁÀÝÁÑ ÓÔÏÒÏÎÕ KM × ÔÏÞËÅ P, Á ÓÔÏÒÏÎÕ LM | ×
ÔÏÞËÅ Q. äÌÉÎÙ ÏÔÒÅÚËÏ× KP, QM É LQ ÓÏÏÔ×ÅÔÓÔ×ÅÎÎÏ ÒÁ×ÎÙ k, m É q.îÁÊÄÉÔÅ ÄÌÉÎÕ ÏÔÒÅÚËÁ
MN.
îÁ ÐÒÏÄÏÌÖÅÎÉÉ ÈÏÒÄÙ KL ÏËÒÕÖÎÏÓÔÉ Ó ÃÅÎÔÒÏÍ O ×ÚÑÔÁ ÔÏÞËÁ A, É ÉÚ ÎÅ£ ÐÒÏ×ÅÄÅÎÙ
ËÁÓÁÔÅÌØÎÙÅ AP É AQ; M | ÓÅÒÅÄÉÎÁ ÏÔÒÅÚËÁ PQ. äÏËÁÖÉÔÅ, ÞÔÏ \MKO = \MLO.
÷ ÔÒÁÐÅÃÉÉ ABCD ÏÓÎÏ×ÁÎÉÅ AD ×Ä×ÏÅ ÂÏÌØÛÅ ÏÓÎÏ×ÁÎÉÑ BC, ÕÇÏÌ A ÒÁ×ÅÎ 45, ÕÇÏÌ
135.44.

÷ ÔÒÅÕÇÏÌØÎÉËÅ

135.45.

135.46.

219

D ÒÁ×ÅÎ 30. îÁ ÄÉÁÇÏÎÁÌÑÈ ÔÒÁÐÅÃÉÉ ËÁË ÎÁ ÄÉÁÍÅÔÒÁÈ ÐÏÓÔÒÏÅÎÙ ÏËÒÕÖÎÏÓÔÉ, ÐÅÒÅÓÅËÁÀÝÉÅÓÑ ×
ÔÏÞËÁÈ M É N. èÏÒÄÁ MN ÐÅÒÅÓÅËÁÅÔ ÏÓÎÏ×ÁÎÉÅ BC × ÔÏÞËÅ F. îÁÊÄÉÔÅ ÏÔÎÏÛÅÎÉÅ BF : FC.
äÏËÁÖÉÔÅ ÆÏÒÍÕÌÕ üÊÌÅÒÁ: O1 O2 = R2 - 2rR, ÇÄÅ O1, O2 | ÃÅÎÔÒÙ ×ÐÉÓÁÎÎÏÊ É
ÏÐÉÓÁÎÎÏÊ ÏËÒÕÖÎÏÓÔÅÊ ÔÒÅÕÇÏÌØÎÉËÁ ABC , Á r É R | ÒÁÄÉÕÓÙ ÜÔÉÈ ÏËÒÕÖÎÏÓÔÅÊ.
ðÑÔÉÕÇÏÌØÎÉË ABCDE ×ÐÉÓÁÎ × ÏËÒÕÖÎÏÓÔØ. òÁÓÓÔÏÑÎÉÑ ÏÔ ÔÏÞËÉ A ÄÏ ÐÒÑÍÙÈ BC, DC
É DE ÒÁ×ÎÙ ÓÏÏÔ×ÅÔÓÔ×ÅÎÎÏ a, b, c. îÁÊÄÉÔÅ ÒÁÓÓÔÏÑÎÉÅ ÏÔ ×ÅÒÛÉÎÙ A ÄÏ ÐÒÑÍÏÊ BE.
ä×Å ÏËÒÕÖÎÏÓÔÉ ÒÁÄÉÕÓÏ× r É R (r < R) ×ÎÅÛÎÉÍ ÏÂÒÁÚÏÍ ËÁÓÁÀÔÓÑ ÄÒÕÇ ÄÒÕÇÁ. ðÒÑÍÁÑ
ËÁÓÁÅÔÓÑ ÜÔÉÈ ÏËÒÕÖÎÏÓÔÅÊ × ÔÏÞËÁÈ M É N. ÷ ÔÏÞËÁÈ A É B ÏËÒÕÖÎÏÓÔÉ ËÁÓÁÀÔÓÑ ×ÎÅÛÎÉÍ ÏÂÒÁÚÏÍ
ÔÒÅÔØÅÊ ÏËÒÕÖÎÏÓÔÉ. ðÒÑÍÙÅ AB É MN ÐÅÒÅÓÅËÁÀÔÓÑ × ÔÏÞËÅ C. éÚ ÔÏÞËÉ C ÐÒÏ×ÅÄÅÎÁ ËÁÓÁÔÅÌØÎÁÑ
Ë ÔÒÅÔØÅÊ ÏËÒÕÖÎÏÓÔÉ (D | ÔÏÞËÁ ËÁÓÁÎÉÑ). îÁÊÄÉÔÅ CD.
135.47.

135.48.

135.49.

135.50.

50. îÁ ÂÏËÏ×ÙÈ ÓÔÏÒÏÎÁÈ ÔÒÁÐÅÃÉÉ ËÁË ÎÁ ÄÉÁÍÅÔÒÁÈ ÐÏÓÔÒÏÅÎÙ ÏËÒÕÖÎÏÓÔÉ. äÏËÁÖÉÔÅ,

ÞÔÏ ÏÔÒÅÚËÉ ËÁÓÁÔÅÌØÎÙÈ, ÐÒÏ×ÅÄÅÎÎÙÈ ÉÚ ÔÏÞËÉ ÐÅÒÅÓÅÞÅÎÉÑ ÄÉÁÇÏÎÁÌÅÊ ÔÒÁÐÅÃÉÉ Ë ÜÔÉÍ ÏËÒÕÖÎÏÓÔÑÍ, ÒÁ×ÎÙ ÍÅÖÄÕ ÓÏÂÏÊ.
135.51.

× ÔÏÞËÁÈ
É

b.

ðÒÏÔÉ×ÏÐÏÌÏÖÎÙÅ ÓÔÏÒÏÎÙ ÞÅÔÙÒÅÈÕÇÏÌØÎÉËÁ, ×ÐÉÓÁÎÎÏÇÏ × ÏËÒÕÖÎÏÓÔØ, ÐÅÒÅÓÅËÁÀÔÓÑ

P É Q. îÁÊÄÉÔÅ PQ, ÅÓÌÉ ËÁÓÁÔÅÌØÎÙÅ Ë ÏËÒÕÖÎÏÓÔÉ, ÐÒÏ×ÅÄ£ÎÎÙÅ ÉÚ ÔÏÞÅË P É Q, ÒÁ×ÎÙ a

l É ÔÏÞËÉ A É B ÐÏ ÏÄÎÕ ÓÔÏÒÏÎÕ ÏÔ ÎÅ£. ó ÐÏÍÏÝØÀ ÃÉÒËÕÌÑ É ÌÉÎÅÊËÉ
ÐÏÓÔÒÏÊÔÅ ÎÁ ÐÒÑÍÏÊ l ÔÏÞËÕ X, ÄÌÑ ËÏÔÏÒÏÊ AX + BX = a, ÇÄÅ a | ÄÁÎÎÁÑ ×ÅÌÉÞÉÎÁ.
135.52.

äÁÎÁ ÐÒÑÍÁÑ

135.53.

(ôÅÏÒÅÍÁ ðÔÏÌÅÍÅÑ) äÏËÁÖÉÔÅ, ÞÔÏ ÅÓÌÉ ÞÅÔÙÒ£ÈÕÇÏÌØÎÉË ×ÐÉÓÁÎ × ÏËÒÕÖÎÏÓÔØ, ÔÏ ÓÕÍÍÁ

ÐÒÏÉÚ×ÅÄÅÎÉÊ ÄÌÉÎ Ä×ÕÈ ÐÁÒ ÅÇÏ ÐÒÏÔÉ×ÏÐÏÌÏÖÎÙÈ ÓÔÏÒÏÎ ÒÁ×ÎÁ ÐÒÏÉÚ×ÅÄÅÎÉÀ ÄÌÉÎ ÅÇÏ ÄÉÁÇÏÎÁÌÅÊ.

220

136. ôÅÏÒÅÍÁ ËÏÓÉÎÕÓÏ×

a, b, c | ÓÔÏÒÏÎÙ ÔÒÅÕÇÏÌØÎÉËÁ; | ÕÇÏÌ, ÐÒÏÔÉ×ÏÌÅÖÁÝÉÊ ÓÔÏÒÏÎÅ a. ôÏÇÄÁ a = b + c2 - 2bc cos .
óÔÏÒÏÎÙ ÔÒÅÕÇÏÌØÎÉËÁ ÒÁ×ÎÙ 5, 8, 10. âÕÄÅÔ ÌÉ ÜÔÏÔ ÔÒÅÕÇÏÌØÎÉË ÏÓÔÒÏÕÇÏÌØÎÙÍ?
136.1.

(ôÅÏÒÅÍÁ ËÏÓÉÎÕÓÏ×) ðÕÓÔØ
2

2

136.2.

136.3.

óÕÍÍÁ Ë×ÁÄÒÁÔÏ× Ä×ÕÈ ÓÔÏÒÏÎ ÔÒÅÕÇÏÌØÎÉËÁ ÂÏÌØÛÅ Ë×ÁÄÒÁÔÁ ÔÒÅÔØÅÊ ÓÔÏÒÏÎÙ.

äÏËÁÖÉÔÅ, ÞÔÏ ÐÒÏÔÉ× ÔÒÅÔØÅÊ ÓÔÏÒÏÎÙ ÌÅÖÉÔ ÏÓÔÒÙÊ ÕÇÏÌ.
136.4.

äÁÎ ÒÁ×ÎÏÓÔÏÒÏÎÎÉÊ ÔÒÅÕÇÏÌØÎÉË ÓÏ ÓÔÏÒÏÎÏÊ

a. îÁÊÄÉÔÅ ÄÌÉÎÕ ÏÔÒÅÚËÁ, ÓÏÅÄÉ-

ÎÑÀÝÅÇÏ ×ÅÒÛÉÎÕ ÔÒÅÕÇÏÌØÎÉËÁ Ó ÔÏÞËÏÊ, ÄÅÌÑÝÅÊ ÐÒÏÔÉ×ÏÐÏÌÏÖÎÕÀ ÓÔÏÒÏÎÕ × ÏÔÎÏÛÅÎÉÉ
2:1.
136.5.

ÒÁ×ÅÎ

60

ïÄÎÁ ÉÚ ÓÔÏÒÏÎ ÔÒÅÕÇÏÌØÎÉËÁ ×Ä×ÏÅ ÂÏÌØÛÅ ÄÒÕÇÏÊ, Á ÕÇÏÌ ÍÅÖÄÕ ÜÔÉÍÉ ÓÔÏÒÏÎÁÍÉ

. äÏËÁÖÉÔÅ, ÞÔÏ ÔÒÅÕÇÏÌØÎÉË ÐÒÑÍÏÕÇÏÌØÎÙÊ.
p

136.6.

óÔÏÒÏÎÁ p
ÔÒÅÕÇÏÌØÎÉËÁ ÒÁ×ÎÁ

ÏÔÎÏÓÑÔÓÑ, ËÁË
136.7.

2 7, Á Ä×Å ÄÒÕÇÉÅ ÓÔÏÒÏÎÙ ÏÂÒÁÚÕÀÔ ÕÇÏÌ × 30 É

1 : 2 3. îÁÊÄÉÔÅ ÜÔÉ ÓÔÏÒÏÎÙ.

ïÄÎÁ ÉÚ ÓÔÏÒÏÎ ÐÁÒÁÌÌÅÌÏÇÒÁÍÍÁ ÒÁ×ÎÁ

ËÏÓÉÎÕÓ ÏÓÔÒÏÇÏ ÕÇÌÁ ÍÅÖÄÕ ÄÉÁÇÏÎÁÌÑÍÉ.

10, Á ÄÉÁÇÏÎÁÌÉ ÒÁ×ÎÙ 20 É 24. îÁÊÄÉÔÅ

ôÏÞËÁ O | ÃÅÎÔÒ ÏËÒÕÖÎÏÓÔÉ, ×ÐÉÓÁÎÎÏÊ × ÔÒÅÕÇÏÌØÎÉË ABC. éÚ×ÅÓÔÎÏ, ÞÔÏ
BC = a, AC = b, \AOB = 120. îÁÊÄÉÔÅ ÓÔÏÒÏÎÕ AB.
õÇÏÌ ÐÒÉ ×ÅÒÛÉÎÅ D ÔÒÁÐÅÃÉÉ ABCD Ó ÏÓÎÏ×ÁÎÉÑÍÉ AD É BC ÒÁ×ÅÎ 60. îÁÊÄÉÔÅ
ÄÉÁÇÏÎÁÌÉ ÔÒÁÐÅÃÉÉ, ÅÓÌÉ AD = 10, BC = 3 É CD = 4.
ïÄÎÁ ÉÚ ÓÔÏÒÏÎ ÔÒÅÕÇÏÌØÎÉËÁ ÒÁ×ÎÁ 6, ×ÔÏÒÁÑ ÓÔÏÒÏÎÁ ÒÁ×ÎÁ 2 7, Á ÐÒÏÔÉ×ÏÌÅÖÁÝÉÊ ÅÊ ÕÇÏÌ ÒÁ×ÅÎ 60. îÁÊÄÉÔÅ ÔÒÅÔØÀ ÓÔÏÒÏÎÕ ÔÒÅÕÇÏÌØÎÉËÁ.
îÁ ÐÒÏÄÏÌÖÅÎÉÉ ÂÏËÏ×ÏÊ ÓÔÏÒÏÎÙ AB ÒÁ×ÎÏÂÅÄÒÅÎÎÏÇÏ ÔÒÅÕÇÏÌØÎÉËÁ ABC ÚÁ
×ÅÒÛÉÎÕ A ×ÚÑÔÁ ÔÏÞËÁ D, ÐÒÉÞÅÍ AD = 2 AB. éÚ×ÅÓÔÎÏ, ÞÔÏ AB = AC = 1, \BAC = 120.
äÏËÁÖÉÔÅ, ÞÔÏ ÔÒÅÕÇÏÌØÎÉË BDC ÒÁ×ÎÏÂÅÄÒÅÎÎÙÊ.
ôÏÞËÉ M É N ÌÅÖÁÔ ÓÏÏÔ×ÅÔÓÔ×ÅÎÎÏ ÎÁ ÓÔÏÒÏÎÁÈ AD É BC ÒÏÍÂÁ ABCD, ÐÒÉÞÅÍ
DM : AM = BN : NC = 2 : 1. îÁÊÄÉÔÅ MN, ÅÓÌÉ ÉÚ×ÅÓÔÎÏ, ÞÔÏ ÓÔÏÒÏÎÁ ÒÏÍÂÁ ÒÁ×ÎÁ a, Á
\BAD = 60.
136.8.

136.9.

p

136.10.

136.11.

136.12.

136.13.

äÏËÁÖÉÔÅ, ÞÔÏ ÓÕÍÍÁ Ë×ÁÄÒÁÔÏ× ÄÉÁÇÏÎÁÌÅÊ ÐÁÒÁÌÌÅÌÏÇÒÁÍÍÁ ÒÁ×ÎÁ ÓÕÍÍÅ Ë×Á-

ÄÒÁÔÏ× ×ÓÅÈ ÅÇÏ ÞÅÔÙÒ£È ÓÔÏÒÏÎ.
136.14.

äÉÁÇÏÎÁÌØ ÐÁÒÁÌÌÅÌÏÇÒÁÍÍÁ, ÒÁ×ÎÁÑ

ÇÒÁÍÍÁ, ÒÁ×ÎÏÊ
136.15.

b,

ÐÅÒÐÅÎÄÉËÕÌÑÒÎÁ ÓÔÏÒÏÎÅ ÐÁÒÁÌÌÅÌÏ-

a. îÁÊÄÉÔÅ ×ÔÏÒÕÀ ÄÉÁÇÏÎÁÌØ ÐÁÒÁÌÌÅÌÏÇÒÁÍÍÁ.

÷ ÒÁ×ÎÏÂÅÄÒÅÎÎÏÍ ÔÒÅÕÇÏÌØÎÉËÅ Ó ÂÏËÏ×ÏÊ ÓÔÏÒÏÎÏÊ, ÒÁ×ÎÏÊ

4, ÐÒÏ×ÅÄÅÎÁ ÍÅÄÉ3.

ÁÎÁ Ë ÂÏËÏ×ÏÊ ÓÔÏÒÏÎÅ. îÁÊÄÉÔÅ ÏÓÎÏ×ÁÎÉÅ ÔÒÅÕÇÏÌØÎÉËÁ, ÅÓÌÉ
ÜÔÁ ÍÅÄÉÁÎÁ ÒÁ×ÎÁ
p
136.16.

4 2, Á ÍÅÄÉÁÎÁ, ÐÒÏ×ÅÄÅÎÎÁÑ Ë
5. îÁÊÄÉÔÅ ÂÏËÏ×ÙÅ ÓÔÏÒÏÎÙ.
óÔÏÒÏÎÙ ÔÒÅÕÇÏÌØÎÉËÁ ÒÁ×ÎÙ a, b É c. îÁÊÄÉÔÅ ÍÅÄÉÁÎÕ, ÐÒÏ×ÅÄÅÎÎÕÀ Ë ÓÔÏÒÏÎÅ,
ïÓÎÏ×ÁÎÉÅ ÒÁ×ÎÏÂÅÄÒÅÎÎÏÇÏ ÔÒÅÕÇÏÌØÎÉËÁ ÒÁ×ÎÏ

ÂÏËÏ×ÏÊ ÓÔÏÒÏÎÅ, ÒÁ×ÎÁ
136.17.

ÒÁ×ÎÏÊ

c.

136.18.

óÔÏÒÏÎÙ ÔÒÅÕÇÏÌØÎÉËÁ ÒÁ×ÎÙ

ÂÏÌØÛÅÊ ÓÔÏÒÏÎÅ.
136.19.

ÒÁ×ÎÁ

11, 13 É 12. îÁÊÄÉÔÅ ÍÅÄÉÁÎÕ, ÐÒÏ×ÅÄÅÎÎÕÀ Ë

÷ ÔÒÅÕÇÏÌØÎÉËÅ Ä×Å ÓÔÏÒÏÎÙ ÒÁ×ÎÙ

10. îÁÊÄÉÔÅ ÔÒÅÔØÀ ÓÔÏÒÏÎÕ.

221

11 É 23, Á ÍÅÄÉÁÎÁ, ÐÒÏ×ÅÄÅÎÎÁÑ Ë ÔÒÅÔØÅÊ,

136.20.

äÏËÁÖÉÔÅ, ÞÔÏ ÏÔÎÏÛÅÎÉÅ ÓÕÍÍÙ Ë×ÁÄÒÁÔÏ× ÍÅÄÉÁÎ ÔÒÅÕÇÏÌØÎÉËÁ Ë ÓÕÍÍÅ Ë×Á-

ÄÒÁÔÏ× ÅÇÏ ÓÔÏÒÏÎ ÒÁ×ÎÏ
136.21.

3=4.

ïËÏÌÏ ÞÅÔÙÒÅÈÕÇÏÌØÎÉËÁ

ABCD

ÍÏÖÎÏ ÏÐÉÓÁÔØ ÏËÒÕÖÎÏÓÔØ. éÚ×ÅÓÔÎÏ, ÞÔÏ

AB = 3, BC = 4, CD = 5 É AD = 2. îÁÊÄÉÔÅ AC.
íÏÖÎÏ ÌÉ ÏËÏÌÏ ÞÅÔÙÒÅÈÕÇÏÌØÎÉËÁ ABCD ÏÐÉÓÁÔØ ÏËÒÕÖÎÏÓÔØ, ÅÓÌÉ \ADC =

30 , AB = 3, BC = 4, AC = 6 ?
136.22.

136.23.

÷ ÒÁ×ÎÏÂÅÄÒÅÎÎÏÍ ÔÒÅÕÇÏÌØÎÉËÅ ÏÓÎÏ×ÁÎÉÅ É ÂÏËÏ×ÁÑ ÓÔÏÒÏÎÁ ÒÁ×ÎÙ ÓÏÏÔ×ÅÔ-

5 É 20. îÁÊÄÉÔÅ ÂÉÓÓÅËÔÒÉÓÕ ÕÇÌÁ ÐÒÉ ÏÓÎÏ×ÁÎÉÉ.
÷ ÔÒÅÕÇÏÌØÎÉËÅ ABC ÉÚ×ÅÓÔÎÙ ÓÔÏÒÏÎÙ AC = 13, AB = 14, BC = 15. îÁ
ÓÔÏÒÏÎÅ BC ×ÚÑÔÁ ÔÏÞËÁ M, ÄÌÑ ËÏÔÏÒÏÊ CM : MB = 1 : 2. îÁÊÄÉÔÅ AM.
÷ ÔÒÅÕÇÏÌØÎÉËÅ ABC ÉÚ×ÅÓÔÎÙ ÓÔÏÒÏÎÙ AB = 12, AC = 15, BC = 18. îÁÊÄÉÔÅ
ÓÔ×ÅÎÎÏ

136.24.

136.25.

ÂÉÓÓÅËÔÒÉÓÕ ÔÒÅÕÇÏÌØÎÉËÁ, ÐÒÏ×ÅÄÅÎÎÕÀ ÉÚ ×ÅÒÛÉÎÙ ÎÁÉÂÏÌØÛÅÇÏ ÕÇÌÁ.
136.26.

É

5.

136.27.

îÁÊÄÉÔÅ ËÏÓÉÎÕÓÙ ÕÇÌÏ× ÔÒÁÐÅÃÉÉ Ó ÏÓÎÏ×ÁÎÉÑÍÉ

3 É 7 É ÂÏËÏ×ÙÍÉ ÓÔÏÒÏÎÁÍÉ 2

ABC, ÐÒÏ×ÅÄÅÎÎÙÅ ÉÚ ×ÅÒÛÉÎ B É C, ÒÁ×ÎÙ 6 É 9 É
M. éÚ×ÅÓÔÎÏ, ÞÔÏ \BMC = 120. îÁÊÄÉÔÅ ÓÔÏÒÏÎÙ ÔÒÅÕÇÏÌØÎÉËÁ.
óÔÏÒÏÎÙ ÐÁÒÁÌÌÅÌÏÇÒÁÍÍÁ ÒÁ×ÎÙ 2 É 4, Á ÕÇÏÌ ÍÅÖÄÕ ÎÉÍÉ ÒÁ×ÅÎ 60 . þÅÒÅÚ
íÅÄÉÁÎÙ ÔÒÅÕÇÏÌØÎÉËÁ

ÐÅÒÅÓÅËÁÀÔÓÑ × ÔÏÞËÅ
136.28.

×ÅÒÛÉÎÕ ÜÔÏÇÏ ÕÇÌÁ ÐÒÏ×ÅÄÅÎÙ ÐÒÑÍÙÅ, ÐÒÏÈÏÄÑÝÉÅ ÞÅÒÅÚ ÓÅÒÅÄÉÎÙ Ä×ÕÈ ÄÒÕÇÉÈ ÓÔÏÒÏÎ
ÐÁÒÁÌÌÅÌÏÇÒÁÍÍÁ. îÁÊÄÉÔÅ ËÏÓÉÎÕÓ ÕÇÌÁ ÍÅÖÄÕ ÜÔÉÍÉ ÐÒÑÍÙÍÉ.

ïËÒÕÖÎÏÓÔØ, ×ÐÉÓÁÎÎÁÑ × ÔÒÅÕÇÏÌØÎÉË ABC, ËÁÓÁÅÔÓÑ ÓÔÏÒÏÎÙ AB × ÔÏÞËÅ M,
AM = 1, BM = 4. îÁÊÄÉÔÅ CM, ÅÓÌÉ ÉÚ×ÅÓÔÎÏ, ÞÔÏ \BAC = 120.
ïÓÎÏ×ÁÎÉÑ ÔÒÁÐÅÃÉÉ ÒÁ×ÎÙ 1 É 6, Á ÄÉÁÇÏÎÁÌÉ | 3 É 5. ðÏÄ ËÁËÉÍ ÕÇÌÏÍ ×ÉÄÎÙ

136.29.

ÐÒÉ ÜÔÏÍ

136.30.

ÏÓÎÏ×ÁÎÉÑ ÉÚ ÔÏÞËÉ ÐÅÒÅÓÅÞÅÎÉÑ ÄÉÁÇÏÎÁÌÅÊ?
136.31.

÷ ×ÙÐÕËÌÏÍ ÞÅÔÙÒÅÈÕÇÏÌØÎÉËÅ ÏÔÒÅÚËÉ, ÓÏÅÄÉÎÑÀÝÉÅ ÓÅÒÅÄÉÎÙ ÐÒÏÔÉ×ÏÐÏÌÏÖ-

ÎÙÈ ÓÔÏÒÏÎ, ÒÁ×ÎÙ ÓÏÏÔ×ÅÔÓÔ×ÅÎÎÏ
ÞÅÔÙÒÅÈÕÇÏÌØÎÉËÁ.
136.32.

a É b É ÐÅÒÅÓÅËÁÀÔÓÑ ÐÏÄ ÕÇÌÏÍ 60. îÁÊÄÉÔÅ ÄÉÁÇÏÎÁÌÉ

äÉÁÇÏÎÁÌÉ ×ÙÐÕËÌÏÇÏ ÞÅÔÙÒÅÈÕÇÏÌØÎÉËÁ ÒÁ×ÎÙ

c

É

d

É ÐÅÒÅÓÅËÁÀÔÓÑ ÐÏÄ

45.

îÁÊÄÉÔÅ ÏÔÒÅÚËÉ, ÓÏÅÄÉÎÑÀÝÉÅ ÓÅÒÅÄÉÎÙ ÐÒÏÔÉ×ÏÐÏÌÏÖÎÙÈ ÓÔÏÒÏÎ ÞÅÔÙÒÅÈ-

136.33.

ãÅÎÔÒ ÏËÒÕÖÎÏÓÔÉ, ×ÐÉÓÁÎÎÏÊ × ÐÒÑÍÏÕÇÏÌØÎÙÊ ÔÒÅÕÇÏÌØÎÉË, ÕÄÁÌÅÎ ÏÔ ×ÅÒÛÉÎ

ÕÇÌÏÍ

ÕÇÏÌØÎÉËÁ.

a É b. îÁÊÄÉÔÅ ÇÉÐÏÔÅÎÕÚÕ.
ôÏÞËÁ M ÌÅÖÉÔ ÎÁ ÓÔÏÒÏÎÅ BC ÐÁÒÁÌÌÅÌÏÇÒÁÍÍÁ ABCD Ó ÕÇÌÏÍ 45 ÐÒÉ ×ÅÒÛÉÎÅ
A, ÐÒÉÞ£Í \AMD = 90 É BM : MC = 2 : 3. îÁÊÄÉÔÅ ÏÔÎÏÛÅÎÉÅ ÓÏÓÅÄÎÉÈ ÓÔÏÒÏÎ

ÏÓÔÒÙÈ ÕÇÌÏ× ÎÁ ÒÁÓÓÔÏÑÎÉÑ
136.34.

ÐÁÒÁÌÌÅÌÏÇÒÁÍÍÁ.

ABCD ÏÓÎÏ×ÁÎÉÅ AD ÒÁ×ÎÏ 16, Á ÂÏËÏ×ÁÑ ÓÔÏÒÏÎÁ CD ÒÁ×ÎÁ 8 3.
ïËÒÕÖÎÏÓÔØ ÐÒÏÈÏÄÑÝÁÑ ÞÅÒÅÚ ÔÏÞËÉ A, B É C, ÐÅÒÅÓÅËÁÅÔ ÐÒÑÍÕÀ AD × ÔÏÞËÅ M; \AMB =
60. îÁÊÄÉÔÅ BM.
îÁ ÂÏËÏ×ÏÊ ÓÔÏÒÏÎÅ BC ÒÁ×ÎÏÂÅÄÒÅÎÎÏÇÏ ÔÒÅÕÇÏÌØÎÉËÁ ABC ËÁË ÎÁ ÄÉÁÍÅÔÒÅ
ÐÏÓÔÒÏÅÎÁ ÏËÒÕÖÎÏÓÔØ, ÐÅÒÅÓÅËÁÀÝÁÑ ÏÓÎÏ×ÁÎÉÅ ÜÔÏÇÏ ÔÒÅÕÇÏÌØÎÉËÁ × ÔÏÞËÅ D. îÁÊÄÉÔÅ
ÒÁÓÓÔÏÑÎÉÅ ÏÔ ×ÅÒÛÉÎÙ A ÄÏ ÃÅÎÔÒÁ ÏËÒÕÖÎÏÓÔÉ, ÅÓÌÉ AD = 3, Á ÕÇÏÌ ABC ÒÁ×ÅÎ 120.
ïËÒÕÖÎÏÓÔØ, ×ÐÉÓÁÎÎÁÑ × ÐÒÑÍÏÕÇÏÌØÎÙÊ ÔÒÅÕÇÏÌØÎÉË Ó ËÁÔÅÔÁÍÉ 6 É 8, ËÁÓÁÅÔÓÑ
ÇÉÐÏÔÅÎÕÚÙ × ÔÏÞËÅ M. îÁÊÄÉÔÅ ÒÁÓÓÔÏÑÎÉÅ ÏÔ ÔÏÞËÉ M ÄÏ ×ÅÒÛÉÎÙ ÐÒÑÍÏÇÏ ÕÇÌÁ.
ôÏÞËÁ M ÌÅÖÉÔ ÎÁ ÓÔÏÒÏÎÅ AC ÒÁ×ÎÏÓÔÏÒÏÎÎÅÇÏ ÔÒÅÕÇÏÌØÎÉËÁ ABC ÓÏ ÓÔÏÒÏÎÏÊ,
ÒÁ×ÎÏÊ 3a, ÐÒÉÞÅÍ AM : MC = 1 : 2. ôÏÞËÉ K É L ÎÁ ÓÔÏÒÏÎÁÈ AB É BC Ñ×ÌÑÀÔÓÑ
136.35.

p

÷ ÔÒÁÐÅÃÉÉ

136.36.

p

136.37.

136.38.

222

MKL. îÁÊÄÉÔÅ ÅÇÏ ÓÔÏÒÏÎÙ.
÷ ÔÒÅÕÇÏÌØÎÉËÅ ABC ÐÒÏ×ÅÄÅÎÙ ×ÙÓÏÔÙ AD É CE. îÁÊÄÉÔÅ AC, ÅÓÌÉ BC = a,
AB = b, DE=AC = k.
÷ ÏËÒÕÖÎÏÓÔÉ ÐÒÏ×ÅÄÅÎÙ ÈÏÒÄÙ AB É BC, ÐÒÉÞÅÍ AB =
3, BC = 3 3,
\ABC = 60. îÁÊÄÉÔÅ ÄÌÉÎÕ ÔÏÊ ÈÏÒÄÙ ÏËÒÕÖÎÏÓÔÉ, ËÏÔÏÒÁÑ ÄÅÌÉÔ ÕÇÏÌ ABC ÐÏÐÏÌÁÍ.
äÁÎ ÔÒÅÕÇÏÌØÎÉË ABC. éÚ×ÅÓÔÎÏ, ÞÔÏ AB = 4, AC = 2 É BC = 3. âÉÓÓÅËÔÒÉÓÁ
ÕÇÌÁ BAC ÐÅÒÅÓÅËÁÅÔ ÓÔÏÒÏÎÕ BC × ÔÏÞËÅ K. ðÒÑÍÁÑ, ÐÒÏÈÏÄÑÝÁÑ ÞÅÒÅÚ ÔÏÞËÕ B ÐÁÒÁÌÌÅÌØÎÏ
AC, ÐÅÒÅÓÅËÁÅÔ ÐÒÏÄÏÌÖÅÎÉÅ ÂÉÓÓÅËÔÒÉÓÙ AK × ÔÏÞËÅ M. îÁÊÄÉÔÅ KM.
÷ ÔÒÅÕÇÏÌØÎÉË ABC ×ÐÉÓÁÎÁ ÏËÒÕÖÎÏÓÔØ, ËÏÔÏÒÁÑ ËÁÓÁÅÔÓÑ ÓÔÏÒÏÎ AB, BC É
AC ÓÏÏÔ×ÅÔÓÔ×ÅÎÎÏ × ÔÏÞËÁÈ M, D É N. îÁÊÄÉÔÅ MD, ÅÓÌÉ ÉÚ×ÅÓÔÎÏ, ÞÔÏ NA = 2, NC = 3,
\BCA = 60.
÷ ÏËÒÕÖÎÏÓÔÉ ÒÁÄÉÕÓÁ R = 4 ÐÒÏ×ÅÄÅÎÙ ÈÏÒÄÁ AB É ÄÉÁÍÅÔÒ AK, ÏÂÒÁÚÕÀÝÉÊ Ó ÈÏÒÄÏÊ ÕÇÏÌ 22;5 . ÷ ÔÏÞËÅ B ÐÒÏ×ÅÄÅÎÁ ËÁÓÁÔÅÌØÎÁÑ Ë ÏËÒÕÖÎÏÓÔÉ, ÐÅÒÅÓÅËÁÀÝÁÑ
ÐÒÏÄÏÌÖÅÎÉÅ ÄÉÁÍÅÔÒÁ AK × ÔÏÞËÅ C. îÁÊÄÉÔÅ ÍÅÄÉÁÎÕ AM ÔÒÅÕÇÏÌØÎÉËÁ ABC.
÷ ÔÒÅÕÇÏÌØÎÉËÅ ABC ÓÔÏÒÏÎÁ AC ÂÏÌØÛÅ ÓÔÏÒÏÎÙ AB. äÏËÁÖÉÔÅ, ÞÔÏ ÍÅÄÉÁÎÁ,
ÐÒÏ×ÅÄÅÎÎÁÑ ÉÚ ×ÅÒÛÉÎÙ B, ÍÅÎØÛÅ ÍÅÄÉÁÎÙ, ÐÒÏ×ÅÄÅÎÎÏÊ ÉÚ ×ÅÒÛÉÎÙ A.
óÔÏÒÏÎÙ ÔÒÅÕÇÏÌØÎÉËÁ ÒÁ×ÎÙ a, b É c. îÁÊÄÉÔÅ ÂÉÓÓÅËÔÒÉÓÕ, ÐÒÏ×ÅÄÅÎÎÕÀ Ë
ÓÔÏÒÏÎÅ a.
×ÅÒÛÉÎÁÍÉ ÄÒÕÇÏÇÏ ÒÁ×ÎÏÓÔÏÒÏÎÎÅÇÏ ÔÒÅÕÇÏÌØÎÉËÁ
136.39.

p

p

136.40.

136.41.

136.42.

136.43.

136.44.

136.45.

136.46.

(ôÅÏÒÅÍÁ ûÔÅÊÎÅÒÁ{ìÅÍÕÓÁ) ä×Å ÂÉÓÓÅËÔÒÉÓÙ ÔÒÅÕÇÏÌØÎËÁ ÒÁ×ÎÙ ÍÅÖÄÕ ÓÏÂÏÊ.

äÏËÁÖÉÔÅ, ÞÔÏ ÔÒÅÕÇÏÌØÎÉË ÒÁ×ÎÏÂÅÄÒÅÎÎÙÊ.

ABCD Ó ÏÓÎÏ×ÁÎÉÑÍÉ AD = 3 39 É BC = 39. ðÒÉ ÜÔÏÍ

\BAD = 30 , É \ADC = 60. þÅÒÅÚ ÔÏÞËÕ D ÐÒÏÈÏÄÉÔ ÐÒÑÍÁÑ, ÄÅÌÑÝÁÑ ÔÒÁÐÅÃÉÀ ÎÁ
136.47.

äÁÎÁ ÔÒÁÐÅÃÉÑ

p

p

Ä×Å ÒÁ×ÎÏ×ÅÌÉËÉÅ ÆÉÇÕÒÙ. îÁÊÄÉÔÅ ÄÌÉÎÕ ×ÓÅÇÏ ÏÔÒÅÚËÁ ÜÔÏÊ ÐÒÑÍÏÊ, ÎÁÈÏÄÑÝÅÇÏÓÑ ×ÎÕÔÒÉ
ÔÒÁÐÅÃÉÉ.
136.48.

äÁÎ ÐÁÒÁÌÌÅÌÏÇÒÁÍÍ

ABCD, × ËÏÔÏÒÏÍ AB = a, BC = b, \ABC = . îÁÊÄÉÔÅ
BCD É DAB.

ÒÁÓÓÔÏÑÎÉÅ ÍÅÖÄÕ ÃÅÎÔÒÁÍÉ ÏËÒÕÖÎÏÓÔÅÊ, ÏÐÉÓÁÎÎÙÈ ÏËÏÌÏ ÔÒÅÕÇÏÌØÎÉËÏ×
136.49.

äÏËÁÖÉÔÅ, ÞÔÏ ÓÕÍÍÁ Ë×ÁÄÒÁÔÏ× ÒÁÓÓÔÏÑÎÉÊ ÏÔ ËÁËÏÊ-ÌÉÂÏ ÔÏÞËÉ ÏËÒÕÖÎÏÓÔÉ ÄÏ

×ÅÒÛÉÎ ÐÒÁ×ÉÌØÎÏÇÏ ×ÐÉÓÁÎÎÏÇÏ ÔÒÅÕÇÏÌØÎÉËÁ ÅÓÔØ ×ÅÌÉÞÉÎÁ ÐÏÓÔÏÑÎÎÁÑ (ÎÅ ÚÁ×ÉÓÑÝÁÑ ÏÔ
ÐÏÌÏÖÅÎÉÑ ÔÏÞËÉ ÎÁ ÏËÒÕÖÎÏÓÔÉ).

îÁ ÏËÒÕÖÎÏÓÔÉ ×ÚÑÔÙ Ä×Å ÄÉÁÍÅÔÒÁÌØÎÏ ÐÒÏÔÉ×ÏÐÏÌÏÖÎÙÅ ÔÏÞËÉ A É C, Á ÔÏÞB ÒÁÓÐÏÌÏÖÅÎÁ ×ÎÅ ÏËÒÕÖÎÏÓÔÉ. ïÔÒÅÚÏË AB ÐÅÒÅÓÅËÁÅÔÓÑ Ó ÏËÒÕÖÎÏÓÔØÀ × ÔÏÞËÅ P,
ÏÔÒÅÚÏË CB | × ÔÏÞËÅ Q. éÚ×ÅÓÔÎÏ, ÞÔÏ \ABC = 45 , AB : AC = 1 : 3. îÁÊÄÉÔÅ ÏÔÎÏÛÅÎÉÅ ÏÔÒÅÚËÏ× CP É AQ.
óÔÏÒÏÎÁ ÒÏÍÂÁ ABCD ÒÁ×ÎÁ a, Á ÏÓÔÒÙÊ ÕÇÏÌ ÒÁ×ÅÎ . îÁ ÏÔÒÅÚËÁÈ AD É BC ËÁË
136.50.

ËÁ

p

136.51.

ÎÁ ÓÔÏÒÏÎÁÈ ÐÏÓÔÒÏÅÎÙ ÐÒÁ×ÉÌØÎÙÅ ÔÒÅÕÇÏÌØÎÉËÉ (×ÎÅ ÒÏÍÂÁ). îÁÊÄÉÔÅ ÒÁÓÓÔÏÑÎÉÅ ÍÅÖÄÕ

ÃÅÎÔÒÁÍÉ ÜÔÉÈ ÔÒÅÕÇÏÌØÎÉËÏ×.

2 ×ÐÉÓÁÎ ÐÒÁ×ÉÌØÎÙÊ ÛÅÓÔÉÕÇÏÌØÎÉË ABCDEF. éÚ ÔÏÞ11 - 1,
ËÉ K, ÌÅÖÁÝÅÊ ÎÁ ÐÒÏÄÏÌÖÅÎÉÉ ÓÔÏÒÏÎÙ AF, ÄÌÑ ËÏÔÏÒÏÊ KA < KF É KA =
ÐÒÏ×ÅÄÅÎÁ ÓÅËÕÝÁÑ KH, ÐÅÒÅÓÅËÁÀÝÁÑ ÏËÒÕÖÎÏÓÔØ × ÔÏÞËÁÈ N É H. éÚ×ÅÓÔÎÏ, ÞÔÏ ×ÎÅÛÎÑÑ
ÞÁÓÔØ ÓÅËÕÝÅÊ KN ÒÁ×ÎÁ 2, Á ÕÇÏÌ NFH | ÔÕÐÏÊ. îÁÊÄÉÔÅ ÕÇÏÌ HKF.
ïËÒÕÖÎÏÓÔØ, ×ÐÉÓÁÎÎÁÑ × ÔÒÅÕÇÏÌØÎÉË ABC ÄÅÌÉÔ ÍÅÄÉÁÎÕ BM ÎÁ ÔÒÉ ÒÁ×ÎÙÅ
ÞÁÓÔÉ. îÁÊÄÉÔÅ ÏÔÎÏÛÅÎÉÅ BC : CA : AB.
íÅÄÉÁÎÁ AD ÏÓÔÒÏÕÇÏÌØÎÏÇÏ ÔÒÅÕÇÏÌØÎÉËÁ ABC ÒÁ×ÎÁ 5. ðÒÏÅËÃÉÉ ÜÔÏÊ ÍÅÄÉÁÎÙ ÎÁ ÓÔÏÒÏÎÙ AB É AC ÒÁ×ÎÙ 4 É 2 5 ÓÏÏÔ×ÅÔÓÔ×ÅÎÎÏ. îÁÊÄÉÔÅ ÓÔÏÒÏÎÕ BC.
136.52.

÷ ÏËÒÕÖÎÏÓÔØ ÒÁÄÉÕÓÁ

p

136.53.

136.54.

p

223

136.55.

(ôÅÏÒÅÍÁ óÔÀÁÒÔÁ) ôÏÞËÁ

äÏËÁÖÉÔÅ, ÞÔÏ

D ÒÁÓÐÏÌÏÖÅÎÁ ÎÁ ÓÔÏÒÏÎÅ BC ÔÒÅÕÇÏÌØÎÉËÁ ABC.

AB2 DC + AC2 BD - AD2 BC = BC DC BD:
4 4
a + b4.
äÁÎÙ ÏÔÒÅÚËÉ a É b. ðÏÓÔÒÏÊÔÅ ÏÔÒÅÚÏË, ÒÁ×ÎÙÊ
ôÏÞËÁ D ÌÅÖÉÔ ÎÁ ÓÔÏÒÏÎÅ AC ÔÒÅÕÇÏÌØÎÉËÁ ABC. ïËÒÕÖÎÏÓÔØ ÒÁÄÉÕÓÁ 2= 3,
×ÐÉÓÁÎÎÁÑ × ÔÒÅÕÇÏÌØÎÉË ABD, ËÁÓÁÅÔÓÑ ÓÔÏÒÏÎÙ AB × ÔÏÞËÅ M, Á ÏËÒÕÖÎÏÓÔØ ÒÁÄÉÕÓÁ 3,
×ÐÉÓÁÎÎÁÑ × ÔÒÅÕÇÏÌØÎÉË BCD, ËÁÓÁÅÔÓÑ ÓÔÏÒÏÎÙ BC × ÔÏÞËÅ N. éÚ×ÅÓÔÎÏ, ÞÔÏ BM = 6,
BN = 5. îÁÊÄÉÔÅ ÓÔÏÒÏÎÙ ÔÒÅÕÇÏÌØÎÉËÁ ABC.
ïËÏÌÏ ÏËÒÕÖÎÏÓÔÉ ÏÐÉÓÁÎÁ ÒÁ×ÎÏÂÅÄÒÅÎÎÁÑ ÔÒÁÐÅÃÉÑ Ó ÏÓÎÏ×ÁÎÉÑÍÉ AD É BC
(AD > BC). ðÒÑÍÁÑ, ÐÁÒÁÌÌÅÌØÎÁÑ ÄÉÁÇÏÎÁÌÉ AC, ÐÅÒÅÓÅËÁÅÔ ÓÔÏÒÏÎÙ AD É CD × ÔÏÞËÁÈ M É N ÓÏÏÔ×ÅÔÓÔ×ÅÎÎÏ É ËÁÓÁÅÔÓÑ ÏËÒÕÖÎÏÓÔÉ × ÔÏÞËÅ P. îÁÊÄÉÔÅ ÕÇÌÙ ÔÒÁÐÅÃÉÉ, ÅÓÌÉ
MP=PN = k (k < 1).
óÔÏÒÏÎÁ BC ÔÒÅÕÇÏÌØÎÉËÁ ABC ÒÁ×ÎÁ 4, ÓÔÏÒÏÎÁ AB ÒÁ×ÎÁ 2 19. éÚ×ÅÓÔÎÏ, ÞÔÏ










p

136.56.

p

136.57.

p

136.58.

p

136.59.

ÃÅÎÔÒ ÏËÒÕÖÎÏÓÔÉ, ÐÒÏÈÏÄÑÝÅÊ ÞÅÒÅÚ ÓÅÒÅÄÉÎÙ ÓÔÏÒÏÎ ÔÒÅÕÇÏÌØÎÉËÁ, ÌÅÖÉÔ ÎÁ ÂÉÓÓÅËÔÒÉÓÅ

ÕÇÌÁ

C. îÁÊÄÉÔÅ AC.

224

137. ôÅÏÒÅÍÁ ÓÉÎÕÓÏ×

a, b, c - ÓÔÏÒÏÎÙ ÔÒÅÕÇÏÌØÎÉËÁ;

ðÕÓÔØ

ÏÐÉÓÁÎÎÏÊ ÏËÒÕÖÎÏÓÔÉ.

,

,

- ÐÒÏÔÉ×ÏÌÅÖÁÝÉÅ ÉÍ ÕÇÌÙ;

ôÅÏÒÅÍÁ ÓÉÎÕÓÏ×.

a = b = c = 2R:
sin
sin
sin
137.1.

ÒÁ×ÅÎ

120

âÏËÏ×ÁÑ ÓÔÏÒÏÎÁ ÒÁ×ÎÏÂÅÄÒÅÎÎÏÇÏ ÔÒÅÕÇÏÌØÎÉËÁ ÒÁ×ÎÁ

. îÁÊÄÉÔÅ ÄÉÁÍÅÔÒ ÏÐÉÓÁÎÎÏÊ ÏËÒÕÖÎÏÓÔÉ.

137.2.

ÎÙÍÉ

2,

Á ÕÇÏÌ ÐÒÉ ×ÅÒÛÉÎÅ

îÁÊÄÉÔÅ ÒÁÄÉÕÓ ÏËÒÕÖÎÏÓÔÉ, ÏÐÉÓÁÎÎÏÊ ÏËÏÌÏ ÔÒÅÕÇÏÌØÎÉËÁ ÓÏ ÓÔÏÒÏÎÁÍÉ, ÒÁ×-

a, a É b.

137.3.

îÁÊÄÉÔÅ ÇÉÐÏÔÅÎÕÚÕ ÐÒÑÍÏÕÇÏÌØÎÏÇÏ ÔÒÅÕÇÏÌØÎÉËÁ Ó ÏÓÔÒÙÍ ÕÇÌÏÍ, ÒÁ×ÎÙÍ

ÅÓÌÉ ÉÚ×ÅÓÔÎÏ, ÞÔÏ ÂÉÓÓÅËÔÒÉÓÁ, ÐÒÏ×ÅÄÅÎÎÁÑ ÉÚ ×ÅÒÛÉÎÙ ÐÒÑÍÏÇÏ ÕÇÌÁ, ÒÁ×ÎÁ
137.4.

ÎÙÍÉ

R - ÒÁÄÉÕÓ

îÁÊÄÉÔÅ ÒÁÄÉÕÓ ÏËÒÕÖÎÏÓÔÉ, ÏÐÉÓÁÎÎÏÊ ÏËÏÌÏ ÔÒÅÕÇÏÌØÎÉËÁ ÓÏ ÓÔÏÒÏÎÁÍÉ, ÒÁ×-

13, 14, 15.

137.5.

âÏËÏ×ÁÑ ÓÔÏÒÏÎÁ ÒÁ×ÎÏÂÏËÏÊ ÔÒÁÐÅÃÉÉ ÒÁ×ÎÁ

ÕÇÌÏ× ÐÒÉ ÂÏÌØÛÅÍ ÏÓÎÏ×ÁÎÉÉ ÒÁ×ÅÎ
ÔÒÁÐÅÃÉÉ.
137.6.

30

a, ÓÒÅÄÎÑÑ ÌÉÎÉÑ ÒÁ×ÎÁ b, Á ÏÄÉÎ

 . îÁÊÄÉÔÅ ÒÁÄÉÕÓ ÏËÒÕÖÎÏÓÔÉ, ÏÐÉÓÁÎÎÏÊ ÏËÏÌÏ ÜÔÏÊ

ïÓÎÏ×ÁÎÉÑ ÒÁ×ÎÏÂÏËÏÊ ÔÒÁÐÅÃÉÉ ÒÁ×ÎÙ

ÏËÒÕÖÎÏÓÔÉ, ÏÐÉÓÁÎÎÏÊ ÏËÏÌÏ ÔÒÁÐÅÃÉÉ.
137.7.

a.

30,

9 É 21, ×ÙÓÏÔÁ ÒÁ×ÎÁ 8. îÁÊÄÉÔÅ ÒÁÄÉÕÓ

ðÒÑÍÁÑ, ÐÅÒÅÓÅËÁÀÝÁÑ ÏÓÎÏ×ÁÎÉÅ ÒÁ×ÎÏÂÅÄÒÅÎÎÏÇÏ ÔÒÅÕÇÏÌØÎÉËÁ É ÐÒÏÈÏÄÑÝÁÑ

ÞÅÒÅÚ ÐÒÏÔÉ×ÏÐÏÌÏÖÎÕÀ ×ÅÒÛÉÎÕ, ÄÅÌÉÔ ÜÔÏÔ ÔÒÅÕÇÏÌØÎÉË ÎÁ Ä×Á. äÏËÁÖÉÔÅ, ÞÔÏ ÒÁÄÉÕÓÙ
ÏËÒÕÖÎÏÓÔÅÊ, ÏÐÉÓÁÎÎÙÈ ÏËÏÌÏ ÜÔÉÈ ÔÒÅÕÇÏÌØÎÉËÏ×, ÒÁ×ÎÙ.
137.8.

ó ÐÏÍÏÝØÀ ÔÅÏÒÅÍÙ ÓÉÎÕÓÏ× ÄÏËÁÖÉÔÅ, ÞÔÏ ÂÉÓÓÅËÔÒÉÓÁ ÔÒÅÕÇÏÌØÎÉËÁ ÄÅÌÉÔ ÅÇÏ

ÓÔÏÒÏÎÕ ÎÁ ÏÔÒÅÚËÉ, ÐÒÏÐÏÒÃÉÏÎÁÌØÎÙÅ Ä×ÕÍ ÄÒÕÇÉÍ ÓÔÏÒÏÎÁÍ.
137.9.

÷ ÔÒÅÕÇÏÌØÎÉËÅ ÉÚ×ÅÓÔÎÙ ÓÔÏÒÏÎÁ

a É Ä×Á ÐÒÉÌÅÖÁÝÉÈ Ë ÎÅÊ ÕÇÌÁ

É

. îÁÊÄÉÔÅ

ÂÉÓÓÅËÔÒÉÓÕ, ÐÒÏ×ÅÄÅÎÎÕÀ ÉÚ ×ÅÒÛÉÎÙ ÔÒÅÔØÅÇÏ ÕÇÌÁ.
137.10.

íÅÄÉÁÎÁ

ÕÇÌÙ, ÒÁ×ÎÙÅ

É

AM ÔÒÅÕÇÏÌØÎÉËÁ ABC ÒÁ×ÎÁ m É ÏÂÒÁÚÕÅÔ ÓÏ ÓÔÏÒÏÎÁÍÉ AB É AC

ÓÏÏÔ×ÅÔÓÔ×ÅÎÎÏ. îÁÊÄÉÔÅ ÜÔÉ ÓÔÏÒÏÎÙ.

äÁÎ ÔÒÅÕÇÏÌØÎÉË ABC, × ËÏÔÏÒÏÍ \A = ; \B = . îÁ ÓÔÏÒÏÎÅ AB ×ÚÑÔÁ ÔÏÞËÁ
D, Á ÎÁ ÓÔÏÒÏÎÅ AC - ÔÏÞËÁ M, ÐÒÉÞÅÍ CD | ÂÉÓÓÅËÔÒÉÓÁ ÔÒÅÕÇÏÌØÎÉËÁ ABC, DMkBC É
AM = a. îÁÊÄÉÔÅ CM.
õÇÌÙ ÔÒÅÕÇÏÌØÎÉËÁ ÒÁ×ÎÙ ,
É , Á ÐÅÒÉÍÅÔÒ ÒÁ×ÅÎ P. îÁÊÄÉÔÅ ÓÔÏÒÏÎÙ
137.11.

137.12.

ÔÒÅÕÇÏÌØÎÉËÁ.
137.13.

ïÄÎÁ ÉÚ ÂÏËÏ×ÙÈ ÓÔÏÒÏÎ ÔÒÁÐÅÃÉÉ ÏÂÒÁÚÕÅÔ Ó ÂÏÌØÛÉÍ ÏÓÎÏ×ÁÎÉÅÍ ÕÇÏÌ, ÒÁ×ÎÙÊ

a É ÏÂÒÁÚÕÅÔ Ó ÍÅÎØÛÉÍ ÏÓÎÏ×ÁÎÉÅÍ ÕÇÏÌ, ÒÁ×ÎÙÊ . îÁÊÄÉÔÅ ÓÒÅÄÎÀÀ
b.
ïÓÎÏ×ÁÎÉÑ ÔÒÁÐÅÃÉÉ ÒÁ×ÎÙ 4 É 16. îÁÊÄÉÔÅ ÒÁÄÉÕÓÙ ÏËÒÕÖÎÏÓÔÅÊ, ×ÐÉÓÁÎÎÏÊ ×

, Á ×ÔÏÒÁÑ ÒÁ×ÎÁ

ÌÉÎÉÀ ÔÒÁÐÅÃÉÉ, ÅÓÌÉ ÍÅÎØÛÅÅ ÏÓÎÏ×ÁÎÉÅ ÒÁ×ÎÏ
137.14.

ÔÒÁÐÅÃÉÀ É ÏÐÉÓÁÎÎÏÊ ÏËÏÌÏ ÎÅÅ, ÅÓÌÉ ÉÚ×ÅÓÔÎÏ, ÞÔÏ ÜÔÉ ÏËÒÕÖÎÏÓÔÉ ÓÕÝÅÓÔ×ÕÀÔ.

AB ÔÒÅÕÇÏÌØÎÉËÁ ABC ×Ï ×ÎÅÛÎÀÀ ÓÔÏÒÏÎÕ ÐÏÓÔÒÏÅÎ ÒÁ×ÎÏÓÔÏÒÏÎÎÉÊ ÔÒÅÕÇÏÌØÎÉË. îÁÊÄÉÔÅ ÒÁÓÓÔÏÑÎÉÅ ÍÅÖÄÕ ÅÇÏ ÃÅÎÔÒÏÍ É ×ÅÒÛÉÎÏÊ C, ÅÓÌÉ AB = c É
\C = 120.
137.15.

îÁ ÓÔÏÒÏÎÅ

225

137.16.

óÔÏÒÏÎÙ ÔÒÅÕÇÏÌØÎÉËÁ ÒÁ×ÎÙ

1 É 2, Á ÕÇÏÌ ÍÅÖÄÕ ÎÉÍÉ ÒÁ×ÅÎ 60. þÅÒÅÚ ÃÅÎÔÒ

×ÐÉÓÁÎÎÏÊ ÏËÒÕÖÎÏÓÔÉ ÜÔÏÇÏ ÔÒÅÕÇÏÌØÎÉËÁ É ËÏÎÃÙ ÔÒÅÔØÅÊ ÓÔÏÒÏÎÙ ÐÒÏ×ÅÄÅÎÁ ÏËÒÕÖÎÏÓÔØ.
îÁÊÄÉÔÅ ÅÅ ÒÁÄÉÕÓ.

a, b É ÐÒÏÔÉ×ÏÌÅÖÁÝÉÅ ÉÍ ÕÇÌÙ É ÔÒÅÕÇÏÌØÎÉËÁ
a= cos = b= cos , ÔÏ ÔÒÅÕÇÏÌØÎÉË ÒÁ×ÎÏÂÅÄÒÅÎÎÙÊ.
ä×Å ÏËÒÕÖÎÏÓÔÉ ÐÅÒÅÓÅËÁÀÔÓÑ × ÔÏÞËÁÈ A É B. ðÒÑÍÁÑ, ÐÒÏÈÏÄÑÝÁÑ ÞÅÒÅÚ ÔÏÞËÕ
A, ×ÔÏÒÉÞÎÏ ÐÅÒÅÓÅËÁÅÔ ÜÔÉ ÏËÒÕÖÎÏÓÔÉ × ÔÏÞËÁÈ C É D, ÐÒÉÞÅÍ ÔÏÞËÁ A ÌÅÖÉÔ ÍÅÖÄÕ
C É D, Á ÈÏÒÄÙ AC É AD ÐÒÏÐÏÒÃÉÏÎÁÌØÎÙ ÒÁÄÉÕÓÁÍ Ó×ÏÉÈ ÏËÒÕÖÎÏÓÔÅÊ. äÏËÁÖÉÔÅ, ÞÔÏ
ÂÉÓÓÅËÔÒÉÓÙ ÕÇÌÏ× ADB É ACB ÐÅÒÅÓÅËÁÀÔÓÑ ÎÁ ÏÔÒÅÚËÅ AB.
137.17.

äÏËÁÖÉÔÅ, ÞÔÏ ÅÓÌÉ ÓÔÏÒÏÎÙ

Ó×ÑÚÁÎÙ ÓÏÏÔÎÏÛÅÎÉÑÍÉ
137.18.

137.19.

÷ ÏËÒÕÖÎÏÓÔØ ×ÐÉÓÁÎÙ Ä×Å ÒÁ×ÎÏÂÅÄÒÅÎÎÙÅ ÔÒÁÐÅÃÉÉ Ó ÓÏÏÔ×ÅÔÓÔ×ÅÎÎÏ ÐÁÒÁÌ-

ÌÅÌØÎÙÍÉ ÓÔÏÒÏÎÁÍÉ. äÏËÁÖÉÔÅ, ÞÔÏ ÄÉÁÇÏÎÁÌØ ÏÄÎÏÊ ÉÚ ÎÉÈ ÒÁ×ÎÁ ÄÉÁÇÏÎÁÌÉ ÄÒÕÇÏÊ ÔÒÁÐÅÃÉÉ.
137.20.

äÏËÁÖÉÔÅ, ÞÔÏ ÄÌÑ ÌÀÂÏÇÏ ÔÒÅÕÇÏÌØÎÉËÁ ÐÒÏÅËÃÉÑ ÄÉÁÍÅÔÒÁ ÏÐÉÓÁÎÎÏÊ ÏËÒÕÖ-

ÎÏÓÔÉ, ÐÅÒÐÅÎÄÉËÕÌÑÒÎÏÇÏ ÏÄÎÏÊ ÓÔÏÒÏÎÅ ÔÒÅÕÇÏÌØÎÉËÁ, ÎÁ ÐÒÑÍÕÀ, ÓÏÄÅÒÖÁÝÕÀ ×ÔÏÒÕÀ
ÓÔÏÒÏÎÕ, ÒÁ×ÎÁ ÔÒÅÔØÅÊ ÓÔÏÒÏÎÅ.
137.21.

ëÁÖÄÏÅ ÉÚ ÏÓÎÏ×ÁÎÉÊ ×ÙÓÏÔ ÔÒÅÕÇÏÌØÎÉËÁ ÐÒÏÅÃÉÒÕÅÔÓÑ ÎÁ ÅÇÏ ÓÔÏÒÏÎÙ. äÏËÁ-

ÖÉÔÅ, ÞÔÏ ÄÌÉÎÁ ÏÔÒÅÚËÁ, ÓÏÅÄÉÎÑÀÝÅÇÏ ÐÒÏÅËÃÉÉ, ÎÅ ÚÁ×ÉÓÉÔ ÏÔ ×ÙÂÏÒÁ ×ÙÓÏÔÙ.
137.22.

îÁ ÏËÒÕÖÎÏÓÔÉ, ÏÐÉÓÁÎÎÏÊ ÏËÏÌÏ ÔÒÅÕÇÏÌØÎÉËÁ

ABC, ÎÁÊÄÉÔÅ ÔÏÞËÕ M ÔÁËÕÀ,

AC É BC ÍÁËÓÉÍÁÌØÎÏ.
÷ÙÓÏÔÙ ÔÒÅÕÇÏÌØÎÉËÁ ABC ÐÅÒÅÓÅËÁÀÔÓÑ × ÔÏÞËÅ H. äÏËÁÖÉÔÅ, ÞÔÏ ÒÁÄÉÕÓÙ
ÏËÒÕÖÎÏÓÔÅÊ, ÏÐÉÓÁÎÎÙÈ ÏËÏÌÏ ÔÒÅÕÇÏÌØÎÉËÏ× ABC, AHB, BHC É AHC, ÒÁ×ÎÙ ÍÅÖÄÕ ÓÏ-

ÞÔÏ ÒÁÓÓÔÏÑÎÉÅ ÍÅÖÄÕ ÅÅ ÐÒÏÅËÃÉÑÍÉ ÎÁ ÐÒÑÍÙÅ
137.23.

ÂÏÊ.

M ÎÁ ÏËÒÕÖÎÏÓÔÉ ÐÒÏ×ÅÄÅÎÙ ÔÒÉ ÈÏÒÄÙ: MN = 1, MP = 6, MQ = 2.
ðÒÉ ÜÔÏÍ ÕÇÌÙ NMP É PMQ ÒÁ×ÎÙ. îÁÊÄÉÔÅ ÒÁÄÉÕÓ ÏËÒÕÖÎÏÓÔÉ.
÷ ÔÒÅÕÇÏÌØÎÉËÅ ABC ÉÚ×ÅÓÔÎÏ, ÞÔÏ AB = 2, AC = 5, BC = 6. îÁÊÄÉÔÅ ÒÁÓÓÔÏÑÎÉÅ ÏÔ ×ÅÒÛÉÎÙ B ÄÏ ÔÏÞËÉ ÐÅÒÅÓÅÞÅÎÉÑ ×ÙÓÏÔ ÔÒÅÕÇÏÌØÎÉËÁ ABC.
÷ ÏÓÔÒÏÕÇÏÌØÎÏÍ ÔÒÅÕÇÏÌØÎÉËÅ ABC ÉÚ ×ÅÒÛÉÎ A É C ÏÐÕÝÅÎÙ ×ÙÓÏÔÙ AP É
CQ ÎÁ ÓÔÏÒÏÎÙ BC É AB. éÚ×ÅÓÔÎÏ, ÞÔÏ
p ÐÌÏÝÁÄØ ÔÒÅÕÇÏÌØÎÉËÁ ABC ÒÁ×ÎÁ 18, ÐÌÏÝÁÄØ
ÔÒÅÕÇÏÌØÎÉËÁ BPQ ÒÁ×ÎÁ 2, Á PQ = 2 2. îÁÊÄÉÔÅ ÒÁÄÉÕÓ ÏËÒÕÖÎÏÓÔÉ, ÏÐÉÓÁÎÎÏÊ ÏËÏÌÏ
ÔÒÅÕÇÏÌØÎÉËÁ ABC.
ïÔÒÅÚËÉ AB É CD | ÄÉÁÍÅÔÒÙ ÏÄÎÏÊ ÏËÒÕÖÎÏÓÔÉ. éÚ ÔÏÞËÉ M ÜÔÏÊ ÏËÒÕÖÎÏÓÔÉ
ÏÐÕÝÅÎÙ ÐÅÒÐÅÎÄÉËÕÌÑÒÙ MP É MQ ÎÁ ÐÒÑÍÙÅ AB É CD. äÏËÁÖÉÔÅ, ÞÔÏ ÄÌÉÎÁ ÏÔÒÅÚËÁ
PQ ÎÅ ÚÁ×ÉÓÉÔ ÏÔ ÐÏÌÏÖÅÎÉÑ ÔÏÞËÉ M.
137.24.

éÚ ÔÏÞËÉ

137.25.

137.26.

137.27.

137.28.

ðÏÓÔÒÏÊÔÅ ÔÒÅÕÇÏÌØÎÉË ÐÏ ÕÇÌÕ É ÒÁÄÉÕÓÁÍ ×ÐÉÓÁÎÎÏÊ É ÏÐÉÓÁÎÎÏÊ ÏËÒÕÖÎÏÓÔÅÊ.

137.29.

þÅÒÅÚ ×ÅÒÛÉÎÙ

A É B ÔÒÅÕÇÏÌØÎÉËÁ ABC ÐÒÏÈÏÄÉÔ ÏËÒÕÖÎÏÓÔØ ÒÁÄÉÕÓÁ r, ÐÅBC × ÔÏÞËÅ D. îÁÊÄÉÔÅ ÒÁÄÉÕÓ ÏËÒÕÖÎÏÓÔÉ, ÐÒÏÈÏÄÑÝÅÊ ÞÅÒÅÚ ÔÏÞËÉ
A, D É C, ÅÓÌÉ AB = c É AC = b.

ÒÅÓÅËÁÀÝÁÑ ÓÔÏÒÏÎÕ
137.30.

õÇÏÌ ÐÒÉ ÏÓÎÏ×ÁÎÉÉ ÒÁ×ÎÏÂÅÄÒÅÎÎÏÇÏ ÔÒÅÕÇÏÌØÎÉËÁ ÒÁ×ÅÎ

. îÁÊÄÉÔÅ ÏÔÎÏÛÅ-

ÎÉÅ ÒÁÄÉÕÓÁ ×ÐÉÓÁÎÎÏÊ × ÄÁÎÎÙÊ ÔÒÅÕÇÏÌØÎÉË ÏËÒÕÖÎÏÓÔÉ Ë ÒÁÄÉÕÓÕ ÏÐÉÓÁÎÎÏÊ ÏËÏÌÏ ÎÅÇÏ
ÏËÒÕÖÎÏÓÔÉ.
137.31.

òÁÄÉÕÓ ÏËÒÕÖÎÏÓÔÉ, ÏÐÉÓÁÎÎÏÊ ÏËÏÌÏ ÏÓÔÒÏÕÇÏÌØÎÏÇÏ ÔÒÅÕÇÏÌØÎÉËÁ

ABC, ÒÁ×ÅÎ

1. éÚ×ÅÓÔÎÏ, ÞÔÏ ÎÁ ÜÔÏÊ ÏËÒÕÖÎÏÓÔÉ ÌÅÖÉÔ ÃÅÎÔÒ ÄÒÕÇÏÊ ÏËÒÕÖÎÏÓÔÉ, ÐÒÏÈÏÄÑÝÅÊ ÞÅÒÅÚ
×ÅÒÛÉÎÙ A, C É ÔÏÞËÕ ÐÅÒÅÓÅÞÅÎÉÑ ×ÙÓÏÔ ÔÒÅÕÇÏÌØÎÉËÁ ABC. îÁÊÄÉÔÅ AC. p
äÁÎ ÔÒÅÕÇÏÌØÎÉË ABC, × ËÏÔÏÒÏÍ \BAC = 75, AB = 1, AC =
6. îÁ ÓÔÏ137.32.

226

BC ×ÙÂÒÁÎÁ ÔÏÞËÁ M ÔÁË, ÞÔÏ \BAM = 30. ðÒÑÍÁÑ AM ÐÅÒÅÓÅËÁÅÔ ÏËÒÕÖÎÏÓÔØ,
ÏÐÉÓÁÎÎÕÀ ÏËÏÌÏ ÔÒÅÕÇÏÌØÎÉËÁ ABC × ÔÏÞËÅ N, ÏÔÌÉÞÎÏÊ ÏÔ A. îÁÊÄÉÔÅ AN.
äÁÎÙ ÏÔÒÅÚÏË AB É ÎÁ ÎÅÍ ÔÏÞËÁ C. îÁÊÄÉÔÅ ÇÅÏÍÅÔÒÉÞÅÓËÏÅ ÍÅÓÔÏ ÔÏÞÅË ÐÅÒÅÓÅÞÅÎÉÑ Ä×ÕÈ ÒÁ×ÎÙÈ ÏËÒÕÖÎÏÓÔÅÊ, ÏÄÎÁ ÉÚ ËÏÔÏÒÙÈ ÐÒÏÈÏÄÉÔ ÞÅÒÅÚ ÔÏÞËÉ A É C, ÄÒÕÇÁÑ
| ÞÅÒÅÚ ÔÏÞËÉ C É B.
ðÒÏÄÏÌÖÅÎÉÑ ×ÙÓÏÔ AM É CN ÏÓÔÒÏÕÇÏÌØÎÏÇÏ ÔÒÅÕÇÏÌØÎÉËÁ ABC ÐÅÒÅÓÅËÁÀÔ
ÏÐÉÓÁÎÎÕÀ ÏËÏÌÏ ÎÅÇÏ ÏËÒÕÖÎÏÓÔØ × ÔÏÞËÁÈ P É Q. îÁÊÄÉÔÅ ÒÁÄÉÕÓ ÏÐÉÓÁÎÎÏÊ ÏËÒÕÖÎÏÓÔÉ,
ÅÓÌÉ AC = a, PQ = 6a=5.
ïÔÒÅÚËÉ, ÓÏÅÄÉÎÑÀÝÉÅ ÏÓÎÏ×ÁÎÉÑ ×ÙÓÏÔ ÏÓÔÒÏÕÇÏÌØÎÏÇÏ ÔÒÅÕÇÏÌØÎÉËÁ, ÒÁ×ÎÙ 8,
15 É 17. îÁÊÄÉÔÅ ÒÁÄÉÕÓ ÏÐÉÓÁÎÎÏÊ ÏËÏÌÏ ÔÒÅÕÇÏÌØÎÉËÁ ÏËÒÕÖÎÏÓÔÉ.
ä×Å ÏËÒÕÖÎÏÓÔÉ ÒÁÄÉÕÓÏ× R É r ÐÅÒÅÓÅËÁÀÔÓÑ × ÔÏÞËÁÈ A É B É ËÁÓÁÀÔÓÑ ÐÒÑÍÏÊ
× ÔÏÞËÁÈ C É D. N | ÔÏÞËÁ ÐÅÒÅÓÅÞÅÎÉÑ ÐÒÑÍÙÈ AB É CD (B ÍÅÖÄÕ A É N). îÁÊÄÉÔÅ:
1) ÒÁÄÉÕÓ ÏËÒÕÖÎÏÓÔÉ, ÏÐÉÓÁÎÎÏÊ ÏËÏÌÏ ÔÒÅÕÇÏÌØÎÉËÁ ACD;
2) ÏÔÎÏÛÅÎÉÅ ×ÙÓÏÔ ÔÒÅÕÇÏÌØÎÉËÏ× NAC É NAD, ÏÐÕÝÅÎÎÙÈ ÉÚ ×ÅÒÛÉÎÙ N.
÷ ÔÒÅÕÇÏÌØÎÉË ABC ÐÏÍÅÝÅÎÙ ÔÒÉ ÒÁ×ÎÙÈ ÏËÒÕÖÎÏÓÔÉ, ËÁÖÄÁÑ ÉÚ ËÏÔÏÒÙÈ
ÒÏÎÅ

137.33.

137.34.

137.35.

137.36.

137.37.

ËÁÓÁÅÔÓÑ Ä×ÕÈ ÓÔÏÒÏÎ ÔÒÅÕÇÏÌØÎÉËÁ. ÷ÓÅ ÔÒÉ ÏËÒÕÖÎÏÓÔÉ ÉÍÅÀÔ ÏÄÎÕ ÏÂÝÕÀ ÔÏÞËÕ. îÁÊÄÉÔÅ

ÒÁÄÉÕÓÙ ÜÔÉÈ ÏËÒÕÖÎÏÓÔÅÊ, ÅÓÌÉ ÒÁÄÉÕÓÙ ×ÐÉÓÁÎÎÏÊ É ÏÐÉÓÁÎÎÏÊ ÏËÒÕÖÎÏÓÔÅÊ ÔÒÅÕÇÏÌØÎÉËÁ

ABC ÒÁ×ÎÙ R É r.

p

ABKC ÓÔÏÒÏÎÁ AB = 3, ÄÉÁÇÏÎÁÌØ BC ÒÁ×ÎÁ

1, Á ÕÇÌÙ ABC, BKA É BKC ÒÁ×ÎÙ 120 , 30 É 60 ÓÏÏÔ×ÅÔÓÔ×ÅÎÎÏ. îÁÊÄÉÔÅ ÓÔÏÒÏÎÕ BK.
÷ ÔÒÅÕÇÏÌØÎÉËÅ ABC ÉÚ×ÅÓÔÎÏ, ÞÔÏ AC = b É \ABC = . îÁÊÄÉÔÅ ÒÁÄÉÕÓ
ÏËÒÕÖÎÏÓÔÉ, ÐÒÏÈÏÄÑÝÅÊ ÞÅÒÅÚ ÃÅÎÔÒ ×ÐÉÓÁÎÎÏÇÏ × ÔÒÅÕÇÏÌØÎÉË ABC ËÒÕÇÁ É ×ÅÒÛÉÎÙ A
É C.
137.38.

÷ ×ÙÐÕËÌÏÍ ÞÅÔÙÒÅÈÕÇÏÌØÎÉËÅ

137.39.

* * *

AD

ABCD

137.40. ÷ ×ÙÐÕËÌÏÍ ÞÅÔÙÒÅÈÕÇÏÌØÎÉËÅ
=

2 \ABD \ACD
ABD ACD
,

=

ÕÇÏÌØÎÉËÉ

É

90p
2

ÐÒÏ×ÅÄÅÎÙ ÄÉÁÇÏÎÁÌÉ

AC BD
É

. éÚ×ÅÓÔÎÏ, ÞÔÏ

, É ÒÁÓÓÔÏÑÎÉÅ ÍÅÖÄÕ ÃÅÎÔÒÁÍÉ ÏËÒÕÖÎÏÓÔÅÊ, ×ÐÉÓÁÎÎÙÈ × ÔÒÅ-

=

, ÒÁ×ÎÏ

. îÁÊÄÉÔÅ

BC

.

137.41. ðÏÓÔÒÏÊÔÅ ÔÒÅÕÇÏÌØÎÉË ÐÏ Ä×ÕÍ ÓÔÏÒÏÎÁÍ ÔÁË, ÞÔÏÂÙ ÍÅÄÉÁÎÁ, ÐÒÏ×ÅÄÅÎÎÁÑ Ë ÔÒÅÔØÅÊ

1 2
ABCD

ÓÔÏÒÏÎÅ, ÄÅÌÉÌÁ ÕÇÏÌ ÔÒÅÕÇÏÌØÎÉËÁ × ÏÔÎÏÛÅÎÉÉ
137.42. ÷ ×ÙÐÕËÌÏÍ ÞÅÔÙÒÅÈÕÇÏÌØÎÉËÅ

\BDC 40 \BDA 70
=

,

=

.

:

.

ÉÚ×ÅÓÔÎÙ ÕÇÌÙ:

BK b CK c
=

,

=

BC

× ÔÏÞËÅ

. îÁÊÄÉÔÅ

=

20 \BCA
,

îÁÊÄÉÔÅ ÕÇÏÌ ÍÅÖÄÕ ÄÉÁÇÏÎÁÌÑÍÉ ÜÔÏÇÏ ÞÅÔÙÒÅÈÕÇÏÌØÎÉËÁ.

137.43. îÁ ÏËÒÕÖÎÏÓÔÉ, ÏÐÉÓÁÎÎÏÊ ÏËÏÌÏ ÔÒÅÕÇÏÌØÎÉËÁ
ÓÅËÁÅÔÓÑ Ó ÐÒÑÍÏÊ

\BAC

BL

L

, Á ÐÒÑÍÁÑ

CM

ABC
AB

| Ó ÐÒÑÍÏÊ

.

227

×ÚÑÔÁ ÔÏÞËÁ
× ÔÏÞËÅ

K

M

. ðÒÑÍÁÑ

=

35

,

MA
AL a

. éÚ×ÅÓÔÎÏ, ÞÔÏ

ÐÅÒÅ=

,

138. ðÌÏÝÁÄØ (2)

a, b, c | ÓÔÏÒÏÎÙ ÔÒÅÕÇÏÌØÎÉËÁ; , , | ÐÒÏÔÉ×ÏÌÅÖÁÝÉÅ ÉÍ ÕÇÌÙ; R | ÒÁÄÉÕÓ
ÏÐÉÓÁÎÎÏÊ ÏËÒÕÖÎÏÓÔÉ; r | ÒÁÄÉÕÓ ×ÐÉÓÁÎÎÏÊ ÏËÒÕÖÎÏÓÔÉ; p | ÐÏÌÕÐÅÒÉÍÅÔÒ.
ðÕÓÔØ

p

æÏÒÍÕÌÙ ÐÌÏÝÁÄÉ ÔÒÅÕÇÏÌØÎÉËÁ.

S = (1=2) ab




sin

çÅÒÏÎÁ).

138.1.

,

S = pr, S = abc=(4R), S = p(p - a)(p - b)(p - c) (ÆÏÒÍÕÌÁ

óÒÅÄÉ ×ÓÅÈ ÔÒÅÕÇÏÌØÎÉËÏ× Ó ÚÁÄÁÎÎÙÍÉ ÓÔÏÒÏÎÁÍÉ

ÒÏÇÏ ÎÁÉÂÏÌØÛÁÑ ÐÌÏÝÁÄØ.
138.2.

ëÁÔÅÔÙ ÐÒÑÍÏÕÇÏÌØÎÏÇÏ ÔÒÅÕÇÏÌØÎÉËÁ ÒÁ×ÎÙ

ÎÁ ÇÉÐÏÔÅÎÕÚÕ.

AB É AC ÎÁÊÄÉÔÅ ÔÏÔ, Õ ËÏÔÏ-

15 É 8. îÁÊÄÉÔÅ ×ÙÓÏÔÕ, ÏÐÕÝÅÎÎÕÀ

ABCD ÕÇÏÌ BAD ÒÁ×ÅÎ 60, Á ÓÔÏÒÏÎÁ AB ÒÁ×ÎÁ 3. âÉÓÓÅËÔÒÉÓÁ ÕÇÌÁ A ÐÅÒÅÓÅËÁÅÔ ÓÔÏÒÏÎÕ BC × ÔÏÞËÅ E. îÁÊÄÉÔÅ ÐÌÏÝÁÄØ ÔÒÅÕÇÏÌØÎÉËÁ ABE.
138.3.

÷ ÐÁÒÁÌÌÅÌÏÇÒÁÍÍÅ

138.4.

äÏËÁÖÉÔÅ, ÞÔÏ ÐÌÏÝÁÄØ ×ÙÐÕËÌÏÇÏ ÞÅÔÙÒÅÈÕÇÏÌØÎÉËÁ ÒÁ×ÎÁ ÐÏÌÏ×ÉÎÅ ÐÒÏÉÚ×ÅÄÅ-

ÎÉÑ ÄÉÁÇÏÎÁÌÅÊ ÎÁ ÓÉÎÕÓ ÕÇÌÁ ÍÅÖÄÕ ÎÉÍÉ.
138.5.

äÏËÁÖÉÔÅ, ÞÔÏ ÅÓÌÉ ÄÉÁÇÏÎÁÌÉ ×ÙÐÕËÌÏÇÏ ÞÅÔÙÒÅÈÕÇÏÌØÎÉËÁ ÒÁ×ÎÙ, ÔÏ ÅÇÏ ÐÌÏ-

ÝÁÄØ ÒÁ×ÎÁ ÐÒÏÉÚ×ÅÄÅÎÉÀ ÄÌÉÎ ÏÔÒÅÚËÏ×, ÓÏÅÄÉÎÑÀÝÉÈ ÓÅÒÅÄÉÎÙ ÐÒÏÔÉ×ÏÐÏÌÏÖÎÙÈ
ÓÔÏÒÏÎ.
p
138.6.

îÁÊÄÉÔÅ ÐÌÏÝÁÄØ ÔÒÅÕÇÏÌØÎÉËÁ, ÅÓÌÉ Ä×Å ÅÇÏ ÓÔÏÒÏÎÙ ÒÁ×ÎÙ

ÐÒÏ×ÅÄÅÎÎÁÑ Ë ÔÒÅÔØÅÊ, ÒÁ×ÎÁ
138.7.

ÒÁ×ÎÏÊ

b.

138.8.

\B =

2.

óÔÏÒÏÎÙ ÔÒÅÕÇÏÌØÎÉËÁ ÒÁ×ÎÙ

a, b, b. îÁÊÄÉÔÅ ×ÙÓÏÔÕ, ÐÒÏ×ÅÄÅÎÎÕÀ Ë ÓÔÏÒÏÎÅ,

îÁÊÄÉÔÅ ÐÌÏÝÁÄØ ÔÒÅÕÇÏÌØÎÉËÁ

.

138.9.

÷ ÔÒÅÕÇÏÌØÎÉË ÓÏ ÓÔÏÒÏÎÁÍÉ

1 É 15, Á ÍÅÄÉÁÎÁ,

ABC, ÅÓÌÉ ÉÚ×ÅÓÔÎÏ, ÞÔÏ AB

a É b É ÕÇÌÏÍ

=

a, \A

=

,

ÍÅÖÄÕ ÎÉÍÉ ×ÐÉÓÁÎ ÐÏÌÕËÒÕÇ Ó

ÄÉÁÍÅÔÒÏÍ ÎÁ ÔÒÅÔØÅÊ ÓÔÏÒÏÎÅ. îÁÊÄÉÔÅ ÅÇÏ ÒÁÄÉÕÓ.
138.10.

Á) ÷ ÔÒÅÕÇÏÌØÎÉËÅ

ÂÉÓÓÅËÔÒÉÓÕ

AM.

ABC ÉÚ×ÅÓÔÎÏ, ÞÔÏ AB = 8, AC = 6, \BAC = 60. îÁÊÄÉÔÅ

Â) óÔÏÒÏÎÙ ÔÒÅÕÇÏÌØÎÉËÁ ÒÁ×ÎÙ

a É b, Á ÕÇÏÌ ÍÅÖÄÕ ÎÉÍÉ ÒÁ×ÅÎ

. îÁÊÄÉÔÅ ÂÉÓÓÅËÔÒÉÓÕ,

ÐÒÏ×ÅÄÅÎÎÕÀ ÉÚ ×ÅÒÛÉÎÙ ÜÔÏÇÏ ÕÇÌÁ.
138.11.

îÁÊÄÉÔÅ ÐÌÏÝÁÄØ ÔÒÁÐÅÃÉÉ

18 É 13 É ÂÏËÏ×ÙÍÉ ÓÔÏÒÏÎÁÍÉ 3 É 4;
Â) Ó ÏÓÎÏ×ÁÎÉÑÍÉ 16 É 44 É ÂÏËÏ×ÙÍÉ ÓÔÏÒÏÎÁÍÉ 17 É 25.
÷ ÔÒÅÕÇÏÌØÎÉËÅ ABC ÉÚ×ÅÓÔÎÏ, ÞÔÏ \BAC = , \BCA =
ÐÌÏÝÁÄØ ÔÒÅÕÇÏÌØÎÉËÁ ABC.
Á) Ó ÏÓÎÏ×ÁÎÉÑÍÉ

138.12.

138.13.

,

AB = c. îÁÊÄÉÔÅ

îÁÊÄÉÔÅ ÐÌÏÝÁÄØ ÔÒÁÐÅÃÉÉ

11 É 4 É ÄÉÁÇÏÎÁÌÑÍÉ 9 É 12;
Â) Ó ÏÓÎÏ×ÁÎÉÑÍÉ 6 É 3 É ÄÉÁÇÏÎÁÌÑÍÉ 7 É 8.
Á) Ó ÏÓÎÏ×ÁÎÉÑÍÉ
138.14.

÷ ÒÁ×ÎÏÂÅÄÒÅÎÎÏÊ ÔÒÁÐÅÃÉÉ ÏÓÎÏ×ÁÎÉÑ ÒÁ×ÎÙ

ÐÅÒÐÅÎÄÉËÕÌÑÒÎÙ. îÁÊÄÉÔÅ ÐÌÏÝÁÄØ ÔÒÁÐÅÃÉÉ.
138.15.
138.16.

40 É 24, Á ÅÅ ÄÉÁÇÏÎÁÌÉ ×ÚÁÉÍÎÏ

ABC ÒÁ×ÎÁ S, \BAC = , AC = b. îÁÊÄÉÔÅ BC.
ä×Å ÓÔÏÒÏÎÙ ÔÒÅÕÇÏÌØÎÉËÁ ÒÁ×ÎÙ 2 2 É 3, ÐÌÏÝÁÄØ ÔÒÅÕÇÏÌØÎÉËÁ ÒÁ×ÎÁ 3. îÁÊðÌÏÝÁÄØ ÔÒÅÕÇÏÌØÎÉËÁ

p

ÄÉÔÅ ÔÒÅÔØÀ ÓÔÏÒÏÎÕ.

228

AN É BM ÔÒÅÕÇÏÌØÎÉËÁ ABC ÒÁ×ÎÙ 6 É 9 ÓÏÏÔ×ÅÔÓÔ×ÅÎÎÏ É ÐÅÒÅÓÅËÁÀÔÓÑ × ÔÏÞËÅ K, ÐÒÉÞÅÍ ÕÇÏÌ AKB ÒÁ×ÅÎ 30 . îÁÊÄÉÔÅ ÐÌÏÝÁÄØ ÔÒÅÕÇÏÌØÎÉËÁ ABC.
òÁÓÓÔÏÑÎÉÑ ÏÔ ÔÏÞËÉ M, ÌÅÖÁÝÅÊ ×ÎÕÔÒÉ ÔÒÅÕÇÏÌØÎÉËÁ ABC, ÄÏ ÅÇÏ ÓÔÏÒÏÎ AC
É BC ÓÏÏÔ×ÅÔÓÔ×ÅÎÎÏ ÒÁ×ÎÙ 2 É 4. îÁÊÄÉÔÅ ÒÁÓÓÔÏÑÎÉÅ ÏÔ ÔÏÞËÉ M ÄÏ ÐÒÑÍÏÊ AB, ÅÓÌÉ
AB = 10, BC = 17, AC = 21.
÷ ÔÒÅÕÇÏÌØÎÉË ×ÐÉÓÁÎ ËÒÕÇ ÒÁÄÉÕÓÁ 4. ïÄÎÁ ÉÚ ÓÔÏÒÏÎ ÔÒÅÕÇÏÌØÎÉËÁ ÒÁÚÄÅÌÅÎÁ
ÔÏÞËÏÊ ËÁÓÁÎÉÑ ÎÁ ÞÁÓÔÉ, ÒÁ×ÎÙÅ 6 É 8. îÁÊÄÉÔÅ Ä×Å ÄÒÕÇÉÅ ÓÔÏÒÏÎÙ ÔÒÅÕÇÏÌØÎÉËÁ.
138.17.

íÅÄÉÁÎÙ

138.18.

138.19.

138.20.

÷ÅÒÛÉÎÙ ÔÒÅÕÇÏÌØÎÉËÁ ÓÏÅÄÉÎÅÎÙ Ó ÃÅÎÔÒÏÍ ×ÐÉÓÁÎÎÏÇÏ ËÒÕÇÁ. ðÒÏ×ÅÄÅÎÎÙÍÉ

ÏÔÒÅÚËÁÍÉ ÐÌÏÝÁÄØ ÜÔÏÇÏ ÔÒÅÕÇÏÌØÎÉËÁ ÒÁÚÄÅÌÉÌÁÓØ ÎÁ ÔÒÉ ÞÁÓÔÉ:
ÓÔÏÒÏÎÙ ÔÒÅÕÇÏÌØÎÉËÁ.
138.21.

ïÓÎÏ×ÁÎÉÅ ÒÁ×ÎÏÂÅÄÒÅÎÎÏÇÏ ÔÒÅÕÇÏÌØÎÉËÁ ÒÁ×ÎÏ

ËÏ×ÕÀ ÓÔÏÒÏÎÕ, ÒÁ×ÎÁ
138.22.

h. îÁÊÄÉÔÅ ÐÌÏÝÁÄØ ÔÒÅÕÇÏÌØÎÉËÁ.

a, Á ×ÙÓÏÔÁ, ÏÐÕÝÅÎÎÁÑ ÎÁ ÂÏ-

õÇÌÙ ÔÒÅÕÇÏÌØÎÉËÁ ÒÁ×ÎÙ

,

É

, Á ÐÌÏÝÁÄØ ÒÁ×ÎÁ

õÇÌÙ ÔÒÅÕÇÏÌØÎÉËÁ ÒÁ×ÎÙ

,

É

, Á ÐÌÏÝÁÄØ ÒÁ×ÎÁ

ÕÇÏÌØÎÉËÁ.
138.23.

ÕÇÏÌØÎÉËÁ.

28, 60 É 80. îÁÊÄÉÔÅ

S. îÁÊÄÉÔÅ ×ÙÓÏÔÙ ÔÒÅ-

S. îÁÊÄÉÔÅ ÓÔÏÒÏÎÙ ÔÒÅ-

ôÏÞËÉ B1 É C1 | ÏÓÎÏ×ÁÎÉÑ ×ÙÓÏÔ ÔÒÅÕÇÏÌØÎÉËÁ ABC, ÐÌÏÝÁÄØ ËÏÔÏÒÏÇÏ ÒÁ×ÎÁ
S, Á ÕÇÏÌ BAC ÒÁ×ÅÎ . îÁÊÄÉÔÅ ÐÌÏÝÁÄØ ÔÒÅÕÇÏÌØÎÉËÁ AB1C1.
îÁÊÄÉÔÅ ÐÌÏÝÁÄØ ÔÒÅÕÇÏÌØÎÉËÁ, ÅÓÌÉ Ä×Å ÅÇÏ ÓÔÏÒÏÎÙ ÒÁ×ÎÙ 35 É 14, Á ÂÉÓÓÅËÔÒÉÓÁ ÕÇÌÁ ÍÅÖÄÕ ÎÉÍÉ ÒÁ×ÎÁ 12.
÷ ÏÓÔÒÏÕÇÏÌØÎÏÍ ÔÒÅÕÇÏÌØÎÉËÅ ABC ÉÚ ×ÅÒÛÉÎ A É C ÏÐÕÝÅÎÙ ×ÙÓÏÔÙ AP É
CQ ÎÁ ÓÔÏÒÏÎÙ BC É AB. éÚ×ÅÓÔÎÏ, ÞÔÏ ÐÌÏÝÁÄØ ÔÒÅÕÇÏÌØÎÉËÁ ABC ÒÁ×ÎÁ 18, ÐÌÏÝÁÄØ
ÔÒÅÕÇÏÌØÎÉËÁ BPQ ÒÁ×ÎÁ 2, Á PQ = 2 2. îÁÊÄÉÔÅ ÒÁÄÉÕÓ ÏËÒÕÖÎÏÓÔÉ, ÏÐÉÓÁÎÎÏÊ ÏËÏÌÏ
ÔÒÅÕÇÏÌØÎÉËÁ ABC.
äÁÎ ÔÒÅÕÇÏÌØÎÉË ABC. éÚ ×ÅÒÛÉÎÙ A ÐÒÏ×ÅÄÅÎÁ ÍÅÄÉÁÎÁ AM, Á ÉÚ ×ÅÒÛÉÎÙ B
| ÍÅÄÉÁÎÁ BP. éÚ×ÅÓÔÎÏ, ÞÔÏ ÕÇÏÌ APB ÒÁ×ÅÎ ÕÇÌÕ BMA. ëÏÓÉÎÕÓ ÕÇÌÁ ACB ÒÁ×ÅÎ 0; 8 É
BP = 1. îÁÊÄÉÔÅ ÐÌÏÝÁÄØ ÔÒÅÕÇÏÌØÎÉËÁ ABC.
÷ ÔÒÁÐÅÃÉÉ ABCD ÄÉÁÇÏÎÁÌÉ AC É BD ×ÚÁÉÍÎÏ ÐÅÒÐÅÎÄÉËÕÌÑÒÎÙ, \BAC =
\CDB. ðÒÏÄÏÌÖÅÎÉÑ ÂÏËÏ×ÙÈ ÓÔÏÒÏÎ AB É DC ÐÅÒÅÓÅËÁÀÔÓÑ × ÔÏÞËÅ K, ÏÂÒÁÚÕÑ ÕÇÏÌ AKD,
ÒÁ×ÎÙÊ 30 . îÁÊÄÉÔÅ ÐÌÏÝÁÄØ ÔÒÅÕÇÏÌØÎÉËÁ AKD, ÅÓÌÉ ÐÌÏÝÁÄØ ÔÒÁÐÅÃÉÉ ÒÁ×ÎÁ P.
÷ ÐÁÒÁÌÌÅÌÏÇÒÁÍÍÅ ABCD ÔÏÞËÁ E ÄÅÌÉÔ ÐÏÐÏÌÁÍ ÓÔÏÒÏÎÕ CD, ÂÉÓÓÅËÔÒÉÓÁ
ÕÇÌÁ ABC ÐÅÒÅÓÅËÁÅÔ × ÔÏÞËÅ O ÏÔÒÅÚÏË AE. îÁÊÄÉÔÅ ÐÌÏÝÁÄØ ÞÅÔÙÒÅÈÕÇÏÌØÎÉËÁ OBCE,
ÚÎÁÑ, ÞÔÏ AD = a, DE = b, \ABO = .
äÉÁÇÏÎÁÌÉ ÔÒÁÐÅÃÉÉ ×ÚÁÉÍÎÏ ÐÅÒÐÅÎÄÉËÕÌÑÒÎÙ. ïÄÎÁ ÉÚ ÎÉÈ ÒÁ×ÎÁ 6. ïÔÒÅÚÏË,
ÓÏÅÄÉÎÑÀÝÉÊ ÓÅÒÅÄÉÎÙ ÏÓÎÏ×ÁÎÉÊ, ÒÁ×ÅÎ 4; 5. îÁÊÄÉÔÅ ÐÌÏÝÁÄØ ÔÒÁÐÅÃÉÉ.
ïËÏÌÏ ÏËÒÕÖÎÏÓÔÉ ÒÁÄÉÕÓÁ R ÏÐÉÓÁÎ ÐÁÒÁÌÌÅÌÏÇÒÁÍÍ. ðÌÏÝÁÄØ ÞÅÔÙÒÅÈÕÇÏÌØÎÉËÁ Ó ×ÅÒÛÉÎÁÍÉ × ÔÏÞËÁÈ ËÁÓÁÎÉÑ ÏËÒÕÖÎÏÓÔÉ É ÐÁÒÁÌÌÅÌÏÇÒÁÍÍÁ ÒÁ×ÎÁ S. îÁÊÔÉ ÄÌÉÎÙ
138.24.

138.25.

138.26.

p

138.27.

138.28.

138.29.

138.30.

138.31.

ÓÔÏÒÏÎ ÐÁÒÁÌÌÅÌÏÇÒÁÍÍÁ.

ABC ÉÚ×ÅÓÔÎÏ, ÞÔÏ AB = 6, BC = 4, AC = 8. âÉÓÓÅËÔÒÉÓÁ
ÕÇÌÁ C ÐÅÒÅÓÅËÁÅÔ ÓÔÏÒÏÎÕ AB × ÔÏÞËÅ D. þÅÒÅÚ ÔÏÞËÉ A, D É C ÐÒÏ×ÅÄÅÎÁ ÏËÒÕÖÎÏÓÔØ,
ÐÅÒÅÓÅËÁÀÝÁÑ ÓÔÏÒÏÎÕ BC × ÔÏÞËÅ E. îÁÊÄÉÔÅ ÐÌÏÝÁÄØ ÔÒÅÕÇÏÌØÎÉËÁ ADE.
÷ ÐÁÒÁÌÌÅÌÏÇÒÁÍÍÅ ABCD ÏÓÔÒÙÊ ÕÇÏÌ BAD ÒÁ×ÅÎ . ðÕÓÔØ O1 , O2 , O3 , O4 |
ÃÅÎÔÒÙ ÏËÒÕÖÎÏÓÔÅÊ, ÏÐÉÓÁÎÎÙÈ ÓÏÏÔ×ÅÔÓÔ×ÅÎÎÏ ÏËÏÌÏ ÔÒÅÕÇÏÌØÎÉËÏ× DAB, DAC, DBC,
ABC. îÁÊÄÉÔÅ ÏÔÎÏÛÅÎÉÅ ÐÌÏÝÁÄÉ ÞÅÔÙÒÅÈÕÇÏÌØÎÉËÁ O1O2O3O4 Ë ÐÌÏÝÁÄÉ ÐÁÒÁÌÌÅÌÏ138.32.

÷ ÔÒÅÕÇÏÌØÎÉËÅ

138.33.

229

ÇÒÁÍÍÁ

ABCD.

138.34.

÷ ÞÅÔÙÒÅÈÕÇÏÌØÎÉËÅ

ABCD ÏÓÔÒÙÊ ÕÇÏÌ ÍÅÖÄÕ ÄÉÁÇÏÎÁÌÑÍÉ ÒÁ×ÅÎ

. þÅÒÅÚ

ËÁÖÄÕÀ ×ÅÒÛÉÎÕ ÐÒÏ×ÅÄÅÎÁ ÐÒÑÍÁÑ, ÐÅÒÐÅÎÄÉËÕÌÑÒÎÁÑ ÄÉÁÇÏÎÁÌÉ, ÎÅ ÓÏÄÅÒÖÁÝÅÊ ÜÔÕ ×ÅÒÛÉÎÕ. îÁÊÄÉÔÅ ÏÔÎÏÛÅÎÉÅ ÐÌÏÝÁÄÉ ÞÅÔÙÒÅÈÕÇÏÌØÎÉËÁ, ÏÇÒÁÎÉÞÅÎÎÏÇÏ ÜÔÉÍÉ ÐÒÑÍÙÍÉ, Ë
ÐÌÏÝÁÄÉ ÞÅÔÙÒÅÈÕÇÏÌØÎÉËÁ
138.35.

éÚ ÔÏÞËÉ

ABCD.

P, ÒÁÓÐÏÌÏÖÅÎÎÏÊ ×ÎÕÔÒÉ ÏÓÔÒÏÕÇÏÌØÎÏÇÏ ÔÒÅÕÇÏÌØÎÉËÁ ABC, ÏÐÕÝÅ-

ÎÙ ÐÅÒÐÅÎÄÉËÕÌÑÒÙ ÎÁ ÅÇÏ ÓÔÏÒÏÎÙ. äÌÉÎÙ ÓÔÏÒÏÎ É ÏÐÕÝÅÎÎÙÈ ÎÁ ÎÉÈ ÐÅÒÐÅÎÄÉËÕÌÑÒÏ×
ÓÏÏÔ×ÅÔÓÔ×ÅÎÎÏ ÒÁ×ÎÙ

a É k, b É m, c É n. îÁÊÄÉÔÅ ÏÔÎÏÛÅÎÉÅ ÐÌÏÝÁÄÉ ÔÒÅÕÇÏÌØÎÉËÁ ABC

Ë ÐÌÏÝÁÄÉ ÔÒÅÕÇÏÌØÎÉËÁ, ×ÅÒÛÉÎÁÍÉ ËÏÔÏÒÏÇÏ ÓÌÕÖÁÔ ÏÓÎÏ×ÁÎÉÑ ÐÅÒÐÅÎÄÉËÕÌÑÒÏ×.
138.36.

ðÅÒÉÍÅÔÒ ×ÙÐÕËÌÏÇÏ ÞÅÔÙÒÅÈÕÇÏÌØÎÉËÁ ÒÁ×ÅÎ

1.

ÐÒÅ×ÏÓÈÏÄÉÔ
138.37.
p

ÈÏÄÉÔ

óÔÏÒÏÎÙ ÔÒÅÕÇÏÌØÎÉËÁ ÎÅ ÐÒÅ×ÏÓÈÏÄÑÔ

3=4.

4. äÏËÁÖÉÔÅ, ÞÔÏ ÅÇÏ ÐÌÏÝÁÄØ ÎÅ

1. äÏËÁÖÉÔÅ, ÞÔÏ ÅÇÏ ÐÌÏÝÁÄØ ÎÅ ÐÒÅ×ÏÓ-

ABC ÏÐÉÓÁÎÁ ÏËÒÕÖÎÏÓÔØ. íÅÄÉÁÎÁ AD ÐÒÏÄÏÌÖÅÎÁ ÄÏ
ÐÅÒÅÓÅÞÅÎÉÑ Ó ÜÔÏÊ ÏËÒÕÖÎÏÓÔØÀ × ÔÏÞËÅ E. éÚ×ÅÓÔÎÏ, ÞÔÏ AB + AD = DE, \BAD = 60,
AE = 6. îÁÊÄÉÔÅ ÐÌÏÝÁÄØ ÔÒÅÕÇÏÌØÎÉËÁ ABC.
138.38.

ïËÏÌÏ ÔÒÅÕÇÏÌØÎÉËÁ

138.39.

÷ÅÒÛÉÎÙ ÒÏÍÂÁ ÒÁÓÐÏÌÏÖÅÎÙ ÎÁ ÓÔÏÒÏÎÁÈ ÐÁÒÁÌÌÅÌÏÇÒÁÍÍÁ, Á ÓÔÏÒÏÎÙ ÒÏÍÂÁ

ÐÁÒÁÌÌÅÌØÎÙ ÄÉÁÇÏÎÁÌÑÍ ÐÁÒÁÌÌÅÌÏÇÒÁÍÍÁ. îÁÊÄÉÔÅ ÏÔÎÏÛÅÎÉÅ ÐÌÏÝÁÄÅÊ ÒÏÍÂÁ É ÐÁÒÁÌ-

k.
äÁÎ ÔÒÅÕÇÏÌØÎÉË ABC. îÁ ÓÔÏÒÏÎÅ BC ×ÚÑÔÁ ÔÏÞËÁ P, Á ÎÁ ÓÔÏÒÏÎÅ AC ×ÚÑÔÁ
ÔÏÞËÁ M ÔÁË, ÞÔÏ \APB = \BMA = 45. ïÔÒÅÚËÉ AP É BM ÐÅÒÅÓÅËÁÀÔÓÑ × ÔÏÞËÅ O.
éÚ×ÅÓÔÎÏ,ÞÔÏ ÐÌÏÝÁÄÉ ÔÒÅÕÇÏÌØÎÉËÏ× BOP É AOM ÒÁ×ÎÙ ÍÅÖÄÕ ÓÏÂÏÊ, BC = 1, BO =
2=2. îÁÊÄÉÔÅ ÐÌÏÝÁÄØ ÔÒÅÕÇÏÌØÎÉËÁ ABC.
ðÕÓÔØ r1 | ÒÁÄÉÕÓ ×ÎÅ×ÐÉÓÁÎÎÏÊ ÏËÒÕÖÎÏÓÔÉ ÔÒÅÕÇÏÌØÎÉËÁ ABC, ËÁÓÁÀÝÅÊÓÑ
ÓÔÏÒÏÎÙ BC = a, p | ÐÏÌÕÐÅÒÉÍÅÔÒ ÔÒÅÕÇÏÌØÎÉËÁ ABC, S | ÅÇÏ ÐÌÏÝÁÄØ. äÏËÁÖÉÔÅ, ÞÔÏ
Á) 1=r = 1=r1 + 1=r2 + 1=r3, ÇÄÅ r | ÒÁÄÉÕÓ ×ÐÉÓÁÎÎÏÊ ÏËÒÕÖÎÏÓÔÉ, Á r1 , r2 É r3 |
ÒÁÄÉÕÓÙ ×ÎÅ×ÐÉÓÁÎÎÙÈ ÏËÒÕÖÎÏÓÔÅÊ ÔÒÅÕÇÏÌØÎÉËÁ ABC; Â) S = r r1 r2 r3.
÷ ÏÓÔÒÏÕÇÏÌØÎÏÍ ÔÒÅÕÇÏÌØÎÉËÅ ABC ÐÒÏ×ÅÄÅÎÙ ×ÙÓÏÔÙ AM É CN, O | ÃÅÎÔÒ
ÏÐÉÓÁÎÎÏÊ ÏËÏÌÏ ABC ÏËÒÕÖÎÏÓÔÉ. éÚ×ÅÓÔÎÏ, ÞÔÏ \ABC = , Á ÐÌÏÝÁÄØ ÞÅÔÙÒÅÈÕÇÏÌØÎÉËÁ
NOMB ÒÁ×ÎÁ S. îÁÊÄÉÔÅ AC.
ä×Å ÏËÒÕÖÎÏÓÔÉ ÐÅÒÅÓÅËÁÀÔÓÑ × ÔÏÞËÁÈ A É K. éÈ ÃÅÎÔÒÙ ÒÁÓÐÏÌÏÖÅÎÙ ÐÏ
ÒÁÚÎÙÅ ÓÔÏÒÏÎÙ ÏÔ ÐÒÑÍÏÊ, ÓÏÄÅÒÖÁÝÅÊ ÏÔÒÅÚÏË AK. ôÏÞËÉ B É C ÌÅÖÁÔ ÎÁ ÒÁÚÎÙÈ ÏËÒÕÖÎÏÓÔÑÈ. ðÒÑÍÁÑ, ÓÏÄÅÒÖÁÝÁÑ ÏÔÒÅÚÏË AB, ËÁÓÁÅÔÓÑ ÏÄÎÏÊ ÏËÒÕÖÎÏÓÔÉ × ÔÏÞËÅ A. ðÒÑÍÁÑ,
ÓÏÄÅÒÖÁÝÁÑ ÏÔÒÅÚÏË AC, ËÁÓÁÅÔÓÑ ÄÒÕÇÏÊ ÏËÒÕÖÎÏÓÔÉ ÔÁËÖÅ × ÔÏÞËÅ A. äÌÉÎÁ ÏÔÒÅÚËÁ BK
ÒÁ×ÎÁ 1, ÄÌÉÎÁ ÏÔÒÅÚËÁ CK ÒÁ×ÎÁ 4, Á ÔÁÎÇÅÎÓ ÕÇÌÁ CAB ÒÁ×ÅÎ 1= 15. îÁÊÄÉÔÅ ÐÌÏÝÁÄØ
ÔÒÅÕÇÏÌØÎÉËÁ ABC.
÷ ÏÓÔÒÏÕÇÏÌØÎÏÍ ÔÒÅÕÇÏÌØÎÉËÅ ABC Ó ÕÇÌÏÍ C, ÒÁ×ÎÙÍ 30, ×ÙÓÏÔÙ ÐÅÒÅÓÅËÁÀÔÓÑ × ÔÏÞËÅ M. îÁÊÄÉÔÅ ÐÌÏÝÁÄØ ÔÒÅÕÇÏÌØÎÉËÁ AMB, ÅÓÌÉ ÒÁÓÓÔÏÑÎÉÅ ÏÔ ÃÅÎÔÒÁ ÏËÒÕÖÎÏÓÔÉ,
ÏÐÉÓÁÎÎÏÊ ÏËÏÌÏ ÔÒÅÕÇÏÌØÎÉËÁ ABC, ÄÏ ÓÔÏÒÏÎ BC É AC ÓÏÏÔ×ÅÔÓÔ×ÅÎÎÏ ÒÁ×ÎÙ 2 É 3=3.
îÁ ÏÔÒÅÚËÅ AB ÌÅÖÁÔ ÔÏÞËÉ C É D, ÐÒÉÞÅÍ ÔÏÞËÁ C | ÍÅÖÄÕ ÔÏÞËÁÍÉ A É D.
ôÏÞËÁ M ×ÚÑÔÁ ÔÁË, ÞÔÏ ÐÒÑÍÙÅ AM É MD ÐÅÒÐÅÎÄÉËÕÌÑÒÎÙ É ÐÒÑÍÙÅ CM É MB ÔÏÖÅ
ÐÅÒÐÅÎÄÉËÕÌÑÒÎÙ. îÁÊÄÉÔÅ ÐÌÏÝÁÄØ ÔÒÅÕÇÏÌØÎÉËÁ AMB, ÅÓÌÉ ÉÚ×ÅÓÔÎÏ, ÞÔÏ ×ÅÌÉÞÉÎÁ ÕÇÌÁ
CMD ÒÁ×ÎÁ , Á ÐÌÏÝÁÄØ ÔÒÅÕÇÏÌØÎÉËÏ× AMD É CMB ÒÁ×ÎÙ S1 É S2 ÓÏÏÔ×ÅÔÓÔ×ÅÎÎÏ.
ÌÅÌÏÇÒÁÍÍÁ, ÅÓÌÉ ÏÔÎÏÛÅÎÉÅ ÄÉÁÇÏÎÁÌÅÊ ÐÁÒÁÌÌÅÌÏÇÒÁÍÍÁ ÒÁ×ÎÏ
138.40.

p

138.41.

p







138.42.

138.43.

p

138.44.

p

p

138.45.

138.46.

(æÏÒÍÕÌÁ âÒÁÈÍÁÇÕÐÔÙ) äÏËÁÖÉÔÅ, ÞÔÏ ÅÓÌÉ ÓÔÏÒÏÎÙ ×ÐÉÓÁÎÎÏÇÏ ÞÅÔÙÒÅÈ-

230

pp

a, b, c É d, ÔÏ ÅÇÏ ÐÌÏÝÁÄØ S ÍÏÖÅÔ ÂÙÔØ ×ÙÞÉÓÌÅÎÁ ÐÏ ÆÏÒÍÕÌÅ:
- a)(p - b)(p - c)(p - d),ÇÄÅ p = (a + b + c + d)=2 | ÐÏÌÕÐÅÒÉÍÅÔÒ ÞÅÔÙÒÅÈ-

ÕÇÏÌØÎÉËÁ ÒÁ×ÎÙ

S=

(

ÕÇÏÌØÎÉËÁ.

138.47.

ïËÒÕÖÎÏÓÔØ, ×ÐÉÓÁÎÎÁÑ × ÔÒÅÕÇÏÌØÎÉË, ÔÏÞËÏÊ ËÁÓÁÎÉÑ ÄÅÌÉÔ ÏÄÎÕ ÉÚ ÓÔÏÒÏÎ ÎÁ

ÏÔÒÅÚËÉ, ÒÁ×ÎÙÅ
ÔÒÅÕÇÏÌØÎÉËÁ.

3 É 4, Á ÐÒÏÔÉ×ÏÌÅÖÁÝÉÊ ÜÔÏÊ ÓÔÏÒÏÎÅ ÕÇÏÌ ÒÁ×ÅÎ 120. îÁÊÄÉÔÅ ÐÌÏÝÁÄØ

ABC ÒÁ×ÎÁ 15 3. õÇÏÌ BAC ÒÁ×ÅÎ 120. õÇÏÌ ABC
ÂÏÌØÛÅ ÕÇÌÁ ACB. òÁÓÓÔÏÑÎÉÅ ÏÔ ×ÅÒÛÉÎÙ A ÄÏ ÃÅÎÔÒÁ ÏËÒÕÖÎÏÓÔÉ, ×ÐÉÓÁÎÎÏÊ × ÔÒÅÕÇÏÌØÎÉË ABC, ÒÁ×ÎÏ 2. îÁÊÄÉÔÅ ÍÅÄÉÁÎÕ ÔÒÅÕÇÏÌØÎÉËÁ ABC, ÐÒÏ×ÅÄÅÎÎÕÀ ÉÚ ×ÅÒÛÉÎÙ B.
÷ ÏËÒÕÖÎÏÓÔØ ÒÁÄÉÕÓÁ 7 ×ÐÉÓÁÎ ÞÅÔÙÒÅÈÕÇÏÌØÎÉË ABCD. éÚ×ÅÓÔÎÏ, ÞÔÏ AB =
BC, ÐÌÏÝÁÄØ ÔÒÅÕÇÏÌØÎÉËÁ BCD × Ä×Á ÒÁÚÁ ÍÅÎØÛÅ ÐÌÏÝÁÄÉ ÔÒÅÕÇÏÌØÎÉËÁ ABD, \ADC =
120. îÁÊÄÉÔÅ ÄÌÉÎÙ ×ÓÅÈ ÓÔÏÒÏÎ ÞÅÔÙÒÅÈÕÇÏÌØÎÉËÁ ABCD.
îÁ ÐÒÑÍÏÊ, ÐÒÏÈÏÄÑÝÅÊ ÞÅÒÅÚ ÃÅÎÔÒ O ÏËÒÕÖÎÏÓÔÉ ÒÁÄÉÕÓÁ 12, ×ÚÑÔÙ ÔÏÞËÉ
A É B ÔÁË, ÞÔÏ OA = 15, AB = 5 É A ÌÅÖÉÔ ÍÅÖÄÕ O É B. éÚ ÔÏÞÅË A É B ÐÒÏ×ÅÄÅÎÙ
ËÁÓÁÔÅÌØÎÙÅ Ë ÏËÒÕÖÎÏÓÔÉ, ÔÏÞËÉ ËÁÓÁÎÉÑ ËÏÔÏÒÙÈ ÌÅÖÁÔ ÐÏ ÏÄÎÕ ÓÔÏÒÏÎÕ ÏÔ ÐÒÑÍÏÊ OB.
îÁÊÄÉÔÅ ÐÌÏÝÁÄØ ÔÒÅÕÇÏÌØÎÉËÁ ABC, ÇÄÅ C | ÔÏÞËÁ ÐÅÒÅÓÅÞÅÎÉÑ ÜÔÉÈ ËÁÓÁÔÅÌØÎÙÈ.
ôÏÞËÉ K, L, M, N É P ÒÁÓÐÏÌÏÖÅÎÙ ÐÏÓÌÅÄÏ×ÁÔÅÌØÎÏ ÎÁ ÏËÒÕÖÎÏÓÔÉ ÒÁÄÉÕÓÁ
2 2. îÁÊÄÉÔÅ ÐÌÏÝÁÄØ ÔÒÅÕÇÏÌØÎÉËÁ KLM, ÅÓÌÉ LM KN, KM NP, MN LP, Á ÕÇÏÌ LOM
ÒÁ×ÅÎ 45, ÇÄÅ O | ÔÏÞËÁ ÐÅÒÅÓÅÞÅÎÉÑ ÈÏÒÄ LN É MP.
÷ ÐÒÑÍÏÕÇÏÌØÎÏÍ ÔÒÅÕÇÏÌØÎÉËÅ ABC Ó ÐÒÑÍÙÍ ÕÇÌÏÍ C, ÕÇÌÏÍ B, ÒÁ×ÎÙÍ 30, É
ËÁÔÅÔÏÍ CA = 1, ÐÒÏ×ÅÄÅÎÁ ÍÅÄÉÁÎÁ CD. ëÒÏÍÅ ÔÏÇÏ, ÉÚ ÔÏÞËÉ D ÐÏÄ ÕÇÌÏÍ 15 Ë ÇÉÐÏÔÅÎÕÚÅ
ÐÒÏ×ÅÄÅÎÁ ÐÒÑÍÁÑ, ÐÅÒÅÓÅËÁÀÝÁÑ ÏÔÒÅÚÏË BC × ÔÏÞËÅ F. îÁÊÄÉÔÅ ÐÌÏÝÁÄØ ÔÒÅÕÇÏÌØÎÉËÁ
CDF.
ïËÒÕÖÎÏÓÔØ ÒÁÄÉÕÓÁ 3 ÐÒÏÈÏÄÉÔ ÞÅÒÅÚ ×ÅÒÛÉÎÕ B, ÓÅÒÅÄÉÎÙ ÓÔÏÒÏÎ AB É BC,
Á ÔÁËÖÅ ËÁÓÁÅÔÓÑ ÓÔÏÒÏÎÙ AC ÔÒÅÕÇÏÌØÎÉËÁ ABC. õÇÏÌ BAC ÏÓÔÒÙÊ, É sin \BAC = 1=3.
îÁÊÄÉÔÅ ÐÌÏÝÁÄØ ÔÒÅÕÇÏÌØÎÉËÁ ABC.
138.48.

p

ðÌÏÝÁÄØ ÔÒÅÕÇÏÌØÎÉËÁ

138.49.

138.50.

p

138.51.

k

k

k

138.52.

138.53.

138.54.

ïÓÔÒÏÕÇÏÌØÎÙÊ ÒÁ×ÎÏÂÅÄÒÅÎÎÙÊ ÔÒÅÕÇÏÌØÎÉË É ÔÒÁÐÅÃÉÑ ×ÐÉÓÁÎÙ × ÏËÒÕÖÎÏÓÔØ.

ïÄÎÏ ÏÓÎÏ×ÁÎÉÅ ÔÒÁÐÅÃÉÉ Ñ×ÌÑÅÔÓÑ ÄÉÁÍÅÔÒÏÍ ÏËÒÕÖÎÏÓÔÉ, Á ÂÏËÏ×ÙÅ ÓÔÏÒÏÎÙ ÐÁÒÁÌÌÅÌØÎÙ
ÂÏËÏ×ÙÍ ÓÔÏÒÏÎÁÍ ÔÒÅÕÇÏÌØÎÉËÁ. äÏËÁÖÉÔÅ, ÞÔÏ ÔÒÁÐÅÃÉÑ É ÔÒÅÕÇÏÌØÎÉË ÒÁ×ÎÏ×ÅÌÉËÉ.
138.55.

ÒÁÓÓÔÏÑÎÉÑ
138.56.
p

1= 3.

138.57.

÷ÎÕÔÒÉ ÐÒÁ×ÉÌØÎÏÇÏ ÔÒÅÕÇÏÌØÎÉËÁ ÉÍÅÅÔÓÑ ÔÏÞËÁ, ÕÄÁÌÅÎÎÁÑ ÏÔ ÅÇÏ ×ÅÒÛÉÎ ÎÁ

5, 6 É 7. îÁÊÄÉÔÅ ÐÌÏÝÁÄØ ÜÔÏÇÏ ÐÒÁ×ÉÌØÎÏÇÏ ÔÒÅÕÇÏÌØÎÉËÁ.
÷ÓÅ ÂÉÓÓÅËÔÒÉÓÙ ÔÒÅÕÇÏÌØÎÉËÁ ÍÅÎØÛÅ 1. äÏËÁÖÉÔÅ, ÞÔÏ ÅÇÏ ÐÌÏÝÁÄØ ÍÅÎØÛÅ
÷ ÔÒÁÐÅÃÉÉ ÏÓÎÏ×ÁÎÉÑ ÒÁ×ÎÙ

ÂÏËÏ×ÙÍÉ ÓÔÏÒÏÎÁÍÉ ÒÁ×ÅÎ
138.58.

a É b, ÄÉÁÇÏÎÁÌÉ ÐÅÒÐÅÎÄÉËÕÌÑÒÎÙ, Á ÕÇÏÌ ÍÅÖÄÕ

. îÁÊÄÉÔÅ ÐÌÏÝÁÄØ ÔÒÁÐÅÃÉÉ.

óÔÏÒÏÎÙ ÞÅÔÙÒÅÈÕÇÏÌØÎÉËÁ ÒÁ×ÎÙ

a, b, c É d. éÚ×ÅÓÔÎÏ, ÞÔÏ × ÜÔÏÔ ÞÅÔÙÒÅÈ-

ÕÇÏÌØÎÉË ÍÏÖÎÏ ×ÐÉÓÁÔØ
ÏËÒÕÖÎÏÓÔØ É ÏËÏÌÏ ÎÅÇÏ ÍÏÖÎÏ ÏÐÉÓÁÔØ ÏËÒÕÖÎÏÓÔØ. äÏËÁÖÉÔÅ,
p

abcd.
a, b, c, d | ÐÏÓÌÅÄÏ×ÁÔÅÌØÎÙÅ ÓÔÏÒÏÎÙ ÞÅÔÙÒÅÈÕÇÏÌØÎÉËÁ. äÏËÁÖÉÔÅ,
ÞÔÏ ÅÓÌÉ S | ÅÇÏ ÐÌÏÝÁÄØ, ÔÏ S 6 (ac + bd)=2, ÐÒÉÞÅÍ ÒÁ×ÅÎÓÔ×Ï ÉÍÅÅÔ ÍÅÓÔÏ ÔÏÌØËÏ ÄÌÑ

ÞÔÏ ÅÇÏ ÐÌÏÝÁÄØ ÒÁ×ÎÁ
138.59.

ðÕÓÔØ

×ÐÉÓÁÎÎÏÇÏ ÞÅÔÙÒÅÈÕÇÏÌØÎÉËÁ, ÄÉÁÇÏÎÁÌÉ ËÏÔÏÒÏÇÏ ×ÚÁÉÍÎÏ ÐÅÒÐÅÎÄÉËÕÌÑÒÎÙ.

ABCDE ÏÔÓÅËÁÅÔ ÏÔ ÎÅÇÏ ÔÒÅÕÇÏÌØÎÉË ÅÄÉÎÉÞÎÏÊ ÐÌÏÝÁÄÉ. ÷ÙÞÉÓÌÉÔÅ ÐÌÏÝÁÄØ ÐÑÔÉÕÇÏÌØÎÉËÁ ABCDE.
÷ ÔÒÅÕÇÏÌØÎÉËÅ ABC ÎÁ ÓÔÏÒÏÎÅ AC ×ÚÑÔÁ ÔÏÞËÁ D. ïËÒÕÖÎÏÓÔÉ, ×ÐÉÓÁÎÎÙÅ ×
138.60.

ëÁÖÄÁÑ ÄÉÁÇÏÎÁÌØ ×ÙÐÕËÌÏÇÏ ÐÑÔÉÕÇÏÌØÎÉËÁ

138.61.

231

ABD É BCD, ËÁÓÁÀÔÓÑ ÓÔÏÒÏÎÙ AC × ÔÏÞËÁÈ M É N ÓÏÏÔ×ÅÔÓÔ×ÅÎÎÏ. éÚ×ÅÓÔÎÏ,
ÞÔÏ AM = 3, MD = 2, DN = 2, NC = 4. îÁÊÄÉÔÅ ÓÔÏÒÏÎÙ ÔÒÅÕÇÏÌØÎÉËÁ ABC.
îÁ ÏÔÒÅÚËÅ AC ×ÚÑÔÁ ÔÏÞËÁ B É ÎÁ ÏÔÒÅÚËÁÈ AB, BC É AC ÐÏÓÔÒÏÅÎÙ ËÁË ÎÁ
ÄÉÁÍÅÔÒÁÈ ÐÏÌÕÏËÒÕÖÎÏÓÔÉ S1, S2 É S3 ÐÏ ÏÄÎÕ ÓÔÏÒÏÎÕ ÏÔ AC. ðÕÓÔØ D | ÔÏÞËÁ ÎÁ S3,
ÐÒÏÅËÃÉÑ ËÏÔÏÒÏÊ ÎÁ AC ÓÏ×ÐÁÄÁÅÔ Ó ÔÏÞËÏÊ B. ïÂÝÁÑ ËÁÓÁÔÅÌØÎÁÑ Ë S1 É S2 ËÁÓÁÅÔÓÑ ÜÔÉÈ
ÐÏÌÕÏËÒÕÖÎÏÓÔÅÊ × ÔÏÞËÁÈ E É F ÓÏÏÔ×ÅÔÓÔ×ÅÎÎÏ.
Á) äÏËÁÖÉÔÅ, ÞÔÏ ÐÒÑÍÁÑ EF ÐÁÒÁÌÌÅÌØÎÁ ËÁÓÁÔÅÌØÎÏÊ Ë S3 , ÐÒÏ×ÅÄÅÎÎÏÊ ÞÅÒÅÚ ÔÏÞËÕ D.
Â) äÏËÁÖÉÔÅ, ÞÔÏ BFDE | ÐÒÑÍÏÕÇÏÌØÎÉË.
ÔÒÅÕÇÏÌØÎÉËÉ
138.62.

×) îÁÊÄÉÔÅ ÒÁÄÉÕÓ ÏËÒÕÖÎÏÓÔÉ, ËÁÓÁÀÝÅÊÓÑ ×ÓÅÈ ÔÒÅÈ ÐÏÌÕÏËÒÕÖÎÏÓÔÅÊ, ÅÓÌÉ ÉÚ×ÅÓÔÎÏ,

ÞÔÏ ÅÅ ÃÅÎÔÒ ÕÄÁÌÅÎ ÏÔ ÐÒÑÍÏÊ
138.63.

AC ÎÁ ÒÁÓÓÔÏÑÎÉÅ a.

äÏËÁÖÉÔÅ, ÞÔÏ ÔÏÞËÁ ÐÅÒÅÓÅÞÅÎÉÑ ÄÉÁÇÏÎÁÌÅÊ ÏÐÉÓÁÎÎÏÇÏ ×ÏËÒÕÇ ÏËÒÕÖÎÏÓÔÉ ÞÅ-

ÔÙÒÅÈÕÇÏÌØÎÉËÁ ÓÏ×ÐÁÄÁÅÔ Ó ÔÏÞËÏÊ ÐÅÒÅÓÅÞÅÎÉÑ ÄÉÁÇÏÎÁÌÅÊ ÞÅÔÙÒÅÈÕÇÏÌØÎÉËÁ, ×ÅÒÛÉÎÁÍÉ
ËÏÔÏÒÏÇÏ ÓÌÕÖÁÔ ÔÏÞËÉ ËÁÓÁÎÉÑ ÓÔÏÒÏÎ ÐÅÒ×ÏÇÏ ÞÅÔÙÒÅÈÕÇÏÌØÎÉËÁ Ó ÏËÒÕÖÎÏÓÔØÀ.

232

139. ðÒÉÍÅÎÅÎÉÅ ÔÒÉÇÏÎÏÍÅÔÒÉÉ

ABC ÕÇÏÌ ABC ÒÁ×ÅÎ

BCA ÒÁ×ÅÎ 2 . ïËÒÕÖÎÏÓÔØ, ÐÒÏÈÏÄÑÝÁÑ ÞÅÒÅÚ ÔÏÞËÉ A, C É ÃÅÎÔÒ ÏÐÉÓÁÎÎÏÊ ÏËÏÌÏ ÔÒÅÕÇÏÌØÎÉËÁ ABC ÏËÒÕÖÎÏÓÔÉ, ÐÅÒÅÓÅËÁÅÔ ÓÔÏÒÏÎÕ AB × ÔÏÞËÅ M. îÁÊÄÉÔÅ ÏÔÎÏÛÅÎÉÅ AM Ë AB.
139.1.

÷ ÔÒÅÕÇÏÌØÎÉËÅ

, ÕÇÏÌ

139.2.

÷ ÒÁ×ÎÏÂÅÄÒÅÎÎÏÊ ÔÒÁÐÅÃÉÉ Ó ÏÓÔÒÙÍ ÕÇÌÏÍ

ÐÒÉ ÏÓÎÏ×ÁÎÉÉ ÏËÒÕÖÎÏÓÔØ, ÐÏ-

ÓÔÒÏÅÎÎÁÑ ÎÁ ÂÏËÏ×ÏÊ ÓÔÏÒÏÎÅ ËÁË ÎÁ ÄÉÁÍÅÔÒÅ, ËÁÓÁÅÔÓÑ ÄÒÕÇÏÊ ÂÏËÏ×ÏÊ ÓÔÏÒÏÎÙ. ÷ ËÁËÏÍ
ÏÔÎÏÛÅÎÉÉ ÏÎÁ ÄÅÌÉÔ ÂÏÌØÛÅÅ ÏÓÎÏ×ÁÎÉÅ ÔÒÁÐÅÃÉÉ?
139.3.

ðÌÏÝÁÄØ ÒÁ×ÎÏÂÅÄÒÅÎÎÏÊ ÔÒÁÐÅÃÉÉ ÒÁ×ÎÁ

20

3. õÇÏÌ ÍÅÖÄÕ ÄÉÁÇÏÎÁÌØÀ É ÏÓÎÏ-

p

 ÂÏÌØÛÅ ÕÇÌÁ ÍÅÖÄÕ ÄÉÁÇÏÎÁÌØÀ É ÂÏËÏ×ÏÊ ÓÔÏÒÏÎÏÊ. îÁÊÄÉÔÅ ÏÓÔÒÙÊ ÕÇÏÌ

×ÁÎÉÅÍ ÎÁ

ÔÒÁÐÅÃÉÉ, ÅÓÌÉ ÅÅ ÄÉÁÇÏÎÁÌØ ÒÁ×ÎÁ

2.

K, L, M, N, P ÒÁÓÐÏÌÏÖÅÎÙ ÐÏÓÌÅÄÏ×ÁÔÅÌØÎÏ ÎÁ ÏËÒÕÖÎÏÓÔÉ ÒÁÄÉÕÓÁ 2 2.
îÁÊÄÉÔÅ ÐÌÏÝÁÄØ ÔÒÅÕÇÏÌØÎÉËÁ KLM, ÅÓÌÉ LM KN, KM NP, MN LP, Á ÕÇÏÌ LOM ÒÁ×ÅÎ
45, ÇÄÅ O | ÔÏÞËÁ ÐÅÒÅÓÅÞÅÎÉÑ ÈÏÒÄ LN É MP.
þÅÒÅÚ ×ÅÒÛÉÎÙ A É B ÔÒÅÕÇÏÌØÎÉËÁ ABC ÐÒÏ×ÅÄÅÎÁ ÏËÒÕÖÎÏÓÔØ, ÐÅÒÅÓÅËÁÀÝÁÑ
ÓÔÏÒÏÎÙ BC É AC × ÔÏÞËÁÈ D É E ÓÏÏÔ×ÅÔÓÔ×ÅÎÎÏ. ðÌÏÝÁÄØ ÔÒÅÕÇÏÌØÎÉËÁ CDE × ÓÅÍØ ÒÁÚ
ÍÅÎØÛÅ ÐÌÏÝÁÄÉ ÞÅÔÙÒÅÈÕÇÏÌØÎÉËÁ ABDE. îÁÊÄÉÔÅ ÈÏÒÄÕ DE É ÒÁÄÉÕÓ ÏËÒÕÖÎÏÓÔÉ, ÅÓÌÉ
AB = 4 É \C = 45.
÷ ÏÓÔÒÏÕÇÏÌØÎÏÍ ÔÒÅÕÇÏÌØÎÉËÅ ABC ÉÚ ÏÓÎÏ×ÁÎÉÑ D ×ÙÓÏÔÙ BD ÏÐÕÝÅÎÙ ÐÅÒÐÅÎÄÉËÕÌÑÒÙ DM É DN ÎÁ ÓÔÏÒÏÎÙ AB É BC. éÚ×ÅÓÔÎÏ, ÞÔÏ MN = a, BD = b. îÁÊÄÉÔÅ
ÕÇÏÌ ABC.
âÉÓÓÅËÔÒÉÓÁ AD ÒÁ×ÎÏÂÅÄÒÅÎÎÏÇÏ ÔÒÅÕÇÏÌØÎÉËÁ ABC (AB = BC) ÄÅÌÉÔ ÓÔÏÒÏÎÕ
BC ÎÁ ÏÔÒÅÚËÉ BD = b É DC = c. îÁÊÄÉÔÅ ÂÉÓÓÅËÔÒÉÓÕ AD.
÷ ÏËÒÕÖÎÏÓÔÉ ÐÒÏ×ÅÄÅÎÙ Ä×Å ÈÏÒÄÙ AB = a É AC = b. äÌÉÎÁ ÄÕÇÉ AC ×Ä×ÏÅ
ÂÏÌØÛÅ ÄÌÉÎÙ ÄÕÇÉ AB. îÁÊÄÉÔÅ ÒÁÄÉÕÓ ÏËÒÕÖÎÏÓÔÉ.
îÁ ÐÒÑÍÏÊ, ÐÒÏÈÏÄÑÝÅÊ ÞÅÒÅÚ ÃÅÎÔÒ O ÏËÒÕÖÎÏÓÔÉ ÒÁÄÉÕÓÁ 12, ×ÚÑÔÙ ÔÏÞËÉ A
É B ÔÁË, ÞÔÏ OA = 15, AB = 5 É A ÌÅÖÉÔ ÍÅÖÄÕ O É B. éÚ ÔÏÞÅË A É B ÐÒÏ×ÅÄÅÎÙ
ËÁÓÁÔÅÌØÎÙÅ Ë ÏËÒÕÖÎÏÓÔÉ, ÔÏÞËÉ ËÁÓÁÎÉÑ ËÏÔÏÒÙÈ ÌÅÖÁÔ ÐÏ ÏÄÎÕ ÓÔÏÒÏÎÕ ÏÔ ÐÒÑÍÏÊ OB.
îÁÊÄÉÔÅ ÐÌÏÝÁÄØ ÔÒÅÕÇÏÌØÎÉËÁ ABC, ÇÄÅ C | ÔÏÞËÁ ÐÅÒÅÓÅÞÅÎÉÑ ÜÔÉÈ ËÁÓÁÔÅÌØÎÙÈ.
139.4.

ôÏÞËÉ

p

k

k

k

139.5.

139.6.

139.7.

139.8.

139.9.

139.10.

âÉÓÓÅËÔÒÉÓÁ ÏÄÎÏÇÏ ÉÚ ÏÓÔÒÙÈ ÕÇÌÏ× ÐÒÑÍÏÕÇÏÌØÎÏÇÏ ÔÒÅÕÇÏÌØÎÉËÁ × ÔÏÞËÅ ÐÅÒÅ-

ÓÅÞÅÎÉÑ
Ó ×ÙÓÏÔÏÊ, ÏÐÕÝÅÎÎÏÊ ÎÁ ÇÉÐÏÔÅÎÕÚÕ, ÄÅÌÉÔÓÑ ÎÁ ÏÔÒÅÚËÉ, ÏÔÎÏÛÅÎÉÅ ËÏÔÏÒÙÈ ÒÁ×ÎÏ
p

1 + 2, ÓÞÉÔÁÑ ÏÔ ×ÅÒÛÉÎÙ. îÁÊÄÉÔÅ ÏÓÔÒÙÅ ÕÇÌÙ ÔÒÅÕÇÏÌØÎÉËÁ.
139.11.

îÁÊÄÉÔÅ ËÏÓÉÎÕÓ ÕÇÌÁ ÐÒÉ ÏÓÎÏ×ÁÎÉÉ ÒÁ×ÎÏÂÅÄÒÅÎÎÏÇÏ ÔÒÅÕÇÏÌØÎÉËÁ, ÅÓÌÉ ÔÏÞËÁ

ÐÅÒÅÓÅÞÅÎÉÑ ÅÇÏ ×ÙÓÏÔ ÌÅÖÉÔ ÎÁ ×ÐÉÓÁÎÎÏÊ × ÔÒÅÕÇÏÌØÎÉË ÏËÒÕÖÎÏÓÔÉ.

ABC ÔÏÞËÁ D ÌÅÖÉÔ ÎÁ ÓÔÏÒÏÎÅ BC, ÐÒÑÍÁÑ AD ÐÅÒÅÓÅËÁÅÔÓÑ
ACB × ÔÏÞËÅ O. éÚ×ÅÓÔÎÏ, ÞÔÏ ÔÏÞËÉ C, D É O ÌÅÖÁÔ ÎÁ ÏËÒÕÖÎÏÓÔÉ,
ÃÅÎÔÒ ËÏÔÏÒÏÊ ÎÁÈÏÄÉÔÓÑ ÎÁ ÓÔÏÒÏÎÅ AC, AC : AB = 3 : 2, Á ×ÅÌÉÞÉÎÁ ÕÇÌÁ DAC × ÔÒÉ ÒÁÚÁ
ÂÏÌØÛÅ ×ÅÌÉÞÉÎÙ ÕÇÌÁ DAB. îÁÊÄÉÔÅ ËÏÓÉÎÕÓ ÕÇÌÁ ACB.
îÁ ÏËÒÕÖÎÏÓÔÉ ÒÁÄÉÕÓÁ 12 Ó ÃÅÎÔÒÏÍ × ÔÏÞËÅ O ÌÅÖÁÔ ÔÏÞËÉ A É B. ðÒÑÍÙÅ
AC É BC ËÁÓÁÀÔÓÑ ÜÔÏÊ ÏËÒÕÖÎÏÓÔÉ. äÒÕÇÁÑ ÏËÒÕÖÎÏÓÔØ Ó ÃÅÎÔÒÏÍ × ÔÏÞËÅ M ×ÐÉÓÁÎÁ ×
ÔÒÅÕÇÏÌØÎÉË ABC É ËÁÓÁÅÔÓÑ ÓÔÏÒÏÎÙ AC × ÔÏÞËÅ K, Á ÓÔÏÒÏÎÙ BC | × ÔÏÞËÅ H. òÁÓÓÔÏÑÎÉÅ
ÏÔ ÔÏÞËÉ M ÄÏ ÐÒÑÍÏÊ KH ÒÁ×ÎÏ 3. îÁÊÄÉÔÅ ×ÅÌÉÞÉÎÕ ÕÇÌÁ AOB.
âÉÓÓÅËÔÒÉÓÁ AE ÕÇÌÁ A ÒÁÓÓÅËÁÅÔ ÞÅÔÙÒÅÈÕÇÏÌØÎÉË ABCD ÎÁ ÒÁ×ÎÏÂÅÄÒÅÎÎÙÊ
139.12.

÷ ÔÒÅÕÇÏÌØÎÉËÅ

Ó ÂÉÓÓÅËÔÒÉÓÏÊ ÕÇÌÁ

139.13.

139.14.

233

ABE (AB = BE) É ÒÏÍÂ AECD. òÁÄÉÕÓ ËÒÕÇÁ, ÏÐÉÓÁÎÎÏÇÏ ÏËÏÌÏ ÔÒÅÕÇÏÌØÎÉËÁ
ECD, × 1; 5 ÒÁÚÁ ÂÏÌØÛÅ ÒÁÄÉÕÓÁ ËÒÕÇÁ, ×ÐÉÓÁÎÎÏÇÏ × ÔÒÅÕÇÏÌØÎÉË ABE. îÁÊÄÉÔÅ ÏÔÎÏÛÅÎÉÅ
ÔÒÅÕÇÏÌØÎÉË

ÐÅÒÉÍÅÔÒÏ× ÜÔÉÈ ÔÒÅÕÇÏÌØÎÉËÏ×.

÷ ÔÒÅÕÇÏÌØÎÉËÅ ABC ÉÚ×ÅÓÔÎÏ, ÞÔÏ AB = 20, AC = 24. éÚ×ÅÓÔÎÏ ÔÁËÖÅ, ÞÔÏ
C, ÃÅÎÔÒ ×ÐÉÓÁÎÎÏÇÏ × ÔÒÅÕÇÏÌØÎÉË ABC ËÒÕÇÁ É ÔÏÞËÁ ÐÅÒÅÓÅÞÅÎÉÑ ÂÉÓÓÅËÔÒÉÓÙ
ÕÇÌÁ A ÓÏ ÓÔÏÒÏÎÏÊ BC ÌÅÖÁÔ ÎÁ ÏËÒÕÖÎÏÓÔÉ, ÃÅÎÔÒ ËÏÔÏÒÏÊ ÌÅÖÉÔ ÎÁ ÓÔÏÒÏÎÅ AC. îÁÊÄÉÔÅ
ÒÁÄÉÕÓ ÏÐÉÓÁÎÎÏÊ ÏËÏÌÏ ÔÒÅÕÇÏÌØÎÉËÁ ABC ÏËÒÕÖÎÏÓÔÉ.
òÁÄÉÕÓ ×ÐÉÓÁÎÎÏÊ × ÔÒÅÕÇÏÌØÎÉË ABC ÏËÒÕÖÎÏÓÔÉ ÒÁ×ÅÎ 4, ÐÒÉÞÅÍ AC = BC.
îÁ ÐÒÑÍÏÊ AB ×ÚÑÔÁ ÔÏÞËÁ D, ÕÄÁÌÅÎÎÁÑ ÏÔ ÐÒÑÍÙÈ AC É BC ÎÁ ÒÁÓÓÔÏÑÎÉÅ 11 É 3 ÓÏÏÔ×ÅÔÓÔ×ÅÎÎÏ. îÁÊÄÉÔÅ ËÏÓÉÎÕÓ ÕÇÌÁ DBC.
139.15.

×ÅÒÛÉÎÁ

139.16.

139.17.

äÁÎÁ ÒÁ×ÎÏÂÅÄÒÅÎÎÁÑ ÔÒÁÐÅÃÉÑ, × ËÏÔÏÒÕÀ ×ÐÉÓÁÎÁ ÏËÒÕÖÎÏÓÔØ É ÏËÏÌÏ ËÏÔÏÒÏÊ

ÏÐÉÓÁÎÁ ÏËÒÕÖÎÏÓÔØ. ïÔÎÏÛÅÎÉÅ ×ÙÓÏÔÙ ÔÒÁÐÅÃÉÉ Ë ÒÁÄÉÕÓÕ ÏÐÉÓÁÎÎÏÊ ÏËÒÕÖÎÏÓÔÉ ÒÁ×ÎÏ

p

2=3. îÁÊÄÉÔÅ ÕÇÌÙ ÔÒÁÐÅÃÉÉ.
139.18.

÷ ÐÒÑÍÏÕÇÏÌØÎÏÍ ÔÒÅÕÇÏÌØÎÉËÅ

ABC

ÉÚ ÔÏÞËÉ

E,

ÒÁÓÐÏÌÏÖÅÎÎÏÊ × ÓÅÒÅÄÉÎÅ

BC, ÏÐÕÝÅÎ ÐÅÒÐÅÎÄÉËÕÌÑÒ EL ÎÁ ÇÉÐÏÔÅÎÕÚÕ AB. îÁÊÄÉÔÅ ÕÇÌÙ ÔÒÅÕÇÏÌØÎÉËÁ ABC,
AE = 10 EL É BC > AC.
òÁ×ÎÏÂÅÄÒÅÎÎÙÅ ÔÒÅÕÇÏÌØÎÉËÉ ABC (AB = BC) É A1 B1C1 (A1 B1 = B1C1 )
ÐÏÄÏÂÎÙ É AC : A1 C1 = 5 :
3. ÷ÅÒÛÉÎÙ A1 É B1 ÒÁÓÐÏÌÏÖÅÎÙ ÓÏÏÔ×ÅÔÓÔ×ÅÎÎÏ ÎÁ ÓÔÏÒÏÎÁÈ AC É BC, Á ×ÅÒÛÉÎÁ C1 | ÎÁ ÐÒÏÄÏÌÖÅÎÉÉ ÓÔÏÒÏÎÙ AB ÚÁ ÔÏÞËÕ B, ÐÒÉÞÅÍ A1 B1
ÐÅÒÐÅÎÄÉËÕÌÑÒÎÏ BC. îÁÊÄÉÔÅ ÕÇÏÌ ABC.
÷ ÐÒÑÍÏÕÇÏÌØÎÏÊ ÔÒÁÐÅÃÉÉ ABCD ÕÇÌÙ A É D ÐÒÑÍÙÅ, ÓÔÏÒÏÎÁ AB ÐÁÒÁÌÌÅÌØÎÁ
ÓÔÏÒÏÎÅ CD; ÄÌÉÎÙ ÓÔÏÒÏÎ ÒÁ×ÎÙ: AB = 1, CD = 4, AD = 5. îÁ ÓÔÏÒÏÎÅ AD ×ÚÑÔÁ ÔÏÞËÁ
M ÔÁË, ÞÔÏ ÕÇÏÌ CMD ×Ä×ÏÅ ÂÏÌØÛÅ ÕÇÌÁ BMA. ÷ ËÁËÏÍ ÏÔÎÏÛÅÎÉÉ ÔÏÞËÁ M ÄÅÌÉÔ ÓÔÏÒÏÎÕ
AD?
õÇÌÙ ÔÒÅÕÇÏÌØÎÉËÁ ABC ÕÄÏ×ÌÅÔ×ÏÒÑÀÔ ÒÁ×ÅÎÓÔ×Õ
2
2
2
cos A + cos B + cos C = 1:
ËÁÔÅÔÁ

p

ÅÓÌÉ



139.19.

p

139.20.

139.21.

îÁÊÄÉÔÅ
ÐÌÏÝÁÄØ
ÜÔÏÇÏ ÔÒÅÕÇÏÌØÎÉËÁ, ÅÓÌÉ ÒÁÄÉÕÓÙ ×ÐÉÓÁÎÎÏÊ É ÏÐÉÓÁÎÎÏÊ ÏËÒÕÖÎÏÓÔÅÊ
p
p

ÒÁ×ÎÙ

3 É 3 2 ÓÏÏÔ×ÅÔÓÔ×ÅÎÎÏ.

ABC ×ÙÓÏÔÁ AD, ÍÅÄÉÁÎÁ BE É ÂÉÓÓÅËÔÒÉÓÁ CF
O. îÁÊÄÉÔÅ \C, ÅÓÌÉ OE = 2 OC.
÷ÙÓÏÔÙ ÒÁ×ÎÏÂÅÄÒÅÎÎÏÇÏ ÏÓÔÒÏÕÇÏÌØÎÏÇÏ ÔÒÅÕÇÏÌØÎÉËÁ, × ËÏÔÏÒÏÍ AB = BC,
ÐÅÒÅÓÅËÁÀÔÓÑ × ÔÏÞËÅ O. îÁÊÄÉÔÅ ÐÌÏÝÁÄØ ÔÒÅÕÇÏÌØÎÉËÁ ABC, ÅÓÌÉ AO = 5, Á ÄÌÉÎÁ
×ÙÓÏÔÙ AD ÒÁ×ÎÁ 8.
ïÔÒÅÚËÉ, ÓÏÅÄÉÎÑÀÝÉÅ ÏÓÎÏ×ÁÎÉÑ ×ÙÓÏÔ ÏÓÔÒÏÕÇÏÌØÎÏÇÏ ÔÒÅÕÇÏÌØÎÉËÁ, ÒÁ×ÎÙ 8,
15 É 17. îÁÊÄÉÔÅ ÐÌÏÝÁÄØ ÔÒÅÕÇÏÌØÎÉËÁ.
îÁ ÏÓÎÏ×ÁÎÉÉ AB ÒÁ×ÎÏÂÅÄÒÅÎÎÏÇÏ ÔÒÅÕÇÏÌØÎÉËÁ ABC ×ÙÂÒÁÎÁ ÔÏÞËÁ D ÔÁË,
ÞÔÏ ÏËÒÕÖÎÏÓÔØ, ×ÐÉÓÁÎÎÁÑ × ÔÒÅÕÇÏÌØÎÉË BCD, ÉÍÅÅÔ ÔÏÔ ÖÅ ÒÁÄÉÕÓ, ÞÔÏ É ÏËÒÕÖÎÏÓÔØ,
ËÁÓÁÀÝÁÑÓÑ ÐÒÏÄÏÌÖÅÎÉÊ ÏÔÒÅÚËÏ× CA É CD É ÏÔÒÅÚËÁ AD (×ÎÅ×ÐÉÓÁÎÎÁÑ ÏËÒÕÖÎÏÓÔØ
ÔÒÅÕÇÏÌØÎÉËÁ ACD). äÏËÁÖÉÔÅ, ÞÔÏ ÜÔÏÔ ÒÁÄÉÕÓ ÒÁ×ÅÎ 1=4 ×ÙÓÏÔÙ ÔÒÅÕÇÏÌØÎÉËÁ, ÏÐÕÝÅÎÎÏÊ
139.22.

÷ ÏÓÔÒÏÕÇÏÌØÎÏÍ ÔÒÅÕÇÏÌØÎÉËÅ

ÐÅÒÅÓÅËÁÀÔÓÑ × ÔÏÞËÅ



139.23.

139.24.

139.25.

ÎÁ ÅÅ ÂÏËÏ×ÕÀ ÓÔÏÒÏÎÕ.
139.26.

äÁÎÙ Ä×Å ÎÅÐÅÒÅÓÅËÁÀÝÉÅÓÑ ÏËÒÕÖÎÏÓÔÉ, Ë ËÏÔÏÒÙÍ ÐÒÏ×ÅÄÅÎÙ Ä×Å ÏÂÝÉÅ ×ÎÅÛ-

ÎÉÅ ËÁÓÁÔÅÌØÎÙÅ. òÁÓÓÍÏÔÒÉÍ ÒÁ×ÎÏÂÅÄÒÅÎÎÙÊ ÔÒÅÕÇÏÌØÎÉË, ÏÓÎÏ×ÁÎÉÅ ËÏÔÏÒÏÇÏ ÌÅÖÉÔ ÎÁ
ÏÄÎÏÊ ËÁÓÁÔÅÌØÎÏÊ, ÐÒÏÔÉ×ÏÐÏÌÏÖÎÁÑ ×ÅÒÛÉÎÁ | ÎÁ ÄÒÕÇÏÊ, Á ËÁÖÄÁÑ ÉÚ ÂÏËÏ×ÙÈ ÓÔÏÒÏÎ
234

ËÁÓÁÅÔÓÑ ÏÄÎÏÊ ÉÚ ÄÁÎÎÙÈ ÏËÒÕÖÎÏÓÔÅÊ. äÏËÁÖÉÔÅ, ÞÔÏ ×ÙÓÏÔÁ ÔÒÅÕÇÏÌØÎÉËÁ ÒÁ×ÎÁ ÓÕÍÍÅ
ÒÁÄÉÕÓÏ× ÏËÒÕÖÎÏÓÔÅÊ.

235

140. ãÅÎÔÒÁÌØÎÁÑ ÓÉÍÍÅÔÒÉÑ

140.1.

äÏËÁÖÉÔÅ, ÞÔÏ ÐÒÉ ÃÅÎÔÒÁÌØÎÏÊ ÓÉÍÍÅÔÒÉÉ ËÁÖÄÙÊ ÌÕÞ ÐÅÒÅÈÏÄÉÔ × ÐÒÏÔÉ×ÏÐÏ-

ÌÏÖÎÏ ÎÁÐÒÁ×ÌÅÎÎÙÊ Ó ÎÉÍ ÌÕÞ.
140.2.

äÏËÁÖÉÔÅ, ÞÔÏ ÞÅÔÙÒÅÈÕÇÏÌØÎÉË, ÉÍÅÀÝÉÊ ÃÅÎÔÒ ÓÉÍÍÅÔÒÉÉ, Ñ×ÌÑÅÔÓÑ ÐÁÒÁÌÌÅ-

ÌÏÇÒÁÍÍÏÍ.
140.3.

îÁ ÐÒÏÔÉ×ÏÐÏÌÏÖÎÙÈ ÓÔÏÒÏÎÁÈ ÐÁÒÁÌÌÅÌÏÇÒÁÍÍÁ ËÁË ÎÁ ÓÔÏÒÏÎÁÈ ÐÏÓÔÒÏÅÎÙ ×ÎÅ

ÐÁÒÁÌÌÅÌÏÇÒÁÍÍÁ Ä×Á Ë×ÁÄÒÁÔÁ. äÏËÁÖÉÔÅ, ÞÔÏ ÐÒÑÍÁÑ, ÓÏÅÄÉÎÑÀÝÁÑ ÉÈ ÃÅÎÔÒÙ, ÐÒÏÈÏÄÉÔ
ÞÅÒÅÚ ÃÅÎÔÒ ÐÁÒÁÌÌÅÌÏÇÒÁÍÍÁ.
äÏËÁÖÉÔÅ, ÞÔÏ ÔÏÞËÉ, ÓÉÍÍÅÔÒÉÞÎÙÅ ÐÒÏÉÚ×ÏÌØÎÏÊ ÔÏÞËÅ ÏÔÎÏÓÉÔÅÌØÎÏ ÓÅÒÅÄÉÎ

140.4.

ÓÔÏÒÏÎ Ë×ÁÄÒÁÔÁ, Ñ×ÌÑÀÔÓÑ ×ÅÒÛÉÎÁÍÉ ÎÅËÏÔÏÒÏÇÏ Ë×ÁÄÒÁÔÁ.

M(x; y) ÐÒÉ ÓÉÍÍÅÔÒÉÉ ÏÔÎÏÓÉÔÅÌØÎÏ Á) ÎÁÞÁÌÁ
A(a; b).
ðÕÓÔØ a É b | ÎÅËÏÔÏÒÙÅ ÞÉÓÌÁ. ëÁÖÄÏÊ ÔÏÞËÅ M(x; y) ËÏÏÒÄÉÎÁÔÎÏÊ ÐÌÏÓËÏÓÔÉ
ÐÏÓÔÁ×ÉÍ × ÓÏÏÔ×ÅÔÓÔ×ÉÅ ÔÏÞËÕ M 0(x 0 ; y 0 ), ÄÌÑ ËÏÔÏÒÏÊ x 0 = 2a - x É y 0 = 2b - y. äÏËÁÖÉÔÅ,
140.5.

îÁÊÄÉÔÅ ËÏÏÒÄÉÎÁÔÙ ÏÂÒÁÚÁ ÔÏÞËÉ

ËÏÏÒÄÉÎÁÔ; Â) ÔÏÞËÉ
140.6.

ÞÔÏ ÜÔÏ ÓÏÏÔ×ÅÔÓÔ×ÉÅ ÅÓÔØ ÃÅÎÔÒÁÌØÎÁÑ ÓÉÍÍÅÔÒÉÑ ÐÌÏÓËÏÓÔÉ. ëÁËÏ×Ù ËÏÏÒÄÉÎÁÔÙ ÃÅÎÔÒÁ
ÓÉÍÍÅÔÒÉÉ?
140.7.

÷ÙÐÕËÌÙÊ ÍÎÏÇÏÕÇÏÌØÎÉË ÉÍÅÅÔ ÃÅÎÔÒ ÓÉÍÍÅÔÒÉÉ. äÏËÁÖÉÔÅ, ÞÔÏ ÓÕÍÍÁ ÅÇÏ ÕÇÌÏ×

ÄÅÌÉÔÓÑ ÎÁ
140.8.

360.

äÁÎ ÕÇÏÌ É ÔÏÞËÁ ×ÎÕÔÒÉ ÎÅÇÏ. ó ÐÏÍÏÝØÀ ÃÅÎÔÒÁÌØÎÏÊ ÓÉÍÍÅÔÒÉÉ ÐÒÏ×ÅÄÉÔÅ

ÞÅÒÅÚ ÄÁÎÎÕÀ ÔÏÞËÕ ÐÒÑÍÕÀ, ÏÔÒÅÚÏË ËÏÔÏÒÏÊ, ÚÁËÌÀÞÅÎÎÙÊ ×ÎÕÔÒÉ ÕÇÌÁ, ÄÅÌÉÌÓÑ ÂÙ ÜÔÏÊ
ÔÏÞËÏÊ ÐÏÐÏÌÁÍ.
140.9.

M ÎÁ ÏÄÎÏÊ ÉÚ ÅÇÏ ÓÔÏÒÏÎ. ðÏÓÔÒÏÊÔÅ ÒÏÍÂ, ÏÄÎÁ
M, Á ÏÓÔÁÌØÎÙÅ ÔÒÉ ×ÅÒÛÉÎÙ ÌÅÖÁÔ ÎÁ ÔÒÅÈ ÄÒÕÇÉÈ ÓÔÏÒÏÎÁÈ

äÁÎ ÐÁÒÁÌÌÅÌÏÇÒÁÍÍ É ÔÏÞËÁ

×ÅÒÛÉÎÁ ËÏÔÏÒÏÇÏ | ÔÏÞËÁ
ÐÁÒÁÌÌÅÌÏÇÒÁÍÍÁ.
140.10.

ðÒÏ×ÅÄÉÔÅ ÞÅÒÅÚ ÏÂÝÕÀ ÔÏÞËÕ

A ÏËÒÕÖÎÏÓÔÅÊ S1 É S2 ÐÒÑÍÕÀ ÔÁË, ÞÔÏÂÙ ÜÔÉ

ÏËÒÕÖÎÏÓÔÉ ×ÙÓÅËÁÌÉ ÎÁ ÎÅÊ ÒÁ×ÎÙÅ ÈÏÒÄÙ.
140.11.

þÅÒÅÚ ÄÁÎÎÕÀ ÔÏÞËÕ ÐÒÏ×ÅÄÉÔÅ ÐÒÑÍÕÀ, ÏÔÒÅÚÏË ËÏÔÏÒÏÊ, ÚÁËÌÀÞÅÎÎÙÊ ÍÅÖÄÕ

Ä×ÕÍÑ ÄÁÎÎÙÍÉ ÏËÒÕÖÎÏÓÔÑÍÉ, ÄÅÌÉÌÓÑ ÂÙ ÜÔÏÊ ÔÏÞËÏÊ ÐÏÐÏÌÁÍ.
140.12.

äÁÎÙ Ä×Å ËÏÎÃÅÎÔÒÉÞÅÓËÉÅ ÏËÒÕÖÎÏÓÔÉ

S1 É S2. ó ÐÏÍÏÝØÀ ÃÉÒËÕÌÑ É ÌÉÎÅÊËÉ

ÐÒÏ×ÅÄÉÔÅ ÐÒÑÍÕÀ, ÎÁ ËÏÔÏÒÏÊ ÜÔÉ ÏËÒÕÖÎÏÓÔÉ ×ÙÓÅËÁÀÔ ÔÒÉ ÒÁ×ÎÙÈ ÏÔÒÅÚËÁ.

ABCD É ÔÏÞËÁ M. þÅÒÅÚ ÔÏÞËÉ A, B, C É D ÐÒÏ×ÅÄÅÎÙ
ÐÒÑÍÙÅ, ÐÁÒÁÌÌÅÌØÎÙÅ ÐÒÑÍÙÍ MC, MD, MA É MB ÓÏÏÔ×ÅÔÓÔ×ÅÎÎÏ. äÏËÁÖÉÔÅ, ÞÔÏ ÏÎÉ
140.13.

äÁÎ ÐÁÒÁÌÌÅÌÏÇÒÁÍÍ

ÐÅÒÅÓÅËÁÀÔÓÑ × ÏÄÎÏÊ ÔÏÞËÅ.
140.14.

ðÒÏÔÉ×ÏÐÏÌÏÖÎÙÅ ÓÔÏÒÏÎÙ ×ÙÐÕËÌÏÇÏ ÛÅÓÔÉÕÇÏÌØÎÉËÁ ÐÏÐÁÒÎÏ ÒÁ×ÎÙ É ÐÁÒÁÌ-

ÌÅÌØÎÙ. äÏËÁÖÉÔÅ, ÞÔÏ ÏÎ ÉÍÅÅÔ ÃÅÎÔÒ ÓÉÍÍÅÔÒÉÉ.
140.15.

ðÒÉ ÓÉÍÍÅÔÒÉÉ ÏÔÎÏÓÉÔÅÌØÎÏ ÔÏÞËÉÐÅÒÅÓÅÞÅÎÉÑ ÍÅÄÉÁÎ ÔÒÅÕÇÏÌØÎÉË

A1B1C1. ôÒÅÕÇÏÌØÎÉËÉ ABC É A1B1C1 ÐÒÉ ÐÅÒÅÓÅÞÅÎÉÉ ÏÂÒÁÚÕÀÔ
KLMNOP. äÏËÁÖÉÔÅ, ÞÔÏ ÄÉÁÇÏÎÁÌÉ KN, LO É MP ÜÔÏÇÏ ÛÅÓÔÉÕÇÏÌØÎÉËÁ

ÒÅÈÏÄÉÔ × ÔÒÅÕÇÏÌØÎÉË
ÛÅÓÔÉÕÇÏÌØÎÉË

ABC ÐÅ-

ÐÅÒÅÓÅËÁÀÔÓÑ × ÏÄÎÏÊ ÔÏÞËÅ É ÎÁÊÄÉÔÅ ÓÔÏÒÏÎÙ ÛÅÓÔÉÕÇÏÌØÎÉËÁ, ÅÓÌÉ ÓÔÏÒÏÎÙ ÔÒÅÕÇÏÌØÎÉËÁ

ABC ÒÁ×ÎÙ a, b É c.
140.16.

äÏËÁÖÉÔÅ, ÞÔÏ ÐÒÏÔÉ×ÏÐÏÌÏÖÎÙÅ ÓÔÏÒÏÎÙ ÛÅÓÔÉÕÇÏÌØÎÉËÁ, ÏÂÒÁÚÏ×ÁÎÎÏÇÏ ÓÔÏÒÏ-

ÎÁÍÉ ÔÒÅÕÇÏÌØÎÉËÁ É ËÁÓÁÔÅÌØÎÙÍÉ Ë ÅÇÏ ×ÐÉÓÁÎÎÏÊ ÏËÒÕÖÎÏÓÔÉ, ÐÁÒÁÌÌÅÌØÎÙÍÉ ÓÔÏÒÏÎÁÍ,

236

ÒÁ×ÎÙ ÍÅÖÄÕ ÓÏÂÏÊ.
140.17.

äÉÁÇÏÎÁÌÉ

AC É BD ÐÁÒÁÌÌÅÌÏÇÒÁÍÍÁ ABCD ÐÅÒÅÓÅËÁÀÔÓÑ × ÔÏÞËÅ O. äÏËÁAOB É COD, ËÁÓÁÀÔÓÑ.

ÖÉÔÅ, ÞÔÏ ÏËÒÕÖÎÏÓÔÉ, ÏÐÉÓÁÎÎÙÅ ÏËÏÌÏ ÔÒÅÕÇÏÌØÎÉËÏ×
140.18.

óÕÝÅÓÔ×ÕÀÔ ÆÉÇÕÒÙ, ÉÍÅÀÝÉÅ ÂÅÓËÏÎÅÞÎÏÅ ÍÎÏÖÅÓÔ×Ï ÃÅÎÔÒÏ× ÓÉÍÍÅÔÒÉÉ (ÎÁ-

ÐÒÉÍÅÒ, ÐÏÌÏÓÁ ÍÅÖÄÕ Ä×ÕÍÑ ÐÁÒÁÌÌÅÌØÎÙÍÉ ÐÒÑÍÙÍÉ). íÏÖÅÔ ÌÉ ÆÉÇÕÒÁ ÉÍÅÔØ ÂÏÌÅÅ ÏÄÎÏÇÏ, ÎÏ ËÏÎÅÞÎÏÅ ÞÉÓÌÏ ÃÅÎÔÒÏ× ÓÉÍÍÅÔÒÉÉ?
140.19.

(ôÅÏÒÅÍÁ íÏÎÖÁ) äÏËÁÖÉÔÅ, ÞÔÏ ÐÒÑÍÙÅ, ÐÒÏ×ÅÄÅÎÎÙÅ ÞÅÒÅÚ ÓÅÒÅÄÉÎÙ ÓÔÏÒÏÎ

×ÐÉÓÁÎÎÏÇÏ ÞÅÔÙÒÅÈÕÇÏÌØÎÉËÁ ÐÅÒÐÅÎÄÉËÕÌÑÒÎÏ ÐÒÏÔÉ×ÏÐÏÌÏÖÎÙÍ ÓÔÏÒÏÎÁÍ, ÐÅÒÅÓÅËÁÀÔÓÑ
× ÏÄÎÏÊ ÔÏÞËÅ.

A É B. þÅÒÅÚ ÔÏÞËÕ A ÐÒÏ×ÅÄÅÎÁ ÐÒÑÍÁÑ,
C, Á ×ÔÏÒÕÀ | × ÔÏÞËÅ D. ðÕÓÔØ M
É N | ÓÅÒÅÄÉÎÙ ÄÕÇ BC É BD, ÎÅ ÓÏÄÅÒÖÁÝÉÈ ÔÏÞËÕ A, Á K | ÓÅÒÅÄÉÎÁ ÏÔÒÅÚËÁ CD.
äÏËÁÖÉÔÅ, ÞÔÏ ÕÇÏÌ MKN ÒÁ×ÅÎ 90. (íÏÖÎÏ ÓÞÉÔÁÔØ, ÞÔÏ ÔÏÞËÉ C É D ÌÅÖÁÔ ÐÏ ÒÁÚÎÙÅ
ÓÔÏÒÏÎÙ ÏÔ ÔÏÞËÉ A).
140.20.

ä×Å ÏËÒÕÖÎÏÓÔÉ ÐÅÒÅÓÅËÁÀÔÓÑ × ÔÏÞËÁÈ

×ÔÏÒÉÞÎÏ ÐÅÒÅÓÅËÁÀÝÁÑ ÐÅÒ×ÕÀ ÏËÒÕÖÎÏÓÔØ × ÔÏÞËÅ

237

141. ïÓÅ×ÁÑ ÓÉÍÍÅÔÒÉÑ

141.1.

äÏËÁÖÉÔÅ, ÞÔÏ ÄÉÁÇÏÎÁÌØ ÒÏÍÂÁ Ñ×ÌÑÅÔÓÑ ÅÇÏ ÏÓØÀ ÓÉÍÍÅÔÒÉÉ.

141.2.

óÕÝÅÓÔ×ÕÅÔ ÌÉ ÆÉÇÕÒÁ, ÎÅ ÉÍÅÀÝÁÑ ÏÓÅÊ ÓÉÍÍÅÔÒÉÉ, ÎÏ ÐÅÒÅÈÏÄÑÝÁÑ × ÓÅÂÑ ÐÒÉ

ÎÅËÏÔÏÒÏÍ ÐÏ×ÏÒÏÔÅ?
141.3.

óÕÝÅÓÔ×ÕÅÔ ÌÉ ÆÉÇÕÒÁ, ÎÅ ÉÍÅÀÝÁÑ ÎÉ ÏÓÅÊ ÓÉÍÍÅÔÒÉÉ, ÎÉ ÃÅÎÔÒÏ× ÓÉÍÍÅÔÒÉÉ,

ÎÏ ÐÅÒÅÈÏÄÑÝÁÑ × ÓÅÂÑ ÐÒÉ ÎÅËÏÔÏÒÏÍ ÐÏ×ÏÒÏÔÅ?
141.4.

îÁÊÄÉÔÅ ËÏÏÒÄÉÎÁÔÙ ÔÏÞËÉ, ÓÉÍÍÅÔÒÉÞÎÏÊ ÔÏÞËÅ

ÏÒÄÉÎÁÔ; Â) ÏÓÉ ÁÂÓÃÉÓÓ; ×) ÐÒÑÍÏÊ

y = -x.

141.5.

M(x; y)

ÏÔÎÏÓÉÔÅÌØÎÏ Á) ÏÓÉ

x = a; Ç) ÐÒÑÍÏÊ y = b; Ä) ÐÒÑÍÏÊ y = x; Å) ÐÒÑÍÏÊ

æÉÇÕÒÁ ÉÍÅÅÔ Ä×Å ÐÅÒÐÅÎÄÉËÕÌÑÒÎÙÅ ÏÓÉ ÓÉÍÍÅÔÒÉÉ. äÏËÁÖÉÔÅ, ÞÔÏ ÏÎÁ ÉÍÅÅÔ

ÃÅÎÔÒ ÓÉÍÍÅÔÒÉÉ.
141.6.

óÕÝÅÓÔ×ÕÅÔ ÌÉ ÆÉÇÕÒÁ, ÉÍÅÀÝÁÑ ÒÏ×ÎÏ Ä×Å ÏÓÉ ÓÉÍÍÅÔÒÉÉ, ÎÏ ÎÅ ÉÍÅÀÝÁÑ ÃÅÎÔÒÁ

ÓÉÍÍÅÔÒÉÉ?
141.7.

þÅÔÙÒÅÈÕÇÏÌØÎÉË ÉÍÅÅÔ ÒÏ×ÎÏ Ä×Å ÏÓÉ ÓÉÍÍÅÔÒÉÉ. ÷ÅÒÎÏ ÌÉ, ÞÔÏ ÏÎ | ÌÉÂÏ

ÐÒÑÍÏÕÇÏÌØÎÉË, ÌÉÂÏ ÒÏÍÂ?
141.8.

íÏÖÅÔ ÌÉ ÐÑÔÉÕÇÏÌØÎÉË ÉÍÅÔØ ÒÏ×ÎÏ Ä×Å ÏÓÉ ÓÉÍÍÅÔÒÉÉ?

141.9.

íÏÖÅÔ ÌÉ ÆÉÇÕÒÁ ÉÍÅÔØ ÃÅÎÔÒ ÓÉÍÍÅÔÒÉÉ É ÒÏ×ÎÏ ÏÄÎÕ ÏÓØ ÓÉÍÍÅÔÒÉÉ?

141.10.

äÏËÁÖÉÔÅ, ÞÔÏ ×ÓÑËÉÊ ×ÙÐÕËÌÙÊ ÞÅÔÙÒÅÈÕÇÏÌØÎÉË Ó ÏÓØÀ ÓÉÍÍÅÔÒÉÉ ÌÉÂÏ ×ÐÉ-

ÓÁÎÎÙÊ, ÌÉÂÏ ÏÐÉÓÁÎÎÙÊ.

A É B ÌÅÖÁÔ ÐÏ ÒÁÚÎÙÅ ÓÔÏÒÏÎÙ ÏÔ ÐÒÑÍÏÊ l. ðÏÓÔÒÏÊÔÅ ÎÁ ÜÔÏÊ ÐÒÑÍÏÊ
ÔÏÞËÕ M ÔÁË, ÞÔÏÂÙ ÐÒÑÍÁÑ l ÄÅÌÉÌÁ ÕÇÏÌ AMB ÐÏÐÏÌÁÍ.
÷ÎÕÔÒÉ ÏÓÔÒÏÇÏ ÕÇÌÁ ÄÁÎÙ ÔÏÞËÉ M É N. ëÁË ÉÚ ÔÏÞËÉ M ÎÁÐÒÁ×ÉÔØ ÌÕÞ Ó×ÅÔÁ,
ÞÔÏÂÙ ÏÎ, ÏÔÒÁÚÉ×ÛÉÓØ ÐÏÓÌÅÄÏ×ÁÔÅÌØÎÏ ÏÔ ÓÔÏÒÏÎ ÕÇÌÁ, ÐÏÐÁÌ × ÔÏÞËÕ N ?
AB - ÄÉÁÍÅÔÒ ÏËÒÕÖÎÏÓÔÉ; C, D, E | ÔÏÞËÉ ÎÁ ÏÄÎÏÊ ÐÏÌÕÏËÒÕÖÎÏÓÔÉ ACDEB.
îÁ ÄÉÁÍÅÔÒÅ AB ×ÚÑÔÙ: ÔÏÞËÁ F ÔÁË, ÞÔÏ \CFA = \DFB, É ÔÏÞËÁ G ÔÁË, ÞÔÏ \DGA =
\EGB. îÁÊÄÉÔÅ \FDG, ÅÓÌÉ ÄÕÇÁ AC ÒÁ×ÎÁ 60, Á ÄÕÇÁ BE ÒÁ×ÎÁ 20.
÷ÎÕÔÒÉ ÏÓÔÒÏÇÏ ÕÇÌÁ ÄÁÎÙ ÔÏÞËÉ M É N. ðÏÓÔÒÏÊÔÅ ÎÁ ÓÔÏÒÏÎÁÈ ÕÇÌÁ ÔÏÞËÉ K
É L ÔÁË, ÞÔÏÂÙ ÐÅÒÉÍÅÔÒ ÞÅÔÙÒÅÈÕÇÏÌØÎÉËÁ MKLN ÂÙÌ ÎÁÉÍÅÎØÛÉÍ.
141.11.

ôÏÞËÉ

141.12.

141.13.

141.14.

141.15.

ðÏÓÔÒÏÊÔÅ ÔÒÅÕÇÏÌØÎÉË ÐÏ ÄÁÎÎÙÍ ÓÅÒÅÄÉÎÁÍ Ä×ÕÈ ÅÇÏ ÓÔÏÒÏÎ É ÐÒÑÍÏÊ, ÎÁ

ËÏÔÏÒÏÊ ÌÅÖÉÔ ÂÉÓÓÅËÔÒÉÓÁ, ÐÒÏ×ÅÄÅÎÎÁÑ Ë ÏÄÎÏÊ ÉÚ ÜÔÉÈ ÓÔÏÒÏÎ.
141.16.

ðÏÓÔÒÏÊÔÅ ÔÒÅÕÇÏÌØÎÉË ÐÏ ÏÓÎÏ×ÁÎÉÑÍ Ä×ÕÈ ÅÇÏ ÂÉÓÓÅËÔÒÉÓ É ÐÒÑÍÏÊ, ÎÁ ËÏÔÏÒÏÊ

ÌÅÖÉÔ ÔÒÅÔØÑ ÂÉÓÓÅËÔÒÉÓÁ.

M É N ÒÁÓÐÏÌÏÖÅÎÙ ÐÏ ÒÁÚÎÙÅ ÓÔÏÒÏÎÙ ÏÔ ÐÒÑÍÏÊ l. ðÏÓÔÒÏÊÔÅ ÎÁ ÐÒÑÍÏÊ
l ÔÁËÕÀ ÔÏÞËÕ K, ÞÔÏÂÙ ÒÁÚÎÏÓÔØ ÏÔÒÅÚËÏ× MK É NK ÂÙÌÁ ÎÁÉÂÏÌØÛÅÊ.
îÁ ÐÌÏÓËÏÓÔÉ ÄÁÎÙ ÐÒÑÍÁÑ l É Ä×Å ÔÏÞËÉ A É B ÐÏ ÏÄÎÕ ÓÔÏÒÏÎÕ ÏÔ ÎÅÅ. îÁ
ÐÒÑÍÏÊ l ×ÙÂÒÁÎÙ ÔÏÞËÁ M, ÓÕÍÍÁ ÒÁÓÓÔÏÑÎÉÊ ÏÔ ËÏÔÏÒÏÊ ÄÏ ÔÏÞÅË A É B ÎÁÉÍÅÎØÛÁÑ, É
ÔÏÞËÁ N, ÄÌÑ ËÏÔÏÒÏÊ ÒÁÓÓÔÏÑÎÉÑ ÏÔ A É B ÒÁ×ÎÙ: AN = BN. äÏËÁÖÉÔÅ, ÞÔÏ ÔÏÞËÉ A, B,
M, N ÌÅÖÁÔ ÎÁ ÏÄÎÏÊ ÏËÒÕÖÎÏÓÔÉ.
ðÏÓÔÒÏÊÔÅ ÞÅÔÙÒÅÈÕÇÏÌØÎÉË ABCD ÐÏ ÞÅÔÙÒÅÍ ÓÔÏÒÏÎÁÍ, ÅÓÌÉ ÉÚ×ÅÓÔÎÏ, ÞÔÏ
ÅÇÏ ÄÉÁÇÏÎÁÌØ AC Ñ×ÌÑÅÔÓÑ ÂÉÓÓÅËÔÒÉÓÏÊ ÕÇÌÁ A.
ðÏÓÔÒÏÊÔÅ ÞÅÔÙÒÅÈÕÇÏÌØÎÉË ABCD ÐÏ Ä×ÕÍ ÓÔÏÒÏÎÁÍ AB É AD É Ä×ÕÍ ÕÇÌÁÍ
B É D, ÅÓÌÉ ÉÚ×ÅÓÔÎÏ, ÞÔÏ × ÎÅÇÏ ÍÏÖÎÏ ×ÐÉÓÁÔØ ÏËÒÕÖÎÏÓÔØ.
141.17.

ôÏÞËÉ

141.18.

141.19.

141.20.

238

141.21.

ðÏÓÔÒÏÊÔÅ ÔÒÅÕÇÏÌØÎÉË, ÅÓÌÉ ÄÁÎÁ ÏÄÎÁ ÅÇÏ ×ÅÒÛÉÎÁ É ÔÒÉ ÐÒÑÍÙÈ, ÎÁ ËÏÔÏÒÙÈ

ÌÅÖÁÔ ÅÇÏ ÂÉÓÓÅËÔÒÉÓÙ.
141.22.

ðÏÓÔÒÏÊÔÅ ÔÒÅÕÇÏÌØÎÉË ÐÏ Ä×ÕÍ ÓÔÏÒÏÎÁÍ É ÒÁÚÎÏÓÔÉ ÕÇÌÏ×, ÐÒÉÌÅÖÁÝÉÈ Ë ÔÒÅ-

ÔØÅÊ.
141.23.

ðÏÓÔÒÏÊÔÅ ÔÒÅÕÇÏÌØÎÉË ÐÏ Ä×ÕÍ ÕÇÌÁÍ É ÒÁÚÎÏÓÔÉ ÐÒÏÔÉ×ÏÌÅÖÁÝÉÈ ÉÍ ÓÔÏÒÏÎ.

141.24.

ðÏÓÔÒÏÊÔÅ ÔÒÅÕÇÏÌØÎÉË ÐÏ ÒÁÚÎÏÓÔÉ Ä×ÕÈ ÓÔÏÒÏÎ, ÕÇÌÕ ÍÅÖÄÕ ÎÉÍÉ É ÓÔÏÒÏÎÅ,

ÐÒÏÔÉ×ÏÌÅÖÁÝÅÊ ÜÔÏÍÕ ÕÇÌÕ.

AD | ÂÉÓÓÅËÔÒÉÓÁ ÕÇÌÁ A × ÔÒÅÕÇÏÌØÎÉËÅ ABC. þÅÒÅÚ ÔÏÞËÕ A ÐÒÏ×ÅÄÅÎÁ ÐÒÑÍÁÑ, ÐÅÒÐÅÎÄÉËÕÌÑÒÎÁÑ Ë AD, É ÉÚ ×ÅÒÛÉÎÙ B ÏÐÕÝÅÎ ÐÅÒÐÅÎÄÉËÕÌÑÒ BB1 ÎÁ ÜÔÕ ÐÒÑÍÕÀ.
äÏËÁÖÉÔÅ, ÞÔÏ ÐÅÒÉÍÅÔÒ ÔÒÅÕÇÏÌØÎÉËÁ BB1C ÂÏÌØÛÅ ÐÅÒÉÍÅÔÒÁ ÔÒÅÕÇÏÌØÎÉËÁ ABC.
óÒÅÄÉ ×ÓÅÈ ÔÒÅÕÇÏÌØÎÉËÏ× ABC Ó ÄÁÎÎÙÍ ÕÇÌÏÍ C É ÓÔÏÒÏÎÏÊ AB ÎÁÊÄÉÔÅ
141.25.

141.26.

ÔÒÅÕÇÏÌØÎÉË Ó ÎÁÉÂÏÌØÛÉÍ ×ÏÚÍÏÖÎÙÍ ÐÅÒÉÍÅÔÒÏÍ.
141.27.

îÁÊÄÉÔÅ ÓÒÅÄÉ ×ÓÅÈ ÔÒÅÕÇÏÌØÎÉËÏ× Ó ÄÁÎÎÙÍ ÏÓÎÏ×ÁÎÉÅÍ É ÄÁÎÎÏÊ ÐÌÏÝÁÄØÀ

ÔÒÅÕÇÏÌØÎÉË ÎÁÉÍÅÎØÛÅÇÏ ÐÅÒÉÍÅÔÒÁ.
141.28.

ðÏÓÔÒÏÊÔÅ ÏÓÔÒÏÕÇÏÌØÎÙÊ ÔÒÅÕÇÏÌØÎÉË ÐÏ ÏÓÎÏ×ÁÎÉÑÍ Ä×ÕÈ ÅÇÏ ×ÙÓÏÔ É ÐÒÑÍÏÊ,

ÓÏÄÅÒÖÁÝÅÊ ÔÒÅÔØÀ ×ÙÓÏÔÕ.
141.29.

ðÏÓÔÒÏÊÔÅ ÔÒÅÕÇÏÌØÎÉË ÐÏ ÃÅÎÔÒÕ ÅÇÏ ÏÐÉÓÁÎÎÏÊ ÏËÒÕÖÎÏÓÔÉ É Ä×ÕÍ ÐÒÑÍÙÍ,

ÎÁ ËÏÔÏÒÙÈ ÌÅÖÁÔ ×ÙÓÏÔÙ.
141.30.

(úÁÄÁÞÁ æÁÎØÑÎÏ) ÷ÐÉÛÉÔÅ × ÄÁÎÎÙÊ ÏÓÔÒÏÕÇÏÌØÎÙÊ ÔÒÅÕÇÏÌØÎÉË

ÕÇÏÌØÎÉË ÎÁÉÍÅÎØÛÅÇÏ ÐÅÒÉÍÅÔÒÁ.
141.31.

ABC

ÔÒÅ-

îÁ ÐÌÏÓËÏÓÔÉ ÚÁÄÁÎÙ Ä×Å ÐÅÒÅÓÅËÁÀÝÉÅÓÑ ÐÒÑÍÙÅ, É ÎÁ ÎÉÈ ÏÔÍÅÞÅÎÏ ÐÏ ÏÄÎÏÊ

D É F). ðÏÓÔÒÏÊÔÅ ÔÒÅÕÇÏÌØÎÉË ABC, Õ ËÏÔÏÒÏÇÏ ÂÉÓÓÅËÔÒÉÓÙ CD É AE ÌÅÖÁÔ ÎÁ
D É F.
÷ÏÓÓÔÁÎÏ×ÉÔÅ ÔÒÅÕÇÏÌØÎÉË, ÅÓÌÉ ÎÁ ÐÌÏÓËÏÓÔÉ ÏÔÍÅÞÅÎÙ ÔÒÉ ÔÏÞËÉ: O | ÃÅÎÔÒ
ÏÐÉÓÁÎÎÏÊ ÏËÒÕÖÎÏÓÔÉ, P | ÔÏÞËÁ ÐÅÒÅÓÅÞÅÎÉÑ ÍÅÄÉÁÎ É H | ÏÓÎÏ×ÁÎÉÅ ÏÄÎÏÊ ÉÚ ×ÙÓÏÔ
ÔÏÞËÅ (

ÄÁÎÎÙÈ ÐÒÑÍÙÈ, Á ÉÈ ÏÓÎÏ×ÁÎÉÑ | ÄÁÎÎÙÅ ÔÏÞËÉ
141.32.

ÜÔÏÇÏ ÔÒÅÕÇÏÌØÎÉËÁ.

îÁÊÄÉÔÅ ÕÇÌÙ ÏÓÔÒÏÕÇÏÌØÎÏÇÏ ÔÒÅÕÇÏÌØÎÉËÁ ABC, ÅÓÌÉ ÉÚ×ÅÓÔÎÏ, ÞÔÏ ÅÇÏ ÂÉÓAD ÒÁ×ÎÁ ÓÔÏÒÏÎÅ AC É ÐÅÒÐÅÎÄÉËÕÌÑÒÎÁ ÏÔÒÅÚËÕ OH, ÇÄÅ O | ÃÅÎÔÒ ÏÐÉÓÁÎÎÏÊ
ÏËÒÕÖÎÏÓÔÉ, H | ÔÏÞËÁ ÐÅÒÅÓÅÞÅÎÉÑ ×ÙÓÏÔ ÔÒÅÕÇÏÌØÎÉËÁ ABC.
äÁÎÙ ÐÒÑÍÁÑ l É ÔÏÞËÉ A É B ÐÏ ÏÄÎÕ ÓÔÏÒÏÎÕ ÏÔ ÎÅÅ. ðÕÓÔØ A1 É B1 | ÐÒÏÅËÃÉÉ
ÜÔÉÈ ÔÏÞÅË ÎÁ ÐÒÑÍÕÀ l. ðÏÓÔÒÏÊÔÅ ÎÁ ÐÒÑÍÏÊ l ÔÁËÕÀ ÔÏÞËÕ M, ÞÔÏÂÙ ÕÇÏÌ AMA1 ÂÙÌ
×Ä×ÏÅ ÍÅÎØÛÅ ÕÇÌÁ BMB1.
îÁ ÐÌÏÓËÏÓÔÉ ÄÁÎ ÔÒÅÕÇÏÌØÎÉË ABC É ÔÏÞËÁ M. éÚ×ÅÓÔÎÏ, ÞÔÏ ÔÏÞËÉ, ÓÉÍÍÅÔÒÉÞÎÙÅ ÔÏÞËÅ M ÏÔÎÏÓÉÔÅÌØÎÏ Ä×ÕÈ ÓÔÏÒÏÎ ÔÒÅÕÇÏÌØÎÉËÁ ABC ÐÏÐÁÄÁÀÔ ÎÁ ÏËÒÕÖÎÏÓÔØ,
ÏÐÉÓÁÎÎÕÀ ÏËÏÌÏ ÔÒÅÕÇÏÌØÎÉËÁ ABC. äÏËÁÖÉÔÅ, ÞÔÏ ÔÏÞËÁ, ÓÉÍÍÅÔÒÉÞÎÁÑ ÔÏÞËÅ M ÏÔÎÏÓÉ141.33.

ÓÅËÔÒÉÓÁ

141.34.

141.35.

ÔÅÌØÎÏ ÔÒÅÔØÅÊ ÓÔÏÒÏÎÙ, ÔÁËÖÅ ÐÏÐÁÄÁÅÔ ÎÁ ÜÔÕ ÏËÒÕÖÎÏÓÔØ.

ABC Ó ÕÇÌÁÍÉ \A = 50, \B = 60, \C = 70 ÄÁÎÁ ÔÏÞËÁ
M ÔÁËÁÑ, ÞÔÏ \AMB = 110, \BMC = 130. îÁÊÄÉÔÅ \MBC.
141.36.

÷ÎÕÔÒÉ ÔÒÅÕÇÏÌØÎÉËÁ

141.37.

ðÏÓÔÒÏÊÔÅ ÒÁ×ÎÏÂÅÄÒÅÎÎÙÊ ÔÒÅÕÇÏÌØÎÉË, ÏÓÎÏ×ÁÎÉÅ ËÏÔÏÒÏÇÏ ÌÅÖÁÌÏ ÂÙ ÎÁ ÏÄ-

ÎÏÊ ÓÔÏÒÏÎÅ ÄÁÎÎÏÇÏ ÏÓÔÒÏÇÏ ÕÇÌÁ, ×ÅÒÛÉÎÁ | ÎÁ ÄÒÕÇÏÊ ÓÔÏÒÏÎÅ ÔÏÇÏ ÖÅ ÕÇÌÁ, Á ÂÏËÏ×ÙÅ
ÓÔÏÒÏÎÙ ÐÒÏÈÏÄÉÌÉ ÂÙ ÞÅÒÅÚ Ä×Å ÄÁÎÎÙÅ ÔÏÞËÉ ×ÎÕÔÒÉ ÜÔÏÇÏ ÕÇÌÁ.

l É ÔÏÞËÉ A É B ÐÏ ÏÄÎÕ ÓÔÏÒÏÎÕ ÏÔ ÎÅÅ. ó ÐÏÍÏÝØÀ ÃÉÒËÕÌÑ É
ÌÉÎÅÊËÉ ÐÏÓÔÒÏÊÔÅ ÎÁ ÐÒÑÍÏÊ l ÔÏÞËÕ X ÔÁËÕÀ, ÞÔÏ AX + BX = a, ÇÄÅ a | ÄÁÎÎÁÑ ×ÅÌÉÞÉÎÁ.
141.38.

äÁÎÁ ÐÒÑÍÁÑ

239

142. ðÏ×ÏÒÏÔ

142.1.

äÏËÁÖÉÔÅ, ÞÔÏ ÔÒÅÕÇÏÌØÎÉË

60
B ÐÅÒÅÈÏÄÉÔ × C.

ËÏÇÄÁ ÐÒÉ ÐÏ×ÏÒÏÔÅ ÎÁ
×ÅÒÛÉÎÁ

142.2.

ABC

Ñ×ÌÑÅÔÓÑ ÐÒÁ×ÉÌØÎÙÍ ÔÏÇÄÁ É ÔÏÌØËÏ ÔÏÇÄÁ,

 (ÌÉÂÏ ÐÏ ÞÁÓÏ×ÏÊ ÓÔÒÅÌËÅ, ÌÉÂÏ ÐÒÏÔÉ×) ÏÔÎÏÓÉÔÅÌØÎÏ ÔÏÞËÉ

A

þÅÒÅÚ ÃÅÎÔÒ Ë×ÁÄÒÁÔÁ ÐÒÏ×ÅÄÅÎÙ Ä×Å ÐÅÒÐÅÎÄÉËÕÌÑÒÎÙÅ ÐÒÑÍÙÅ. äÏËÁÖÉÔÅ, ÞÔÏ

ÉÈ ÔÏÞËÉ ÐÅÒÅÓÅÞÅÎÉÑ ÓÏ ÓÔÏÒÏÎÁÍÉ Ë×ÁÄÒÁÔÁ ÏÂÒÁÚÕÀÔ Ë×ÁÄÒÁÔ.
142.3.

ðÕÓÔØ Ä×Å ÐÒÑÍÙÅ ÐÅÒÅÓÅËÁÀÔÓÑ × ÔÏÞËÅ

ÐÏ×ÏÒÏÔÅ ÎÁ ÕÇÏÌ

O

ÐÏÄ ÕÇÌÏÍ

. äÏËÁÖÉÔÅ, ÞÔÏ ÐÒÉ

(× ÏÄÎÏÍ ÉÚ ÎÁÐÒÁ×ÌÅÎÉÊ) ÏÔÎÏÓÉÔÅÌØÎÏ ÐÒÏÉÚ×ÏÌØÎÏÊ ÔÏÞËÉ, ÏÔÌÉÞÎÏÊ

O, ÏÄÎÁ ÉÚ ÜÔÉÈ ÐÒÑÍÙÈ ÐÅÒÅÊÄÅÔ × ÐÒÑÍÕÀ, ÐÁÒÁÌÌÅÌØÎÕÀ ÄÒÕÇÏÊ.
îÁ ÓÔÏÒÏÎÁÈ BC É CD ÐÁÒÁÌÌÅÌÏÇÒÁÍÍÁ ABCD ÐÏÓÔÒÏÊÔÅ ÔÏÞËÉ M É N ÔÁË,
ÞÔÏÂÙ ÕÇÏÌ ÐÒÉ ×ÅÒÛÉÎÅ A ÒÁ×ÎÏÂÅÄÒÅÎÎÏÇÏ ÔÒÅÕÇÏÌØÎÉËÁ MAN ÉÍÅÌ ÄÁÎÎÕÀ ×ÅÌÉÞÉÎÕ .
ðÕÓÔØ M É N | ÓÅÒÅÄÉÎÙ ÓÔÏÒÏÎ CD É DE ÐÒÁ×ÉÌØÎÏÇÏ ÛÅÓÔÉÕÇÏÌØÎÉËÁ
ABCDEF. îÁÊÄÉÔÅ ×ÅÌÉÞÉÎÕ ÕÇÌÁ ÍÅÖÄÕ ÐÒÑÍÙÍÉ AM É BN.
ûÅÓÔÉÕÇÏÌØÎÉË ABCDEF | ÐÒÁ×ÉÌØÎÙÊ, K É M | ÓÅÒÅÄÉÎÙ ÏÔÒÅÚËÏ× BD É EF.
äÏËÁÖÉÔÅ, ÞÔÏ ÔÒÅÕÇÏÌØÎÉË AMK ÐÒÁ×ÉÌØÎÙÊ.
ðÏÓÔÒÏÊÔÅ ÒÁ×ÎÏÓÔÏÒÏÎÎÉÊ ÔÒÅÕÇÏÌØÎÉË ABC ÔÁË, ÞÔÏÂÙ ÅÇÏ ×ÅÒÛÉÎÙ ÌÅÖÁÌÉ ÎÁ

ÏÔ

142.4.

142.5.

142.6.

142.7.

ÔÒÅÈ ÄÁÎÎÙÈ ÐÁÒÁÌÌÅÌØÎÙÈ ÐÒÑÍÙÈ.
142.8.

ðÏÓÔÒÏÊÔÅ ÒÁ×ÎÏÓÔÏÒÏÎÎÉÊ ÔÒÅÕÇÏÌØÎÉË, ÏÄÎÁ ×ÅÒÛÉÎÁ ËÏÔÏÒÏÇÏ ÌÅÖÁÌÁ ÂÙ ÎÁ

ÄÁÎÎÏÊ ÏËÒÕÖÎÏÓÔÉ, ÄÒÕÇÁÑ | ÎÁ ÄÁÎÎÏÊ ÐÒÑÍÏÊ, Á ÔÒÅÔØÑ | × ÄÁÎÎÏÊ ÔÏÞËÅ.
142.9.

ðÏÓÔÒÏÊÔÅ Ë×ÁÄÒÁÔ ÔÒÉ ×ÅÒÛÉÎÙ ËÏÔÏÒÏÇÏ ÌÅÖÁÌÉ ÂÙ ÎÁ ÔÒÅÈ ÄÁÎÎÙÈ ÐÁÒÁÌÌÅÌØ-

ÎÙÈ ÐÒÑÍÙÈ.
142.10.

ðÏÓÔÒÏÊÔÅ ÒÁ×ÎÏÂÅÄÒÅÎÎÙÊ ÐÒÑÍÏÕÇÏÌØÎÙÊ ÔÒÅÕÇÏÌØÎÉË Ó ×ÅÒÛÉÎÏÊ ÐÒÑÍÏÇÏ

ÕÇÌÁ × ÄÁÎÎÏÊ ÔÏÞËÅ É Ó ×ÅÒÛÉÎÁÍÉ ÏÓÔÒÙÈ ÕÇÌÏ× ÎÁ Ä×ÕÈ ÄÁÎÎÙÈ ÏËÒÕÖÎÏÓÔÑÈ.
142.11.

ôÏÞËÁ

P ÌÅÖÉÔ ×ÎÕÔÒÉ ÒÁ×ÎÏÓÔÏÒÏÎÎÅÇÏ ÔÒÅÕÇÏÌØÎÉËÁ ABC. äÏËÁÖÉÔÅ, ÞÔÏ ÓÕPA, PD É PC.

ÝÅÓÔ×ÕÅÔ ÔÒÅÕÇÏÌØÎÉË ÓÔÏÒÏÎÙ ËÏÔÏÒÏÇÏ ÒÁ×ÎÙ ÏÔÒÅÚËÁÍ
142.12.

÷ÐÉÛÉÔÅ Ë×ÁÄÒÁÔ × ÄÁÎÎÙÊ ÐÁÒÁÌÌÅÌÏÇÒÁÍÍ.

142.13.

îÁ ÏÔÒÅÚËÅ

AE ÐÏ ÏÄÎÕ ÓÔÏÒÏÎÕ ÏÔ ÎÅÇÏ ÐÏÓÔÒÏÅÎÙ ÒÁ×ÎÏÓÔÏÒÏÎÎÉÅ ÔÒÅÕÇÏÌØÎÉËÉ
ABC É CDE; M É P | ÓÅÒÅÄÉÎÙ ÏÔÒÅÚËÏ× AD É BE. äÏËÁÖÉÔÅ, ÞÔÏ ÔÒÅÕÇÏÌØÎÉË CPM

ÒÁ×ÎÏÓÔÏÒÏÎÎÉÊ.

ABCD Ó ÏÓÔÒÙÍ ÕÇÌÏÍ A, ÒÁ×ÎÙÍ 60. ðÒÑÍÁÑ MN ÏÔÓÅËÁÅÔ ÏÔ ÓÔÏÒÏÎ
AB É BC ÏÔÒÅÚËÉ MB É NB, ÓÕÍÍÁ ËÏÔÏÒÙÈ ÒÁ×ÎÁ ÓÔÏÒÏÎÅ ÒÏÍÂÁ. îÁÊÄÉÔÅ ÕÇÌÙ ÔÒÅÕÇÏÌØÎÉËÁ MDN.
îÁ ÄÕÇÅ BC ÏËÒÕÖÎÏÓÔÉ, ÏÐÉÓÁÎÎÏÊ ÏËÏÌÏ ÒÁ×ÎÏÓÔÏÒÏÎÎÅÇÏ ÔÒÅÕÇÏÌØÎÉËÁ ABC,
×ÚÑÔÁ ÐÒÏÉÚ×ÏÌØÎÁÑ ÔÏÞËÁ M. äÏËÁÖÉÔÅ Ó ÐÏÍÏÝØÀ ÐÏ×ÏÒÏÔÁ, ÞÔÏ AM = BM + CM.
ä×Á Ë×ÁÄÒÁÔÁ BCDA É BKMN ÉÍÅÀÔ ÏÂÝÕÀ ×ÅÒÛÉÎÕ B. äÏËÁÖÉÔÅ Ó ÐÏÍÏÝØÀ
ÐÏ×ÏÒÏÔÁ, ÞÔÏ ÍÅÄÉÁÎÁ BE ÔÒÅÕÇÏÌØÎÉËÁ ABK É ×ÙÓÏÔÁ BF ÔÒÅÕÇÏÌØÎÉËÁ CBN ÌÅÖÁÔ ÎÁ
142.14.

äÁÎ ÒÏÍÂ

142.15.

142.16.

ÏÄÎÏÊ ÐÒÑÍÏÊ. (÷ÅÒÛÉÎÙ ÏÂÏÉÈ Ë×ÁÄÒÁÔÏ× ÐÅÒÅÞÉÓÌÅÎÙ ÐÏ ÞÁÓÏ×ÏÊ ÓÔÒÅÌËÅ.)

BC É CD Ë×ÁÄÒÁÔÁ ABCD ×ÚÑÔÙ ÔÏÞËÉ M É K ÓÏÏÔ×ÅÔÓÔ×ÅÎÎÏ,
ÐÒÉÞÅÍ \BAM = \MAK. äÏËÁÖÉÔÅ, ÞÔÏ BM + KD = AK.
äÁÎ ÐÒÁ×ÉÌØÎÙÊ ÔÒÅÕÇÏÌØÎÉË ABC. îÅËÏÔÏÒÁÑ ÐÒÑÍÁÑ, ÐÁÒÁÌÌÅÌØÎÁÑ ÐÒÑÍÏÊ
AC, ÐÅÒÅÓÅËÁÅÔ ÐÒÑÍÙÅ AB É BC × ÔÏÞËÁÈ M É P, ÓÏÏÔ×ÅÔÓÔ×ÅÎÎÏ. ôÏÞËÁ D | ÃÅÎÔÒ
142.17.

îÁ ÓÔÏÒÏÎÁÈ

142.18.

240

ÐÒÁ×ÉÌØÎÏÇÏ ÔÒÅÕÇÏÌØÎÉËÁ
ÕÇÏÌØÎÉËÁ

DEC.

PMB, ÔÏÞËÁ E | ÓÅÒÅÄÉÎÁ ÏÔÒÅÚËÁ AP. ïÐÒÅÄÅÌÉÔÅ ÕÇÌÙ ÔÒÅ-

ABC ×ÎÅÛÎÉÍ ÏÂÒÁÚÏÍ ÐÏÓÔÒÏÅÎÙ ÐÒÁ×ÉÌØÎÙÅ ÔÒÅÕÇÏÌØÎÉËÉ ABC1 , AB1 C É A1 BC. ðÕÓÔØ P É Q | ÓÅÒÅÄÉÎÙ ÏÔÒÅÚËÏ× A1 B1 É A1 C1 . äÏËÁÖÉÔÅ,
ÞÔÏ ÔÒÅÕÇÏÌØÎÉË APQ ÐÒÁ×ÉÌØÎÙÊ.
÷ÎÕÔÒÉ Ë×ÁÄÒÁÔÁ A1 A2 A3 A4 ×ÚÑÔÁ ÔÏÞËÁ P. éÚ ×ÅÒÛÉÎÙ A1 ÏÐÕÝÅÎ ÐÅÒÐÅÎÄÉËÕÌÑÒ ÎÁ A2 P, ÉÚ A2 | ÎÁ A3 P, ÉÚ A3 | ÎÁ A4 P, ÉÚ A4 | ÎÁ A1 P. äÏËÁÖÉÔÅ, ÞÔÏ ×ÓÅ
142.19.

îÁ ÓÔÏÒÏÎÁÈ ÔÒÅÕÇÏÌØÎÉËÁ

142.20.

ÞÅÔÙÒÅ ÐÅÒÐÅÎÄÉËÕÌÑÒÁ (ÉÌÉ ÉÈ ÐÒÏÄÏÌÖÅÎÉÑ) ÐÅÒÅÓÅËÁÅÔÓÑ × ÏÄÎÏÊ ÔÏÞËÅ.

A Ë×ÁÄÒÁÔÁ ABCD ×ÎÕÔÒØ Ë×ÁÄÒÁÔÁ ÐÒÏ×ÅÄÅÎÙ Ä×Á ÌÕÞÁ, ÎÁ ËÏÔÏÒÙÅ
ÏÐÕÝÅÎÙ ÐÅÒÐÅÎÄÉËÕÌÑÒÙ BK, BL, DM, DN ÉÚ ×ÅÒÛÉÎ B É D. äÏËÁÖÉÔÅ, ÞÔÏ ÏÔÒÅÚËÉ KL
É MN ÒÁ×ÎÙ É ÐÅÒÐÅÎÄÉËÕÌÑÒÎÙ ÄÒÕÇ ÄÒÕÇÕ.
142.21.

142.22.

éÚ ×ÅÒÛÉÎÙ

äÁÎÙ Ä×Å ÔÏÞËÉ É ÏËÒÕÖÎÏÓÔØ. ÞÅÒÅÚ ÄÁÎÎÙÅ ÔÏÞËÉ ÐÒÏ×ÅÄÉÔÅ Ä×Å ÓÅËÕÝÉÅ,

ÏÔÒÅÚËÉ ËÏÔÏÒÙÈ ×ÎÕÔÒÉ ÄÁÎÎÏÊ ÏËÒÕÖÎÏÓÔÉ ÂÙÌÉ ÂÙ ÒÁ×ÎÙ É ÐÅÒÅÓÅËÁÌÉÓØ ÂÙ ÐÏÄ ÄÁÎÎÙÍ
ÕÇÌÏÍ

.

142.23.

ABC ÐÏÓÔÒÏÅÎÙ ×ÎÅ ÔÒÅÕÇÏÌØÎÉËÁ ÒÁ×ÎÏÓÔÏÒÏÎÎÉÅ
BCA1, CAB1, ABC1, É ÐÒÏ×ÅÄÅÎÙ ÏÔÒÅÚËÉ AA1, BB1 É CC1. äÏËÁÖÉÔÅ, ÞÔÏ

îÁ ÓÔÏÒÏÎÁÈ ÔÒÅÕÇÏÌØÎÉËÁ

ÔÒÅÕÇÏÌØÎÉËÉ

ÜÔÉ ÏÔÒÅÚËÉ ÒÁ×ÎÙ ÍÅÖÄÕ ÓÏÂÏÊ.

M ÌÅÖÉÔ ×ÎÕÔÒÉ Ë×ÁÄÒÁÔÁ ABCD, Á ÔÏÞËÁ K | ×ÎÅ, ÐÒÉÞÅÍ ÔÒÅÕÇÏÌØÎÉËÉ
AMD É CKD | ÒÁ×ÎÏÓÔÏÒÏÎÎÉÅ. äÏËÁÖÉÔÅ, ÞÔÏ ÔÏÞËÉ B, M É K ÌÅÖÁÔ ÎÁ ÏÄÎÏÊ ÐÒÑÍÏÊ.
ôÏÞËÁ P ÒÁÓÐÏÌÏÖÅÎÁ ×ÎÕÔÒÉ Ë×ÁÄÒÁÔÁ ABCD, ÐÒÉÞÅÍ AP : BP : CP = 1 : 2 : 3.
îÁÊÄÉÔÅ ÕÇÏÌ APB.
142.24.

ôÏÞËÁ

142.25.

142.26.

÷ÏËÒÕÇ Ë×ÁÄÒÁÔÁ ÏÐÉÓÁÎ ÐÁÒÁÌÌÅÌÏÇÒÁÍÍ (×ÅÒÛÉÎÙ Ë×ÁÄÒÁÔÁ ÌÅÖÁÔ ÎÁ ÒÁÚÎÙÈ

ÓÔÏÒÏÎÁÈ ÐÁÒÁÌÌÅÌÏÇÒÁÍÍÁ). äÏËÁÖÉÔÅ, ÞÔÏ ÐÅÒÐÅÎÄÉËÕÌÑÒÙ, ÏÐÕÝÅÎÎÙÅ ÉÈ ×ÅÒÛÉÎ ÐÁÒÁÌÌÅÌÏÇÒÁÍÍÁ ÎÁ ÓÔÏÒÏÎÙ Ë×ÁÄÒÁÔÁ, ÏÂÒÁÚÕÀÔ ÎÏ×ÙÊ Ë×ÁÄÒÁÔ.

äÁÎ ÔÒÅÕÇÏÌØÎÉË ABC. îÁ ÅÇÏ ÓÔÏÒÏÎÁÈ AB É BC ÐÏÓÔÒÏÅÎÙ ×ÎÅÛÎÉÍ ÏÂÒÁÚÏÍ
ABMN É BCPQ. äÏËÁÖÉÔÅ, ÞÔÏ ÃÅÎÔÒÙ ÜÔÉÈ Ë×ÁÄÒÁÔÏ× É ÓÅÒÅÄÉÎÙ ÏÔÒÅÚËÏ× MQ

142.27.

Ë×ÁÄÒÁÔÙ
É

AC ÏÂÒÁÚÕÀÔ Ë×ÁÄÒÁÔ.
142.28.

(úÁÄÁÞÁ

æÅÒÍÁ) ÷ÎÕÔÒÉ ÏÓÔÒÏÕÇÏÌØÎÏÇÏ ÔÒÅÕÇÏÌØÎÉËÁ ÎÁÊÄÉÔÅ ÔÏÞËÕ, ÓÕÍÍÁ ÒÁÓ-

ÓÔÏÑÎÉÊ ÏÔ ËÏÔÏÒÏÊ ÄÏ ×ÅÒÛÉÎ ÍÉÎÉÍÁÌØÎÁ.

241

143. ðÁÒÁÌÌÅÌØÎÙÊ ÐÅÒÅÎÏÓ

143.1.

äÁÎ ÕÇÏÌ

ÓÔÏÒÏÎÙ ÕÇÌÁ

ABC É ÐÒÑÍÁÑ l. ðÁÒÁÌÌÅÌØÎÏ ÐÒÑÍÏÊ l ÐÒÏ×ÅÄÉÔÅ ÐÒÑÍÕÀ, ÎÁ ËÏÔÏÒÏÊ

ABC ×ÙÓÅËÁÀÔ ÏÔÒÅÚÏË ÄÁÎÎÏÊ ÄÌÉÎÙ.

143.2.

ðÏÓÔÒÏÊÔÅ ÈÏÒÄÕ ÄÁÎÎÏÊ ÏËÒÕÖÎÏÓÔÉ, ÒÁ×ÎÕÀ É ÐÁÒÁÌÌÅÌØÎÕÀ ÄÁÎÎÏÍÕ ÏÔÒÅÚËÕ.

143.3.

ðÏÓÔÒÏÊÔÅ ÏÔÒÅÚÏË, ÒÁ×ÎÙÊ É ÐÁÒÁÌÌÅÌØÎÙÊ ÄÁÎÎÏÍÕ, ÔÁË, ÞÔÏÂÙ ÅÇÏ ËÏÎÃÙ ÌÅ-

ÖÁÌÉ ÎÁ ÄÁÎÎÏÊ ÐÒÑÍÏÊ É ÎÁ ÄÁÎÎÏÊ ÏËÒÕÖÎÏÓÔÉ.
143.4.

ðÏÓÔÒÏÊÔÅ ÏÔÒÅÚÏË, ÒÁ×ÎÙÊ É ÐÁÒÁÌÌÅÌØÎÙÊ ÄÁÎÎÏÍÕ, ÔÁË, ÞÔÏÂÙ ÅÇÏ ËÏÎÃÙ ÌÅ-

ÖÁÌÉ ÎÁ Ä×ÕÈ ÄÁÎÎÙÈ ÏËÒÕÖÎÏÓÔÑÈ.

ABCD ×ÚÑÔÁ ÔÏÞËÁ M. äÏËÁÖÉÔÅ, ÞÔÏ ÓÕÝÅÓÔ×ÕÅÔ ×ÙAB É BC, ÓÔÏÒÏÎÙ
ËÏÔÏÒÏÇÏ ÒÁ×ÎÙ AM, BM, CM, DM.
ä×Å ÏËÒÕÖÎÏÓÔÉ ÒÁÄÉÕÓÁ R ËÁÓÁÀÔÓÑ × ÔÏÞËÅ K. îÁ ÏÄÎÏÊ ÉÚ ÎÉÈ ×ÚÑÔÁ ÔÏÞËÁ A,
Á ÎÁ ÄÒÕÇÏÊ | ÔÏÞËÁ B, ÐÒÉÞÅÍ \AKB = 90. äÏËÁÖÉÔÅ, ÞÔÏ AB = 2R.
ä×Å ÏËÒÕÖÎÏÓÔÉ ÒÁÄÉÕÓÁ R ÐÅÒÅÓÅËÁÀÔÓÑ × ÔÏÞËÁÈ M É N. ðÕÓÔØ A É B | ÔÏÞËÉ
ÐÅÒÅÓÅÞÅÎÉÑ ÓÅÒÅÄÉÎÎÏÇÏ ÐÅÒÐÅÎÄÉËÕÌÑÒÁ Ë ÏÔÒÅÚËÕ MN Ó ÜÔÉÍÉ ÏËÒÕÖÎÏÓÔÑÍÉ, ÌÅÖÁÝÉÅ
ÐÏ ÏÄÎÕ ÓÔÏÒÏÎÕ ÏÔ ÐÒÑÍÏÊ MN. äÏËÁÖÉÔÅ, ÞÔÏ MN2 + AB2 = 4R2 .
÷ ËÁËÏÍ ÍÅÓÔÅ ÓÌÅÄÕÅÔ ÐÏÓÔÒÏÉÔØ ÍÏÓÔ MN ÞÅÒÅÚ ÒÅËÕ, ÒÁÚÄÅÌÑÀÝÕÀ Ä×Å ÄÁÎÎÙÅ
ÄÅÒÅ×ÎÉ A É B, ÞÔÏÂÙ ÐÕÔØ AMNB ÉÚ ÄÅÒÅ×ÎÉ A × ÄÅÒÅ×ÎÀ B ÂÙÌ ËÒÁÔÞÁÊÛÉÍ (ÂÅÒÅÇÁ
143.5.

÷ÎÕÔÒÉ ÐÒÑÍÏÕÇÏÌØÎÉËÁ

ÐÕËÌÙÊ ÞÅÔÙÒÅÈÕÇÏÌØÎÉË Ó ÐÅÒÐÅÎÄÉËÕÌÑÒÎÙÍÉ ÄÉÁÇÏÎÁÌÑÍÉ ÄÌÉÎÙ
143.6.

143.7.

143.8.

ÒÅËÉ ÓÞÉÔÁÀÔÓÑ ÐÁÒÁÌÌÅÌØÎÙÍÉ ÐÒÑÍÙÍÉ, ÍÏÓÔ ÐÒÅÄÐÏÌÁÇÁÅÔÓÑ ÐÅÒÐÅÎÄÉËÕÌÑÒÎÙÍ Ë ÒÅËÅ).
143.9.

þÅÒÅÚ ÔÏÞËÕ ÐÅÒÅÓÅÞÅÎÉÑ Ä×ÕÈ ÏËÒÕÖÎÏÓÔÅÊ ÐÒÏ×ÅÄÉÔÅ ÓÅËÕÝÕÀ ÔÁË, ÞÔÏÂÙ ÞÁÓÔØ

ÅÅ, ÚÁËÌÀÞÅÎÎÁÑ ×ÎÕÔÒÉ ÏËÒÕÖÎÏÓÔÅÊ, ÉÍÅÌÁ ÄÁÎÎÕÀ ÄÌÉÎÕ.
143.10.

ðÁÒÁÌÌÅÌØÎÏ ÄÁÎÎÏÊ ÐÒÑÍÏÊ ÐÒÏ×ÅÄÉÔÅ ÐÒÑÍÕÀ, ÎÁ ËÏÔÏÒÏÊ Ä×Å ÄÁÎÎÙÅ ÏËÒÕÖ-

ÎÏÓÔÉ ×ÙÓÅËÁÌÉ ÂÙ ÈÏÒÄÙ ÒÁ×ÎÏÊ ÄÌÉÎÙ.
143.11.

ðÁÒÁÌÌÅÌØÎÏ ÄÁÎÎÏÊ ÐÒÑÍÏÊ ÐÒÏ×ÅÄÉÔÅ ÐÒÑÍÕÀ, ÎÁ ËÏÔÏÒÏÊ Ä×Å ÄÁÎÎÙÅ ÏËÒÕÖ-

ÎÏÓÔÉ ×ÙÓÅËÁÌÉ ÂÙ ÈÏÒÄÙ, ÓÕÍÍÁ (ÉÌÉ ÒÁÚÎÏÓÔØ) ÄÌÉÎ ËÏÔÏÒÙÈ ÉÍÅÌÁ ÂÙ ÚÁÄÁÎÎÕÀ ×ÅÌÉÞÉÎÕ

a.

ABCD ÐÏ ÞÅÔÙÒÅÍ ÕÇÌÁÍ É ÓÔÏÒÏÎÁÍ AB = a É

143.12.

ðÏÓÔÒÏÊÔÅ ÞÅÔÙÒÅÈÕÇÏÌØÎÉË

143.13.

ðÏÓÔÒÏÊÔÅ ÞÅÔÙÒÅÈÕÇÏÌØÎÉË ÐÏ ÔÒÅÍ ÓÔÏÒÏÎÁÍ É ÕÇÌÁÍ, ÐÒÉÌÅÖÁÝÉÍ Ë ÞÅÔ×ÅÒ-

CD = b.
ÔÏÊ.

143.14.

ðÏÓÔÒÏÊÔÅ ÞÅÔÙÒÅÈÕÇÏÌØÎÉË ÐÏ ÄÉÁÇÏÎÁÌÑÍ, ÕÇÌÕ ÍÅÖÄÕ ÎÉÍÉ É Ä×ÕÍ ËÁËÉÍ-

ÎÉÂÕÄØ ÓÔÏÒÏÎÁÍ.
143.15.

ðÏÓÔÒÏÊÔÅ ×ÙÐÕËÌÙÊ ÞÅÔÙÒÅÈÕÇÏÌØÎÉË ÐÏ ÞÅÔÙÒÅÍ ÓÔÏÒÏÎÁÍ É ÏÔÒÅÚËÕ, ÓÏÅÄÉ-

ÎÑÀÝÅÍÕ ÓÅÒÅÄÉÎÙ Ä×ÕÈ ÐÒÏÔÉ×ÏÐÏÌÏÖÎÙÈ ÓÔÏÒÏÎ.
143.16.

óÒÅÄÉ ×ÓÅÈ ÞÅÔÙÒÅÈÕÇÏÌØÎÉËÏ× Ó ÄÁÎÎÙÍÉ ÄÉÁÇÏÎÁÌÑÍÉ É ÄÁÎÎÙÍ ÕÇÌÏÍ ÍÅÖÄÕ

ÎÉÍÉ ÎÁÊÄÉÔÅ ÞÅÔÙÒÅÈÕÇÏÌØÎÉË ÎÁÉÍÅÎØÛÅÇÏ ÐÅÒÉÍÅÔÒÁ.

äÁÎ ÏÔÒÅÚÏË AB. îÁÊÄÉÔÅ ÎÁ ÐÌÏÓËÏÓÔÉ ÍÎÏÖÅÓÔ×Ï ÔÏÞÅË C ÔÁËÉÈ, ÞÔÏ × ÔÒÅABC ÍÅÄÉÁÎÁ ÐÒÏ×ÅÄÅÎÎÁÑ ÉÚ ×ÅÒÛÉÎÙ A, ÒÁ×ÎÁ ×ÙÓÏÔÅ, ÐÒÏ×ÅÄÅÎÎÏÊ ÉÚ ×ÅÒÛÉÎÙ

143.17.

ÕÇÏÌØÎÉËÅ

B.

B ÐÁÒÁÌÌÅÌÏÇÒÁÍÍÁ ABCD ÐÒÏ×ÅÄÅÎÙ ÅÇÏ ×ÙÓÏÔÙ BK É BH. éÚ×ÅÓÔÎÙ ÏÔÒÅÚËÉ KH = a É BD = b. îÁÊÄÉÔÅ ÒÁÓÓÔÏÑÎÉÅ ÏÔ ÔÏÞËÉ B ÄÏ ÔÏÞËÉ ÐÅÒÅÓÅÞÅÎÉÑ
×ÙÓÏÔ ÔÒÅÕÇÏÌØÎÉËÁ BKH.
143.18.

éÚ ×ÅÒÛÉÎÙ

242

AB É CD ÎÅËÏÔÏÒÏÊ ÏËÒÕÖÎÏÓÔÉ. ðÏÓÔÒÏÊÔÅ ÎÁ
ÜÔÏÊ ÏËÒÕÖÎÏÓÔÉ ÔÁËÕÀ ÔÏÞËÕ X, ÞÔÏÂÙ ÈÏÒÄÙ AX É BX ×ÙÓÅËÁÌÉ ÎÁ ÈÏÒÄÅ CD ÏÔÒÅÚÏË EF,
ÉÍÅÀÝÉÊ ÄÁÎÎÕÀ ÄÌÉÎÕ a.
äÁÎÙ ÏËÒÕÖÎÏÓÔØ, Ä×Å ÔÏÞËÉ P É Q ÜÔÏÊ ÏËÒÕÖÎÏÓÔÉ É ÐÒÑÍÁÑ. îÁÊÄÉÔÅ ÎÁ
ÏËÒÕÖÎÏÓÔÉ ÔÁËÕÀ ÔÏÞËÕ M, ÞÔÏÂÙ ÐÒÑÍÙÅ MP É MQ ÏÔÓÅËÁÌÉ ÎÁ ÄÁÎÎÏÊ ÐÒÑÍÏÊ ÏÔÒÅÚÏË
AB ÄÁÎÎÏÊ ×ÅÌÉÞÉÎÙ.
143.19.

äÁÎÙ ÎÅÐÅÒÅÓÅËÁÀÝÉÅÓÑ ÈÏÒÄÙ

143.20.

143.21.

þÅÒÅÚ ÄÁÎÎÕÀ ÔÏÞËÕ ÐÒÏ×ÅÄÉÔÅ ÐÒÑÍÕÀ, ÎÁ ËÏÔÏÒÏÊ Ä×Å ÄÁÎÎÙÅ ÏËÒÕÖÎÏÓÔÉ

×ÙÓÅËÁÌÉ ÂÙ ÒÁ×ÎÙÅ ÈÏÒÄÙ.

243

144. çÏÍÏÔÅÔÉÑ

AB É AC ÔÒÅÕÇÏÌØÎÉËÁ ABC ×ÚÑÔÙ ÔÏÞËÉ M É N ÔÁË, ÞÔÏ MN k BC.
MN ×ÚÑÔÁ ÔÏÞËÁ P ÔÁË, ÞÔÏ MP = MN=3. ðÒÑÍÁÑ AP ÐÅÒÅÓÅËÁÅÔ ÓÔÏÒÏÎÕ BC ×
ÔÏÞËÅ Q. äÏËÁÖÉÔÅ, ÞÔÏ BQ = BC=3.
144.1.

îÁ ÓÔÏÒÏÎÁÈ

îÁ ÏÔÒÅÚËÅ
144.2.

ðÏÓÔÒÏÊÔÅ ÐÒÑÍÏÕÇÏÌØÎÙÊ ÔÒÅÕÇÏÌØÎÉË ÐÏ ÇÉÐÏÔÅÎÕÚÅ É ÏÔÎÏÛÅÎÉÀ ËÁÔÅÔÏ×.

144.3.

ðÏÓÔÒÏÊÔÅ ÐÒÑÍÏÕÇÏÌØÎÙÊ ÔÒÅÕÇÏÌØÎÉË ÐÏ ÄÁÎÎÏÍÕ ÏÔÎÏÛÅÎÉÀ ÏÄÎÏÇÏ ËÁÔÅÔÁ Ë

ÇÉÐÏÔÅÎÕÚÅ É ×ÔÏÒÏÍÕ ËÁÔÅÔÕ.

A É B É ÐÒÑÍÁÑ l. ðÏ ËÁËÏÊ ÔÒÁÅËÔÏÒÉÉ Ä×ÉÖÅÔÓÑ ÔÏÞËÁ
ÐÅÒÅÓÅÞÅÎÉÑ ÍÅÄÉÁÎ ÔÒÅÕÇÏÌØÎÉËÏ× ABC, ÅÓÌÉ ÔÏÞËÁ C Ä×ÉÖÅÔÓÑ ÐÏ ÐÒÑÍÏÊ l?
144.4.

îÁ ÐÌÏÓËÏÓÔÉ ÄÁÎÙ ÔÏÞËÉ

144.5.

îÁ ÏÓÎÏ×ÁÎÉÑÈ ÔÒÁÐÅÃÉÉ ËÁË ÎÁ ÓÔÏÒÏÎÁÈ ÐÏÓÔÒÏÅÎÙ ×Ï ×ÎÅÛÎÀÀ ÓÔÏÒÏÎÕ Ä×Á

Ë×ÁÄÒÁÔÁ. äÏËÁÖÉÔÅ, ÞÔÏ ÏÔÒÅÚÏË, ÓÏÅÄÉÎÑÀÝÉÊ ÃÅÎÔÒÙ Ë×ÁÄÒÁÔÏ×, ÐÒÏÈÏÄÉÔ ÞÅÒÅÚ ÔÏÞËÕ
ÐÅÒÅÓÅÞÅÎÉÑ ÄÉÁÇÏÎÁÌÅÊ ÔÒÁÐÅÃÉÉ.
144.6.

ðÏÓÔÒÏÊÔÅ ÔÒÁÐÅÃÉÀ, ÅÓÌÉ ÄÁÎÙ: ÏÔÎÏÛÅÎÉÅ ÅÅ ÏÓÎÏ×ÁÎÉÊ, Ä×Á ÕÇÌÁ ÐÒÉ ÏÄÎÏÍ ÉÚ

ÜÔÉÈ ÏÓÎÏ×ÁÎÉÊ É ×ÙÓÏÔÁ.
144.7.

äÁÎÙ Ä×Á ÏÔÒÅÚËÁ, ÌÅÖÁÝÉÅ ÌÉÂÏ ÎÁ ÏÄÎÏÊ ÐÒÑÍÏÊ, ÌÉÂÏ ÎÁ Ä×ÕÈ ÐÁÒÁÌÌÅÌØÎÙÈ.

óËÏÌØËÏ ÓÕÝÅÓÔ×ÕÅÔ ÇÏÍÏÔÅÔÉÊ, ÐÅÒÅ×ÏÄÑÝÉÈ ÜÔÉ ÏÔÒÅÚËÉ ÄÒÕÇ × ÄÒÕÇÁ?
144.8.

äÏËÁÖÉÔÅ, ÞÔÏ Ä×Å ËÁÓÁÀÝÉÅÓÑ ÏËÒÕÖÎÏÓÔÉ ÇÏÍÏÔÅÔÉÞÎÙ ÏÔÎÏÓÉÔÅÌØÎÏ ÉÈ ÔÏÞËÉ

ËÁÓÁÎÉÑ.
144.9.

ä×Å ÏËÒÕÖÎÏÓÔÉ ËÁÓÁÀÔÓÑ × ÔÏÞËÅ

A É B. äÏËÁÖÉÔÅ, ÞÔÏ ËÁÓÁÔÅÌØÎÙÅ Ë ÏËÒÕÖÎÏÓÔÑÍ, ÐÒÏ×ÅA É B, ÐÁÒÁÌÌÅÌØÎÙ.

ÓÅËÁÅÔ ÜÔÉ ÏËÒÕÖÎÏÓÔÉ × ÔÏÞËÁÈ
ÄÅÎÎÙÅ ÞÅÒÅÚ ÔÏÞËÉ

K. ðÒÑÍÁÑ, ÐÒÏÈÏÄÑÝÁÑ ÞÅÒÅÚ ÔÏÞËÕ K, ÐÅÒÅ-

144.10.

óËÏÌØËÏ ÓÕÝÅÓÔ×ÕÅÔ ÇÏÍÏÔÅÔÉÊ, ÐÅÒÅ×ÏÄÑÝÉÈ ÄÒÕÇ × ÄÒÕÇÁ Ä×Å ÏËÒÕÖÎÏÓÔÉ?

144.11.

ðÏÓÔÒÏÊÔÅ ÏÂÝÕÀ ËÁÓÁÔÅÌØÎÕÀ Ë Ä×ÕÍ ÄÁÎÎÙÍ ÏËÒÕÖÎÏÓÔÑÍ.

144.12.

îÁÊÄÉÔÅ ÇÅÏÍÅÔÒÉÞÅÓËÏÅ ÍÅÓÔÏ ÓÅÒÅÄÉÎ ÏÔÒÅÚËÏ×, ÓÏÅÄÉÎÑÀÀÝÉÈ ÄÁÎÎÕÀ ÔÏÞËÕ,

ÌÅÖÁÝÕÀ ×ÎÅ ÄÁÎÎÏÊ ÏËÒÕÖÎÏÓÔÉ, Ó ÔÏÞËÁÍÉ ÜÔÏÊ ÏËÒÕÖÎÏÓÔÉ.
144.13.

þÅÒÅÚ ÄÁÎÎÕÀ ÔÏÞËÕ ÐÒÏ×ÅÄÉÔÅ ÓÅËÕÝÕÀ Ë ÄÁÎÎÏÊ ÏËÒÕÖÎÏÓÔÉ ÔÁË, ÞÔÏÂÙ ×ÎÅÛ-

ÎÑÑ ÞÁÓÔØ ÓÅËÕÝÅÊ ÂÙÌÁ ÒÁ×ÎÁ ×ÎÕÔÒÅÎÎÅÊ.
144.14.

ó ÐÏÍÏÝØÀ ÃÉÒËÕÌÑ É ÌÉÎÅÊËÉ ÐÏÓÔÒÏÊÔÅ ÈÏÒÄÕ ÄÁÎÎÏÊ ÏËÒÕÖÎÏÓÔÉ, ËÏÔÏÒÕÀ

Ä×Á ÄÁÎÎÙÈ ÒÁÄÉÕÓÁ ÒÁÚÄÅÌÉÌÉ ÂÙ ÎÁ ÔÒÉ ÒÁ×ÎÙÅ ÞÁÓÔÉ.
144.15.

144.16.

M, ÌÅÖÁÝÕÀ ÎÁ ÄÁÎÎÏÊ
AB ÄÅÌÉÌÁÓØ ÂÙ ÐÏÐÏÌÁÍ.

þÅÒÅÚ ÔÏÞËÕ

ÄÁÎÎÏÊ ÈÏÒÄÏÊ

ÏËÒÕÖÎÏÓÔÉ, ÐÒÏ×ÅÄÉÔÅ ÈÏÒÄÕ, ËÏÔÏÒÁÑ

ðÏÓÔÒÏÊÔÅ ÏËÒÕÖÎÏÓÔØ, ËÁÓÁÀÝÕÀÓÑ ÄÁÎÎÏÊ ÐÒÑÍÏÊ É ÄÁÎÎÏÊ ÏËÒÕÖÎÏÓÔÉ ×

ÄÁÎÎÏÊ ÎÁ ÎÅÊ ÔÏÞËÅ.
144.17.

ðÏÓÔÒÏÊÔÅ ÏËÒÕÖÎÏÓÔØ, ËÁÓÁÀÝÕÀÓÑ ÄÁÎÎÏÊ ÏËÒÕÖÎÏÓÔÉ É ÄÁÎÎÏÊ ÐÒÑÍÏÊ ×

ÄÁÎÎÏÊ ÎÁ ÎÅÊ ÔÏÞËÅ.
144.18.

þÅÒÅÚ ÄÁÎÎÕÀ ÔÏÞËÕ ÐÒÏ×ÅÄÉÔÅ ÏËÒÕÖÎÏÓÔØ, ËÁÓÁÀÝÕÀÓÑ ÄÁÎÎÙÈ ÐÒÑÍÏÊ É

ÏËÒÕÖÎÏÓÔÉ.
144.19.

÷ ÄÁÎÎÙÊ ÔÒÅÕÇÏÌØÎÉË ×ÐÉÛÉÔÅ Ë×ÁÄÒÁÔ ÔÁË, ÞÔÏÂÙ ÏÄÎÁ ÓÔÏÒÏÎÁ Ë×ÁÄÒÁÔÁ ÂÙ-

ÌÁ ÒÁÓÐÏÌÏÖÅÎÁ ÎÁ ÓÔÏÒÏÎÅ ÔÒÅÕÇÏÌØÎÉËÁ, Á ÏÓÔÁÌØÎÙÅ ×ÅÒÛÉÎÙ Ë×ÁÄÒÁÔÁ ÌÅÖÁÌÉ ÎÁ Ä×ÕÈ
ÄÒÕÇÉÈ ÓÔÏÒÏÎÁÈ ÔÒÅÕÇÏÌØÎÉËÁ.

A1B1C1 É A2B2C2, ÐÌÏÝÁÄÉ ËÏÔÏÒÙÈ ÒÁ×ÎÙ S1 É S2, ÒÁÓÐÏÌÏÖÅÎÙ ÔÁË, ÞÔÏ ÌÕÞÉ A1 B1 É A2 B2 , B1C1 É B2 C2 , C1 A1 É C2A2 ÐÁÒÁÌÌÅÌØÎÙ, ÎÏ ÐÒÏÔÉ144.20.

ä×Á ÔÒÅÕÇÏÌØÎÉËÁ

244

×ÏÐÏÌÏÖÎÏ ÎÁÐÒÁ×ÌÅÎÙ. îÁÊÄÉÔÅ ÐÌÏÝÁÄØ ÔÒÅÕÇÏÌØÎÉËÁ Ó ×ÅÒÛÉÎÁÍÉ × ÓÅÒÅÄÉÎÁÈ ÏÔÒÅÚËÏ×

A1A2 , B1B2, C1C2.

A É B, Á ÔÏÞËÁ C Ä×ÉÖÅÔÓÑ ÐÏ ÜÔÏÊ ÏËÒÕÖÎÏÓÔÉ. îÁÊÄÉÔÅ ÇÅÏÍÅÔÒÉÞÅÓËÏÅ ÍÅÓÔÏ ÔÏÞÅË ÐÅÒÅÓÅÞÅÎÉÑ ÍÅÄÉÁÎ ÔÒÅÕÇÏÌØÎÉËÏ× ABC.
÷ÅÒÛÉÎÙ K É N ÔÒÅÕÇÏÌØÎÉËÁ KMN ÐÅÒÅÍÅÝÁÀÔÓÑ ÓÏÏÔ×ÅÔÓÔ×ÅÎÎÏ ÐÏ ÓÔÏÒÏÎÁÍ AB É AC ÕÇÌÁ BAC, Á ÓÔÏÒÏÎÙ ÔÒÅÕÇÏÌØÎÉËÁ KMN ÓÏÏÔ×ÅÔÓÔ×ÅÎÎÏ ÐÁÒÁÌÌÅÌØÎÙ ÔÒÅÍ
ÄÁÎÎÙÍ ÐÒÑÍÙÍ. îÁÊÄÉÔÅ ÇÅÏÍÅÔÒÉÞÅÓËÏÅ ÍÅÓÔÏ ×ÅÒÛÉÎ M.
þÅÒÅÚ ÔÏÞËÕ D, ×ÚÑÔÕÀ ÎÁ ÓÔÏÒÏÎÅ AB ÔÒÅÕÇÏÌØÎÉËÁ ABC, ÐÒÏ×ÅÄÅÎÁ ÐÒÑÍÁÑ,
ÐÁÒÁÌÌÅÌØÎÁÑ AC, É ÐÅÒÅÓÅËÁÀÝÁÑ ÓÔÏÒÏÎÕ BC × ÔÏÞËÅ E. äÏËÁÖÉÔÅ, ÞÔÏ AE, CD É ÍÅÄÉÁÎÁ,
ÐÒÏ×ÅÄÅÎÎÁÑ ÞÅÒÅÚ ×ÅÒÛÉÎÕ B, ÐÅÒÅÓÅËÁÀÔÓÑ × ÏÄÎÏÊ ÔÏÞËÅ.
144.21.

îÁ ÏËÒÕÖÎÏÓÔÉ ÆÉËÓÉÒÏ×ÁÎÙ ÔÏÞËÉ

144.22.

144.23.

144.24.

÷ ÄÁÎÎÙÊ ÐÁÒÁÌÌÅÌÏÇÒÁÍÍ ×ÐÉÛÉÔÅ ÒÏÍ ÔÁË, ÞÔÏÂÙ ÓÔÏÒÏÎÙ ÒÏÍÂÁ ÂÙÌÉ ÐÁ-

ÒÁÌÌÅÌØÎÙ ÄÉÁÇÏÎÁÌÑÍ ÐÁÒÁÌÌÅÌÏÇÒÁÍÍÁ, Á ×ÅÒÛÉÎÙ ÒÏÍÂÁ ÌÅÖÁÌÉ ÂÙ ÎÁ ÓÔÏÒÏÎÁÈ ÐÁÒÁÌÌÅÌÏÇÒÁÍÍÁ.
144.25.

÷ ÄÁÎÎÙÊ ÔÒÅÕÇÏÌØÎÉË ×ÐÉÛÉÔÅ ÄÒÕÇÏÊ ÔÒÅÕÇÏÌØÎÉË, ÓÔÏÒÏÎÙ ËÏÔÏÒÏÇÏ ÓÏÏÔ×ÅÔ-

ÓÔ×ÅÎÎÏ ÐÁÒÁÌÌÅÌØÎÙ ÔÒÅÍ ÄÁÎÎÙÍ ÐÒÑÍÙÍ.

AB É AC ÔÒÅÕÇÏÌØÎÉËÁ ABC ÐÏÓÔÒÏÊÔÅ ÓÏÏÔ×ÅÔÓÔ×ÅÎÎÏ ÔÏÞËÉ M É
N ÔÁË, ÞÔÏ AM = MN = NC.
144.26.

îÁ ÓÔÏÒÏÎÁÈ

144.27.

÷ÐÉÛÉÔÅ × ÔÒÅÕÇÏÌØÎÉË Ä×Å ÒÁ×ÎÙÅ ÏËÒÕÖÎÏÓÔÉ, ËÁÖÄÁÑ ÉÚ ËÏÔÏÒÙÈ ËÁÓÁÅÔÓÑ

Ä×ÕÈ ÓÔÏÒÏÎ ÔÒÅÕÇÏÌØÎÉËÁ É ÄÒÕÇÏÊ ÏËÒÕÖÎÏÓÔÉ.

ðÏÓÔÒÏÊÔÅ ÔÒÅÕÇÏÌØÎÉË ABC, ÅÓÌÉ ÚÁÄÁÎÙ ÅÇÏ ÎÁÉÍÅÎØÛÉÊ ÕÇÏÌ \A É ÏÔÒÅÚËÉ
d = AB - BC É e = AC - BC.

144.28.

ÄÌÉÎÙ

144.29.

÷ÐÉÛÉÔÅ × ÄÁÎÎÙÊ ÕÇÏÌ ÏËÒÕÖÎÏÓÔØ, ÐÒÏÈÏÄÑÝÕÀ ÞÅÒÅÚ ÄÁÎÎÕÀ ×ÎÕÔÒÉ ÕÇÌÁ

ÔÏÞËÕ.
144.30.

÷ÐÉÛÉÔÅ × ÄÁÎÎÙÊ ÕÇÏÌ ÏËÒÕÖÎÏÓÔØ, ËÁÓÁÀÝÕÀÓÑ ÄÁÎÎÏÊ ÏËÒÕÖÎÏÓÔÉ.

144.31.

(ðÒÑÍÁÑ

üÊÌÅÒÁ) äÏËÁÖÉÔÅ, ÞÔÏ ÔÏÞËÁ ÐÅÒÅÓÅÞÅÎÉÑ ÍÅÄÉÁÎ, ÏÒÔÏÃÅÎÔÒ É ÃÅÎÔÒ

ÏÐÉÓÁÎÎÏÊ ÏËÒÕÖÎÏÓÔÉ ÔÒÅÕÇÏÌØÎÉËÁ ÌÅÖÁÔ ÎÁ ÏÄÎÏÊ ÐÒÑÍÏÊ. (òÁÓÓÍÏÔÒÉÔÅ
ÔÏÞËÁ ÐÅÒÅÓÅÞÅÎÉÑ ÍÅÄÉÁÎ).

H1=2
M , ÇÄÅ M |

AA1, BB1 É CC1 ÔÒÅÕÇÏÌØÎÉËÁ ABC ÐÅÒÅÓÅËÁÀÔÓÑ × ÔÏÞËÅ M; P |
ÐÒÏÉÚ×ÏÌØÎÁÑ ÔÏÞËÁ. ðÒÑÍÁÑ la ÐÒÏÈÏÄÉÔ ÞÅÒÅÚ ÔÏÞËÕ A ÐÁÒÁÌÌÅÌØÎÏ ÐÒÑÍÏÊ PA1, ÐÒÑÍÙÅ
lb É lc ÏÐÒÅÄÅÌÑÀÔÓÑ ÁÎÁÌÏÇÉÞÎÏ. äÏËÁÖÉÔÅ, ÞÔÏ
Á) ÐÒÑÍÙÅ la , lb É lc ÐÅÒÅÓÅËÁÀÔÓÑ × ÏÄÎÏÊ ÔÏÞËÅ Q;
Â) ÔÏÞËÁ M ÌÅÖÉÔ ÎÁ ÏÔÒÅÚËÅ PQ, ÐÒÉÞÅÍ PM : MQ = 1 : 2.
óÔÏÒÏÎÁ C1C2 ÐÒÑÍÏÕÇÏÌØÎÉËÁ C1C2 PR ÒÁÓÐÏÌÏÖÅÎÁ ÎÁ ÓÔÏÒÏÎÅ AB ÔÒÅÕÇÏÌØÎÉËÁ ABC, ÓÔÏÒÏÎÁ B1 B2 ÐÒÑÍÏÕÇÏÌØÎÉËÁ B1 B2QP | ÎÁ ÓÔÏÒÏÎÅ AC, ÓÔÏÒÏÎÁ A1 A2 ÐÒÑÍÏÕÇÏÌØÎÉËÁ A1 A2 RQ | ÎÁ ÓÔÏÒÏÎÅ BC, ÐÒÉÞÅÍ ÔÏÞËÉ P, Q É R ÌÅÖÁÔ ×ÎÕÔÒÉ ÔÒÅÕÇÏÌØÎÉËÁ
ABC. äÏËÁÖÉÔÅ, ÞÔÏ ÐÒÑÍÙÅ AP, BR É CQ ÐÅÒÅÓÅËÁÀÔÓÑ × ÏÄÎÏÊ ÔÏÞËÅ.
÷ÐÉÓÁÎÎÁÑ ÏËÒÕÖÎÏÓÔØ ÔÒÅÕÇÏÌØÎÉËÁ ABC ËÁÓÁÅÔÓÑ ÓÔÏÒÏÎÙ AC × ÔÏÞËÅ D, DM
| ÅÅ ÄÉÁÍÅÔÒ. ðÒÑÍÁÑ BM ÐÅÒÅÓÅËÁÅÔ ÓÔÏÒÏÎÕ AC × ÔÏÞËÅ K. äÏËÁÖÉÔÅ, ÞÔÏ AK = DC.
÷ ÔÒÅÕÇÏÌØÎÉËÅ ABC ÞÅÒÅÚ ÓÅÒÅÄÉÎÕ M ÓÔÏÒÏÎÙ BC É ÃÅÎÔÒ O ×ÐÉÓÁÎÎÏÊ × ÜÔÏÔ
ÔÒÅÕÇÏÌØÎÉË ÏËÒÕÖÎÏÓÔÉ ÐÒÏ×ÅÄÅÎÁ ÐÒÑÍÁÑ MO, ËÏÔÏÒÁÑ ÐÅÒÅÓÅËÁÅÔ ×ÙÓÏÔÕ AH × ÔÏÞËÅ E.
äÏËÁÖÉÔÅ, ÞÔÏ ÏÔÒÅÚÏË AE ÒÁ×ÅÎ ÒÁÄÉÕÓÕ ×ÐÉÓÁÎÎÏÊ ÏËÒÕÖÎÏÓÔÉ.
144.32.

íÅÄÉÁÎÙ

144.33.

144.34.

144.35.

144.36.

òÁÓÓÍÏÔÒÉÍ ×ÓÅ×ÏÚÍÏÖÎÙÅ ÐÁÒÙ ËÁÓÁÀÝÉÈÓÑ ×ÎÅÛÎÉÍ ÏÂÒÁÚÏÍ ÏËÒÕÖÎÏÓÔÅÊ,

×ÐÉÓÁÎÎÙÈ × ËÒÕÇÏ×ÏÊ ÓÅÇÍÅÎÔ. äÏËÁÖÉÔÅ, ÞÔÏ ÏÂÝÉÅ ×ÎÕÔÒÅÎÎÉÅ ËÁÓÁÔÅÌØÎÙÅ ËÁÖÄÏÊ ÔÁËÏÊ
ÐÁÒÙ ÐÒÏÈÏÄÑÔ ÞÅÒÅÚ ÏÄÎÕ ÔÏÞËÕ.

245

r É R (r < R) ×ÎÅÛÎÉÍ ÏÂÒÁÚÏÍ ËÁÓÁÀÔÓÑ ÄÒÕÇ ÄÒÕÇÁ.
ðÒÑÍÁÑ ËÁÓÁÅÔÓÑ ÜÔÉÈ ÏËÒÕÖÎÏÓÔÅÊ × ÔÏÞËÁÈ M É N. ÷ ÔÏÞËÁÈ A É B ÏËÒÕÖÎÏÓÔÉ ËÁÓÁÀÔÓÑ
×ÎÅÛÎÉÍ ÏÂÒÁÚÏÍ ÔÒÅÔØÅÊ ÏËÒÕÖÎÏÓÔÉ. ðÒÑÍÙÅ AB É MN ÐÅÒÅÓÅËÁÀÔÓÑ × ÔÏÞËÅ C. éÚ ÔÏÞËÉ
C ÐÒÏ×ÅÄÅÎÁ ËÁÓÁÔÅÌØÎÁÑ Ë ÔÒÅÔØÅÊ ÏËÒÕÖÎÏÓÔÉ (D | ÔÏÞËÁ ËÁÓÁÎÉÑ). îÁÊÄÉÔÅ CD.
÷ ÐÏÌÕËÒÕÇ ÐÏÍÅÝÅÎÙ Ä×Å ÏËÒÕÖÎÏÓÔÉ ÄÉÁÍÅÔÒÏÍ d É D (d < D) ÔÁË, ÞÔÏ ËÁÖÄÁÑ
144.37.

ä×Å ÏËÒÕÖÎÏÓÔÉ ÒÁÄÉÕÓÏ×

144.38.

ÏËÒÕÖÎÏÓÔØ ËÁÓÁÅÔÓÑ ÄÕÇÉ É ÄÉÁÍÅÔÒÁ ÐÏÌÕËÒÕÇÁ, Á ÔÁËÖÅ ÄÒÕÇÏÊ ÏËÒÕÖÎÏÓÔÉ. þÅÒÅÚ ÃÅÎÔÒÙ

M.
M ÐÒÏ×ÅÄÅÎÁ ËÁÓÁÔÅÌØÎÁÑ Ë ÄÕÇÅ ÐÏÌÕËÒÕÇÁ (N | ÔÏÞËÁ ËÁÓÁÎÉÑ). îÁÊÄÉÔÅ ÄÌÉÎÕ
ÏÔÒÅÚËÁ MN.

ÏËÒÕÖÎÏÓÔÅÊ ÐÒÏ×ÅÄÅÎÁ ÐÒÑÍÁÑ, ÐÅÒÅÓÅËÁÀÝÁÑ ÐÒÏÄÏÌÖÅÎÉÅ ÄÉÁÍÅÔÒÁ ÐÏÌÕËÒÕÇÁ × ÔÏÞËÅ
éÚ ÔÏÞËÉ

144.39.

þÅÒÅÚ ÄÁÎÎÕÀ ÔÏÞËÕ ÐÒÏ×ÅÄÉÔÅ ÐÒÑÍÕÀ, ÏÔÓÅËÁÀÝÕÀ ÏÔ ÄÁÎÎÏÇÏ ÕÇÌÁ ÔÒÅÕÇÏÌØ-

ÎÉË ÎÁÉÍÅÎØÛÅÇÏ ×ÏÚÍÏÖÎÏÇÏ ÐÅÒÉÍÅÔÒÁ.

ïÐÕÓÔÉÍ ÉÚ ÌÀÂÏÊ ÔÏÞËÉ P ÂÉÓÓÅËÔÒÉÓÙ ÕÇÌÁ A ÔÒÅÕÇÏÌØÎÉËÁ ABC ÐÅÒÐÅÎÄÉPA1, PB1, PC1 ÎÁ ÅÇÏ ÓÔÏÒÏÎÙ BC, CA É AB ÓÏÏÔ×ÅÔÓÔ×ÅÎÎÏ. ðÕÓÔØ R | ÔÏÞËÁ
ÐÅÒÅÓÅÞÅÎÉÑ ÐÒÑÍÙÈ PA1 É B1 C1 . äÏËÁÖÉÔÅ, ÞÔÏ ÐÒÑÍÁÑ AR ÄÅÌÉÔ ÓÔÏÒÏÎÕ BC ÐÏÐÏÌÁÍ.
òÁ×ÎÙÅ ÏËÒÕÖÎÏÓÔÉ S1 É S2 ËÁÓÁÀÔÓÑ ×ÎÕÔÒÅÎÎÉÍ ÏÂÒÁÚÏÍ ÏËÒÕÖÎÏÓÔÉ S × ÔÏÞËÁÈ A1 É A2 . ðÕÓÔØ C | ÎÅËÏÔÏÒÁÑ ÔÏÞËÁ ÏËÒÕÖÎÏÓÔÉ S, ÐÒÑÍÙÅ A1 C É A2 C ÐÅÒÅÓÅËÁÀÔ
ÏËÒÕÖÎÏÓÔÉ S1 É S2 × ÔÏÞËÁÈ B1 É B2 ÓÏÏÔ×ÅÔÓÔ×ÅÎÎÏ. äÏËÁÖÉÔÅ, ÞÔÏ B1 B2 = (?)A1 A2 .
42. îÁ ÐÌÏÓËÏÓÔÉ ÒÁÓÐÏÌÏÖÅÎÙ ÔÒÉ ÏËÒÕÖÎÏÓÔÉ S1, S2 , S3 ÒÁÄÉÕÓÏ× r1, r2 , r3 |
ËÁÖÄÁÑ ×ÎÅ Ä×ÕÈ ÄÒÕÇÉÈ, ÐÒÉÞÅÍ r1 > r2 É r1 > r3 . éÚ ÔÏÞËÉ ÐÅÒÅÓÅÞÅÎÉÑ ×ÎÅÛÎÉÈ ËÁÓÁÔÅÌØÎÙÈ Ë ÏËÒÕÖÎÏÓÔÑÍ S1 É S2 ÐÒÏ×ÅÄÅÎÙ ËÁÓÁÔÅÌØÎÙÅ Ë ÏËÒÕÖÎÏÓÔÉ S3 , Á ÉÚ ÔÏÞËÉ ÐÅÒÅÓÅÞÅÎÉÑ
×ÎÅÛÎÉÈ ËÁÓÁÔÅÌØÎÙÈ Ë ÏËÒÕÖÎÏÓÔÑÍ S1 É S3 ÐÒÏ×ÅÄÅÎÙ ËÁÓÁÔÅÌØÎÙÅ Ë ÏËÒÕÖÎÏÓÔÉ S2 . äÏ144.40.

ËÕÌÑÒÙ

144.41.

144.42.

ËÁÖÉÔÅ, ÞÔÏ ÐÏÓÌÅÄÎÉÅ Ä×Å ÐÁÒÙ ËÁÓÁÔÅÌØÎÙÈ ÏÂÒÁÚÕÀÔ ÞÅÔÙÒÅÈÕÇÏÌØÎÉË, × ËÏÔÏÒÙÊ ÍÏÖÎÏ

×ÐÉÓÁÔØ ÏËÒÕÖÎÏÓÔØ, É ÎÁÊÄÉÔÅ ÅÅ ÒÁÄÉÕÓ.

AB, BC É CA ÔÒÅÕÇÏÌØÎÉËÁ ABC ÐÏÓÔÒÏÅÎÙ ×Ï ×ÎÅÛÎÀÀ ÓÔÏÒÏÎÕ
Ë×ÁÄÒÁÔÙ ABB1 A2 , BCC1B2 É CAA1 C2. äÏËÁÖÉÔÅ, ÞÔÏ ÐÅÒÐÅÎÄÉËÕÌÑÒÙ Ë ÏÔÒÅÚËÁÍ A1 A2 ,
B1B2 É C1C2, ×ÏÓÓÔÁ×ÌÅÎÎÙÅ × ÉÈ ÓÅÒÅÄÉÎÁÈ, ÐÅÒÅÓÅËÁÀÔÓÑ × ÏÄÎÏÊ ÔÏÞËÅ.
144.43.

îÁ ÏÔÒÅÚËÁÈ

246

145. ëÏÎÔÒÏÌØÎÁÑ ÒÁÂÏÔÁ. ðÁÒÁÌÌÅÌÏÇÒÁÍÍ, ÐÒÑÍÏÕÇÏÌØÎÉË, ÒÏÍÂ,
Ë×ÁÄÒÁÔ

÷ÁÒÉÁÎÔ 1

âÉÓÓÅËÔÒÉÓÁ ÕÇÌÁ A ÐÁÒÁÌÌÅÌÏÇÒÁÍÍÁ ABCD ÐÅÒÅÓÅËÁÅÔ ÓÔÏÒÏÎÕ BC × ÔÏÞËÅ K, Á
ÐÒÏÄÏÌÖÅÎÉÅ ÓÔÏÒÏÎÙ CD | × ÔÏÞËÅ M. éÚ×ÅÓÔÎÏ, ÞÔÏ CM = 1, BK = 3. îÁÊÄÉÔÅ ÓÔÏÒÏÎÙ
ÐÁÒÁÌÌÅÌÏÇÒÁÍÍÁ.
145.2. éÚ ÐÒÏÉÚ×ÏÌØÎÏÊ ÔÏÞËÉ ÏÓÎÏ×ÁÎÉÑ ÒÁ×ÎÏÂÅÄÒÅÎÎÏÇÏ ÔÒÅÕÇÏÌØÎÉËÁ Ó ÂÏËÏ×ÏÊ ÓÔÏÒÏÎÏÊ, ÒÁ×ÎÏÊ 5, ÐÒÏ×ÅÄÅÎÙ ÐÒÑÍÙÅ, ÐÁÒÁÌÌÅÌØÎÙÅ ÂÏËÏ×ÙÍ ÓÔÏÒÏÎÁÍ. îÁÊÄÉÔÅ ÐÅÒÉÍÅÔÒ
ÐÏÌÕÞÉ×ÛÅÇÏÓÑ ÞÅÔÙÒÅÈÕÇÏÌØÎÉËÁ.
145.3. îÁ ÓÔÏÒÏÎÁÈ AB É CD ÐÒÑÍÏÕÇÏÌØÎÉËÁ ABCD ×ÚÑÔÙ ÔÏÞËÉ K É M ÔÁË, ÞÔÏ AKCM
| ÒÏÍÂ. äÉÁÇÏÎÁÌØ AC ÓÏÓÔÁ×ÌÑÅÔ ÓÏ ÓÔÏÒÏÎÏÊ AB ÕÇÏÌ 30. îÁÊÄÉÔÅ ÓÔÏÒÏÎÕ ÒÏÍÂÁ, ÅÓÌÉ
ÎÁÉÂÏÌØÛÁÑ ÓÔÏÒÏÎÁ ÐÒÑÍÏÕÇÏÌØÎÉËÁ ABCD ÒÁ×ÎÁ 3.
145.1.

145.4. ÷ ÐÒÑÍÏÕÇÏÌØÎÉËÅ

CD P
,

ABCD

* * *

M
DM BN

ÔÏÞËÁ

| ÔÏÞËÁ ÐÅÒÅÓÅÞÅÎÉÑ ÏÔÒÅÚËÏ×

| ÓÅÒÅÄÉÎÁ ÓÔÏÒÏÎÙ

É

BC

N
MAN

, ÔÏÞËÁ

. äÏËÁÖÉÔÅ, ÞÔÏ ÕÇÏÌ

| ÓÅÒÅÄÉÎÁ ÓÔÏÒÏÎÙ

ÒÁ×ÅÎ ÕÇÌÕ

BPM

.

÷ÁÒÉÁÎÔ 2

âÉÓÓÅËÔÒÉÓÁ ÕÇÌÁ A ÐÁÒÁÌÌÅÌÏÇÒÁÍÍÁ ABCD ÓÏ ÓÔÏÒÏÎÁÍÉ AB = 5, AD = 8
ÐÅÒÅÓÅËÁÅÔ ÓÔÏÒÏÎÕ BC × ÔÏÞËÅ P, Á ÐÒÏÄÏÌÖÅÎÉÅ ÓÔÏÒÏÎÙ CD | × ÔÏÞËÅ Q. îÁÊÄÉÔÅ CQ
É BP.
145.6. éÚ ÐÒÏÉÚ×ÏÌØÎÏÊ ÔÏÞËÉ ÏÓÎÏ×ÁÎÉÑ ÒÁ×ÎÏÂÅÄÒÅÎÎÏÇÏ ÔÒÅÕÇÏÌØÎÉËÁ Ó ÐÅÒÉÍÅÔÒÏÍ,
ÒÁ×ÎÙÍ 25, ÐÒÏ×ÅÄÅÎÙ ÐÒÑÍÙÅ, ÐÁÒÁÌÌÅÌØÎÙÅ ÂÏËÏ×ÙÍ ÓÔÏÒÏÎÁÍ. ðÅÒÉÍÅÔÒ ÐÏÌÕÞÉ×ÛÅÇÏÓÑ
ÞÅÔÙÒÅÈÕÇÏÌØÎÉËÁ ÒÁ×ÅÎ 16. îÁÊÄÉÔÅ ÓÔÏÒÏÎÙ ÔÒÅÕÇÏÌØÎÉËÁ.
145.7. þÅÒÅÚ ÓÅÒÅÄÉÎÕ ÄÉÁÇÏÎÁÌÉ KM ÐÒÑÍÏÕÇÏÌØÎÉËÁ KLMN ÐÅÒÐÅÎÄÉËÕÌÑÒÎÏ ÜÔÏÊ
ÄÉÁÇÏÎÁÌÉ ÐÒÏ×ÅÄÅÎÁ ÐÒÑÍÁÑ, ÐÅÒÅÓÅËÁÀÝÁÑ ÓÔÏÒÏÎÙ KL É MN × ÔÏÞËÁÈ A É B ÓÏÏÔ×ÅÔÓÔ×ÅÎÎÏ. éÚ×ÅÓÔÎÏ, ÞÔÏ AB = BM = 6. îÁÊÄÉÔÅ ÎÁÉÂÏÌØÛÕÀ ÓÔÏÒÏÎÕ ÐÒÑÍÏÕÇÏÌØÎÉËÁ.
145.5.

145.8. ÷ ÐÒÑÍÏÕÇÏÌØÎÉËÅ

CD P
,

ABCD

* * *

M
DM BN

ÔÏÞËÁ

| ÔÏÞËÁ ÐÅÒÅÓÅÞÅÎÉÑ ÏÔÒÅÚËÏ×

| ÓÅÒÅÄÉÎÁ ÓÔÏÒÏÎÙ

É

BC

N
MAN

, ÔÏÞËÁ

. äÏËÁÖÉÔÅ, ÞÔÏ ÕÇÏÌ

| ÓÅÒÅÄÉÎÁ ÓÔÏÒÏÎÙ

ÒÁ×ÅÎ ÕÇÌÕ

BPM

.

÷ÁÒÉÁÎÔ 3
145.9. óÔÏÒÏÎÁ BC ÐÁÒÁÌÌÅÌÏÇÒÁÍÍÁ ABCD ×Ä×ÏÅ ÂÏÌØÛÅ ÓÔÏÒÏÎÙ BC. âÉÓÓÅËÔÒÉÓÙ
ÕÇÌÏ× A É B ÐÅÒÅÓÅËÁÀÔ ÐÒÑÍÕÀ CD × ÔÏÞËÁÈ M É N, ÐÒÉÞÅÍ MN = 12. îÁÊÄÉÔÅ ÓÔÏÒÏÎÙ
ÐÁÒÁÌÌÅÌÏÇÒÁÍÍÁ.
145.10. éÚ ÐÒÏÉÚ×ÏÌØÎÏÊ ÔÏÞËÉ ÏÓÎÏ×ÁÎÉÑ ÒÁ×ÎÏÂÅÄÒÅÎÎÏÇÏ ÔÒÅÕÇÏÌØÎÉËÁ ÐÒÏ×ÅÄÅÎÙ
ÐÒÑÍÙÅ, ÐÁÒÁÌÌÅÌØÎÙÅ ÂÏËÏ×ÙÍ ÓÔÏÒÏÎÁÍ. ðÅÒÉÍÅÔÒ ÐÏÌÕÞÉ×ÛÅÇÏÓÑ ÞÅÔÙÒÅÈÕÇÏÌØÎÉËÁ ÒÁ×ÅÎ 12. îÁÊÄÉÔÅ ÂÏËÏ×ÙÅ ÓÔÏÒÏÎÙ ÔÒÅÕÇÏÌØÎÉËÁ.

247

ðÒÑÍÁÑ, ÐÒÏÈÏÄÑÝÁÑ ÞÅÒÅÚ ÃÅÎÔÒ ÐÒÑÍÏÕÇÏÌØÎÉËÁ ÐÅÒÐÅÎÄÉËÕÌÑÒÎÏ ÄÉÁÇÏÎÁÌÉ,
ÐÅÒÅÓÅËÁÅÔ ÂÏÌØÛÕÀ ÓÔÏÒÏÎÕ ÐÒÑÍÏÕÇÏÌØÎÉËÁ ÐÏÄ ÕÇÌÏÍ, ÒÁ×ÎÙÍ 60. ïÔÒÅÚÏË ÜÔÏÊ ÐÒÑÍÏÊ,
ÚÁËÌÀÞÅÎÎÙÊ ×ÎÕÔÒÉ ÐÒÑÍÏÕÇÏÌØÎÉËÁ, ÒÁ×ÅÎ 10. îÁÊÄÉÔÅ ÂÏÌØÛÕÀ ÓÔÏÒÏÎÕ ÐÒÑÍÏÕÇÏÌØÎÉËÁ.
145.11.

248

146. ëÏÎÔÒÏÌØÎÁÑ ÒÁÂÏÔÁ. ðÌÏÝÁÄØ, 1

÷ÁÒÉÁÎÔ 1
146.1. ôÏÞËÉ M É N ÒÁÓÐÏÌÏÖÅÎÙ ÎÁ ÓÔÏÒÏÎÅ BC ÔÒÅÕÇÏÌØÎÉËÁ ABC, Á ÔÏÞËÁ K | ÎÁ
ÓÔÏÒÏÎÅ AC, ÐÒÉÞÅÍ BM : MN : NC = 1 : 1 : 2 É CK : AK = 1 : 4. éÚ×ÅÓÔÎÏ, ÞÔÏ ÐÌÏÝÁÄØ
ÔÒÅÕÇÏÌØÎÉËÁ ABC ÒÁ×ÎÁ 1. îÁÊÄÉÔÅ ÐÌÏÝÁÄØ ÞÅÔÙÒÅÈÕÇÏÌØÎÉËÁ AMNK.
146.2. ðÒÏÅËÃÉÑ ×ÅÒÛÉÎÙ ÍÅÎØÛÅÇÏ ÏÓÎÏ×ÁÎÉÑ ÒÁ×ÎÏÂÅÄÒÅÎÎÏÊ ÔÒÁÐÅÃÉÉ ÎÁ ÅÅ ÂÏÌØÛÅÅ
ÏÓÎÏ×ÁÎÉÅ ÄÅÌÉÔ ÅÇÏ ÎÁ ÏÔÒÅÚËÉ, ÒÁ×ÎÙÅ a É b. îÁÊÄÉÔÅ ÐÌÏÝÁÄØ ÔÒÁÐÅÃÉÉ, ÅÓÌÉ ÕÇÏÌ ÐÒÉ
ÅÅ ÍÅÎØÛÅÍ ÏÓÎÏ×ÁÎÉÉ ÒÁ×ÅÎ 150 .
146.3. ïÔÒÅÚËÉ, ÓÏÅÄÉÎÑÀÝÉÅ ÓÅÒÅÄÉÎÙ ÐÒÏÔÉ×ÏÐÏÌÏÖÎÙÈ ÓÔÏÒÏÎ ×ÙÐÕËÌÏÇÏ ÞÅÔÙÒÅÈÕÇÏÌØÎÉËÁ, ÒÁ×ÎÙ ÍÅÖÄÕ ÓÏÂÏÊ. îÁÊÄÉÔÅ ÐÌÏÝÁÄØ ÞÅÔÙÒÅÈÕÇÏÌØÎÉËÁ, ÅÓÌÉ ÅÇÏ ÄÉÁÇÏÎÁÌÉ
ÒÁ×ÎÙ 8 É 12.

* * *

ABCD
ABC

146.4. ÷ ×ÙÐÕËÌÏÍ ÞÅÔÙÒÅÈÕÇÏÌØÎÉËÅ
éÚ×ÅÓÔÎÏ, ÞÔÏ ÐÌÏÝÁÄØ ÔÒÅÕÇÏÌØÎÉËÁ

BCD
ABC ABD ACD BCD
ÔÒÅÕÇÏÌØÎÉËÁ
,

, ÐÌÏÝÁÄØ ËÏÔÏÒÏÇÏ ÒÁ×ÎÁ

×ÔÒÏÅ ÂÏÌØÛÅ ÐÌÏÝÁÄÉ ÔÒÅÕÇÏÌØÎÉËÁ

,

É

25

, ÐÒÏ×ÅÄÅÎÙ ÄÉÁÇÏÎÁÌÉ.

×Ä×ÏÅ ÂÏÌØÛÅ ÐÌÏÝÁÄÉ ÔÒÅÕÇÏÌØÎÉËÁ

ADC

ABD

, Á ÐÌÏÝÁÄØ

. îÁÊÄÉÔÅ ÐÌÏÝÁÄÉ ÔÒÅÕÇÏÌØÎÉËÏ×

.

÷ÁÒÉÁÎÔ 2

ôÏÞËÉ M É N ÒÁÓÐÏÌÏÖÅÎÙ ÎÁ ÓÔÏÒÏÎÅ AC ÔÒÅÕÇÏÌØÎÉËÁ ABC, Á ÔÏÞËÉ K É L |
ÎÁ ÓÔÏÒÏÎÅ AB, ÐÒÉÞÅÍ AM : MN : NC = 1 : 3 : 1 É AK = KL = LB. éÚ×ÅÓÔÎÏ, ÞÔÏ ÐÌÏÝÁÄØ
ÔÒÅÕÇÏÌØÎÉËÁ ABC ÒÁ×ÎÁ 1. îÁÊÄÉÔÅ ÐÌÏÝÁÄØ ÞÅÔÙÒÅÈÕÇÏÌØÎÉËÁ KLNM.
146.6. îÁÊÄÉÔÅ ÕÇÏÌ ÐÒÉ ÏÓÎÏ×ÁÎÉÉ ÒÁ×ÎÏÂÅÄÒÅÎÎÏÊ ÔÒÁÐÅÃÉÉ, ÅÓÌÉ ÉÚ×ÅÓÔÎÏ, ÞÔÏ ÅÅ ÐÌÏÝÁÄØ ÒÁ×ÎÁ ÐÒÏÉÚ×ÅÄÅÎÉÀ ÏÔÒÅÚËÏ×, ÎÁ ËÏÔÏÒÙÅ ×ÙÓÏÔÁ, ÐÒÏ×ÅÄÅÎÎÁÑ ÉÚ ×ÅÒÛÉÎÙ ÍÅÎØÛÅÇÏ
ÏÓÎÏ×ÁÎÉÑ, ÄÅÌÉÔ ÂÏÌØÛÅÅ ÏÓÎÏ×ÁÎÉÅ.
146.7. ôÏÞËÉ M É N ÒÁÓÐÏÌÏÖÅÎÙ ÓÏÏÔ×ÅÔÓÔ×ÅÎÎÏ ÎÁ ÓÔÏÒÏÎÁÈ AB É BC ÔÒÅÕÇÏÌØÎÉËÁ
ABC. ïÔÒÅÚÏË, ÓÏÅÄÉÎÑÀÝÉÊ ÓÅÒÅÄÉÎÙ MN É AC, ÒÁ×ÅÎ ÏÔÒÅÚËÕ, ÓÏÅÄÉÎÑÀÝÅÍÕ ÓÅÒÅÄÉÎÙ
AN É CM. îÁÊÄÉÔÅ ÐÌÏÝÁÄØ ÔÒÅÕÇÏÌØÎÉËÁ ABC, ÅÓÌÉ AB = a, BC = b.
146.5.

146.8. äÉÁÇÏÎÁÌÉ
ÐÅÒÅÓÅËÁÀÔÓÑ × ÔÏÞËÅ

AC BD
O
AOB 2
É

BOC 18
×

ABCD
AOB BOC COD DOA
COD

×ÙÐÕËÌÏÇÏ ÞÅÔÙÒÅÈÕÇÏÌØÎÉËÁ

. îÁÊÄÉÔÅ ÐÌÏÝÁÄÉ ÔÒÅÕÇÏÌØÎÉËÏ×

ÞÔÏ ÐÌÏÝÁÄØ ÔÒÅÕÇÏÌØÎÉËÁ
ÎÉËÁ

* * *

×

, ÐÌÏÝÁÄØ ËÏÔÏÒÏÇÏ ÒÁ×ÎÁ 28,

,

,

ÒÁÚÁ ÂÏÌØÛÅ ÐÌÏÝÁÄÉ ÔÒÅÕÇÏÌØÎÉËÁ

ÒÁÚ ÂÏÌØÛÅ ÐÌÏÝÁÄÉ ÔÒÅÕÇÏÌØÎÉËÁ

DOA

É

, ÅÓÌÉ ÉÚ×ÅÓÔÎÏ,

, Á ÐÌÏÝÁÄØ ÔÒÅÕÇÏÌØ-

.

÷ÁÒÉÁÎÔ 3
146.9. ôÏÞËÉ E, F, M ÒÁÓÐÏÌÏÖÅÎÙ ÓÏÏÔ×ÅÔÓÔ×ÅÎÎÏ ÎÁ ÓÔÏÒÏÎÁÈ AB, BC, AC ÔÒÅÕÇÏÌØÎÉËÁ ABC. ïÔÒÅÚÏË AE ÓÏÓÔÁ×ÌÑÅÔ ÏÄÎÕ ÔÒÅÔØ ÓÔÏÒÏÎÙ AB, ÏÔÒÅÚÏË BF ÓÏÓÔÁ×ÌÑÅÔ ÏÄÎÕ
ÛÅÓÔÕÀ ÓÔÏÒÏÎÙ BC, ÏÔÒÅÚÏË AM ÓÏÓÔÁ×ÌÑÅÔ Ä×Å ÐÑÔÙÈ ÓÔÏÒÏÎÙ AC. îÁÊÄÉÔÅ ÏÔÎÏÛÅÎÉÅ
ÐÌÏÝÁÄÉ ÔÒÅÕÇÏÌØÎÉËÁ EFM Ë ÐÌÏÝÁÄÉ ÔÒÅÕÇÏÌØÎÉËÁ ABC.

249

÷ ×ÙÐÕËÌÏÍ ÞÅÔÙÒÅÈÕÇÏÌØÎÉËÅ ABCD ÔÏÞËÁ L Ñ×ÌÑÅÔÓÑ ÓÅÒÅÄÉÎÏÊ ÓÔÏÒÏÎÙ BC,
ÔÏÞËÁ M Ñ×ÌÑÅÔÓÑ ÓÅÒÅÄÉÎÏÊ AD, ÔÏÞËÁ N Ñ×ÌÑÅÔÓÑ ÓÅÒÅÄÉÎÏÊ ÓÔÏÒÏÎÙ AB. îÁÊÄÉÔÅ ÏÔÏÎÏÛÅÎÉÅ ÐÌÏÝÁÄÉ ÔÒÅÕÇÏÌØÎÉËÁ LMN Ë ÐÌÏÝÁÄÉ ÞÅÔÙÒÅÈÕÇÏÌØÎÉËÁ ABCD.
146.11. îÁ ÓÔÏÒÏÎÁÈ ×ÙÐÕËÌÏÇÏ ÞÅÔÙÒÅÈÕÇÏÌØÎÉËÁ ABCD, ÐÌÏÝÁÄØ ËÏÔÏÒÏÇÏ ÒÁ×ÎÁ 1,
×ÚÑÔÙ ÔÏÞËÉ: K | ÎÁ AB, L | ÎÁ BC, M | ÎÁ CD, N | ÎÁ AD. ðÒÉ ÜÔÏÍ AK=KB =
2, BL=LC = 1=3, CM=MD = 1, DN=NA = 1=5. îÁÊÄÉÔÅ ÐÌÏÝÁÄØ ÛÅÓÔÉÕÇÏÌØÎÉËÁ
AKLCMN.
146.10.

ïÔ×ÅÔÙ

146.1: 13=20; 146.2: ab=2; 146.3: 48; 146.4: 20, 10, 15, 5; 146.5: 7=15; 146.6: 45 ; 146.7:
ab=2; 146.8: 6, 18, 3, 1.

250

147. ëÏÎÔÒÏÌØÎÁÑ ÒÁÂÏÔÁ. óÒÅÄÎÑÑ ÌÉÎÉÑ ÔÒÅÕÇÏÌØÎÉËÁ

÷ÁÒÉÁÎÔ 1
147.1. ïÔÒÅÚËÉ, ÓÏÅÄÉÎÑÀÝÉÅ ÓÅÒÅÄÉÎÙ ÐÒÏÔÉ×ÏÐÏÌÏÖÎÙÈ ÓÔÏÒÏÎ ×ÙÐÕËÌÏÇÏ ÞÅÔÙÒÅÈÕÇÏÌØÎÉËÁ ×ÚÁÉÍÎÏ ÐÅÒÐÅÎÄÉËÕÌÑÒÎÙ É ÒÁ×ÎÙ ÍÅÖÄÕ ÓÏÂÏÊ. ïÄÎÁ ÉÚ ÄÉÁÇÏÎÁÌÅÊ ÞÅÔÙÒÅÈÕÇÏÌØÎÉËÁ ÒÁ×ÎÁ 10. îÁÊÄÉÔÅ ×ÔÏÒÕÀ ÄÉÁÇÏÎÁÌØ.

147.2. íÅÄÉÁÎÁ BD ÔÒÅÕÇÏÌØÎÉËÁ ABC ÒÁ×ÎÁ 14 É ÏÂÒÁÚÕÅÔ ÕÇÏÌ 30 ÓÏ ÓÔÏÒÏÎÏÊ BC.
îÁÊÄÉÔÅ ×ÙÓÏÔÕ, ÐÒÏ×ÅÄÅÎÎÕÀ ÉÚ ×ÅÒÛÉÎÙ A.
147.3. îÁÊÄÉÔÅ ÐÅÒÉÍÅÔÒ ÔÒÅÕÇÏÌØÎÉËÁ Ó ×ÅÒÛÉÎÁÍÉ × ÓÅÒÅÄÉÎÁÈ ÍÅÄÉÁÎ ÔÒÅÕÇÏÌØÎÉËÁ
Ó ÐÅÒÉÍÅÔÒÏÍ, ÒÁ×ÎÙÍ 32.

147.4. äÉÁÇÏÎÁÌØ
ÎÅÊ ÐÏÄ ÕÇÌÏÍ ×

AC = 24

60

AC

* * *

ÐÁÒÁÌÌÅÌÏÇÒÁÍÍÁ

ABCD

×ÔÒÏÅ ÂÏÌØÛÅ ÄÉÁÇÏÎÁÌÉ

. îÁÊÄÉÔÅ ÏÔÒÅÚÏË, ÓÏÅÄÉÎÑÀÝÉÊ ×ÅÒÛÉÎÕ

D

BD

É ÐÅÒÅÓÅËÁÅÔÓÑ Ó

Ó ÓÅÒÅÄÉÎÏÊ ÓÔÏÒÏÎÙ

BC

, ÅÓÌÉ

.

÷ÁÒÉÁÎÔ 2
147.5. ïÔÒÅÚËÉ, ÓÏÅÄÉÎÑÀÝÉÅ ÓÅÒÅÄÉÎÙ ÐÒÏÔÉ×ÏÐÏÌÏÖÎÙÈ ÓÔÏÒÏÎ ×ÙÐÕËÌÏÇÏ ÞÅÔÙÒÅÈÕÇÏÌØÎÉËÁ, ÒÁ×ÎÙ 6 É ÐÅÒÅÓÅËÁÀÔÓÑ ÐÏÄ ÕÇÌÏÍ 60. îÁÊÄÉÔÅ ÍÅÎØÛÕÀ ÄÉÁÇÏÎÁÌØ ÞÅÔÙÒÅÈÕÇÏÌØÎÉËÁ.

147.6. ïÓÔÒÙÊ ÕÇÏÌ A ÒÏÍÂÁ ABCD ÒÁ×ÅÎ 45 , ÐÒÏÅËÃÉÑ ÓÔÏÒÏÎÙ AB ÎÁ ÓÔÏÒÏÎÕ AD
ÒÁ×ÎÁ 12. îÁÊÄÉÔÅ ÒÁÓÓÔÏÑÎÉÅ ÏÔ ÃÅÎÔÒÁ ÒÏÍÂÁ ÄÏ ÓÔÏÒÏÎÙ CD.
147.7. ÷ÙÓÏÔÙ ÏÓÔÒÏÕÇÏÌØÎÏÇÏ ÔÒÅÕÇÏÌØÎÉËÁ ABC, ÐÒÏ×ÅÄÅÎÎÙÅ ÉÚ ×ÅÒÛÉÎ B É C, ÒÁ×ÎÙ
7 É 9, Á ÍÅÄÉÁÎÁ AM ÒÁ×ÎÁ 8. ôÏÞËÉ P É Q ÓÉÍÍÅÔÒÉÞÎÙ ÔÏÞËÅ M ÏÔÎÏÓÉÔÅÌØÎÏ ÓÔÏÒÏÎ AC
É AB ÓÏÏÔ×ÅÔÓÔ×ÅÎÎÏ. îÁÊÄÉÔÅ ÐÅÒÉÍÅÔÒ ÞÅÔÙÒÅÈÕÇÏÌØÎÉËÁ APMQ.

147.8. äÉÁÇÏÎÁÌØ
ÎÅÊ ÐÏÄ ÕÇÌÏÍ ×

AC = 24

60

.

AC

* * *

ÐÁÒÁÌÌÅÌÏÇÒÁÍÍÁ

ABCD

×ÔÒÏÅ ÂÏÌØÛÅ ÄÉÁÇÏÎÁÌÉ

îÁÊÄÉÔÅ ÏÔÒÅÚÏË, ÓÏÅÄÉÎÑÀÝÉÊ ×ÅÒÛÉÎÕ

.

251

D

BD

É ÐÅÒÅÓÅËÁÅÔÓÑ Ó

Ó ÓÅÒÅÄÉÎÏÊ ÓÔÏÒÏÎÙ

BC

, ÅÓÌÉ

148. ëÏÎÔÒÏÌØÎÁÑ ÒÁÂÏÔÁ. ôÒÁÐÅÃÉÑ

÷ÁÒÉÁÎÔ 1
148.1. ÷ÙÓÏÔÁ ÒÁ×ÎÏÂÅÄÒÅÎÎÏÊ ÔÒÁÐÅÃÉÉ, ÐÒÏ×ÅÄÅÎÎÁÑ ÉÚ ×ÅÒÛÉÎÙ ÍÅÎØÛÅÇÏ ÏÓÎÏ×ÁÎÉÑ,
ÄÅÌÉÔ ÅÅ ÂÏÌØÛÅÅ ÏÓÎÏ×ÁÎÉÅ ÎÁ ÏÔÒÅÚËÉ, ÒÁ×ÎÙÅ 4 É 8. îÁÊÄÉÔÅ ÏÓÎÏ×ÁÎÉÑ ÔÒÁÐÅÃÉÉ.
148.2. íÅÎØÛÁÑ ÂÏËÏ×ÁÑ ÓÔÏÒÏÎÁ ÐÒÑÍÏÕÇÏÌØÎÏÊ ÔÒÁÐÅÃÉÉ ÒÁ×ÎÁ 3, Á ÂÏÌØÛÁÑ ÏÂÒÁÚÕÅÔ
ÕÇÏÌ, ÒÁ×ÎÙÊ 30, Ó ÏÄÎÉÍ ÉÚ ÏÓÎÏ×ÁÎÉÊ. îÁÊÄÉÔÅ ÜÔÏ ÏÓÎÏ×ÁÎÉÅ, ÅÓÌÉ ÎÁ ÎÅÍ ÌÅÖÉÔ ÔÏÞËÁ
ÐÅÒÓÅÞÅÎÉÑ ÂÉÓÓÅËÔÒÉÓ ÕÇÌÏ× ÐÒÉ ÄÒÕÇÏÍ ÏÓÎÏ×ÁÎÉÉ.

148.3. óÒÅÄÎÑÑ ÌÉÎÉÑ ÔÒÁÐÅÃÉÉ ÒÁ×ÎÁ 4, Á ÕÇÌÙ ÐÒÉ ÏÄÎÏÍ ÉÚ ÏÓÎÏ×ÁÎÉÊ ÒÁ×ÎÙ 40 É

50 . îÁÊÄÉÔÅ ÏÓÎÏ×ÁÎÉÑ ÔÒÁÐÅÃÉÉ, ÅÓÌÉ ÏÔÒÅÚÏË, ÓÏÅÄÉÎÑÀÝÉÊ ÓÅÒÅÄÉÎÙ ÏÓÎÏ×ÁÎÉÊ, ÒÁ×ÅÎ
1.

* * *

148.4. ÷ ×ÙÐÕËÌÏÍ ÞÅÔÙÒÅÈÕÇÏÌØÎÉËÅ

É

CD

, ÒÁ×ÎÁ

1

. åÓÌÉ ÓÔÏÒÏÎÙ

ÐÒÑÍÙÍÉ ÂÕÄÅÔ ÒÁ×ÅÎ

90

BC AD
É

ABCD

ÄÌÉÎÁ ÏÔÒÅÚËÁ, ÓÏÅÄÉÎÑÀÝÅÇÏ ÓÅÒÅÄÉÎÙ ÓÔÏÒÏÎ

AB

ÐÒÏÄÏÌÖÉÔØ ÄÏ ÉÈ ÐÅÒÅÓÅÞÅÎÉÑ, ÔÏ ÕÇÏÌ, ÏÂÒÁÚÏ×ÁÎÎÙÊ ÜÔÉÍÉ

. îÁÊÄÉÔÅ ÄÌÉÎÕ ÏÔÒÅÚËÁ, ÓÏÅÄÉÎÑÀÝÅÇÏ ÓÅÒÅÄÉÎÙ ÄÉÁÇÏÎÁÌÅÊ.

÷ÁÒÉÁÎÔ 2

îÁÊÄÉÔÅ ÍÅÎØÛÅÅ ÏÓÎÏ×ÁÎÉÅ ÒÁ×ÎÏÂÅÄÒÅÎÎÏÊ ÔÒÁÐÅÃÉÉ, ÅÓÌÉ ×ÙÓÏÔÁ, ÐÒÏ×ÅÄÅÎÎÁÑ
ÉÚ ×ÅÒÛÉÎÙ ÍÅÎØÛÅÇÏ ÏÓÎÏ×ÁÎÉÑ, ÄÅÌÉÔ ÂÏÌØÛÅÅ ÏÓÎÏ×ÁÎÉÅ ÎÁ ÏÔÒÅÚËÉ, ÏÄÉÎ ÉÚ ËÏÔÏÒÙÈ ÎÁ
5 ÂÏÌØÛÅ ÄÒÕÇÏÇÏ.

148.6. ïÄÉÎ ÉÚ ÕÇÌÏ× ÐÒÑÍÏÕÇÏÌØÎÏÊ ÔÒÁÐÅÃÉÉ ÒÁ×ÅÎ 120 , ÂÏÌØÛÅÅ ÏÓÎÏ×ÁÎÉÅ ÒÁ×ÎÏ 12.
îÁÊÄÉÔÅ ÏÔÒÅÚÏË, ÓÏÅÄÉÎÑÀÝÉÊ ÓÅÒÅÄÉÎÙ ÄÉÁÇÏÎÁÌÅÊ, ÅÓÌÉ ÉÚ×ÅÓÔÎÏ, ÞÔÏ ÍÅÎØÛÁÑ ÄÉÁÇÏÎÁÌØ
ÔÒÁÐÅÃÉÉ ÒÁ×ÎÁ ÅÅ ÂÏÌØÛÅÍÕ ÏÓÎÏ×ÁÎÉÀ.


148.7. ÷ ÔÒÁÐÅÃÉÉ ÕÇÌÙ ÐÒÉ ÏÄÎÏÍ ÉÚ ÏÓÎÏ×ÁÎÉÊ ÒÁ×ÎÙ 20 É 70 , Á ÏÔÒÅÚÏË, ÓÏÅÄÉÎÑÀÝÉÊ
ÓÅÒÅÄÉÎÙ ÏÓÎÏ×ÁÎÉÊ, ÒÁ×ÅÎ 2. îÁÊÄÉÔÅ ÏÓÎÏ×ÁÎÉÑ ÔÒÁÐÅÃÉÉ, ÅÓÌÉ ÅÅ ÓÒÅÄÎÑÑ ÌÉÎÉÑ ÒÁ×ÎÁ 4.
148.5.

* * *

148.8. ÷ ×ÙÐÕËÌÏÍ ÞÅÔÙÒÅÈÕÇÏÌØÎÉËÅ

ABCD

ÄÌÉÎÁ ÏÔÒÅÚËÁ, ÓÏÅÄÉÎÑÀÝÅÇÏ ÓÅÒÅÄÉÎÙ ÄÉÁÇÏ-

ÎÁÌÅÊ, ÒÁ×ÎÁ ÄÌÉÎÅ ÏÔÒÅÚËÁ, ÓÏÅÄÉÎÑÀÝÅÇÏ ÓÅÒÅÄÉÎÙ ÓÔÏÒÏÎ
ÏÂÒÁÚÏ×ÁÎÎÏÇÏ ÐÒÏÄÏÌÖÅÎÉÅÍ ÓÔÏÒÏÎ

AB CD
É

.

252

AD BC
É

. îÁÊÄÉÔÅ ×ÅÌÉÞÉÎÕ ÕÇÌÁ,